6Июл

Π‘Ρ…Π΅ΠΌΠ° ΠΏΠΎΠ΄ΠΊΠ»ΡŽΡ‡Π΅Π½ΠΈΡ Ρ‚ΡƒΠΌΠ°Π½ΠΎΠΊ Ρ‡Π΅Ρ€Π΅Π· Ρ€Π΅Π»Π΅ ΠΈ ΠΊΠ½ΠΎΠΏΠΊΡƒ: НичСго Π½Π΅ Π½Π°ΠΉΠ΄Π΅Π½ΠΎ для% request_words%

Π‘Ρ…Π΅ΠΌΠ° ΠΏΠΎΠ΄ΠΊΠ»ΡŽΡ‡Π΅Π½ΠΈΡ ΠΏΡ€ΠΎΡ‚ΠΈΠ²ΠΎΡ‚ΡƒΠΌΠ°Π½ΠΎΠΊ Ρ‡Π΅Ρ€Π΅Π· Ρ€Π΅Π»Π΅ ΠΈ ΠΊΠ½ΠΎΠΏΠΊΡƒ

Π‘ΠΎΠ΄Π΅Ρ€ΠΆΠ°Π½ΠΈΠ΅

  • Π˜Π½ΡΡ‚Ρ€ΡƒΠΊΡ†ΠΈΡ β€” ΠΊΠ°ΠΊ ΠΏΠΎΠ΄ΠΊΠ»ΡŽΡ‡ΠΈΡ‚ΡŒ ΠΏΡ€ΠΎΡ‚ΠΈΠ²ΠΎΡ‚ΡƒΠΌΠ°Π½ΠΊΠΈ Ρ‡Π΅Ρ€Π΅Π· Ρ€Π΅Π»Π΅ ΠΈ ΠΊΠ½ΠΎΠΏΠΊΡƒ Π½Π° ΠΏΡ€ΠΈΠΌΠ΅Ρ€Π΅ Π¨Π΅Π²Ρ€ΠΎΠ»Π΅ Нива
    • ΠŸΠΎΠ»Π΅Π·Π½Ρ‹Π΅ ΠΏΡ€Π°Π²ΠΈΠ»Π° β€” Π½Π°Π΄ΠΎ Π·Π½Π°Ρ‚ΡŒ
  • Для Ρ‡Π΅Π³ΠΎ Π½ΡƒΠΆΠ½Ρ‹ ΠΏΡ€ΠΎΡ‚ΠΈΠ²ΠΎΡ‚ΡƒΠΌΠ°Π½ΠΊΠΈ
  • ВрСбования ΠΊ установкС ΠΏΡ€ΠΎΡ‚ΠΈΠ²ΠΎΡ‚ΡƒΠΌΠ°Π½Π½Ρ‹Ρ… Ρ„Π°Ρ€
  • ΠŸΠΎΠ΄ΠΊΠ»ΡŽΡ‡Π΅Π½ΠΈΠ΅ ПВЀ Ρ‡Π΅Ρ€Π΅Π· Ρ€Π΅Π»Π΅ ΠΈ ΠΊΠ½ΠΎΠΏΠΊΡƒ
  • Π‘Ρ…Π΅ΠΌΠ° ΠΏΠ°Ρ€Π°Π»Π»Π΅Π»ΡŒΠ½ΠΎΠ³ΠΎ Π²ΠΊΠ»ΡŽΡ‡Π΅Π½ΠΈΡ ΠΏΡ€ΠΎΡ‚ΠΈΠ²ΠΎΡ‚ΡƒΠΌΠ°Π½ΠΎΠΊ

Π’ΠΎ врСмя Π½Π΅Π½Π°ΡΡ‚ΡŒΡ ΡΡ„Ρ„Π΅ΠΊΡ‚ΠΈΠ²Π½ΠΎΡΡ‚ΡŒ Ρ„Π°Ρ€ Π³ΠΎΠ»ΠΎΠ²Π½ΠΎΠ³ΠΎ свСта Π·Π°ΠΌΠ΅Ρ‚Π½ΠΎ сниТаСтся, Ρ‡Ρ‚ΠΎ Π½Π΅ Ρ‚ΠΎΠ»ΡŒΠΊΠΎ Π΄Π΅Π»Π°Π΅Ρ‚ ΡƒΠΏΡ€Π°Π²Π»Π΅Π½ΠΈΠ΅ Π°Π²Ρ‚ΠΎΠΌΠΎΠ±ΠΈΠ»Π΅ΠΌ ΠΌΠ΅Π½Π΅Π΅ ΠΊΠΎΠΌΡ„ΠΎΡ€Ρ‚Π½Ρ‹ΠΌ, Π½ΠΎ ΠΈ сказываСтся Π½Π° бСзопасности ΠΏΠΎΠ΅Π·Π΄ΠΊΠΈ.

Π‘Π²Π΅Ρ‚ΠΎΠ²ΠΎΠΉ Π»ΡƒΡ‡ Ρ„Π°Ρ€ Π±Π»ΠΈΠΆΠ½Π΅Π³ΠΎ ΠΈ дальнСго свСта отраТаСтся ΠΎΡ‚ капСль доТдя ΠΈ частичСк водяного ΠΏΠ°Ρ€Π° Π²ΠΎ врСмя Ρ‚ΡƒΠΌΠ°Π½Π°, образуя ΠΏΠ»ΠΎΡ‚Π½ΡƒΡŽ Π±Π΅Π»ΡƒΡŽ ΠΏΠ΅Π»Π΅Π½Ρƒ.

Настоящим спасСниСм для водитСля Π² Ρ‚Π°ΠΊΠΈΡ… ΠΏΠΎΠ³ΠΎΠ΄Π½Ρ‹Ρ… условиях станут ΠΏΡ€ΠΎΡ‚ΠΈΠ²ΠΎΡ‚ΡƒΠΌΠ°Π½Π½Ρ‹Π΅ Ρ„Π°Ρ€Ρ‹ (ПВЀ), ΠΊΠΎΡ‚ΠΎΡ€Ρ‹Π΅ Π΄Π°ΡŽΡ‚ плоский ΠΈ ΡˆΠΈΡ€ΠΎΠΊΠΈΠΉ Π³ΠΎΡ€ΠΈΠ·ΠΎΠ½Ρ‚Π°Π»ΡŒΠ½Ρ‹ΠΉ Π»ΡƒΡ‡ свСта.

Он стСлСтся Π½Π°Π΄ Π΄ΠΎΡ€ΠΎΠ³ΠΎΠΉ, Ρ…ΠΎΡ€ΠΎΡˆΠΎ подсвСчивая ΠΎΠ±ΠΎΡ‡ΠΈΠ½Ρƒ, ΡƒΠ»ΡƒΡ‡ΡˆΠ°Ρ Π²ΠΈΠ΄ΠΈΠΌΠΎΡΡ‚ΡŒ самого автомобиля для Π²ΠΎΠ΄ΠΈΡ‚Π΅Π»Π΅ΠΉ встрСчных Π°Π²Ρ‚ΠΎ.

Π¨Ρ‚Π°Ρ‚Π½Ρ‹Π΅ ПВЀ Ρ€Π΅Π΄ΠΊΠΎ Π²ΡΡ‚Ρ€Π΅Ρ‡Π°ΡŽΡ‚ΡΡ Π² Π±Π°Π·ΠΎΠ²Ρ‹Ρ… комплСктациях Π°Π²Ρ‚ΠΎΠΌΠΎΠ±ΠΈΠ»Π΅ΠΉ, ΠΎΠ΄Π½Π°ΠΊΠΎ ΠΏΡ€ΠΈ ΠΆΠ΅Π»Π°Π½ΠΈΠΈ ΠΏΡ€ΠΎΡ‚ΠΈΠ²ΠΎΡ‚ΡƒΠΌΠ°Π½ΠΊΠΈ всСгда ΠΌΠΎΠΆΠ½ΠΎ приобрСсти ΠΎΡ‚Π΄Π΅Π»ΡŒΠ½ΠΎ ΠΈ ΡƒΡΡ‚Π°Π½ΠΎΠ²ΠΈΡ‚ΡŒ ΡΠ°ΠΌΠΎΡΡ‚ΠΎΡΡ‚Π΅Π»ΡŒΠ½ΠΎ. Для этого достаточно ΠΈΠΌΠ΅Ρ‚ΡŒ Π±Π°Π·ΠΎΠ²Ρ‹Π΅ знания Π² области элСктротСхники ΠΈ элСктроники.

Π’ΠΎΡ‚ Ρ‚Π°ΠΊ выглядит схСма ΠΏΠΎΠ΄ΠΊΠ»ΡŽΡ‡Π΅Π½ΠΈΡ ΠΊΠ½ΠΎΠΏΠΊΠΈ ПВЀ Ρ‡Π΅Ρ€Π΅Π· Ρ€Π΅Π»Π΅:

На Π½Π΅ΠΉ всС ΠΏΡ€Π΅Π΄Π΅Π»ΡŒΠ½ΠΎ просто ΠΈ понятно.

  1. Π‘ аккумуляторной Π±Π°Ρ‚Π°Ρ€Π΅ΠΈ плюс Ρ‡Π΅Ρ€Π΅Π· ΠΏΡ€Π΅Π΄ΠΎΡ…Ρ€Π°Π½ΠΈΡ‚Π΅Π»ΡŒ ΠΈΠ΄Π΅Ρ‚ Π½Π° ΠΊΠΎΠ½Ρ‚Π°ΠΊΡ‚ Ρ€Π΅Π»Π΅ (30) ΠΈ Π΄Π°Π»Π΅Π΅ с ΠΊΠΎΠ½Ρ‚Π°ΠΊΡ‚Π° (85) ΠΈΠ΄Π΅Ρ‚ Π½Π° ΠΎΠ΄ΠΈΠ½ ΠΊΠΎΠ½Ρ‚Π°ΠΊΡ‚ ΠΊΠ½ΠΎΠΏΠΊΠΈ Π²ΠΊΠ»ΡŽΡ‡Π΅Π½ΠΈΡ/Π²Ρ‹ΠΊΠ»ΡŽΡ‡Π΅Π½ΠΈΡ ПВЀ.
  2. На Π²Ρ‚ΠΎΡ€ΠΎΠΉ ΠΊΠΎΠ½Ρ‚Π°ΠΊΡ‚ ΠΊΠ½ΠΎΠΏΠΊΠΈ подаСтся минус.
  3. Π”Π°Π»Π΅Π΅ с ΠΊΠΎΠ½Ρ‚Π°ΠΊΡ‚Π° Ρ€Π΅Π»Π΅ (87) плюс ΠΈΠ΄Π΅Ρ‚ Π½Π° Π»Π°ΠΌΠΏΡ‹ ΠΏΡ€ΠΎΡ‚ΠΈΠ²ΠΎΡ‚ΡƒΠΌΠ°Π½Π½Ρ‹Ρ… Ρ„Π°Ρ€.
  4. К ΠΊΠΎΠ½Ρ‚Π°ΠΊΡ‚Ρƒ (86) ΠΏΠΎΠ΄ΠΊΠ»ΡŽΡ‡Π°Π΅Ρ‚ΡΡ минус.

ΠžΠ±Ρ€Π°Ρ‚ΠΈΡ‚Π΅ Π²Π½ΠΈΠΌΠ°Π½ΠΈΠ΅ Π½Π° Ρ‚ΠΎ, Ρ‡Ρ‚ΠΎ каТдая модСль автомобиля ΠΈΠΌΠ΅Π΅Ρ‚ свои особСнности, поэтому схСма ΠΌΠΎΠΆΠ΅Ρ‚ ΠΊΠΎΡ€Ρ€Π΅ΠΊΡ‚ΠΈΡ€ΠΎΠ²Π°Ρ‚ΡŒΡΡ.

Π˜Π½ΡΡ‚Ρ€ΡƒΠΊΡ†ΠΈΡ β€” ΠΊΠ°ΠΊ ΠΏΠΎΠ΄ΠΊΠ»ΡŽΡ‡ΠΈΡ‚ΡŒ ΠΏΡ€ΠΎΡ‚ΠΈΠ²ΠΎΡ‚ΡƒΠΌΠ°Π½ΠΊΠΈ Ρ‡Π΅Ρ€Π΅Π· Ρ€Π΅Π»Π΅ ΠΈ ΠΊΠ½ΠΎΠΏΠΊΡƒ Π½Π° ΠΏΡ€ΠΈΠΌΠ΅Ρ€Π΅ Π¨Π΅Π²Ρ€ΠΎΠ»Π΅ Нива

Как Π²ΠΈΠ΄Π½ΠΎ ΠΈΠ· прСдставлСнной Π²Ρ‹ΡˆΠ΅ схСмы, для ΠΏΠΎΠ΄ΠΊΠ»ΡŽΡ‡Π΅Π½ΠΈΡ ΠΏΡ€ΠΎΡ‚ΠΈΠ²ΠΎΡ‚ΡƒΠΌΠ°Π½ΠΎΠΊ понадобятся:

  • сами ПВЀ;
  • Ρ€Π΅Π»Π΅ ΠΏΡ€ΠΎΡ‚ΠΈΠ²ΠΎΡ‚ΡƒΠΌΠ°Π½Π½Ρ‹Ρ… Ρ„Π°Ρ€;
  • ΠΊΠ½ΠΎΠΏΠΊΠ° Π²ΠΊΠ»ΡŽΡ‡Π΅Π½ΠΈΡ/Π²Ρ‹ΠΊΠ»ΡŽΡ‡Π΅Π½ΠΈΡ Ρ„Π°Ρ€;
  • ΠΏΡ€Π΅Π΄ΠΎΡ…Ρ€Π°Π½ΠΈΡ‚Π΅Π»ΡŒ 10 А;
  • ΠΏΡ€ΠΎΠ²ΠΎΠ΄Π° ΠΈ ΡΠΎΠ΅Π΄ΠΈΠ½ΠΈΡ‚Π΅Π»ΡŒΠ½Ρ‹Π΅ ΠΊΠ»Π΅ΠΌΠΌΡ‹.

Для Ρ€Π°Π±ΠΎΡ‚Ρ‹ Ρ‚Π°ΠΊΠΆΠ΅ понадобятся Π½Π΅ΠΊΠΎΡ‚ΠΎΡ€Ρ‹Π΅ инструмСнты: ΠΎΡ‚Π²Π΅Ρ€Ρ‚ΠΊΠΈ, острый Π½ΠΎΠΆ для зачистки ΠΏΡ€ΠΎΠ²ΠΎΠ΄ΠΎΠ² ΠΈ ΠΈΠ·ΠΎΠ»Π΅Π½Ρ‚Π°.


Π Π°Π±ΠΎΡ‚Ρƒ ΠΏΠΎ установкС ΠΈ ΠΏΠΎΠ΄ΠΊΠ»ΡŽΡ‡Π΅Π½ΠΈΡŽ ΠΏΡ€ΠΎΡ‚ΠΈΠ²ΠΎΡ‚ΡƒΠΌΠ°Π½ΠΎΠΊ Ρ‡Π΅Ρ€Π΅Π· Ρ€Π΅Π»Π΅ ΠΈ ΠΊΠ½ΠΎΠΏΠΊΡƒ выполняСм Π² ΡΠ»Π΅Π΄ΡƒΡŽΡ‰Π΅ΠΉ ΠΏΠΎΡΠ»Π΅Π΄ΠΎΠ²Π°Ρ‚Π΅Π»ΡŒΠ½ΠΎΡΡ‚ΠΈ (рассмотрим Π½Π° ΠΏΡ€ΠΈΠΌΠ΅Ρ€Π΅ Π¨Π΅Π²Ρ€ΠΎΠ»Π΅ Нива):

  1. ΠžΠΏΡ€Π΅Π΄Π΅Π»ΡΠ΅ΠΌ мСсто располоТСния Ρ€Π΅Π»Π΅. ΠŸΠΎΡΠΊΠΎΠ»ΡŒΠΊΡƒ Π΄Π°Π½Π½Ρ‹ΠΉ ΠΊΠΎΠΌΠΏΠΎΠ½Π΅Π½Ρ‚ ΠΈΠΌΠ΅Π΅Ρ‚ нСбольшиС Ρ€Π°Π·ΠΌΠ΅Ρ€Ρ‹, Π΅Π³ΠΎ ΠΌΠΎΠΆΠ½ΠΎ Π±Π΅Π· Ρ‚Ρ€ΡƒΠ΄Π° ΡΠΏΡ€ΡΡ‚Π°Ρ‚ΡŒ Π·Π° ΠΏΡ€ΠΈΠ±ΠΎΡ€Π½ΠΎΠΉ панСлью.
  2. ΠžΠΏΡ€Π΅Π΄Π΅Π»ΡΠ΅ΠΌ мСсто располоТСния ΠΊΠ½ΠΎΠΏΠΊΠΈ Π²ΠΊΠ»ΡŽΡ‡Π΅Π½ΠΈΡ/Π²Ρ‹ΠΊΠ»ΡŽΡ‡Π΅Π½ΠΈΡ ПВЀ;
  3. ΠžΡ‚ΠΌΠ΅Ρ€ΡΠ΅ΠΌ Π½Π΅ΠΎΠ±Ρ…ΠΎΠ΄ΠΈΠΌΡƒΡŽ для ΠΏΠΎΠ΄ΠΊΠ»ΡŽΡ‡Π΅Π½ΠΈΡ Π΄Π»ΠΈΠ½Ρƒ ΠΏΡ€ΠΎΠ²ΠΎΠ΄ΠΎΠ²;
  4. Π”Π°Π»Π΅Π΅ протягиваСм ΠΌΠ΅Π΄Π½Ρ‹ΠΉ ΠΏΡ€ΠΎΠ²ΠΎΠ΄ ΠΎΡ‚ ΠΊΠΎΠ½Ρ‚Π°ΠΊΡ‚Π° 30 ΠΊ аккумулятору (+), ΠΏΡ€Π΅Π΄Π²Π°Ρ€ΠΈΡ‚Π΅Π»ΡŒΠ½ΠΎ установив Π² ΡƒΠ΄ΠΎΠ±Π½ΠΎΠΌ мСстС ΠΏΡ€Π΅Π΄ΠΎΡ…Ρ€Π°Π½ΠΈΡ‚Π΅Π»ΡŒ Π½Π° 10 АмпСр;
  5. ΠŸΠΎΠ΄ΠΊΠ»ΡŽΡ‡Π°Π΅ΠΌ ΠΊΠ½ΠΎΠΏΠΊΡƒ ΠΏΡ€ΠΎΡ‚ΠΈΠ²ΠΎΡ‚ΡƒΠΌΠ°Π½Π½Ρ‹Ρ… Ρ„Π°Ρ€ ΠΊ ΠΊΠΎΠ½Ρ‚Π°ΠΊΡ‚Ρƒ 85 ΠΈ устанавливаСм Π² Π²Ρ‹Π±Ρ€Π°Π½Π½ΠΎΠ΅ мСсто. Как ΠΏΡ€Π°Π²ΠΈΠ»ΠΎ, ΠΊΠ½ΠΎΠΏΠΊΠ° ΠΏΠ΅Ρ€Π΅Π΄Π½ΠΈΡ… ПВЀ устанавливаСтся Π½Π° мСсто ΡˆΡ‚Π°Ρ‚Π½Ρ‹Ρ… ΠΊΠ½ΠΎΠΏΠΎΠΊ, Π³Π΄Π΅ стоит Π·Π°Π³Π»ΡƒΡˆΠΊΠ°;
  6. ΠšΠΎΠ½Ρ‚Π°ΠΊΡ‚ Ρ€Π΅Π»Π΅ 86 Π² любом ΡƒΠ΄ΠΎΠ±Π½ΠΎΠΌ мСстС соСдиняСм с массой;
  7. УстанавливаСм ΠΏΡ€ΠΎΡ‚ΠΈΠ²ΠΎΡ‚ΡƒΠΌΠ°Π½Π½Ρ‹Π΅ Ρ„Π°Ρ€Ρ‹. ΠŸΡ€ΠΎΠΈΠ·Π²ΠΎΠ΄ΠΈΡ‚Π΅Π»ΠΈ ΠΏΠΎΠ·Π°Π±ΠΎΡ‚ΠΈΠ»ΠΈΡΡŒ ΠΎ Ρ‚ΠΎΠΌ, Ρ‡Ρ‚ΠΎΠ±Ρ‹ Ρ‚Π΅ Π²Π»Π°Π΄Π΅Π»ΡŒΡ†Ρ‹, ΠΊΠΎΡ‚ΠΎΡ€Ρ‹Π΅ ΠΏΠΎΠΆΠ΅Π»Π°ΡŽΡ‚ ΡƒΡΡ‚Π°Π½ΠΎΠ²ΠΈΡ‚ΡŒ ΠΏΡ€ΠΎΡ‚ΠΈΠ²ΠΎΡ‚ΡƒΠΌΠ°Π½Π½Ρ‹Π΅ Ρ„Π°Ρ€Ρ‹, Π½Π΅ Π²Ρ‹Π΄ΡƒΠΌΡ‹Π²Π°Π»ΠΈ «вСлосипСд». Π’ ΠΏΠ΅Ρ€Π΅Π΄Π½ΠΈΡ… Π±Π°ΠΌΠΏΠ΅Ρ€Π°Ρ… Π¨Π΅Π²Ρ€ΠΎΠ»Π΅ Нива для этого прСдусмотрСны ΡΠΏΠ΅Ρ†ΠΈΠ°Π»ΡŒΠ½Ρ‹Π΅ ниши, ΠΊΠΎΡ‚ΠΎΡ€Ρ‹Π΅ Π·Π°ΠΊΡ€Ρ‹Ρ‚Ρ‹ Π·Π°Π³Π»ΡƒΡˆΠΊΠ°ΠΌΠΈ. Для Ρ‚ΠΎΠ³ΠΎ Ρ‡Ρ‚ΠΎΠ±Ρ‹ ΠΈΡ… ΡΠ½ΡΡ‚ΡŒ Π½Π΅ΠΎΠ±Ρ…ΠΎΠ΄ΠΈΠΌΠΎ с Π²Π½ΡƒΡ‚Ρ€Π΅Π½Π½Π΅ΠΉ стороны Π±Π°ΠΌΠΏΠ΅Ρ€Π° ΠΎΡ‚ΠΊΡ€ΡƒΡ‚ΠΈΡ‚ΡŒ саморСзы. ВставляСм Ρ„Π°Ρ€Ρ‹ ΠΈ смотрим Π½Π° обозначСния ΠΏΡ€ΠΎΠ²ΠΎΠ΄ΠΎΠ².
  8. Плюсовой ΠΏΡ€ΠΎΠ²ΠΎΠ΄ Π½Π° Π½ΠΈΡ… ΠΏΠΎΠ΄Π°Π΅ΠΌ с ΠΊΠΎΠ½Ρ‚Π°ΠΊΡ‚Π° 87, минус β€” с ΠΊΡƒΠ·ΠΎΠ²Π° Π°Π²Ρ‚ΠΎ;
  9. ΠŸΡ€ΠΎΠ²ΠΎΠ΄ΠΈΠΌ тСстированиС.

Π’Π˜Π”Π•Πž ИНБВРУКЦИЯ
» alt=»»>

ΠŸΠΎΠ»Π΅Π·Π½Ρ‹Π΅ ΠΏΡ€Π°Π²ΠΈΠ»Π° β€” Π½Π°Π΄ΠΎ Π·Π½Π°Ρ‚ΡŒ

ΠŸΡ€ΠΈ Π²Ρ‹Π±ΠΎΡ€Π΅ самих ΠΏΡ€ΠΎΡ‚ΠΈΠ²ΠΎΡ‚ΡƒΠΌΠ°Π½ΠΎΠΊ ΠΈ мСста для ΠΈΡ… располоТСния слСдуСт Π²Π½ΠΈΠΌΠ°Ρ‚Π΅Π»ΡŒΠ½ΠΎ ΠΎΠ·Π½Π°ΠΊΠΎΠΌΠΈΡ‚ΡŒΡΡ с принятыми Π½ΠΎΡ€ΠΌΠ°ΠΌΠΈ, ΠΊΠΎΡ‚ΠΎΡ€Ρ‹Π΅ строго прописаны Π² ΠŸΠ”Π”.

Π’Π°ΠΊ, Π½Π° Ρ‚Π΅Ρ€Ρ€ΠΈΡ‚ΠΎΡ€ΠΈΠΈ России допускаСтся установка Ρ‚ΠΎΠ»ΡŒΠΊΠΎ заводских ПВЀ, ΠΏΡ€ΠΎΡˆΠ΅Π΄ΡˆΠΈΡ… ΡΠ΅Ρ€Ρ‚ΠΈΡ„ΠΈΠΊΠ°Ρ†ΠΈΡŽ. ΠŸΠΎΠ΄Ρ‚Π²Π΅Ρ€ΠΆΠ΄Π΅Π½ΠΈΠ΅ΠΌ этого являСтся Π·Π½Π°ΠΊ Π•22 Π² ΠΊΡ€ΡƒΠ³Π΅, ΠΊΠΎΡ‚ΠΎΡ€Ρ‹ΠΉ наносится Π½Π° корпус ПВЀ.

  1. Π‘Π°ΠΌΠΈΡ… ΠΏΡ€ΠΎΡ‚ΠΈΠ²ΠΎΡ‚ΡƒΠΌΠ°Π½ΠΎΠΊ Π΄ΠΎΠ»ΠΆΠ½ΠΎ Π±Ρ‹Ρ‚ΡŒ Π΄Π²Π΅ ΡˆΡ‚ΡƒΠΊΠΈ β€” Π½Π΅ большС ΠΈ Π½Π΅ мСньшС.
  2. УстановлСны ΠΎΠ½ΠΈ Π΄ΠΎΠ»ΠΆΠ½Ρ‹ Π±Ρ‹Ρ‚ΡŒ Π½Π° расстоянии Π½Π΅ Π±ΠΎΠ»Π΅Π΅ 40 см ΠΎΡ‚ ΠΊΡ€ΠΎΠΌΠΊΠΈ рассСиватСля Π±ΠΎΠΊΠΎΠ²ΠΎΠ³ΠΎ Π³Π°Π±Π°Ρ€ΠΈΡ‚Π° ΠΈ Π½Π΅ Π½ΠΈΠΆΠ΅ 25 см ΠΎΡ‚ уровня повСрхности Π΄ΠΎΡ€ΠΎΠΆΠ½ΠΎΠ³ΠΎ покрытия. Π£ Π±ΠΎΠ»ΡŒΡˆΠΈΠ½ΡΡ‚Π²Π° соврСмСнных Π°Π²Ρ‚ΠΎΠΌΠΎΠ±ΠΈΠ»Π΅ΠΉ Π½Π° Π±Π°ΠΌΠΏΠ΅Ρ€Π΅ прСдусмотрСны ΡˆΡ‚Π°Ρ‚Π½Ρ‹Π΅ мСста для установки ПВЀ.
  3. Если Π΄Π°Π½Π½ΠΎΠΉ ΠΊΠΎΠΌΠΏΠ»Π΅ΠΊΡ‚Π°Ρ†ΠΈΠ΅ΠΉ ΠΏΡ€ΠΎΡ‚ΠΈΠ²ΠΎΡ‚ΡƒΠΌΠ°Π½ΠΊΠΈ Π½Π΅ прСдусмотрСны, Π½Π° ΠΈΡ… мСсто ставятся Π·Π°Π³Π»ΡƒΡˆΠΊΠΈ, ΠΊΠΎΡ‚ΠΎΡ€Ρ‹Π΅ Π»Π΅Π³ΠΊΠΎ ΠΈΠ·Π²Π»Π΅ΠΊΠ°ΡŽΡ‚ΡΡ ΠΈ Π½Π° ΠΈΡ… мСсто ΠΌΠΎΠ½Ρ‚ΠΈΡ€ΡƒΡŽΡ‚ΡΡ ПВЀ.

Π’ Π±ΠΎΠ»ΡŒΡˆΠΈΠ½ΡΡ‚Π²Π΅ ΠΏΡ€ΠΎΡ‚ΠΈΠ²ΠΎΡ‚ΡƒΠΌΠ°Π½Π½Ρ‹Ρ… Ρ„Π°Ρ€, прСдставлСнных Π½Π° Ρ€Ρ‹Π½ΠΊΠ΅, примСняСтся стандартная однонитСвая Π»Π°ΠΌΠΏΠ° ΠΊΠ°Ρ‚Π΅Π³ΠΎΡ€ΠΈΠΈ Н1. Π’ΠΊΠ»ΡŽΡ‡Π°Ρ‚ΡŒΡΡ ПВЀ Π΄ΠΎΠ»ΠΆΠ½Ρ‹ Ρ‚ΠΎΠ»ΡŒΠΊΠΎ вмСстС с Π³Π°Π±Π°Ρ€ΠΈΡ‚Π½Ρ‹ΠΌΠΈ огнями.

ΠΠ΅ΠΏΡ€Π°Π²ΠΈΠ»ΡŒΠ½Π°Ρ установка ΠΏΡ€ΠΎΡ‚ΠΈΠ²ΠΎΡ‚ΡƒΠΌΠ°Π½Π½Ρ‹Π΅ Ρ„Π°Ρ€ ΠΌΠΎΠΆΠ΅Ρ‚ привСсти ΠΊ возникновСнию Π”Π’ΠŸ ΠΈΠ»ΠΈ Π±ΡƒΠ΄Π΅Ρ‚ Π²Ρ‹Π·Ρ‹Π²Π°Ρ‚ΡŒ дискомфорт Ρƒ Π²ΠΎΠ΄ΠΈΡ‚Π΅Π»Π΅ΠΉ встрСчных Π°Π²Ρ‚ΠΎΠΌΠΎΠ±ΠΈΠ»Π΅ΠΉ. Π‘ΠΎΠ±Π»ΡŽΠ΄Π°ΠΉΡ‚Π΅ всС ΠΏΡ€Π°Π²ΠΈΠ»Π°, ΠΈ Π²Ρ‹ Π½ΠΈΠΊΠΎΠ³Π΄Π° Π½Π΅ ΠΏΠΎΠΏΠ°Π΄Π΅Ρ‚Π΅ Π² Π½Π΅ΠΏΡ€ΠΈΡΡ‚Π½ΡƒΡŽ ΡΠΈΡ‚ΡƒΠ°Ρ†ΠΈΡŽ.

ПослС ΠΏΡƒΠ±Π»ΠΈΠΊΠ°Ρ†ΠΈΠΈ ΡΡ‚Π°Ρ‚ΡŒΠΈ ΠΎΠ± установкС ΠΈ ΠΏΠΎΠ΄ΠΊΠ»ΡŽΡ‡Π΅Π½ΠΈΠΈ ΠΏΡ€ΠΎΡ‚ΠΈΠ²ΠΎΡ‚ΡƒΠΌΠ°Π½Π½Ρ‹Ρ… Ρ„Π°Ρ€ Π½Π° ВАЗ 2109 ΠΌΠ½ΠΎΠ³ΠΈΠ΅ Ρ‡ΠΈΡ‚Π°Ρ‚Π΅Π»ΠΈ Π·Π°Π΄Π°Π²Π°Π»ΠΈ ΠΌΠ½Π΅ вопрос: Ρ‡Ρ‚ΠΎ Π΄Π΅Π»Π°Ρ‚ΡŒ Ссли ΡˆΡ‚Π°Ρ‚Π½ΠΎΠΉ ΠΏΡ€ΠΎΠ²ΠΎΠ΄ΠΊΠΈ для ПВЀ ΠΈ ΠΊΠ½ΠΎΠΏΠΊΠΈ Π²ΠΊΠ»ΡŽΡ‡Π΅Π½ΠΈΡ ПВЀ Π² салонС Π½Π΅Ρ‚, Ρ‚ΠΎ Π΅ΡΡ‚ΡŒ ΠΏΠΎΠ»Π½ΠΎΡΡ‚ΡŒΡŽ отсутствуСт ΠΏΠΎΠ΄Π³ΠΎΡ‚ΠΎΠ²ΠΊΠ° для установки ΠΏΡ€ΠΎΡ‚ΠΈΠ²ΠΎΡ‚ΡƒΠΌΠ°Π½ΠΎΠΊ, Ρ‡Ρ‚ΠΎ встрСчаСтся Π½Π° Π±ΠΎΠ»Π΅Π΅ Ρ€Π°Π½Π½ΠΈΡ… вСрсиях ВАЗ 2108 ΠΈ ВАЗ 2109?

Π’ ΠΎΡ‚Π²Π΅Ρ‚ Π½Π° это вопрос я ΠΏΡƒΠ±Π»ΠΈΠΊΡƒΡŽ эту ΡΡ‚Π°Ρ‚ΡŒΡŽ, Π² ΠΊΠΎΡ‚ΠΎΡ€ΠΎΠΉ ΠΌΡ‹ рассмотрим, ΠΊΠ°ΠΊ ΠΏΠΎΠ΄ΠΊΠ»ΡŽΡ‡Π°Ρ‚ΡŒ ΠΏΡ€ΠΎΡ‚ΠΈΠ²ΠΎΡ‚ΡƒΠΌΠ°Π½Π½Ρ‹Π΅ Ρ„Π°Ρ€Ρ‹, ΠΊΠ°ΠΊ говорится, «с нуля», ΠΏΡ€ΠΈ ΠΏΠΎΠ»Π½ΠΎΠΌ отсутствии ΠΏΡ€ΠΎΠ²ΠΎΠ΄ΠΊΠΈ.

Рассмотрим для ΠΎΠ±Ρ‰Π΅Π³ΠΎ понятия схСму ΠΏΠΎΠ΄ΠΊΠ»ΡŽΡ‡Π΅Π½ΠΈΡ любого Π΄ΠΎΠΏΠΎΠ»Π½ΠΈΡ‚Π΅Π»ΡŒΠ½ΠΎΠ³ΠΎ оборудования Π½Π° Π°Π²Ρ‚ΠΎΠΌΠΎΠ±ΠΈΠ»Π΅ с использованиСм Ρ€Π΅Π»Π΅:

К слову, ΠΏΠΎ Ρ‚Π°ΠΊΠΎΠΉ ΠΆΠ΅ схСмС ΠΏΠΎΠ΄ΠΊΠ»ΡŽΡ‡Π°Π΅Ρ‚ΡΡ ΠΈ Π·Π²ΡƒΠΊΠΎΠ²ΠΎΠΉ сигнал ΠΎΡ‚ Π’ΠΎΠ»Π³ΠΈ, ΠΈ ΠΎΠ±ΠΎΠ³Ρ€Π΅Π² Π½Π°Ρ€ΡƒΠΆΠ½Ρ‹Ρ… Π·Π΅Ρ€ΠΊΠ°Π».

Номинал прСдохранитСля Π½Π° этой схСмС выбираСтся исходя ΠΈΠ· мощности ΠΏΠΎΠ΄ΠΊΠ»ΡŽΡ‡Π°Π΅ΠΌΠΎΠΉ Π½Π°Π³Ρ€ΡƒΠ·ΠΊΠΈ. НапримСр, Π² ПВЀ ВАЗ 2109 стоят 2 Π»Π°ΠΌΠΏΡ‹ ΠΏΠΎ 55 Π’Ρ‚. Π’ΠΎΠ³Π΄Π° Ρ‚ΠΎΠΊ Π² Ρ†Π΅ΠΏΠΈ Π±ΡƒΠ΄Π΅Ρ‚ Ρ€Π°Π²Π΅Π½ 55 Π’Ρ‚ * 2 / 12Π’ = 9,1 А. ΠŸΡ€Π΅Π΄ΠΎΡ…Ρ€Π°Π½ΠΈΡ‚Π΅Π»ΡŒ возьмСм с нСбольшим запасом, Ρ‚.Π΅ 15А.

Но такая схСма Π² чистом Π²ΠΈΠ΄Π΅ для ΠΏΠΎΠ΄ΠΊΠ»ΡŽΡ‡Π΅Π½ΠΈΡ ΠΏΡ€ΠΎΡ‚ΠΈΠ²ΠΎΡ‚ΡƒΠΌΠ°Π½ΠΎΠΊ Π½Π΅ ΠΏΠΎΠ΄Ρ…ΠΎΠ΄ΠΈΡ‚. ΠžΠ±Ρ€Π°Ρ‰Π°Π΅ΠΌ Π²Π½ΠΈΠΌΠ°Π½ΠΈΠ΅ Π½Π° ΠŸΡ€Π°Π²ΠΈΠ»Π°, Π² ΠΊΠΎΡ‚ΠΎΡ€Ρ‹Ρ… говорится, Ρ‡Ρ‚ΠΎ ПВЀ Π΄ΠΎΠ»ΠΆΠ½Ρ‹ Π²ΠΊΠ»ΡŽΡ‡Π°Ρ‚ΡŒΡΡ Ρ‚ΠΎΠ»ΡŒΠΊΠΎ послС Π²ΠΊΠ»ΡŽΡ‡Π΅Π½ΠΈΡ Π³Π°Π±Π°Ρ€ΠΈΡ‚Π½Ρ‹Ρ… ΠΎΠ³Π½Π΅ΠΉ. БоотвСтствСнно, схСму Π½ΡƒΠΆΠ½ΠΎ Π½Π΅ΠΌΠ½ΠΎΠ³ΠΎ Π΄ΠΎΡ€Π°Π±ΠΎΡ‚Π°Ρ‚ΡŒ, ΠΈ ΠΎΠ½Π° ΠΏΡ€ΠΈΠΎΠ±Ρ€Π΅Ρ‚Π°Π΅Ρ‚ Π²ΠΎΡ‚ Ρ‚Π°ΠΊΠΎΠΉ Π²ΠΈΠ΄:

Π—Π΄Π΅ΡΡŒ Π±ΡƒΠ΄Π΅Ρ‚ Ρ€Π°Π±ΠΎΡ‚Π°Ρ‚ΡŒ Ρ‚Π°ΠΊΠΆΠ΅ ΠΈ индикация Π²ΠΊΠ»ΡŽΡ‡Π΅Π½ΠΈΡ ΠΏΡ€ΠΎΡ‚ΠΈΠ²ΠΎΡ‚ΡƒΠΌΠ°Π½Π½Ρ‹Ρ… Ρ„Π°Ρ€, Ρ‚.Π΅. Π±ΡƒΠ΄Π΅Ρ‚ Π·Π°Π³ΠΎΡ€Π°Ρ‚ΡŒΡΡ Π»Π°ΠΌΠΏΠΎΡ‡ΠΊΠ° ΠΏΡ€ΠΈ Π²ΠΊΠ»ΡŽΡ‡Π΅Π½ΠΈΠΈ ПВЀ.

ЕдинствСнноС, Ρ‡Π΅Π³ΠΎ Π½Π΅ ΠΎΡ‚Ρ€Π°ΠΆΠ΅Π½ΠΎ Π½Π° схСмС, Ρ‚Π°ΠΊ это подсвСтка самой ΠΊΠ½ΠΎΠΏΠΊΠΈ ΠΏΡ€ΠΈ Π²ΠΊΠ»ΡŽΡ‡Π΅Π½ΠΈΠΈ Π³Π°Π±Π°Ρ€ΠΈΡ‚ΠΎΠ². Но Π½Π° Π½Π΅ΠΊΠΎΡ‚ΠΎΡ€Ρ‹Ρ… ΠΊΠ½ΠΎΠΏΠΊΠ°Ρ…, Π² Ρ‚ΠΎΠΌ числС Π½Π° старых ВАЗ 2108 ΠΎΠ½Π° попросту отсутствуСт. А Ссли Π½Π΅ΠΎΠ±Ρ…ΠΎΠ΄ΠΈΠΌΠΎ, ΠΈ ΠΊΠ½ΠΎΠΏΠΊΠ° позволяСт ΡΠ΄Π΅Π»Π°Ρ‚ΡŒ подсвСтку, Ρ‚ΠΎ это ΠΎΡ‡Π΅Π½ΡŒ просто ΡΠ΄Π΅Π»Π°Ρ‚ΡŒ, ΠΏΠΎΠ΄Π°Π² Π½Π° ΠΎΠ΄ΠΈΠ½ ΠΊΠΎΠ½Ρ‚Π°ΠΊΡ‚ Π»Π°ΠΌΠΏΡ‹ подсвСтки плюс ΠΎΡ‚ любого ΠΏΡ€ΠΎΠ²ΠΎΠ΄Π°, Π½Π° ΠΊΠΎΡ‚ΠΎΡ€ΠΎΠΌ появляСтся напряТСниС ΠΏΡ€ΠΈ Π²ΠΊΠ»ΡŽΡ‡Π΅Π½ΠΈΠΈ Π³Π°Π±Π°Ρ€ΠΈΡ‚ΠΎΠ² (подсвСтка ΠΏΠ°Π½Π΅Π»ΠΈ, сосСднСй ΠΊΠ½ΠΎΠΏΠΊΠΈ), Π° Π΄Ρ€ΡƒΠ³ΠΎΠΉ ΠΏΠΎΠ΄ΠΊΠ»ΡŽΡ‡ΠΈΠ² Π½Π° массу.

Ну Π° напослСдок ΠΏΡ€ΠΈΠ²Π΅Π΄Ρƒ ΠΏΠ°Ρ€Ρƒ Ρ‚ΠΈΠΏΠΎΠ²Ρ‹Ρ… схСм ΠΏΠΎΠ΄ΠΊΠ»ΡŽΡ‡Π΅Π½ΠΈΡ ПВЀ Π½Π° ВАЗ 2108 ΠΈ 2109, ΠΌΠΎΠΆΠ΅Ρ‚ ΠΊΠΎΠΌΡƒ пригодятся!.

ΠŸΡ€ΠΎΡ‚ΠΈΠ²ΠΎΡ‚ΡƒΠΌΠ°Π½Π½Ρ‹Π΅ Ρ„Π°Ρ€Ρ‹ Π½Π΅ относятся ΠΊ ΠΎΠ±ΡΠ·Π°Ρ‚Π΅Π»ΡŒΠ½ΠΎΠΌΡƒ ΠΎΠ±ΠΎΡ€ΡƒΠ΄ΠΎΠ²Π°Π½ΠΈΡŽ автомобиля. Однако ΠΈΡ… Π½Π°Π»ΠΈΡ‡ΠΈΠ΅ сдСлаСт ΠΏΠΎΠ΅Π·Π΄ΠΊΠΈ Π±ΠΎΠ»Π΅Π΅ ΠΊΠΎΠΌΡ„ΠΎΡ€Ρ‚Π½Ρ‹ΠΌΠΈ ΠΈ бСзопасными. Для ΠΌΠΎΠ½Ρ‚Π°ΠΆΠ° Π½Π΅ Ρ‚Ρ€Π΅Π±ΡƒΡŽΡ‚ΡΡ особыС Π½Π°Π²Ρ‹ΠΊΠΈ. Установка ΠΈ подсоСдинСниС ПВЀ ΠΊ ΠΏΡ€ΠΎΠ²ΠΎΠ΄ΠΊΠ΅ Π²ΠΏΠΎΠ»Π½Π΅ осущСствимо Π² Π΄ΠΎΠΌΠ°ΡˆΠ½ΠΈΡ… условиях. Для этого понадобится ΠΌΠΎΠ½Ρ‚Π°ΠΆΠ½Ρ‹ΠΉ Π½Π°Π±ΠΎΡ€ ΠΏΠΎΠ΄ΠΊΠ»ΡŽΡ‡Π΅Π½ΠΈΡ ΠΏΡ€ΠΎΡ‚ΠΈΠ²ΠΎΡ‚ΡƒΠΌΠ°Π½Π½Ρ‹Ρ… Ρ„Π°Ρ€, ΠΏΡ€ΠΎΡΡ‚Π΅ΠΉΡˆΠΈΠ΅ слСсарныС инструмСнты ΠΈ изоляция.

Для Ρ‡Π΅Π³ΠΎ Π½ΡƒΠΆΠ½Ρ‹ ΠΏΡ€ΠΎΡ‚ΠΈΠ²ΠΎΡ‚ΡƒΠΌΠ°Π½ΠΊΠΈ

Π‘Ρ‚Π°Π½Π΄Π°Ρ€Ρ‚Π½ΠΎΠ΅ Π³ΠΎΠ»ΠΎΠ²Π½ΠΎΠ΅ освСщСниС автомобиля обСспСчиваСт Π΄ΠΎΡΡ‚Π°Ρ‚ΠΎΡ‡Π½ΡƒΡŽ Π²ΠΈΠ΄ΠΈΠΌΠΎΡΡ‚ΡŒ ΠΏΡ€ΠΈ ясной ΠΏΠΎΠ³ΠΎΠ΄Π΅. Однако Π² Π½Π΅Π½Π°ΡΡ‚ΡŒΠ΅ ΠΎΡ‚ Π½Π΅Π³ΠΎ Π½Π΅ΠΌΠ½ΠΎΠ³ΠΎ Ρ‚ΠΎΠ»ΠΊΡƒ. ΠŸΡ€ΠΎΠ±Π»Π΅ΠΌΠ° Π² Ρ‚ΠΎΠΌ, Ρ‡Ρ‚ΠΎ Π»ΡƒΡ‡ΠΈ Π»Π°ΠΌΠΏ Π±Π»ΠΈΠΆΠ½Π΅Π³ΠΎ ΠΈ дальнСго свСта отраТаСтся ΠΎΡ‚ капСль Ρ‚ΡƒΠΌΠ°Π½Π° ΠΈΠ»ΠΈ снСТинок, создавая Β«Π±Π΅Π»ΡƒΡŽ ΠΏΠ΅Π»Π΅Π½ΡƒΒ» Π²ΠΏΠ΅Ρ€Π΅Π΄ΠΈ ΠΌΠ°ΡˆΠΈΠ½Ρ‹.

Π­Ρ‚ΠΎ слСдствиС Π΄Π²ΡƒΡ… Ρ„Π°ΠΊΡ‚ΠΎΡ€ΠΎΠ²:

  • ΡƒΠ·ΠΊΠΈΠ΅ оптичСский ΠΏΡƒΡ‡ΠΎΠΊ Π»ΡƒΡ‡ΡˆΠ΅ отраТаСтся ΠΎΡ‚ повСрхности;
  • высоко установлСнныС Ρ„Π°Ρ€Ρ‹ ΠΏΠΎΠ΄ΡΠ²Π΅Ρ‡ΠΈΠ²Π°ΡŽΡ‚ участок с большой ΠΊΠΎΠ½Ρ†Π΅Π½Ρ‚Ρ€Π°Ρ†ΠΈΠ΅ΠΉ капСль Ρ‚ΡƒΠΌΠ°Π½Π°.

ПВЀ ΡΠΏΠ΅Ρ†ΠΈΠ°Π»ΡŒΠ½ΠΎ спроСктированы для эксплуатации Π² тяТСлых ΠΏΠΎΠ³ΠΎΠ΄Π½Ρ‹Ρ… условиях. И Π΄Π΅Π»ΠΎ Ρ‚ΡƒΡ‚ Π½Π΅ Π² ΠΆΠ΅Π»Ρ‚Ρ‹Ρ… ΡΠ²Π΅Ρ‚ΠΎΡ„ΠΈΠ»ΡŒΡ‚Ρ€Π°Ρ…, ΠΊΠΎΡ‚ΠΎΡ€Ρ‹Π΅ Π½Π΅ΠΊΠΎΡ‚ΠΎΡ€Ρ‹Π΅ Π²ΠΎΠ΄ΠΈΡ‚Π΅Π»ΠΈ ΡΡ‡ΠΈΡ‚Π°ΡŽΡ‚ ΠΏΠ°Π½Π°Ρ†Π΅Π΅ΠΉ Π² Ρ‚ΡƒΠΌΠ°Π½Π½ΡƒΡŽ ΠΏΠΎΠ³ΠΎΠ΄Ρƒ. НС зря ΠŸΠ”Π” Π΄ΠΎΠΏΡƒΡΠΊΠ°ΡŽΡ‚ установку ПВЀ Π΄Π²ΡƒΡ… Ρ†Π²Π΅Ρ‚ΠΎΠ² β€” Π±Π΅Π»ΠΎΠ³ΠΎ ΠΈ ΠΆΠ΅Π»Ρ‚ΠΎΠ³ΠΎ.

Π’Π°ΠΊΠΈΠ΅ Ρ„Π°Ρ€Ρ‹ ΡΠΏΡ€Π°Π²Π»ΡΡŽΡ‚ΡΡ с Π½Π΅ΠΏΠΎΠ³ΠΎΠ΄ΠΎΠΉ благодаря ΠΏΠ°Ρ€Π΅ Π²Π°ΠΆΠ½Ρ‹Ρ… свойств:

  • Π¨ΠΈΡ€ΠΈΠ½Π° свСтового ΠΏΡƒΡ‡ΠΊΠ°. Π¨ΠΈΡ€ΠΎΠΊΠΈΠΉ Π»ΡƒΡ‡ мСньшС отраТаСтся ΠΎΡ‚ Π²ΠΎΠ΄Ρ‹ ΠΈ льдинок ΠΈ Π½Π΅ создаСт Β«Π±Π΅Π»ΡƒΡŽ ΠΏΠ΅Π»Π΅Π½ΡƒΒ».
  • Высота крСплСния. ΠšΠΎΠ½Ρ†Π΅Π½Ρ‚Ρ€Π°Ρ†ΠΈΡ Ρ‚ΡƒΠΌΠ°Π½Π° Π²Π±Π»ΠΈΠ·ΠΈ Π·Π΅ΠΌΠ»ΠΈ Π½ΠΈΠΆΠ΅. ΠŸΠΎΡΡ‚ΠΎΠΌΡƒ Π½ΠΈΠ·ΠΊΠΎΠ΅ располоТСниС позволяСт ΠΏΠΎΠ΄ΡΠ²Π΅Ρ‡ΠΈΠ²Π°Ρ‚ΡŒ ΠΏΠΎΠ΄ ΠΏΠ΅Π»Π΅Π½Ρƒ.

ПослСдний ΠΌΠΎΠΌΠ΅Π½Ρ‚ особСнно Π²Π°ΠΆΠ΅Π½. Если ΡƒΡΡ‚Π°Π½ΠΎΠ²ΠΈΡ‚ΡŒ ПВЀ Π½Π° ΡƒΡ€ΠΎΠ²Π½Π΅ Ρ„Π°Ρ€ Π³ΠΎΠ»ΠΎΠ²Π½ΠΎΠ³ΠΎ освСщСния, ΠΎΠ½ΠΈ Π½Π΅ Π±ΡƒΠ΄ΡƒΡ‚ ΡΠΏΡ€Π°Π²Π»ΡΡ‚ΡŒΡΡ со своСй Π·Π°Π΄Π°Ρ‡Π΅ΠΉ.

ΠšΡ€ΠΎΠΌΠ΅ ΠΏΠ΅Ρ€Π΅Π΄Π½ΠΈΡ… ПВЀ, ΡƒΠ»ΡƒΡ‡ΡˆΠ°ΡŽΡ‰ΠΈΡ… Π²ΠΈΠ΄ΠΈΠΌΠΎΡΡ‚ΡŒ с Π²ΠΎΠ΄ΠΈΡ‚Π΅Π»ΡŒΡΠΊΠΎΠ³ΠΎ мСста, ΠΈΡΠΏΠΎΠ»ΡŒΠ·ΡƒΡŽΡ‚ΡΡ Π·Π°Π΄Π½ΠΈΠ΅ ΠΏΡ€ΠΎΡ‚ΠΈΠ²ΠΎΡ‚ΡƒΠΌΠ°Π½Π½Ρ‹Π΅ Ρ„Π°Ρ€Ρ‹, Π΄ΡƒΠ±Π»ΠΈΡ€ΡƒΡŽΡ‰ΠΈΠ΅ Π³Π°Π±Π°Ρ€ΠΈΡ‚Ρ‹. Они Ρ‡Π΅Ρ‚ΠΊΠΎ ΠΎΠ±ΠΎΠ·Π½Π°Ρ‡Π°ΡŽΡ‚ Π°Π²Ρ‚ΠΎ Π² условиях ΠΏΠ»ΠΎΡ…ΠΎΠΉ видимости, Π½ΠΎ ΠΎΠ±Π»Π°Π΄Π°ΡŽΡ‚ ΠΈΠ·Π±Ρ‹Ρ‚ΠΎΡ‡Π½ΠΎΠΉ ΡΡ€ΠΊΠΎΡΡ‚ΡŒΡŽ. ΠŸΠΎΡΡ‚ΠΎΠΌΡƒ ΠΈΡ… Π½Π΅ слСдуСт Π²ΠΊΠ»ΡŽΡ‡Π°Ρ‚ΡŒ Π² ΡΡΠ½ΡƒΡŽ ΠΏΠΎΠ³ΠΎΠ΄Ρƒ.

ВрСбования ΠΊ установкС ΠΏΡ€ΠΎΡ‚ΠΈΠ²ΠΎΡ‚ΡƒΠΌΠ°Π½Π½Ρ‹Ρ… Ρ„Π°Ρ€

ПВЀ состоят ΠΈΠ· Ρ‚Ρ€Π΅Ρ… корпуса с Ρ†ΠΎΠΊΠΎΠ»Π΅ΠΌ для крСплСния Π»Π°ΠΌΠΏΡ‹, отраТатСля ΠΈ стСклянного рассСиватСля, ΠΊΠΎΡ‚ΠΎΡ€Ρ‹Π΅ ΠΎΠ±Π΅ΡΠΏΠ΅Ρ‡ΠΈΠ²Π°ΡŽΡ‚ Π½ΡƒΠΆΠ½ΡƒΡŽ Ρ„ΠΎΡ€ΠΌΡƒ свСтового ΠΏΡƒΡ‡ΠΊΠ°. ΠžΡΠΎΠ±Π΅Π½Π½ΠΎΡΡ‚ΠΈ эксплуатации ΠΈ Π½Π°Π·Π½Π°Ρ‡Π΅Π½ΠΈΠ΅ ΠΏΡ€ΠΎΡ‚ΠΈΠ²ΠΎΡ‚ΡƒΠΌΠ°Π½ΠΎΠΊ Π½Π°Π»Π°Π³Π°Π΅Ρ‚ Π½Π° Π½ΠΈΡ… особыС трСбования, ΠΊΠΎΡ‚ΠΎΡ€Ρ‹Π΅ нашли ΠΎΡ‚Ρ€Π°ΠΆΠ΅Π½ΠΈΠ΅ Π² Π΄Π΅ΠΉΡΡ‚Π²ΡƒΡŽΡ‰ΠΈΡ… ΠΏΡ€Π°Π²ΠΈΠ»Π°Ρ… Π΄ΠΎΡ€ΠΎΠΆΠ½ΠΎΠ³ΠΎ двиТСния:

  • ΡƒΡΡ‚Π°Π½Π°Π²Π»ΠΈΠ²Π°Ρ‚ΡŒ ΠΌΠΎΠΆΠ½ΠΎ Π½Π΅ Π±ΠΎΠ»Π΅Π΅ 2 Ρ„Π°Ρ€;
  • ПВЀ Π΄ΠΎΠ»ΠΆΠ½Ρ‹ ΠΊΡ€Π΅ΠΏΠΈΡ‚ΡŒΡΡ с отступом 40 см ΠΎΡ‚ Π±ΠΎΠΊΠΎΠ²Ρ‹Ρ… ΠΊΡ€Π°Π΅Π² Π°Π²Ρ‚ΠΎ ΠΈ Π²Ρ‹ΡˆΠ΅ 25 см ΠΎΡ‚ повСрхности Π΄ΠΎΡ€ΠΎΠ³ΠΈ;
  • Π»ΡƒΡ‡ Π΄ΠΎΠ»ΠΆΠ΅Π½ Π±Ρ‹Ρ‚ΡŒ Π½Π°ΠΏΡ€Π°Π²Π»Π΅Π½ Π½ΠΈΠΆΠ΅, Ρ‡Π΅ΠΌ Ρ„Π°Ρ€Ρ‹ Π±Π»ΠΈΠΆΠ½Π΅Π³ΠΎ свСта;
  • ΡƒΠ³Π»Ρ‹ видимости Π½Π΅ ΠΌΠΎΠ³ΡƒΡ‚ Π²Ρ‹Ρ…ΠΎΠ΄ΠΈΡ‚ΡŒ Π·Π° ΠΏΡ€Π΅Π΄Π΅Π»Ρ‹ ΡΠ»Π΅Π΄ΡƒΡŽΡ‰ΠΈΡ… Π·Π½Π°Ρ‡Π΅Π½ΠΈΠΉ: -10…+15 градусов ΠΈ -10. .+45 ΠΏΠΎ Π²Π΅Ρ€Ρ‚ΠΈΠΊΠ°Π»ΠΈ ΠΈ Π³ΠΎΡ€ΠΈΠ·ΠΎΠ½Ρ‚Π°Π»ΠΈ соотвСтствСнно.

Π’ Π±Π°ΠΌΠΏΠ΅Ρ€Π΅ соврСмСнных Π»Π΅Π³ΠΊΠΎΠ²ΡƒΡˆΠ΅ΠΊ прСдусмотрСны ΡΠΏΠ΅Ρ†ΠΈΠ°Π»ΡŒΠ½Ρ‹Π΅ мСста для ΠΌΠΎΠ½Ρ‚Π°ΠΆΠ° ΠΏΡ€ΠΎΡ‚ΠΈΠ²ΠΎΡ‚ΡƒΠΌΠ°Π½Π½Ρ‹Ρ… Ρ„Π°Ρ€. ΠŸΡ€ΠΈ ΠΈΡ… отсутствии ΠΏΡ€ΠΎΡ‚ΠΈΠ²ΠΎΡ‚ΡƒΠΌΠ°Π½ΠΊΠΈ слСдуСт ΠΊΡ€Π΅ΠΏΠΈΡ‚ΡŒ согласно Ρ€Π΅ΠΊΠΎΠΌΠ΅Π½Π΄Π°Ρ†ΠΈΠΉ производитСля. Π“Π»Π°Π²Π½Ρ‹Π΅ трСбования:

  • ТСсткоС ΠΊΡ€Π΅ΠΏΠ»Π΅Π½ΠΈΠ΅ ПВЀ Π½Π° ΠΊΡ€ΠΎΠ½ΡˆΡ‚Π΅ΠΉΠ½Π°Ρ…;
  • Π·Π°Ρ‰ΠΈΡ‚Π° элСктропитания ΠΏΡ€Π΅Π΄ΠΎΡ…Ρ€Π°Π½ΠΈΡ‚Π΅Π»Π΅ΠΌ;
  • ΠΏΡ€Π°Π²ΠΈΠ»ΡŒΠ½Ρ‹Π΅ ΡƒΠ³Π»Ρ‹ направлСния свСтовых ΠΏΡƒΡ‡ΠΊΠΎΠ².

ΠŸΠΎΠ΄ΠΊΠ»ΡŽΡ‡Π΅Π½ΠΈΠ΅ ПВЀ Ρ‡Π΅Ρ€Π΅Π· Ρ€Π΅Π»Π΅ ΠΈ ΠΊΠ½ΠΎΠΏΠΊΡƒ

БущСствуСт нСсколько Π²Π°Ρ€ΠΈΠ°Π½Ρ‚ΠΎΠ² сопряТСния ПВЀ с Π±ΠΎΡ€Ρ‚ΠΎΠ²ΠΎΠΉ ΡΠ΅Ρ‚ΡŒΡŽ автомобиля. ΠŸΠΎΠ΄ΠΊΠ»ΡŽΡ‡Π΅Π½ΠΈΠ΅ ΠΏΡ€ΠΎΡ‚ΠΈΠ²ΠΎΡ‚ΡƒΠΌΠ°Π½Π½Ρ‹Ρ… Ρ„Π°Ρ€ Ρ‡Π΅Ρ€Π΅Π· Ρ€Π΅Π»Π΅ с использованиСм ΠΎΡ‚Π΄Π΅Π»ΡŒΠ½ΠΎΠΉ ΠΊΠ½ΠΎΠΏΠΊΠΈ β€” самоС ΡƒΠ΄ΠΎΠ±Π½ΠΎΠ΅, бСзопасноС ΠΈ ΠΏΡ€Π°Π²ΠΈΠ»ΡŒΠ½ΠΎΠ΅ Ρ€Π΅ΡˆΠ΅Π½ΠΈΠ΅. Для выполнСния Ρ€Π°Π±ΠΎΡ‚Ρ‹ Π²Π°ΠΌ понадобятся:

  • ΠΏΡ€ΠΎΡ‚ΠΈΠ²ΠΎΡ‚ΡƒΠΌΠ°Π½ΠΊΠΈ;
  • ΡΠΎΠ΅Π΄ΠΈΠ½ΠΈΡ‚Π΅Π»ΡŒΠ½Ρ‹Π΅ ΠΊΠ»Π΅ΠΌΠΌΡ‹;
  • ΠΏΡ€Π΅Π΄ΠΎΡ…Ρ€Π°Π½ΠΈΡ‚Π΅Π»ΡŒ Π½Π° 15 Π°ΠΌΠΏΠ΅Ρ€;
  • Ρ‡Π΅Ρ‚Ρ‹Ρ€Π΅Ρ…ΠΊΠΎΠ½Ρ‚Π°ΠΊΡ‚Π½ΠΎΠ΅ Ρ€Π΅Π»Π΅;
  • ΠΊΠΎΠ»ΠΎΠ΄ΠΊΠ° ΠΏΠΎΠ΄ΠΊΠ»ΡŽΡ‡Π΅Π½ΠΈΡ;
  • ΠΏΡ€ΠΎΠ²ΠΎΠ΄Π°;
  • изоляция ΠΈΠ»ΠΈ тСрмоусадочная Ρ‚Ρ€ΡƒΠ±ΠΊΠ°.

ΠœΠ°Ρ‚Π΅Ρ€ΠΈΠ°Π»Ρ‹ входят Π² ΠΌΠΎΠ½Ρ‚Π°ΠΆΠ½Ρ‹ΠΉ ΠΊΠΎΠΌΠΏΠ»Π΅ΠΊΡ‚ ΠΈ ΠΏΡ€ΠΎΠ΄Π°ΡŽΡ‚ΡΡ Π² ΠΎΠ΄Π½ΠΎΠΌ Π½Π°Π±ΠΎΡ€Π΅ с ПВЀ.

Из инструмСнта Π½ΡƒΠΆΠ½Ρ‹ ΠΎΡ‚Π²Π΅Ρ€Ρ‚ΠΊΠΈ, пассатиТи ΠΈ Π½ΠΎΠΆ для зачистки изоляции ΠΏΡ€ΠΎΠ²ΠΎΠ΄ΠΎΠ².

Π Π°Π·Π²ΠΎΠ΄ΠΊΠ° Π΄ΠΎΠ»ΠΆΠ½Π° Π±Ρ‹Ρ‚ΡŒ Π²Ρ‹ΠΏΠΎΠ»Π½Π΅Π½Π° согласно ΠΏΠ»Π°Π½Π° ΠΏΠ°Ρ€Π°Π»Π»Π΅Π»ΡŒΠ½ΠΎΠ³ΠΎ ΠΈΠ»ΠΈ ΠΏΠΎΡΠ»Π΅Π΄ΠΎΠ²Π°Ρ‚Π΅Π»ΡŒΠ½ΠΎΠ³ΠΎ соСдинСния Π»Π°ΠΌΠΏ.

Π‘Ρ…Π΅ΠΌΠ° ΠΏΠΎΠ΄ΠΊΠ»ΡŽΡ‡Π΅Π½ΠΈΡ ΠΏΡ€ΠΎΡ‚ΠΈΠ²ΠΎΡ‚ΡƒΠΌΠ°Π½Π½Ρ‹Ρ… Ρ„Π°Ρ€ ΠΏΡ€ΠΈΠ²Π΅Π΄Π΅Π½Π° Π½Π° рисункС

ΠžΠΏΠ΅Ρ€Π°Ρ†ΠΈΠΈ ΠΏΠΎ ΠΏΠΎΠ΄ΠΊΠ»ΡŽΡ‡Π΅Π½ΠΈΡŽ ПВЀ Π²Ρ‹ΠΏΠΎΠ»Π½ΡΡŽΡ‚ΡΡ Π² Ρ‚Π°ΠΊΠΎΠΉ ΠΏΠΎΡΠ»Π΅Π΄ΠΎΠ²Π°Ρ‚Π΅Π»ΡŒΠ½ΠΎΡΡ‚ΠΈ:

  • ΠžΡ‚ΡΠΎΠ΅Π΄ΠΈΠ½ΠΈΡ‚ΡŒ ΠΌΠΈΠ½ΡƒΡΠΎΠ²ΡƒΡŽ ΠΊΠ»Π΅ΠΌΠΌΡƒ ΠΎΡ‚ ΠΠšΠ‘;
  • НадСТно Π·Π°Ρ„ΠΈΠΊΡΠΈΡ€ΠΎΠ²Π°Ρ‚ΡŒ Ρ€Π΅Π»Π΅ Π² салонС ΠΌΠ°ΡˆΠΈΠ½Ρ‹. Для этого ΠΏΠΎΠ΄ΠΎΠΉΠ΄Π΅Ρ‚ пространство ΠΏΠΎΠ΄ панСлью ΠΏΡ€ΠΈΠ±ΠΎΡ€ΠΎΠ².
  • Π—Π°ΠΊΡ€Π΅ΠΏΠΈΡ‚ΡŒ ΠΊΠ½ΠΎΠΏΠΊΡƒ для Π²ΠΊΠ»ΡŽΡ‡Π΅Π½ΠΈΡ ΠΏΡ€ΠΎΡ‚ΠΈΠ²ΠΎΡ‚ΡƒΠΌΠ°Π½Π½Ρ‹Ρ… Ρ„Π°Ρ€ Π½Π° Β«Ρ‚ΠΎΡ€ΠΏΠ΅Π΄Π΅Β» (ΠΎΠ±Ρ‹Ρ‡Π½ΠΎ для Π½Π΅Π΅ прСдусмотрСно ΠΎΡ‚Π΄Π΅Π»ΡŒΠ½ΠΎΠ΅ мСсто, ΠΏΡ€ΠΈΠΊΡ€Ρ‹Ρ‚ΠΎΠ΅ Π·Π°Π³Π»ΡƒΡˆΠΊΠΎΠΉ).
  • ΠŸΡ€ΠΎΠ»ΠΎΠΆΠΈΡ‚ΡŒ ΠΏΡ€ΠΎΠ²ΠΎΠ΄ ΠΎΡ‚ аккумулятора ΠΊ Ρ€Π΅Π»Π΅, Π·Π°ΠΊΡ€Π΅ΠΏΠΈΡ‚ΡŒ Π½Π° Π½Π΅ΠΌ ΠΊΠ»Π΅ΠΌΠΌΡƒ ΠΈ ΠΏΡ€ΠΈΡΠΎΠ΅Π΄ΠΈΠ½ΠΈΡ‚ΡŒ ΠΊ ΠΊΠΎΠ½Ρ‚Π°ΠΊΡ‚Ρƒ Β«30Β».

Π’Π°ΠΆΠ½ΠΎ: Π² ΡΠ»Π΅ΠΊΡ‚Ρ€ΠΎΡ†Π΅ΠΏΡŒ ΠΌΠ΅ΠΆΠ΄Ρƒ ΠΠšΠ‘ ΠΈ Ρ€Π΅Π»Π΅ Π½Π΅ΠΎΠ±Ρ…ΠΎΠ΄ΠΈΠΌΠΎ Π²ΡΡ‚Π°Π²ΠΈΡ‚ΡŒ ΠΏΡ€Π΅Π΄ΠΎΡ…Ρ€Π°Π½ΠΈΡ‚Π΅Π»ΡŒ, ΠΊΠΎΡ‚ΠΎΡ€Ρ‹ΠΉ Π»ΡƒΡ‡ΡˆΠ΅ Ρ€Π°ΡΠΏΠΎΠ»ΠΎΠΆΠΈΡ‚ΡŒ Π² ΠΏΠΎΠ΄ΠΊΠ°ΠΏΠΎΡ‚Π½ΠΎΠΌ пространствС ΠΏΠΎΠ±Π»ΠΈΠΆΠ΅ ΠΊ аккумуляторной Π±Π°Ρ‚Π°Ρ€Π΅Π΅. Π­Ρ‚ΠΎ ΠΏΡ€Π΅Π΄ΠΎΡ‚Π²Ρ€Π°Ρ‚ΠΈΡ‚ Π²ΠΎΠ·Π³ΠΎΡ€Π°Π½ΠΈΠ΅ ΠΏΡ€ΠΎΠ²ΠΎΠ΄ΠΊΠΈ ΠΏΡ€ΠΈ Π·Π°ΠΌΡ‹ΠΊΠ°Π½ΠΈΠΈ Π½Π° корпус.

  • Π‘ΠΎΠ΅Π΄ΠΈΠ½ΠΈΡ‚ΡŒ ΠΊΠ½ΠΎΠΏΠΊΡƒ ΠΈ 85-ΠΉ Π²Ρ‹Π²ΠΎΠ΄ Ρ€Π΅Π»Π΅.
  • ΠŸΠΎΠ΄ΠΊΠ»ΡŽΡ‡ΠΈΡ‚ΡŒ плюсовой кабСль ΠΊΠΎ Π²Ρ…ΠΎΠ΄Ρƒ ΠΊΠ½ΠΎΠΏΠΊΠΈ, протянув Π΅Π³ΠΎ ΠΎΡ‚ Π·Π°ΠΌΠΊΠ° ΠΊΠΎΠ½Ρ‚Π°ΠΊΡ‚ΠΎΠ² Π½Π° Π·Π°ΠΌΠΊΠ΅ заТигания. ΠŸΠΎΠ΄ΠΊΠ»ΡŽΡ‡Π°Ρ‚ΡŒ ΠΏΠΈΡ‚Π°Π½ΠΈΠ΅ слСдуСт Ρ‚ΡƒΠ΄Π°, Π³Π΄Π΅ напряТСниС подаСтся лишь послС ΠΏΠΎΠ²ΠΎΡ€ΠΎΡ‚Π° ΠΊΠ»ΡŽΡ‡Π°. Π­Ρ‚ΠΎ Π½Π΅ даст Π²Π°ΠΌ ΠΎΡΡ‚Π°Π²ΠΈΡ‚ΡŒ ПВЀ горящими ΠΏΡ€ΠΈ постановкС ΠΌΠ°ΡˆΠΈΠ½Ρ‹ Π½Π° стоянку.
  • Π‘ΠΎΠ΅Π΄ΠΈΠ½ΠΈΡ‚ΡŒ ΠΊΠΎΠ½Ρ‚Π°ΠΊΡ‚ Β«86Β» с массой, ΠΈΡΠΏΠΎΠ»ΡŒΠ·ΡƒΡ для этого блиТайший Π±ΠΎΠ»Ρ‚ ΠΈΠ»ΠΈ саморСз.
  • Π—Π°ΠΊΡ€Π΅ΠΏΠΈΡ‚ΡŒ ПВЀ Π½Π° ΠΏΠ΅Ρ€Π΅Π΄Π½Π΅ΠΌ Π±Π°ΠΌΠΏΠ΅Ρ€Π΅, ΡƒΠ΄Π°Π»ΠΈΠ² пластиковыС Π·Π°Π³Π»ΡƒΡˆΠΊΠΈ. Иногда ΡΠΏΠ΅Ρ†ΠΈΠ°Π»ΡŒΠ½Ρ‹Ρ… мСст для крСплСния ПВЀ Π½Π΅ прСдусмотрСно. Π’ΠΎΠ³Π΄Π° придСтся ΠΏΡ€ΠΎΡ€Π΅Π·Π°Ρ‚ΡŒ ΠΈΡ… Π»ΠΎΠ±Π·ΠΈΠΊΠΎΠΌ ΠΈΠ»ΠΈ Π·Π°Ρ„ΠΈΠΊΡΠΈΡ€ΠΎΠ²Π°Ρ‚ΡŒ Ρ„Π°Ρ€Ρ‹ Π½Π° ΡΡ‚Π°Π»ΡŒΠ½Ρ‹Ρ… ΠΊΡ€ΠΎΠ½ΡˆΡ‚Π΅ΠΉΠ½Π°Ρ… Π½Π°Π΄ Π±Π°ΠΌΠΏΠ΅Ρ€ΠΎΠΌ.
  • ΠŸΡ€ΠΎΠ»ΠΎΠΆΠΈΡ‚ΡŒ кабСля ΠΎΡ‚ 87-Π³ΠΎ ΠΊΠΎΠ½Ρ‚Π°ΠΊΡ‚Π° Ρ€Π΅Π»Π΅ ΠΊ ΠΏΡ€ΠΎΡ‚ΠΈΠ²ΠΎΡ‚ΡƒΠΌΠ°Π½ΠΊΠ°ΠΌ, ΠΏΠΎΠ΄ΠΊΠ»ΡŽΡ‡ΠΈΠ² ΠΊ цоколям.
  • Π‘ΠΎΠ΅Π΄ΠΈΠ½ΠΈΡ‚ΡŒ Π²Ρ‹Ρ…ΠΎΠ΄ массы ПВЀ ΠΊ ΠΊΡƒΠ·ΠΎΠ²Ρƒ.
  • ΠΠ°Π΄Π΅Ρ‚ΡŒ ΠΈ Π·Π°ΠΊΡ€Π΅ΠΏΠΈΡ‚ΡŒ ΠΌΠΈΠ½ΡƒΡΠΎΠ²ΡƒΡŽ ΠΊΠ»Π΅ΠΌΠΌΡƒ Π½Π° ΠΠšΠ‘.
  • ΠŸΠΎΠ²Π΅Ρ€Π½ΡƒΡ‚ΡŒ ΠΊΠ»ΡŽΡ‡ заТигания ΠΈ ΡƒΠ±Π΅Π΄ΠΈΡ‚ΡŒΡΡ Π² работоспособности ΠΏΡ€ΠΎΡ‚ΠΈΠ²ΠΎΡ‚ΡƒΠΌΠ°Π½Π½Ρ‹Ρ… Ρ„Π°Ρ€.

ΠŸΡ€ΠΈ ΠΌΠΎΠ½Ρ‚Π°ΠΆΠ΅ слСдуСт ΠΈΠ·ΠΎΠ»ΠΈΡ€ΠΎΠ²Π°Ρ‚ΡŒ мСста соСдинСния ΠΏΡ€ΠΎΠ²ΠΎΠ΄ΠΎΠ² ΠΈ ΠΊΠ»Π΅ΠΌΠΌΡ‹ ΠΏΡ€ΠΈ ΠΏΠΎΠΌΠΎΡ‰ΠΈ тСрмоусадочных Ρ‚Ρ€ΡƒΠ±ΠΎΠΊ ΠΈΠ»ΠΈ ΠΈΠ·ΠΎΠ»Π΅Π½Ρ‚Ρ‹.

Π‘Ρ…Π΅ΠΌΠ° ΠΏΠ°Ρ€Π°Π»Π»Π΅Π»ΡŒΠ½ΠΎΠ³ΠΎ Π²ΠΊΠ»ΡŽΡ‡Π΅Π½ΠΈΡ ΠΏΡ€ΠΎΡ‚ΠΈΠ²ΠΎΡ‚ΡƒΠΌΠ°Π½ΠΎΠΊ

ЭлСктросхСмы ΠΌΠΎΠ½Ρ‚Π°ΠΆΠ° ΠΏΡ€ΠΎΡ‚ΠΈΠ²ΠΎΡ‚ΡƒΠΌΠ°Π½Π½Ρ‹Ρ… Ρ„Π°Ρ€ ΠΎΡ‚Π»ΠΈΡ‡Π°ΡŽΡ‚ΡΡ лишь количСством ΠΏΠ»ΡŽΡΠΎΠ²Ρ‹Ρ… ΠΏΡ€ΠΎΠ²ΠΎΠ΄ΠΎΠ², протянутых ΠΊ цоколям Π»Π°ΠΌΠΏ. ΠŸΠΎΡΠ»Π΅Π΄ΠΎΠ²Π°Ρ‚Π΅Π»ΡŒΠ½Π°Ρ схСма прСдусматриваСт ΠΏΡ€ΠΎΠΊΠ»Π°Π΄ΠΊΡƒ ΠΎΠ΄Π½ΠΎΠ³ΠΎ кабСля ΠΎΡ‚ ΠΊΠΎΠ½Ρ‚Π°ΠΊΡ‚Π° Β«87Β» ΠΊ Π»Π΅Π²ΠΎΠΉ Ρ„Π°Ρ€Π΅. Π’ мСстС крСплСния ΠΊΠ»Π΅ΠΌΠΌΡ‹ ΠΊ Π½Π΅ΠΌΡƒ ΠΏΠΎΠ΄ΠΊΠ»ΡŽΡ‡Π°Π΅Ρ‚ΡΡ Π²Ρ‚ΠΎΡ€ΠΎΠΉ ΠΏΡ€ΠΎΠ²ΠΎΠ΄, ΠΊΠΎΡ‚ΠΎΡ€Ρ‹ΠΉ тянСтся ΠΊ Ρ†ΠΎΠΊΠΎΠ»ΡŽ ΠΏΡ€Π°Π²ΠΎΠΉ Π»Π°ΠΌΠΏΡ‹.

ΠŸΠ°Ρ€Π°Π»Π»Π΅Π»ΡŒΠ½ΠΎΠ΅ Π²ΠΊΠ»ΡŽΡ‡Π΅Π½ΠΈΠ΅ ПВЀ ΠΏΡ€Π΅Π΄ΠΏΠΎΠ»Π°Π³Π°Π΅Ρ‚ ΡƒΠΊΠ»Π°Π΄ΠΊΡƒ ΠΏΠ°Ρ€Ρ‹ ΠΎΡ‚Π΄Π΅Π»ΡŒΠ½Ρ‹Ρ… ΠΊΠ°Π±Π΅Π»Π΅ΠΉ ΠΎΡ‚ Ρ€Π΅Π»Π΅ ΠΊ ΠΊΠ°ΠΆΠ΄ΠΎΠΉ ΠΏΡ€ΠΎΡ‚ΠΈΠ²ΠΎΡ‚ΡƒΠΌΠ°Π½ΠΊΠ΅.

ΠšΡ€ΠΎΠΌΠ΅ Π΄Π²ΡƒΡ… ΠΏΡ€ΠΎΠ²ΠΎΠ΄ΠΎΠ² ΠΊ Π»Π°ΠΌΠΏΠ°ΠΌ ПВЀ, ΠΌΠΎΠΆΠ½ΠΎ провСсти Ρ‚Ρ€Π΅Ρ‚ΠΈΠΉ, ΠΈΠ΄ΡƒΡ‰ΠΈΠΉ ΠΊ Β«ΠΊΠΎΠ½Ρ‚Ρ€ΠΎΠ»ΡŒΠΊΠ΅Β», ΡΠΈΠ³Π½Π°Π»ΠΈΠ·ΠΈΡ€ΡƒΡŽΡ‰Π΅ΠΉ ΠΎΠ± Π°ΠΊΡ‚ΠΈΠ²Π°Ρ†ΠΈΠΈ ΠΏΡ€ΠΎΡ‚ΠΈΠ²ΠΎΡ‚ΡƒΠΌΠ°Π½Π½Ρ‹Ρ… Ρ„Π°Ρ€. Она устанавливаСтся Π½Π° ΠΏΠ°Π½Π΅Π»ΠΈ ΠΏΡ€ΠΈΠ±ΠΎΡ€ΠΎΠ². ΠœΠ΅ΡΡ‚ΠΎ для Π½Π΅Π΅ прСдусмотрСно Π½Π° всСх соврСмСнных автомобилях. ЕдинствСнноС Ρ‡Ρ‚ΠΎ понадобится для ΠΏΠΎΠ΄ΠΊΠ»ΡŽΡ‡Π΅Π½ΠΈΡ β€” ΠΊΡƒΠΏΠΈΡ‚ΡŒ Ρ†ΠΎΠΊΠΎΠ»ΡŒ ΠΈ Π»Π°ΠΌΠΏΡƒ (накаливания ΠΈΠ»ΠΈ ΡΠ²Π΅Ρ‚ΠΎΠ΄ΠΈΠΎΠ΄Π½ΡƒΡŽ) ΡΠΎΠΎΡ‚Π²Π΅Ρ‚ΡΡ‚Π²ΡƒΡŽΡ‰Π΅Π³ΠΎ Ρ€Π°Π·ΠΌΠ΅Ρ€Π°.

ΠŸΡ€ΠΎΠ²ΠΎΠ΄Π° питания слСдуСт ΠΏΡ€ΠΎΠΊΠ»Π°Π΄Ρ‹Π²Π°Ρ‚ΡŒ, Ρ‡Ρ‚ΠΎΠ±Ρ‹ ΠΎΠ½ΠΈ Π½Π΅ ΠΏΡ€ΠΎΡ…ΠΎΠ΄ΠΈΠ»ΠΈ Π²Π±Π»ΠΈΠ·ΠΈ горячих ΠΈΠ»ΠΈ острых Π΄Π΅Ρ‚Π°Π»Π΅ΠΉ ΠΌΠ°ΡˆΠΈΠ½Ρ‹. Π’ мСстах, Π³Π΄Π΅ Π²ΠΎΠ·ΠΌΠΎΠΆΠ΅Π½ ΠΏΠ΅Ρ€Π΅Π³ΠΈΠ± ΠΏΡ€ΠΎΠ²ΠΎΠ΄Π° ΠΈΠ»ΠΈ ΠΏΠΎΠ²Ρ€Π΅ΠΆΠ΄Π΅Π½ΠΈΠ΅ изоляции, ΠΈΡ… слСдуСт Π·Π°Ρ‰ΠΈΡ‚ΠΈΡ‚ΡŒ Π³ΠΎΡ„Ρ€ΠΎΠΉ ΠΈΠ»ΠΈ Π΄ΠΎΠΏΠΎΠ»Π½ΠΈΡ‚Π΅Π»ΡŒΠ½Ρ‹ΠΌ слоСм ΠΈΠ·ΠΎΠ»Π΅Π½Ρ‚Ρ‹.

ПослС ΠΌΠΎΠ½Ρ‚Π°ΠΆΠ° ΠΈ ΠΏΠΎΠ΄ΠΊΠ»ΡŽΡ‡Π΅Π½ΠΈΡ ПВЀ Π½Π΅ΠΎΠ±Ρ…ΠΎΠ΄ΠΈΠΌΠΎ ΠΎΡ‚Ρ€Π΅Π³ΡƒΠ»ΠΈΡ€ΠΎΠ²Π°Ρ‚ΡŒ ΠΏΠΎΠ»ΠΎΠΆΠ΅Π½ΠΈΠ΅ Π»ΡƒΡ‡Π΅ΠΉ. Π€Π°Ρ€Ρ‹ Π½Π΅ Π΄ΠΎΠ»ΠΆΠ½Ρ‹ ΡΠ²Π΅Ρ‚ΠΈΡ‚ΡŒ Π²Π²Π΅Ρ€Ρ…. Π˜Π½Π°Ρ‡Π΅ ΠΎΠ½ΠΈ Π±ΡƒΠ΄ΡƒΡ‚ ΡΠ»Π΅ΠΏΠΈΡ‚ΡŒ Π²ΠΎΠ΄ΠΈΡ‚Π΅Π»Π΅ΠΉ встрСчных Π°Π²Ρ‚ΠΎ, Π½ΠΎ Π½Π΅ ΠΏΠΎΠΌΠΎΠ³ΡƒΡ‚ Π²Π°ΠΌ Ρ€Π°Π·Π³Π»ΡΠ΄Π΅Ρ‚ΡŒ Π΄ΠΎΡ€ΠΎΠ³Ρƒ Π² Ρ‚ΡƒΠΌΠ°Π½Π΅. Π’Π°ΠΊΠΆΠ΅ слСдуСт ΠΏΠΎΠ·Π°Π±ΠΎΡ‚ΠΈΡ‚ΡŒΡΡ ΠΎ Ρ‚ΠΎΠΌ, Ρ‡Ρ‚ΠΎΠ±Ρ‹ Π»ΡƒΡ‡ Ρ‡Π΅Ρ‚ΠΊΠΎ освСщал ΠΎΠ±ΠΎΡ‡ΠΈΠ½Ρƒ. Π­Ρ‚ΠΎ ΠΏΠΎΠΌΠΎΠΆΠ΅Ρ‚ Π²Π°ΠΌ Π±Π΅Π· ΠΏΡ€ΠΎΠ±Π»Π΅ΠΌ ΠΏΡ€ΠΎΠΉΡ‚ΠΈ тСхосмотр ΠΈ обСспСчит Ρ…ΠΎΡ€ΠΎΡˆΡƒΡŽ Π²ΠΈΠ΄ΠΈΠΌΠΎΡΡ‚ΡŒ Π΄Π°ΠΆΠ΅ Π² Ρ‚ΡƒΠΌΠ°Π½ ΠΈΠ»ΠΈ ΠΏΡ€ΠΈ снСгопадС.

ΠžΡΠΎΠ±Π΅Π½Π½ΠΎΡΡ‚ΡŒ ΠΏΠΎΠ΄ΠΊΠ»ΡŽΡ‡Π΅Π½ΠΈΡ ΠΏΡ€ΠΎΡ‚ΠΈΠ²ΠΎΡ‚ΡƒΠΌΠ°Π½Π½Ρ‹Ρ… Ρ„Π°Ρ€ Ρ‡Π΅Ρ€Π΅Π· Ρ€Π΅Π»Π΅ ΠΈ ΠΊΠ½ΠΎΠΏΠΊΡƒ

Π‘ΠΎΠ΄Π΅Ρ€ΠΆΠ°Π½ΠΈΠ΅

  1. Для Ρ‡Π΅Π³ΠΎ Π½ΡƒΠΆΠ½Ρ‹
  2. ВрСбования ΠΊ установкС
  3. ΠŸΠΎΠ΄ΠΊΠ»ΡŽΡ‡Π΅Π½ΠΈΠ΅ Ρ‡Π΅Ρ€Π΅Π· Ρ€Π΅Π»Π΅ ΠΈ ΠΊΠ½ΠΎΠΏΠΊΡƒ
  4. Π‘Ρ…Π΅ΠΌΠ° ΠΏΠ°Ρ€Π°Π»Π»Π΅Π»ΡŒΠ½ΠΎΠ³ΠΎ Π²ΠΊΠ»ΡŽΡ‡Π΅Π½ΠΈΡ ΠΏΡ€ΠΎΡ‚ΠΈΠ²ΠΎΡ‚ΡƒΠΌΠ°Π½ΠΎΠΊ

ΠŸΡ€ΠΎΡ‚ΠΈΠ²ΠΎΡ‚ΡƒΠΌΠ°Π½Π½Ρ‹Π΅ Ρ„Π°Ρ€Ρ‹ Π½Π΅ относятся ΠΊ ΠΎΠ±ΡΠ·Π°Ρ‚Π΅Π»ΡŒΠ½ΠΎΠΌΡƒ ΠΎΠ±ΠΎΡ€ΡƒΠ΄ΠΎΠ²Π°Π½ΠΈΡŽ автомобиля. Однако ΠΈΡ… Π½Π°Π»ΠΈΡ‡ΠΈΠ΅ сдСлаСт ΠΏΠΎΠ΅Π·Π΄ΠΊΠΈ Π±ΠΎΠ»Π΅Π΅ ΠΊΠΎΠΌΡ„ΠΎΡ€Ρ‚Π½Ρ‹ΠΌΠΈ ΠΈ бСзопасными. Для ΠΌΠΎΠ½Ρ‚Π°ΠΆΠ° Π½Π΅ Ρ‚Ρ€Π΅Π±ΡƒΡŽΡ‚ΡΡ особыС Π½Π°Π²Ρ‹ΠΊΠΈ. Установка ΠΈ подсоСдинСниС ПВЀ ΠΊ ΠΏΡ€ΠΎΠ²ΠΎΠ΄ΠΊΠ΅ Π²ΠΏΠΎΠ»Π½Π΅ осущСствимо Π² Π΄ΠΎΠΌΠ°ΡˆΠ½ΠΈΡ… условиях. Для этого понадобится ΠΌΠΎΠ½Ρ‚Π°ΠΆΠ½Ρ‹ΠΉ Π½Π°Π±ΠΎΡ€ ΠΏΠΎΠ΄ΠΊΠ»ΡŽΡ‡Π΅Π½ΠΈΡ ΠΏΡ€ΠΎΡ‚ΠΈΠ²ΠΎΡ‚ΡƒΠΌΠ°Π½Π½Ρ‹Ρ… Ρ„Π°Ρ€, ΠΏΡ€ΠΎΡΡ‚Π΅ΠΉΡˆΠΈΠ΅ слСсарныС инструмСнты ΠΈ изоляция.

Для Ρ‡Π΅Π³ΠΎ Π½ΡƒΠΆΠ½Ρ‹

Π‘Ρ‚Π°Π½Π΄Π°Ρ€Ρ‚Π½ΠΎΠ΅ Π³ΠΎΠ»ΠΎΠ²Π½ΠΎΠ΅ освСщСниС автомобиля обСспСчиваСт Π΄ΠΎΡΡ‚Π°Ρ‚ΠΎΡ‡Π½ΡƒΡŽ Π²ΠΈΠ΄ΠΈΠΌΠΎΡΡ‚ΡŒ ΠΏΡ€ΠΈ ясной ΠΏΠΎΠ³ΠΎΠ΄Π΅. Однако Π² Π½Π΅Π½Π°ΡΡ‚ΡŒΠ΅ ΠΎΡ‚ Π½Π΅Π³ΠΎ Π½Π΅ΠΌΠ½ΠΎΠ³ΠΎ Ρ‚ΠΎΠ»ΠΊΡƒ. ΠŸΡ€ΠΎΠ±Π»Π΅ΠΌΠ° Π² Ρ‚ΠΎΠΌ, Ρ‡Ρ‚ΠΎ Π»ΡƒΡ‡ΠΈ Π»Π°ΠΌΠΏ Π±Π»ΠΈΠΆΠ½Π΅Π³ΠΎ ΠΈ дальнСго свСта отраТаСтся ΠΎΡ‚ капСль Ρ‚ΡƒΠΌΠ°Π½Π° ΠΈΠ»ΠΈ снСТинок, создавая Β«Π±Π΅Π»ΡƒΡŽ ΠΏΠ΅Π»Π΅Π½ΡƒΒ» Π²ΠΏΠ΅Ρ€Π΅Π΄ΠΈ ΠΌΠ°ΡˆΠΈΠ½Ρ‹.

Π­Ρ‚ΠΎ слСдствиС Π΄Π²ΡƒΡ… Ρ„Π°ΠΊΡ‚ΠΎΡ€ΠΎΠ²:

  • ΡƒΠ·ΠΊΠΈΠ΅ оптичСский ΠΏΡƒΡ‡ΠΎΠΊ Π»ΡƒΡ‡ΡˆΠ΅ отраТаСтся ΠΎΡ‚ повСрхности;
  • высоко установлСнныС Ρ„Π°Ρ€Ρ‹ ΠΏΠΎΠ΄ΡΠ²Π΅Ρ‡ΠΈΠ²Π°ΡŽΡ‚ участок с большой ΠΊΠΎΠ½Ρ†Π΅Π½Ρ‚Ρ€Π°Ρ†ΠΈΠ΅ΠΉ капСль Ρ‚ΡƒΠΌΠ°Π½Π°.

ПВЀ ΡΠΏΠ΅Ρ†ΠΈΠ°Π»ΡŒΠ½ΠΎ спроСктированы для эксплуатации Π² тяТСлых ΠΏΠΎΠ³ΠΎΠ΄Π½Ρ‹Ρ… условиях. И Π΄Π΅Π»ΠΎ Ρ‚ΡƒΡ‚ Π½Π΅ Π² ΠΆΠ΅Π»Ρ‚Ρ‹Ρ… ΡΠ²Π΅Ρ‚ΠΎΡ„ΠΈΠ»ΡŒΡ‚Ρ€Π°Ρ…, ΠΊΠΎΡ‚ΠΎΡ€Ρ‹Π΅ Π½Π΅ΠΊΠΎΡ‚ΠΎΡ€Ρ‹Π΅ Π²ΠΎΠ΄ΠΈΡ‚Π΅Π»ΠΈ ΡΡ‡ΠΈΡ‚Π°ΡŽΡ‚ ΠΏΠ°Π½Π°Ρ†Π΅Π΅ΠΉ Π² Ρ‚ΡƒΠΌΠ°Π½Π½ΡƒΡŽ ΠΏΠΎΠ³ΠΎΠ΄Ρƒ. НС зря ΠŸΠ”Π” Π΄ΠΎΠΏΡƒΡΠΊΠ°ΡŽΡ‚ установку ПВЀ Π΄Π²ΡƒΡ… Ρ†Π²Π΅Ρ‚ΠΎΠ² β€” Π±Π΅Π»ΠΎΠ³ΠΎ ΠΈ ΠΆΠ΅Π»Ρ‚ΠΎΠ³ΠΎ.

Π’Π°ΠΊΠΈΠ΅ Ρ„Π°Ρ€Ρ‹ ΡΠΏΡ€Π°Π²Π»ΡΡŽΡ‚ΡΡ с Π½Π΅ΠΏΠΎΠ³ΠΎΠ΄ΠΎΠΉ благодаря ΠΏΠ°Ρ€Π΅ Π²Π°ΠΆΠ½Ρ‹Ρ… свойств:

  • Π¨ΠΈΡ€ΠΈΠ½Π° свСтового ΠΏΡƒΡ‡ΠΊΠ°. Π¨ΠΈΡ€ΠΎΠΊΠΈΠΉ Π»ΡƒΡ‡ мСньшС отраТаСтся ΠΎΡ‚ Π²ΠΎΠ΄Ρ‹ ΠΈ льдинок ΠΈ Π½Π΅ создаСт Β«Π±Π΅Π»ΡƒΡŽ ΠΏΠ΅Π»Π΅Π½ΡƒΒ».
  • Высота крСплСния. ΠšΠΎΠ½Ρ†Π΅Π½Ρ‚Ρ€Π°Ρ†ΠΈΡ Ρ‚ΡƒΠΌΠ°Π½Π° Π²Π±Π»ΠΈΠ·ΠΈ Π·Π΅ΠΌΠ»ΠΈ Π½ΠΈΠΆΠ΅. ΠŸΠΎΡΡ‚ΠΎΠΌΡƒ Π½ΠΈΠ·ΠΊΠΎΠ΅ располоТСниС позволяСт ΠΏΠΎΠ΄ΡΠ²Π΅Ρ‡ΠΈΠ²Π°Ρ‚ΡŒ ΠΏΠΎΠ΄ ΠΏΠ΅Π»Π΅Π½Ρƒ.

ПослСдний ΠΌΠΎΠΌΠ΅Π½Ρ‚ особСнно Π²Π°ΠΆΠ΅Π½. Если ΡƒΡΡ‚Π°Π½ΠΎΠ²ΠΈΡ‚ΡŒ ПВЀ Π½Π° ΡƒΡ€ΠΎΠ²Π½Π΅ Ρ„Π°Ρ€ Π³ΠΎΠ»ΠΎΠ²Π½ΠΎΠ³ΠΎ освСщСния, ΠΎΠ½ΠΈ Π½Π΅ Π±ΡƒΠ΄ΡƒΡ‚ ΡΠΏΡ€Π°Π²Π»ΡΡ‚ΡŒΡΡ со своСй Π·Π°Π΄Π°Ρ‡Π΅ΠΉ.

ΠšΡ€ΠΎΠΌΠ΅ ΠΏΠ΅Ρ€Π΅Π΄Π½ΠΈΡ… ПВЀ, ΡƒΠ»ΡƒΡ‡ΡˆΠ°ΡŽΡ‰ΠΈΡ… Π²ΠΈΠ΄ΠΈΠΌΠΎΡΡ‚ΡŒ с Π²ΠΎΠ΄ΠΈΡ‚Π΅Π»ΡŒΡΠΊΠΎΠ³ΠΎ мСста, ΠΈΡΠΏΠΎΠ»ΡŒΠ·ΡƒΡŽΡ‚ΡΡ Π·Π°Π΄Π½ΠΈΠ΅ ΠΏΡ€ΠΎΡ‚ΠΈΠ²ΠΎΡ‚ΡƒΠΌΠ°Π½Π½Ρ‹Π΅ Ρ„Π°Ρ€Ρ‹, Π΄ΡƒΠ±Π»ΠΈΡ€ΡƒΡŽΡ‰ΠΈΠ΅ Π³Π°Π±Π°Ρ€ΠΈΡ‚Ρ‹. Они Ρ‡Π΅Ρ‚ΠΊΠΎ ΠΎΠ±ΠΎΠ·Π½Π°Ρ‡Π°ΡŽΡ‚ Π°Π²Ρ‚ΠΎ Π² условиях ΠΏΠ»ΠΎΡ…ΠΎΠΉ видимости, Π½ΠΎ ΠΎΠ±Π»Π°Π΄Π°ΡŽΡ‚ ΠΈΠ·Π±Ρ‹Ρ‚ΠΎΡ‡Π½ΠΎΠΉ ΡΡ€ΠΊΠΎΡΡ‚ΡŒΡŽ. ΠŸΠΎΡΡ‚ΠΎΠΌΡƒ ΠΈΡ… Π½Π΅ слСдуСт Π²ΠΊΠ»ΡŽΡ‡Π°Ρ‚ΡŒ Π² ΡΡΠ½ΡƒΡŽ ΠΏΠΎΠ³ΠΎΠ΄Ρƒ.

ВрСбования ΠΊ установкС

ПВЀ состоят ΠΈΠ· Ρ‚Ρ€Π΅Ρ… корпуса с Ρ†ΠΎΠΊΠΎΠ»Π΅ΠΌ для крСплСния Π»Π°ΠΌΠΏΡ‹, отраТатСля ΠΈ стСклянного рассСиватСля, ΠΊΠΎΡ‚ΠΎΡ€Ρ‹Π΅ ΠΎΠ±Π΅ΡΠΏΠ΅Ρ‡ΠΈΠ²Π°ΡŽΡ‚ Π½ΡƒΠΆΠ½ΡƒΡŽ Ρ„ΠΎΡ€ΠΌΡƒ свСтового ΠΏΡƒΡ‡ΠΊΠ°. ΠžΡΠΎΠ±Π΅Π½Π½ΠΎΡΡ‚ΠΈ эксплуатации ΠΈ Π½Π°Π·Π½Π°Ρ‡Π΅Π½ΠΈΠ΅ ΠΏΡ€ΠΎΡ‚ΠΈΠ²ΠΎΡ‚ΡƒΠΌΠ°Π½ΠΎΠΊ Π½Π°Π»Π°Π³Π°Π΅Ρ‚ Π½Π° Π½ΠΈΡ… особыС трСбования, ΠΊΠΎΡ‚ΠΎΡ€Ρ‹Π΅ нашли ΠΎΡ‚Ρ€Π°ΠΆΠ΅Π½ΠΈΠ΅ Π² Π΄Π΅ΠΉΡΡ‚Π²ΡƒΡŽΡ‰ΠΈΡ… ΠΏΡ€Π°Π²ΠΈΠ»Π°Ρ… Π΄ΠΎΡ€ΠΎΠΆΠ½ΠΎΠ³ΠΎ двиТСния:

  • ΡƒΡΡ‚Π°Π½Π°Π²Π»ΠΈΠ²Π°Ρ‚ΡŒ ΠΌΠΎΠΆΠ½ΠΎ Π½Π΅ Π±ΠΎΠ»Π΅Π΅ 2 Ρ„Π°Ρ€;
  • ПВЀ Π΄ΠΎΠ»ΠΆΠ½Ρ‹ ΠΊΡ€Π΅ΠΏΠΈΡ‚ΡŒΡΡ с отступом 40 см ΠΎΡ‚ Π±ΠΎΠΊΠΎΠ²Ρ‹Ρ… ΠΊΡ€Π°Π΅Π² Π°Π²Ρ‚ΠΎ ΠΈ Π²Ρ‹ΡˆΠ΅ 25 см ΠΎΡ‚ повСрхности Π΄ΠΎΡ€ΠΎΠ³ΠΈ;
  • Π»ΡƒΡ‡ Π΄ΠΎΠ»ΠΆΠ΅Π½ Π±Ρ‹Ρ‚ΡŒ Π½Π°ΠΏΡ€Π°Π²Π»Π΅Π½ Π½ΠΈΠΆΠ΅, Ρ‡Π΅ΠΌ Ρ„Π°Ρ€Ρ‹ Π±Π»ΠΈΠΆΠ½Π΅Π³ΠΎ свСта;
  • ΡƒΠ³Π»Ρ‹ видимости Π½Π΅ ΠΌΠΎΠ³ΡƒΡ‚ Π²Ρ‹Ρ…ΠΎΠ΄ΠΈΡ‚ΡŒ Π·Π° ΠΏΡ€Π΅Π΄Π΅Π»Ρ‹ ΡΠ»Π΅Π΄ΡƒΡŽΡ‰ΠΈΡ… Π·Π½Π°Ρ‡Π΅Π½ΠΈΠΉ: -10…+15 градусов ΠΈ -10. .+45 ΠΏΠΎ Π²Π΅Ρ€Ρ‚ΠΈΠΊΠ°Π»ΠΈ ΠΈ Π³ΠΎΡ€ΠΈΠ·ΠΎΠ½Ρ‚Π°Π»ΠΈ соотвСтствСнно.

Π’ Π±Π°ΠΌΠΏΠ΅Ρ€Π΅ соврСмСнных Π»Π΅Π³ΠΊΠΎΠ²ΡƒΡˆΠ΅ΠΊ прСдусмотрСны ΡΠΏΠ΅Ρ†ΠΈΠ°Π»ΡŒΠ½Ρ‹Π΅ мСста для ΠΌΠΎΠ½Ρ‚Π°ΠΆΠ° ΠΏΡ€ΠΎΡ‚ΠΈΠ²ΠΎΡ‚ΡƒΠΌΠ°Π½Π½Ρ‹Ρ… Ρ„Π°Ρ€. ΠŸΡ€ΠΈ ΠΈΡ… отсутствии ΠΏΡ€ΠΎΡ‚ΠΈΠ²ΠΎΡ‚ΡƒΠΌΠ°Π½ΠΊΠΈ слСдуСт ΠΊΡ€Π΅ΠΏΠΈΡ‚ΡŒ согласно Ρ€Π΅ΠΊΠΎΠΌΠ΅Π½Π΄Π°Ρ†ΠΈΠΉ производитСля.

Π“Π»Π°Π²Π½Ρ‹Π΅ трСбования:

  • ТСсткоС ΠΊΡ€Π΅ΠΏΠ»Π΅Π½ΠΈΠ΅ ПВЀ Π½Π° ΠΊΡ€ΠΎΠ½ΡˆΡ‚Π΅ΠΉΠ½Π°Ρ…;
  • Π·Π°Ρ‰ΠΈΡ‚Π° элСктропитания ΠΏΡ€Π΅Π΄ΠΎΡ…Ρ€Π°Π½ΠΈΡ‚Π΅Π»Π΅ΠΌ;
  • ΠΏΡ€Π°Π²ΠΈΠ»ΡŒΠ½Ρ‹Π΅ ΡƒΠ³Π»Ρ‹ направлСния свСтовых ΠΏΡƒΡ‡ΠΊΠΎΠ².

ΠŸΠΎΠ΄ΠΊΠ»ΡŽΡ‡Π΅Π½ΠΈΠ΅ Ρ‡Π΅Ρ€Π΅Π· Ρ€Π΅Π»Π΅ ΠΈ ΠΊΠ½ΠΎΠΏΠΊΡƒ

БущСствуСт нСсколько Π²Π°Ρ€ΠΈΠ°Π½Ρ‚ΠΎΠ² сопряТСния ПВЀ с Π±ΠΎΡ€Ρ‚ΠΎΠ²ΠΎΠΉ ΡΠ΅Ρ‚ΡŒΡŽ автомобиля. ΠŸΠΎΠ΄ΠΊΠ»ΡŽΡ‡Π΅Π½ΠΈΠ΅ ΠΏΡ€ΠΎΡ‚ΠΈΠ²ΠΎΡ‚ΡƒΠΌΠ°Π½Π½Ρ‹Ρ… Ρ„Π°Ρ€ Ρ‡Π΅Ρ€Π΅Π· Ρ€Π΅Π»Π΅ с использованиСм ΠΎΡ‚Π΄Π΅Π»ΡŒΠ½ΠΎΠΉ ΠΊΠ½ΠΎΠΏΠΊΠΈ β€” самоС ΡƒΠ΄ΠΎΠ±Π½ΠΎΠ΅, бСзопасноС ΠΈ ΠΏΡ€Π°Π²ΠΈΠ»ΡŒΠ½ΠΎΠ΅ Ρ€Π΅ΡˆΠ΅Π½ΠΈΠ΅.

Для выполнСния Ρ€Π°Π±ΠΎΡ‚Ρ‹ Π²Π°ΠΌ понадобятся:

  • ΠΏΡ€ΠΎΡ‚ΠΈΠ²ΠΎΡ‚ΡƒΠΌΠ°Π½ΠΊΠΈ;
  • ΡΠΎΠ΅Π΄ΠΈΠ½ΠΈΡ‚Π΅Π»ΡŒΠ½Ρ‹Π΅ ΠΊΠ»Π΅ΠΌΠΌΡ‹;
  • ΠΏΡ€Π΅Π΄ΠΎΡ…Ρ€Π°Π½ΠΈΡ‚Π΅Π»ΡŒ Π½Π° 15 Π°ΠΌΠΏΠ΅Ρ€;
  • Ρ‡Π΅Ρ‚Ρ‹Ρ€Π΅Ρ…ΠΊΠΎΠ½Ρ‚Π°ΠΊΡ‚Π½ΠΎΠ΅ Ρ€Π΅Π»Π΅;
  • ΠΊΠΎΠ»ΠΎΠ΄ΠΊΠ° ΠΏΠΎΠ΄ΠΊΠ»ΡŽΡ‡Π΅Π½ΠΈΡ;
  • ΠΏΡ€ΠΎΠ²ΠΎΠ΄Π°;
  • изоляция ΠΈΠ»ΠΈ тСрмоусадочная Ρ‚Ρ€ΡƒΠ±ΠΊΠ°.

ΠœΠ°Ρ‚Π΅Ρ€ΠΈΠ°Π»Ρ‹ входят Π² ΠΌΠΎΠ½Ρ‚Π°ΠΆΠ½Ρ‹ΠΉ ΠΊΠΎΠΌΠΏΠ»Π΅ΠΊΡ‚ ΠΈ ΠΏΡ€ΠΎΠ΄Π°ΡŽΡ‚ΡΡ Π² ΠΎΠ΄Π½ΠΎΠΌ Π½Π°Π±ΠΎΡ€Π΅ с ПВЀ. Из инструмСнта Π½ΡƒΠΆΠ½Ρ‹ ΠΎΡ‚Π²Π΅Ρ€Ρ‚ΠΊΠΈ, пассатиТи ΠΈ Π½ΠΎΠΆ для зачистки изоляции ΠΏΡ€ΠΎΠ²ΠΎΠ΄ΠΎΠ².

Π Π°Π·Π²ΠΎΠ΄ΠΊΠ° Π΄ΠΎΠ»ΠΆΠ½Π° Π±Ρ‹Ρ‚ΡŒ Π²Ρ‹ΠΏΠΎΠ»Π½Π΅Π½Π° согласно ΠΏΠ»Π°Π½Π° ΠΏΠ°Ρ€Π°Π»Π»Π΅Π»ΡŒΠ½ΠΎΠ³ΠΎ ΠΈΠ»ΠΈ ΠΏΠΎΡΠ»Π΅Π΄ΠΎΠ²Π°Ρ‚Π΅Π»ΡŒΠ½ΠΎΠ³ΠΎ соСдинСния Π»Π°ΠΌΠΏ.

ΠžΠΏΠ΅Ρ€Π°Ρ†ΠΈΠΈ ΠΏΠΎ ΠΏΠΎΠ΄ΠΊΠ»ΡŽΡ‡Π΅Π½ΠΈΡŽ ПВЀ Π²Ρ‹ΠΏΠΎΠ»Π½ΡΡŽΡ‚ΡΡ Π² Ρ‚Π°ΠΊΠΎΠΉ ΠΏΠΎΡΠ»Π΅Π΄ΠΎΠ²Π°Ρ‚Π΅Π»ΡŒΠ½ΠΎΡΡ‚ΠΈ:

  • ΠžΡ‚ΡΠΎΠ΅Π΄ΠΈΠ½ΠΈΡ‚ΡŒ ΠΌΠΈΠ½ΡƒΡΠΎΠ²ΡƒΡŽ ΠΊΠ»Π΅ΠΌΠΌΡƒ ΠΎΡ‚ ΠΠšΠ‘;
  • НадСТно Π·Π°Ρ„ΠΈΠΊΡΠΈΡ€ΠΎΠ²Π°Ρ‚ΡŒ Ρ€Π΅Π»Π΅ Π² салонС ΠΌΠ°ΡˆΠΈΠ½Ρ‹. Для этого ΠΏΠΎΠ΄ΠΎΠΉΠ΄Π΅Ρ‚ пространство ΠΏΠΎΠ΄ панСлью ΠΏΡ€ΠΈΠ±ΠΎΡ€ΠΎΠ².
  • Π—Π°ΠΊΡ€Π΅ΠΏΠΈΡ‚ΡŒ ΠΊΠ½ΠΎΠΏΠΊΡƒ для Π²ΠΊΠ»ΡŽΡ‡Π΅Π½ΠΈΡ ΠΏΡ€ΠΎΡ‚ΠΈΠ²ΠΎΡ‚ΡƒΠΌΠ°Π½Π½Ρ‹Ρ… Ρ„Π°Ρ€ Π½Π° Β«Ρ‚ΠΎΡ€ΠΏΠ΅Π΄Π΅Β» (ΠΎΠ±Ρ‹Ρ‡Π½ΠΎ для Π½Π΅Π΅ прСдусмотрСно ΠΎΡ‚Π΄Π΅Π»ΡŒΠ½ΠΎΠ΅ мСсто, ΠΏΡ€ΠΈΠΊΡ€Ρ‹Ρ‚ΠΎΠ΅ Π·Π°Π³Π»ΡƒΡˆΠΊΠΎΠΉ).
  • ΠŸΡ€ΠΎΠ»ΠΎΠΆΠΈΡ‚ΡŒ ΠΏΡ€ΠΎΠ²ΠΎΠ΄ ΠΎΡ‚ аккумулятора ΠΊ Ρ€Π΅Π»Π΅, Π·Π°ΠΊΡ€Π΅ΠΏΠΈΡ‚ΡŒ Π½Π° Π½Π΅ΠΌ ΠΊΠ»Π΅ΠΌΠΌΡƒ ΠΈ ΠΏΡ€ΠΈΡΠΎΠ΅Π΄ΠΈΠ½ΠΈΡ‚ΡŒ ΠΊ ΠΊΠΎΠ½Ρ‚Π°ΠΊΡ‚Ρƒ Β«30Β».

Π’Π°ΠΆΠ½ΠΎ: Π² ΡΠ»Π΅ΠΊΡ‚Ρ€ΠΎΡ†Π΅ΠΏΡŒ ΠΌΠ΅ΠΆΠ΄Ρƒ ΠΠšΠ‘ ΠΈ Ρ€Π΅Π»Π΅ Π½Π΅ΠΎΠ±Ρ…ΠΎΠ΄ΠΈΠΌΠΎ Π²ΡΡ‚Π°Π²ΠΈΡ‚ΡŒ ΠΏΡ€Π΅Π΄ΠΎΡ…Ρ€Π°Π½ΠΈΡ‚Π΅Π»ΡŒ, ΠΊΠΎΡ‚ΠΎΡ€Ρ‹ΠΉ Π»ΡƒΡ‡ΡˆΠ΅ Ρ€Π°ΡΠΏΠΎΠ»ΠΎΠΆΠΈΡ‚ΡŒ Π² ΠΏΠΎΠ΄ΠΊΠ°ΠΏΠΎΡ‚Π½ΠΎΠΌ пространствС ΠΏΠΎΠ±Π»ΠΈΠΆΠ΅ ΠΊ аккумуляторной Π±Π°Ρ‚Π°Ρ€Π΅Π΅. Π­Ρ‚ΠΎ ΠΏΡ€Π΅Π΄ΠΎΡ‚Π²Ρ€Π°Ρ‚ΠΈΡ‚ Π²ΠΎΠ·Π³ΠΎΡ€Π°Π½ΠΈΠ΅ ΠΏΡ€ΠΎΠ²ΠΎΠ΄ΠΊΠΈ ΠΏΡ€ΠΈ Π·Π°ΠΌΡ‹ΠΊΠ°Π½ΠΈΠΈ Π½Π° корпус.

  • Π‘ΠΎΠ΅Π΄ΠΈΠ½ΠΈΡ‚ΡŒ ΠΊΠ½ΠΎΠΏΠΊΡƒ ΠΈ 85-ΠΉ Π²Ρ‹Π²ΠΎΠ΄ Ρ€Π΅Π»Π΅.
  • ΠŸΠΎΠ΄ΠΊΠ»ΡŽΡ‡ΠΈΡ‚ΡŒ плюсовой кабСль ΠΊΠΎ Π²Ρ…ΠΎΠ΄Ρƒ ΠΊΠ½ΠΎΠΏΠΊΠΈ, протянув Π΅Π³ΠΎ ΠΎΡ‚ Π·Π°ΠΌΠΊΠ° ΠΊΠΎΠ½Ρ‚Π°ΠΊΡ‚ΠΎΠ² Π½Π° Π·Π°ΠΌΠΊΠ΅ заТигания. ΠŸΠΎΠ΄ΠΊΠ»ΡŽΡ‡Π°Ρ‚ΡŒ ΠΏΠΈΡ‚Π°Π½ΠΈΠ΅ слСдуСт Ρ‚ΡƒΠ΄Π°, Π³Π΄Π΅ напряТСниС подаСтся лишь послС ΠΏΠΎΠ²ΠΎΡ€ΠΎΡ‚Π° ΠΊΠ»ΡŽΡ‡Π°. Π­Ρ‚ΠΎ Π½Π΅ даст Π²Π°ΠΌ ΠΎΡΡ‚Π°Π²ΠΈΡ‚ΡŒ ПВЀ горящими ΠΏΡ€ΠΈ постановкС ΠΌΠ°ΡˆΠΈΠ½Ρ‹ Π½Π° стоянку.
  • Π‘ΠΎΠ΅Π΄ΠΈΠ½ΠΈΡ‚ΡŒ ΠΊΠΎΠ½Ρ‚Π°ΠΊΡ‚ Β«86Β» с массой, ΠΈΡΠΏΠΎΠ»ΡŒΠ·ΡƒΡ для этого блиТайший Π±ΠΎΠ»Ρ‚ ΠΈΠ»ΠΈ саморСз.
  • Π—Π°ΠΊΡ€Π΅ΠΏΠΈΡ‚ΡŒ ПВЀ Π½Π° ΠΏΠ΅Ρ€Π΅Π΄Π½Π΅ΠΌ Π±Π°ΠΌΠΏΠ΅Ρ€Π΅, ΡƒΠ΄Π°Π»ΠΈΠ² пластиковыС Π·Π°Π³Π»ΡƒΡˆΠΊΠΈ. Иногда ΡΠΏΠ΅Ρ†ΠΈΠ°Π»ΡŒΠ½Ρ‹Ρ… мСст для крСплСния ПВЀ Π½Π΅ прСдусмотрСно. Π’ΠΎΠ³Π΄Π° придСтся ΠΏΡ€ΠΎΡ€Π΅Π·Π°Ρ‚ΡŒ ΠΈΡ… Π»ΠΎΠ±Π·ΠΈΠΊΠΎΠΌ ΠΈΠ»ΠΈ Π·Π°Ρ„ΠΈΠΊΡΠΈΡ€ΠΎΠ²Π°Ρ‚ΡŒ Ρ„Π°Ρ€Ρ‹ Π½Π° ΡΡ‚Π°Π»ΡŒΠ½Ρ‹Ρ… ΠΊΡ€ΠΎΠ½ΡˆΡ‚Π΅ΠΉΠ½Π°Ρ… Π½Π°Π΄ Π±Π°ΠΌΠΏΠ΅Ρ€ΠΎΠΌ.
  • ΠŸΡ€ΠΎΠ»ΠΎΠΆΠΈΡ‚ΡŒ кабСля ΠΎΡ‚ 87-Π³ΠΎ ΠΊΠΎΠ½Ρ‚Π°ΠΊΡ‚Π° Ρ€Π΅Π»Π΅ ΠΊ ΠΏΡ€ΠΎΡ‚ΠΈΠ²ΠΎΡ‚ΡƒΠΌΠ°Π½ΠΊΠ°ΠΌ, ΠΏΠΎΠ΄ΠΊΠ»ΡŽΡ‡ΠΈΠ² ΠΊ цоколям.
  • Π‘ΠΎΠ΅Π΄ΠΈΠ½ΠΈΡ‚ΡŒ Π²Ρ‹Ρ…ΠΎΠ΄ массы ПВЀ ΠΊ ΠΊΡƒΠ·ΠΎΠ²Ρƒ.
  • ΠΠ°Π΄Π΅Ρ‚ΡŒ ΠΈ Π·Π°ΠΊΡ€Π΅ΠΏΠΈΡ‚ΡŒ ΠΌΠΈΠ½ΡƒΡΠΎΠ²ΡƒΡŽ ΠΊΠ»Π΅ΠΌΠΌΡƒ Π½Π° ΠΠšΠ‘.
  • ΠŸΠΎΠ²Π΅Ρ€Π½ΡƒΡ‚ΡŒ ΠΊΠ»ΡŽΡ‡ заТигания ΠΈ ΡƒΠ±Π΅Π΄ΠΈΡ‚ΡŒΡΡ Π² работоспособности ΠΏΡ€ΠΎΡ‚ΠΈΠ²ΠΎΡ‚ΡƒΠΌΠ°Π½Π½Ρ‹Ρ… Ρ„Π°Ρ€.

ΠŸΡ€ΠΈ ΠΌΠΎΠ½Ρ‚Π°ΠΆΠ΅ слСдуСт ΠΈΠ·ΠΎΠ»ΠΈΡ€ΠΎΠ²Π°Ρ‚ΡŒ мСста соСдинСния ΠΏΡ€ΠΎΠ²ΠΎΠ΄ΠΎΠ² ΠΈ ΠΊΠ»Π΅ΠΌΠΌΡ‹ ΠΏΡ€ΠΈ ΠΏΠΎΠΌΠΎΡ‰ΠΈ тСрмоусадочных Ρ‚Ρ€ΡƒΠ±ΠΎΠΊ ΠΈΠ»ΠΈ ΠΈΠ·ΠΎΠ»Π΅Π½Ρ‚Ρ‹.

Π‘Ρ…Π΅ΠΌΠ° ΠΏΠ°Ρ€Π°Π»Π»Π΅Π»ΡŒΠ½ΠΎΠ³ΠΎ Π²ΠΊΠ»ΡŽΡ‡Π΅Π½ΠΈΡ ΠΏΡ€ΠΎΡ‚ΠΈΠ²ΠΎΡ‚ΡƒΠΌΠ°Π½ΠΎΠΊ

ЭлСктросхСмы ΠΌΠΎΠ½Ρ‚Π°ΠΆΠ° ΠΏΡ€ΠΎΡ‚ΠΈΠ²ΠΎΡ‚ΡƒΠΌΠ°Π½Π½Ρ‹Ρ… Ρ„Π°Ρ€ ΠΎΡ‚Π»ΠΈΡ‡Π°ΡŽΡ‚ΡΡ лишь количСством ΠΏΠ»ΡŽΡΠΎΠ²Ρ‹Ρ… ΠΏΡ€ΠΎΠ²ΠΎΠ΄ΠΎΠ², протянутых ΠΊ цоколям Π»Π°ΠΌΠΏ. ΠŸΠΎΡΠ»Π΅Π΄ΠΎΠ²Π°Ρ‚Π΅Π»ΡŒΠ½Π°Ρ схСма прСдусматриваСт ΠΏΡ€ΠΎΠΊΠ»Π°Π΄ΠΊΡƒ ΠΎΠ΄Π½ΠΎΠ³ΠΎ кабСля ΠΎΡ‚ ΠΊΠΎΠ½Ρ‚Π°ΠΊΡ‚Π° Β«87Β» ΠΊ Π»Π΅Π²ΠΎΠΉ Ρ„Π°Ρ€Π΅. Π’ мСстС крСплСния ΠΊΠ»Π΅ΠΌΠΌΡ‹ ΠΊ Π½Π΅ΠΌΡƒ ΠΏΠΎΠ΄ΠΊΠ»ΡŽΡ‡Π°Π΅Ρ‚ΡΡ Π²Ρ‚ΠΎΡ€ΠΎΠΉ ΠΏΡ€ΠΎΠ²ΠΎΠ΄, ΠΊΠΎΡ‚ΠΎΡ€Ρ‹ΠΉ тянСтся ΠΊ Ρ†ΠΎΠΊΠΎΠ»ΡŽ ΠΏΡ€Π°Π²ΠΎΠΉ Π»Π°ΠΌΠΏΡ‹.

ΠŸΠ°Ρ€Π°Π»Π»Π΅Π»ΡŒΠ½ΠΎΠ΅ Π²ΠΊΠ»ΡŽΡ‡Π΅Π½ΠΈΠ΅ ПВЀ ΠΏΡ€Π΅Π΄ΠΏΠΎΠ»Π°Π³Π°Π΅Ρ‚ ΡƒΠΊΠ»Π°Π΄ΠΊΡƒ ΠΏΠ°Ρ€Ρ‹ ΠΎΡ‚Π΄Π΅Π»ΡŒΠ½Ρ‹Ρ… ΠΊΠ°Π±Π΅Π»Π΅ΠΉ ΠΎΡ‚ Ρ€Π΅Π»Π΅ ΠΊ ΠΊΠ°ΠΆΠ΄ΠΎΠΉ ΠΏΡ€ΠΎΡ‚ΠΈΠ²ΠΎΡ‚ΡƒΠΌΠ°Π½ΠΊΠ΅.

ΠšΡ€ΠΎΠΌΠ΅ Π΄Π²ΡƒΡ… ΠΏΡ€ΠΎΠ²ΠΎΠ΄ΠΎΠ² ΠΊ Π»Π°ΠΌΠΏΠ°ΠΌ ПВЀ, ΠΌΠΎΠΆΠ½ΠΎ провСсти Ρ‚Ρ€Π΅Ρ‚ΠΈΠΉ, ΠΈΠ΄ΡƒΡ‰ΠΈΠΉ ΠΊ Β«ΠΊΠΎΠ½Ρ‚Ρ€ΠΎΠ»ΡŒΠΊΠ΅Β», ΡΠΈΠ³Π½Π°Π»ΠΈΠ·ΠΈΡ€ΡƒΡŽΡ‰Π΅ΠΉ ΠΎΠ± Π°ΠΊΡ‚ΠΈΠ²Π°Ρ†ΠΈΠΈ ΠΏΡ€ΠΎΡ‚ΠΈΠ²ΠΎΡ‚ΡƒΠΌΠ°Π½Π½Ρ‹Ρ… Ρ„Π°Ρ€. Она устанавливаСтся Π½Π° ΠΏΠ°Π½Π΅Π»ΠΈ ΠΏΡ€ΠΈΠ±ΠΎΡ€ΠΎΠ². ΠœΠ΅ΡΡ‚ΠΎ для Π½Π΅Π΅ прСдусмотрСно Π½Π° всСх соврСмСнных автомобилях. ЕдинствСнноС Ρ‡Ρ‚ΠΎ понадобится для ΠΏΠΎΠ΄ΠΊΠ»ΡŽΡ‡Π΅Π½ΠΈΡ β€” ΠΊΡƒΠΏΠΈΡ‚ΡŒ Ρ†ΠΎΠΊΠΎΠ»ΡŒ ΠΈ Π»Π°ΠΌΠΏΡƒ (накаливания ΠΈΠ»ΠΈ ΡΠ²Π΅Ρ‚ΠΎΠ΄ΠΈΠΎΠ΄Π½ΡƒΡŽ) ΡΠΎΠΎΡ‚Π²Π΅Ρ‚ΡΡ‚Π²ΡƒΡŽΡ‰Π΅Π³ΠΎ Ρ€Π°Π·ΠΌΠ΅Ρ€Π°.

ΠŸΡ€ΠΎΠ²ΠΎΠ΄Π° питания слСдуСт ΠΏΡ€ΠΎΠΊΠ»Π°Π΄Ρ‹Π²Π°Ρ‚ΡŒ, Ρ‡Ρ‚ΠΎΠ±Ρ‹ ΠΎΠ½ΠΈ Π½Π΅ ΠΏΡ€ΠΎΡ…ΠΎΠ΄ΠΈΠ»ΠΈ Π²Π±Π»ΠΈΠ·ΠΈ горячих ΠΈΠ»ΠΈ острых Π΄Π΅Ρ‚Π°Π»Π΅ΠΉ ΠΌΠ°ΡˆΠΈΠ½Ρ‹. Π’ мСстах, Π³Π΄Π΅ Π²ΠΎΠ·ΠΌΠΎΠΆΠ΅Π½ ΠΏΠ΅Ρ€Π΅Π³ΠΈΠ± ΠΏΡ€ΠΎΠ²ΠΎΠ΄Π° ΠΈΠ»ΠΈ ΠΏΠΎΠ²Ρ€Π΅ΠΆΠ΄Π΅Π½ΠΈΠ΅ изоляции, ΠΈΡ… слСдуСт Π·Π°Ρ‰ΠΈΡ‚ΠΈΡ‚ΡŒ Π³ΠΎΡ„Ρ€ΠΎΠΉ ΠΈΠ»ΠΈ Π΄ΠΎΠΏΠΎΠ»Π½ΠΈΡ‚Π΅Π»ΡŒΠ½Ρ‹ΠΌ слоСм ΠΈΠ·ΠΎΠ»Π΅Π½Ρ‚Ρ‹.

ПослС ΠΌΠΎΠ½Ρ‚Π°ΠΆΠ° ΠΈ ΠΏΠΎΠ΄ΠΊΠ»ΡŽΡ‡Π΅Π½ΠΈΡ ПВЀ Π½Π΅ΠΎΠ±Ρ…ΠΎΠ΄ΠΈΠΌΠΎ ΠΎΡ‚Ρ€Π΅Π³ΡƒΠ»ΠΈΡ€ΠΎΠ²Π°Ρ‚ΡŒ ΠΏΠΎΠ»ΠΎΠΆΠ΅Π½ΠΈΠ΅ Π»ΡƒΡ‡Π΅ΠΉ. Π€Π°Ρ€Ρ‹ Π½Π΅ Π΄ΠΎΠ»ΠΆΠ½Ρ‹ ΡΠ²Π΅Ρ‚ΠΈΡ‚ΡŒ Π²Π²Π΅Ρ€Ρ…. Π˜Π½Π°Ρ‡Π΅ ΠΎΠ½ΠΈ Π±ΡƒΠ΄ΡƒΡ‚ ΡΠ»Π΅ΠΏΠΈΡ‚ΡŒ Π²ΠΎΠ΄ΠΈΡ‚Π΅Π»Π΅ΠΉ встрСчных Π°Π²Ρ‚ΠΎ, Π½ΠΎ Π½Π΅ ΠΏΠΎΠΌΠΎΠ³ΡƒΡ‚ Π²Π°ΠΌ Ρ€Π°Π·Π³Π»ΡΠ΄Π΅Ρ‚ΡŒ Π΄ΠΎΡ€ΠΎΠ³Ρƒ Π² Ρ‚ΡƒΠΌΠ°Π½Π΅. Π’Π°ΠΊΠΆΠ΅ слСдуСт ΠΏΠΎΠ·Π°Π±ΠΎΡ‚ΠΈΡ‚ΡŒΡΡ ΠΎ Ρ‚ΠΎΠΌ, Ρ‡Ρ‚ΠΎΠ±Ρ‹ Π»ΡƒΡ‡ Ρ‡Π΅Ρ‚ΠΊΠΎ освСщал ΠΎΠ±ΠΎΡ‡ΠΈΠ½Ρƒ. Π­Ρ‚ΠΎ ΠΏΠΎΠΌΠΎΠΆΠ΅Ρ‚ Π²Π°ΠΌ Π±Π΅Π· ΠΏΡ€ΠΎΠ±Π»Π΅ΠΌ ΠΏΡ€ΠΎΠΉΡ‚ΠΈ тСхосмотр ΠΈ обСспСчит Ρ…ΠΎΡ€ΠΎΡˆΡƒΡŽ Π²ΠΈΠ΄ΠΈΠΌΠΎΡΡ‚ΡŒ Π΄Π°ΠΆΠ΅ Π² Ρ‚ΡƒΠΌΠ°Π½ ΠΈΠ»ΠΈ ΠΏΡ€ΠΈ снСгопадС.

Π‘Ρ…Π΅ΠΌΠ° ΠΏΠΎΠ΄ΠΊΠ»ΡŽΡ‡Π΅Π½ΠΈΡ ΠΏΡ€ΠΎΡ‚ΠΈΠ²ΠΎΡ‚ΡƒΠΌΠ°Π½Π½Ρ‹Ρ… Ρ„Π°Ρ€ Ρ‡Π΅Ρ€Π΅Π· Ρ€Π΅Π»Π΅ ΠΈ ΠΊΠ½ΠΎΠΏΠΊΡƒ, ΠΏΡ€Π°Π²ΠΈΠ»Π° установки ΠΈ Π²ΠΎΠ·ΠΌΠΎΠΆΠ½Ρ‹Π΅ трудности

ΠŸΡ€ΠΎΡ‚ΠΈΠ²ΠΎΡ‚ΡƒΠΌΠ°Π½Π½Ρ‹Π΅ Ρ„Π°Ρ€Ρ‹, ΠΏΡ€Π΅Π΄Π½Π°Π·Π½Π°Ρ‡Π΅Π½Ρ‹ для формирования плоского ΠΈ ΡˆΠΈΡ€ΠΎΠΊΠΎΠ³ΠΎ ΠΏΠΎΡ‚ΠΎΠΊΠ° свСта, Π½Π°ΠΏΡ€Π°Π²Π»Π΅Π½Π½ΠΎΠ³ΠΎ Π½Π°Π΄ ΠΏΠΎΠ²Π΅Ρ€Ρ…Π½ΠΎΡΡ‚ΡŒΡŽ Π΄ΠΎΡ€ΠΎΠ³ΠΈ. Π’Π°ΠΊΠΎΠ΅ Π½Π°ΠΏΡ€Π°Π²Π»Π΅Π½ΠΈΠ΅ ΠΏΡƒΡ‡ΠΊΠ° свСта позволяСт ΠΌΠΈΠ½ΠΈΠΌΠΈΠ·ΠΈΡ€ΠΎΠ²Π°Ρ‚ΡŒ освСщСниС объСма Ρ‚ΡƒΠΌΠ°Π½Π° ΠΏΠΎ Ρ‚ΠΎΠ»Ρ‰ΠΈΠ½Π΅ ΠΈ ΡƒΠ»ΡƒΡ‡ΡˆΠ°Π΅Ρ‚ Π²ΠΈΠ΄ΠΈΠΌΠΎΡΡ‚ΡŒ с мСста водитСля. НСсоблюдСниС ΠΏΡ€Π°Π²ΠΈΠ» ΠΈ схСмы ΠΏΠΎΠ΄ΠΊΠ»ΡŽΡ‡Π΅Π½ΠΈΡ ΠΏΡ€ΠΎΡ‚ΠΈΠ²ΠΎΡ‚ΡƒΠΌΠ°Π½Π½Ρ‹Ρ… Ρ„Π°Ρ€ ΠΏΡ€ΠΈ ΠΈΡ… ΡΠ°ΠΌΠΎΡΡ‚ΠΎΡΡ‚Π΅Π»ΡŒΠ½ΠΎΠΉ установкС ΠΌΠΎΠΆΠ΅Ρ‚ привСсти ΠΊ авариям.

НазначСниС ΠΈ прСимущСства ПВЀ

ΠŸΡ€ΠΎΡ‚ΠΈΠ²ΠΎΡ‚ΡƒΠΌΠ°Π½ΠΊΠΈ Π½ΡƒΠΆΠ½Ρ‹ для освСщСния Π΄ΠΎΡ€ΠΎΠΆΠ½ΠΎΠ³ΠΎ покрытия Π²ΠΎ врСмя доТдя, Ρ‚ΡƒΠΌΠ°Π½Π° ΠΈ Ρ‚. ΠΏ. Π”Ρ€ΡƒΠ³ΠΈΠ΅ Ρ†Π΅Π»ΠΈ примСнСния ПВЀ:

  • для подсвСтки участков ΠΏΡ€Π°Π²ΠΎΠ³ΠΎ края ΠΎΠ±ΠΎΡ‡ΠΈΠ½Ρ‹;
  • для подсвСтки участка, располоТСнного прямо ΠΏΠ΅Ρ€Π΅Π΄ ΠΊΠ°ΠΏΠΎΡ‚ΠΎΠΌ ΠΌΠ°ΡˆΠΈΠ½Ρ‹;
  • позволяСт ΠΏΠΎΠ»ΡƒΡ‡ΠΈΡ‚ΡŒ максимум Π²ΠΈΠ·ΡƒΠ°Π»ΡŒΠ½ΠΎΠΉ ΠΈΠ½Ρ„ΠΎΡ€ΠΌΠ°Ρ†ΠΈΠΈ ΠΎ состояния Π΄ΠΎΡ€ΠΎΠ³ΠΈ;
  • ΠΊΠ°ΠΊ Π΄Π½Π΅Π²Π½Ρ‹Π΅ Ρ…ΠΎΠ΄ΠΎΠ²Ρ‹Π΅ Ρ„Π°Ρ€Ρ‹.

ΠŸΡ€Π΅ΠΈΠΌΡƒΡ‰Π΅ΡΡ‚Π²Π° примСнСния ПВЀ:

  • ΡˆΠΈΡ€ΠΎΠΊΠΈΠΉ Ρ„ΡƒΠ½ΠΊΡ†ΠΈΠΎΠ½Π°Π» β€” ΠΈΡ… Π²ΠΊΠ»ΡŽΡ‡Π΅Π½ΠΈΠ΅ позволяСт Ρ€Π΅ΡˆΠΈΡ‚ΡŒ сразу нСсколько Π·Π°Π΄Π°Ρ‡, связанных с подсвСткой;
  • Π»ΡƒΡ‡ΡˆΠΈΠΉ просмотр Π³Π°Π±Π°Ρ€ΠΈΡ‚ΠΎΠ² Π°Π²Ρ‚ΠΎ Π΄Π°ΠΆΠ΅ Π½Π° большом ΡƒΠ΄Π°Π»Π΅Π½ΠΈΠΈ.

ΠŸΡ€Π°Π²ΠΈΠ»Π° установки ПВЀ зависят ΠΎΡ‚ ΠΈΡ… Ρ‚ΠΈΠΏΠ°. Π‘ΡƒΡ‰Π΅ΡΡ‚Π²ΡƒΡŽΡ‚ ΠΎΠΏΡ€Π΅Π΄Π΅Π»Π΅Π½Π½Ρ‹Π΅ схСмы ΠΏΠΎΠ΄ΠΊΠ»ΡŽΡ‡Π΅Π½ΠΈΡ, ΠΊΠΎΡ‚ΠΎΡ€Ρ‹Ρ… слСдуСт ΠΏΡ€ΠΈΠ΄Π΅Ρ€ΠΆΠΈΠ²Π°Ρ‚ΡŒΡΡ Π²ΠΎ врСмя ΠΌΠΎΠ½Ρ‚Π°ΠΆΠ°.

Достоинства Ρ„ΡƒΠ½ΠΊΡ†ΠΈΠΎΠ½Π°Π»ΡŒΠ½ΠΎΠ³ΠΎ ΠΎΡΠ²Π΅Ρ‚ΠΈΡ‚Π΅Π»ΡŒΠ½ΠΎΠ³ΠΎ оборудования

ΠŸΡ€ΠΎΡ‚ΠΈΠ²ΠΎΡ‚ΡƒΠΌΠ°Π½Π½Ρ‹Π΅ Ρ„Π°Ρ€Ρ‹ ΠΏΠΎΠ·Π²ΠΎΠ»ΡΡŽΡ‚ частично ΡΠ½ΡΡ‚ΡŒ Π½Π°Π³Ρ€ΡƒΠ·ΠΊΡƒ с Π³Π»Π°Π· водитСля Π²ΠΎ врСмя Π²Ρ‹Π½ΡƒΠΆΠ΄Π΅Π½Π½ΠΎΠΉ ΠΏΠΎΠ΅Π·Π΄ΠΊΠΈ Π² слоТных ΠΏΠΎΠ³ΠΎΠ΄Π½Ρ‹Ρ… условиях. Π‘ ΠΈΡ… ΠΏΠΎΠΌΠΎΡ‰ΡŒΡŽ удаСтся Π΄ΠΎΠ±ΠΈΡ‚ΡŒΡΡ Π»ΡƒΡ‡ΡˆΠ΅Π³ΠΎ освСщСния Π΄ΠΎΡ€ΠΎΠΆΠ½ΠΎΠ³ΠΎ ΠΏΠΎΠ»ΠΎΡ‚Π½Π°. НаибольшСС Π·Π½Π°Ρ‡Π΅Π½ΠΈΠ΅ Ρ„Π°Ρ€ кроСтся Π² Ρ‚ΠΎΡ‡Π½ΠΎΠΉ ΠΏΠΎΠ΄Π°Ρ‡Π΅ свСта, ΠΎΡ‚ Ρ‡Π΅Π³ΠΎ зависит освСщСниС Π΄ΠΎΡ€ΠΎΠ³ΠΈ.

ΠŸΡ€ΠΈΠ±ΠΎΡ€Ρ‹ с Ρ‚ΠΎΡ‡Π½ΠΎΠΉ Ρ€Π΅Π³ΡƒΠ»ΠΈΡ€ΠΎΠ²ΠΊΠΎΠΉ ΠΎΡΠ²Π΅Ρ‰Π°ΡŽΡ‚ Π΄ΠΎ 10 ΠΌΠ΅Ρ‚Ρ€ΠΎΠ² пространства ΠΏΠ΅Ρ€Π΅Π΄ Π°Π²Ρ‚ΠΎΠΌΠΎΠ±ΠΈΠ»Π΅ΠΌ, Ρ‡Π΅Π³ΠΎ Π²ΠΏΠΎΠ»Π½Π΅ достаточно для бСзопасного двиТСния Π² слоТных ΠΏΠΎΠ³ΠΎΠ΄Π½Ρ‹Ρ… условиях. РазумССтся, для этого понадобится ΠΏΡ€ΠΈΠ΄Π΅Ρ€ΠΆΠΈΠ²Π°Ρ‚ΡŒΡΡ ΠΎΠΏΡ€Π΅Π΄Π΅Π»Π΅Π½Π½ΠΎΠ³ΠΎ скоростного Ρ€Π΅ΠΆΠΈΠΌΠ°. Π Π΅Π³ΡƒΠ»ΠΈΡ€ΠΎΠ²Π°Π½ΠΈΠ΅ Ρ„Π°Ρ€ осущСствляСтся Π² процСссС установки оборудования. ΠšΠ°Ρ‡Π΅ΡΡ‚Π²ΠΎ освСщСния Π΄ΠΎΡ€ΠΎΠΆΠ½ΠΎΠ³ΠΎ ΠΏΠΎΠ»ΠΎΡ‚Π½Π° ΠΏΠΎΠ»Π½ΠΎΡΡ‚ΡŒΡŽ зависит ΠΎΡ‚ настройки ΡƒΠ³Π»Π° падСния свСтового ΠΏΠΎΡ‚ΠΎΠΊΠ°.

ВрСбования ΠΊ ΠΌΠΎΠ½Ρ‚Π°ΠΆΡƒ

Π‘ΠΎΠ»ΡŒΡˆΠΈΠ½ΡΡ‚Π²ΠΎ автомобилистов, ΠΏΡ€ΠΈΠ»Π΅ΠΆΠ½ΠΎ ΠΈΠ·ΡƒΡ‡Π°Π²ΡˆΠΈΡ… тСорСтичСский курс Π² Π°Π²Ρ‚ΠΎΡˆΠΊΠΎΠ»Π΅, Π·Π½Π°ΡŽΡ‚, Ρ‡Ρ‚ΠΎ нСльзя ΠΏΠΎ своСму ТСланию ΡΡ‚Π°Π²ΠΈΡ‚ΡŒ Π½Π° Π°Π²Ρ‚ΠΎ приспособлСния, Π½Π΅ прСдусмотрСнныС ΠΏΡ€ΠΎΠΈΠ·Π²ΠΎΠ΄ΠΈΡ‚Π΅Π»Π΅ΠΌ, Π½ΠΎ ΠΌΠΎΠ½Ρ‚Π°ΠΆ ΠΏΡ€ΠΎΡ‚ΠΈΠ²ΠΎΡ‚ΡƒΠΌΠ°Π½ΠΎΠΊ прописан Π² ΠŸΠ”Π”.

Π”Π°ΠΆΠ΅ Ссли Π°Π²Ρ‚ΠΎ ΠΎΡ‚ производитСля Π½Π΅ оснащСно ΠΏΡ€ΠΎΡ‚ΠΈΠ²ΠΎΡ‚ΡƒΠΌΠ°Π½ΠΊΠ°ΠΌΠΈ, ΠΊΠ°ΠΆΠ΄Ρ‹ΠΉ Π°Π²Ρ‚ΠΎΠ²Π»Π°Π΄Π΅Π»Π΅Ρ† ΠΌΠΎΠΆΠ΅Ρ‚ ΠΈΡ… ΡƒΡΡ‚Π°Π½ΠΎΠ²ΠΈΡ‚ΡŒ, вынСся ΠΎΡ‚Π΄Π΅Π»ΡŒΠ½ΠΎ ΠΈΠ»ΠΈ ΠΏΠ΅Ρ€Π΅Π΄Π΅Π»Π°Π² Π±Π°ΠΌΠΏΠ΅Ρ€. НуТно ΠΏΠΎΠΌΠ½ΠΈΡ‚ΡŒ, Ρ‡Ρ‚ΠΎ ΠΏΡ€ΠΈ ΠΏΠ΅Ρ€Π΅Π΄Π΅Π»ΠΊΠ΅ вносят ΡΠ΅Ρ€ΡŒΠ΅Π·Π½Ρ‹Π΅ измСнСния Π² ΠΊΠΎΠ½ΡΡ‚Ρ€ΡƒΠΊΡ†ΠΈΡŽ ΠΌΠ°ΡˆΠΈΠ½Ρ‹, Π° это ΡƒΠΆΠ΅ считаСтся частичным ΠΏΠ΅Ρ€Π΅ΠΎΠ±ΠΎΡ€ΡƒΠ΄ΠΎΠ²Π°Π½ΠΈΠ΅ΠΌ.

ΠŸΡ€Π°Π²ΠΈΠ»Π° установки:

  • Ρ€Π°Π·Ρ€Π΅ΡˆΠ°Π΅Ρ‚ΡΡ ΡƒΡΡ‚Π°Π½ΠΎΠ²ΠΈΡ‚ΡŒ Π΄Π²Π΅ ΠΏΡ€ΠΎΡ‚ΠΈΠ²ΠΎΡ‚ΡƒΠΌΠ°Π½Π½Ρ‹Π΅ Ρ„Π°Ρ€Ρ‹;
  • максимальноС расстояниС ΠΌΠ΅ΠΆΠ΄Ρƒ Ρ„Π°Ρ€ΠΎΠΉ ΠΈ Π±ΠΎΠΊΠΎΠ²ΠΎΠΉ ΠΏΠΎΠ²Π΅Ρ€Ρ…Π½ΠΎΡΡ‚ΡŒΡŽ β€” 0,4 ΠΌ;
  • минимальноС расстояниС ΠΎΡ‚ ПВЀ Π΄ΠΎ Π΄ΠΎΡ€ΠΎΠ³ΠΈ β€” 25 см.

ΠœΠΎΠ½Ρ‚Π°ΠΆ ПВЀ проводится Π² соотвСтствии с прСдусмотрСнными Π½ΠΎΡ€ΠΌΠ°ΠΌΠΈ.
На Π·Π°ΠΌΠ΅Ρ‚ΠΊΡƒ!
ПВЀ, Π²ΠΎΠ·ΠΌΠΎΠΆΠ½ΠΎ, понадобится Ρ€Π΅Π³ΡƒΠ»ΠΈΡ€ΠΎΠ²ΠΊΠ°, Ρ‚. ΠΊ. ΡƒΠ³ΠΎΠ» падСния свСтопотока Π½Π° Π΄ΠΎΡ€ΠΎΠΆΠ½ΠΎΠ΅ ΠΏΠΎΠΊΡ€Ρ‹Ρ‚ΠΈΠ΅ Π΄ΠΎΠ»ΠΆΠ΅Π½ ΡΠΎΠΎΡ‚Π²Π΅Ρ‚ΡΡ‚Π²ΠΎΠ²Π°Ρ‚ΡŒ ΠΏΡ€Π°Π²ΠΈΠ»Π°ΠΌ.

Как выглядит ΠΈΠ½Π΄ΠΈΠΊΠ°Ρ‚ΠΎΡ€ Π²ΠΊΠ»ΡŽΡ‡Π΅Π½ΠΈΡ ΠΏΠ΅Ρ€Π΅Π΄Π½ΠΈΡ… ΠΏΡ€ΠΎΡ‚ΠΈΠ²ΠΎΡ‚ΡƒΠΌΠ°Π½ΠΎΠΊ

Π’Π½Π΅ΡˆΠ½Π΅ ΠΏΠΈΠΊΡ‚ΠΎΠ³Ρ€Π°ΠΌΠΌΠ° ΠΏΠ΅Ρ€Π΅Π΄Π½ΠΈΡ… ΠΏΡ€ΠΎΡ‚ΠΈΠ²ΠΎΡ‚ΡƒΠΌΠ°Π½Π½Ρ‹Ρ… Ρ„Π°Ρ€ выглядит ΠΊΠ°ΠΊ ΠΏΠΎΠ»ΡƒΠΊΡ€ΡƒΠ³ (Ρ„Π°Ρ€Π°), рядом с ΠΊΠΎΡ‚ΠΎΡ€Ρ‹ΠΌ нарисованы косыС ΠΏΠ΅Ρ€Π΅Ρ‡Ρ‘Ρ€ΠΊΠ½ΡƒΡ‚Ρ‹Π΅ Π»ΠΈΠ½ΠΈΠΈ.

Π‘Ρ‚ΠΎΠΈΡ‚ ΠΎΡ‚ΠΌΠ΅Ρ‚ΠΈΡ‚ΡŒ, Ρ‡Ρ‚ΠΎ Π² ΠΏΠ΅Ρ€Π΅Π΄Π½Π΅ΠΉ ΠΎΠΏΡ‚ΠΈΠΊΠ΅ ΠΈΡΠΏΠΎΠ»ΡŒΠ·ΡƒΠ΅Ρ‚ΡΡ ΠΈΠΌΠ΅Π½Π½ΠΎ свСт ΠΏΠΎΠ΄ Π½Π°ΠΊΠ»ΠΎΠ½ΠΎΠΌ, ΠΊΠ°ΠΊ ΠΎΠ½ ΠΈ нарисован Π½Π° Π·Π½Π°Ρ‡ΠΊΠ΅ Π²ΠΊΠ»ΡŽΡ‡Π΅Π½ΠΈΡ. Π­Ρ‚ΠΎ связано с Π½ΠΎΡ€ΠΌΠ°Ρ‚ΠΈΠ²Π°ΠΌΠΈ ΠŸΠ”Π”.

Π’ частности, Π² Π½ΠΎΡ€ΠΌΠ΅ β„– 19.4 ΠŸΠ”Π” сказано, Ρ‡Ρ‚ΠΎ ΠΏΡ€ΠΎΡ‚ΠΈΠ²ΠΎΡ‚ΡƒΠΌΠ°Π½Π½Ρ‹Π΅ Ρ„Π°Ρ€Ρ‹ Π΄ΠΎΠ»ΠΆΠ½Ρ‹ ΠΈΡΠΏΠΎΠ»ΡŒΠ·ΠΎΠ²Π°Ρ‚ΡŒΡΡ Π² Π½ΠΎΡ‡Π½ΠΎΠ΅ врСмя суток ΠΈΠ»ΠΈ Π² сумСрках, совмСстно с Π³ΠΎΠ»ΠΎΠ²Π½ΠΎΠΉ ΠΎΠΏΡ‚ΠΈΠΊΠΎΠΉ. Π˜Ρ… Ρ€Π°Π·Ρ€Π΅ΡˆΠ΅Π½ΠΎ Π²ΠΊΠ»ΡŽΡ‡Π°Ρ‚ΡŒ Ρ‚ΠΎΠ»ΡŒΠΊΠΎ Π½Π° нСосвСщённых участках Π΄ΠΎΡ€ΠΎΠ³ΠΈ ΠΈ Π² слоТных климатичСских условиях с ΠΎΠ³Ρ€Π°Π½ΠΈΡ‡Π΅Π½Π½ΠΎΠΉ Π²ΠΈΠ΄ΠΈΠΌΠΎΡΡ‚ΡŒΡŽ. Π’Π°ΠΊΠΆΠ΅ ΠΈΡ… ΠΌΠΎΠΆΠ½ΠΎ ΠΈΡΠΏΠΎΠ»ΡŒΠ·ΠΎΠ²Π°Ρ‚ΡŒ Π΄Π½Ρ‘ΠΌ, вмСсто Ρ…ΠΎΠ΄ΠΎΠ²Ρ‹Ρ… ΠΎΠ³Π½Π΅ΠΉ.

ΠŸΡ€ΠΈ этом ΡΡƒΡ‰Π΅ΡΡ‚Π²ΡƒΡŽΡ‚ тСхничСскиС Π½ΠΎΡ€ΠΌΠ°Ρ‚ΠΈΠ²Ρ‹, согласно ΠΊΠΎΡ‚ΠΎΡ€Ρ‹ΠΌ Π²Π΅Ρ€Ρ‚ΠΈΠΊΠ°Π»ΡŒΠ½Ρ‹ΠΉ ΡƒΠ³ΠΎΠ» Π»ΡƒΡ‡Π° ΠΎΡ‚ ΠΏΡ€ΠΎΡ‚ΠΈΠ²ΠΎΡ‚ΡƒΠΌΠ°Π½Π½Ρ‹Ρ… Ρ„Π°Ρ€ Π΄ΠΎΠ»ΠΆΠ΅Π½ Π½Π°Ρ…ΠΎΠ΄ΠΈΡ‚ΡŒΡΡ Π² ΠΏΡ€Π΅Π΄Π΅Π»Π°Ρ… ΠΎΡ‚ -10 Π΄ΠΎ +15 градусов. Π‘Π°ΠΌΠΈ ΠΏΡ€ΠΎΡ‚ΠΈΠ²ΠΎΡ‚ΡƒΠΌΠ°Π½ΠΊΠΈ Π΄ΠΎΠ»ΠΆΠ½Ρ‹ Π±Ρ‹Ρ‚ΡŒ Π½Π°ΠΏΡ€Π°Π²Π»Π΅Π½Ρ‹ большС Π² ΠΏΡ€Π°Π²ΡƒΡŽ сторону, Ρ‡Ρ‚ΠΎΠ±Ρ‹ свСт ΠΎΡ‚ Π½ΠΈΡ… ΠΌΠΎΠ³ Β«Π²Ρ‹Ρ…Π²Π°Ρ‚ΠΈΡ‚ΡŒΒ» ΠΈΠ΄ΡƒΡ‰Π΅Π³ΠΎ ΠΏΠ΅ΡˆΠ΅Ρ…ΠΎΠ΄Π°, ΠΈ Π½Π΅ слСпил ΠΌΠ°ΡˆΠΈΠ½Ρ‹, Π΄Π²ΠΈΠ³Π°ΡŽΡ‰ΠΈΠ΅ΡΡ ΠΏΠΎ встрСчной полосС.

Π’Ρ‹Π±ΠΎΡ€ ПВЀ

Π‘Π½Π°Ρ‡Π°Π»Π° ΠΎΠΏΡ€Π΅Π΄Π΅Π»ΡΡŽΡ‚ мСсто ΠΈΡ… ΠΌΠΎΠ½Ρ‚Π°ΠΆΠ° Π½Π° Π°Π²Ρ‚ΠΎ. Часто Π² ΠΈΠ½ΠΎΠΌΠ°Ρ€ΠΊΠ°Ρ… Π½Π° Π±Π°ΠΌΠΏΠ΅Ρ€Π΅ Π΅ΡΡ‚ΡŒ ниши ΠΏΠΎΠ΄ установку Π΄ΠΎΠΏΠΎΠ»Π½ΠΈΡ‚Π΅Π»ΡŒΠ½Ρ‹Ρ… ΠΎΠ³Π½Π΅ΠΉ. Π’ΠΎΠ³Π΄Π° ΠΏΡ€ΠΎΡ‚ΠΈΠ²ΠΎΡ‚ΡƒΠΌΠ°Π½ΠΊΠΈ ΠΏΠΎΠ΄Π±ΠΈΡ€Π°ΡŽΡ‚ Ρ‚Π°ΠΊ, Ρ‡Ρ‚ΠΎΠ±Ρ‹ ΠΈΡ… Ρ„ΠΎΡ€ΠΌΠ° ΠΈ Ρ€Π°Π·ΠΌΠ΅Ρ€Ρ‹ соотвСтствовали этим нишам Π² Π±Π°ΠΌΠΏΠ΅Ρ€Π΅.

Π’Π°ΠΆΠ½ΠΎ! На качСство выполнСния Ρ„ΡƒΠ½ΠΊΡ†ΠΈΠΉ ПВЀ практичСски Π½Π΅ влияСт ΠΈΡ… ΠΌΠΎΡ‰Π½ΠΎΡΡ‚ΡŒ. ΠŸΠΎΡΡ‚ΠΎΠΌΡƒ цСлСсообразно Π²Ρ‹Π±ΠΈΡ€Π°Ρ‚ΡŒ источники свСта ΠΌΠ°Π»ΠΎΠΉ мощности β€” свСтодиодныС Π»Π°ΠΌΠΏΡ‹.

На Π²Π½Π΅Π΄ΠΎΡ€ΠΎΠΆΠ½ΠΈΠΊΠ°Ρ… ПВЀ ΡƒΡΡ‚Π°Π½Π°Π²Π»ΠΈΠ²Π°ΡŽΡ‚ Π½Π° ΡƒΡ€ΠΎΠ²Π½Π΅ ΠΎΠ±Ρ‹Ρ‡Π½Ρ‹Ρ… Ρ„Π°Ρ€ Π»ΠΈΠ±ΠΎ Π½Π° ΠΊΡ€Ρ‹ΡˆΠ΅ ΠΌΠ°ΡˆΠΈΠ½Ρ‹. Π’ этой ситуации походят ПВЀ с Π»ΡŽΠ±Ρ‹ΠΌΠΈ Ρ€Π°Π·ΠΌΠ΅Ρ€Π°ΠΌΠΈ ΠΈ Ρ„ΠΎΡ€ΠΌΠΎΠΉ. Π“Π»Π°Π²Π½ΠΎΠ΅, Ρ‡Ρ‚ΠΎΠ±Ρ‹ ΠΎΠ½ΠΈ ΠΊΠΎΠΌΠΏΠ»Π΅ΠΊΡ‚ΠΎΠ²Π°Π»ΠΈΡΡŒ ΠΊΡ€ΠΎΠ½ΡˆΡ‚Π΅ΠΉΠ½Π°ΠΌΠΈ, ΠΎΠ±Π΅ΡΠΏΠ΅Ρ‡ΠΈΠ²Π°ΡŽΡ‰ΠΈΠΌΠΈ Π½Π°Π΄Π΅ΠΆΠ½ΠΎΠ΅ ΠΊΡ€Π΅ΠΏΠ»Π΅Π½ΠΈΠ΅.

Π“Π»Π°Π²Π½ΠΎΠ΅ ΠΎΡ‚Π»ΠΈΡ‡ΠΈΠ΅ ΠΏΡ€ΠΎΡ‚ΠΈΠ²ΠΎΡ‚ΡƒΠΌΠ°Π½ΠΎΠΊ β€” ΠΈΡΠΏΠΎΠ»ΡŒΠ·ΡƒΠ΅ΠΌΡ‹Π΅ Π² Π½ΠΈΡ… Π»Π°ΠΌΠΏΡ‹. Π Π°Π·Π»ΠΈΡ‡Π°ΡŽΡ‚ Ρ‚Π°ΠΊΠΈΠ΅ Π²ΠΈΠ΄Ρ‹ ПВЀ:

  1. Π“Π°Π»ΠΎΠ³Π΅Π½Π½Ρ‹Π΅. Π‘Π°ΠΌΡ‹ΠΉ доступный Π²Π°Ρ€ΠΈΠ°Π½Ρ‚. ΠœΠΈΠ½ΡƒΡ β€” нСвысокий рСсурс ΠΏΠΎ сравнСнии с Π΄Ρ€ΡƒΠ³ΠΈΠΌΠΈ Π²ΠΈΠ΄Π°ΠΌΠΈ Π»Π°ΠΌΠΏ. ΠŸΡ€Π΅ΠΆΠ΄Π΅ Ρ‡Π΅ΠΌ ΠΊΡƒΠΏΠΈΡ‚ΡŒ Ρ‚Π°ΠΊΠΈΠ΅ Ρ„Π°Ρ€Ρ‹, ΡƒΠ±Π΅Π΄ΠΈΡ‚Π΅ΡΡŒ, Ρ‡Ρ‚ΠΎ Π²Ρ‹ смоТСтС ΠΈΡ… Π±Π΅Π· Ρ‚Ρ€ΡƒΠ΄Π° ΠΏΠΎΠΌΠ΅Π½ΡΡ‚ΡŒ, ΠΊΠΎΠ³Π΄Π° ΠΎΠ½ΠΈ Π²Ρ‹ΠΉΠ΄ΡƒΡ‚ ΠΈΠ· строя.
  2. Π‘Π²Π΅Ρ‚ΠΎΠ΄ΠΈΠΎΠ΄Π½Ρ‹Π΅. Π˜Ρ… Π³Π»Π°Π²Π½Ρ‹Π΅ прСимущСства β€” Π΄ΠΎΠ»Π³ΠΎΠ²Π΅Ρ‡Π½ΠΎΡΡ‚ΡŒ ΠΈ ΡΠΊΠΎΠ½ΠΎΠΌΠΈΡ‡Π½ΠΎΡΡ‚ΡŒ. ΠœΠΈΠ½ΡƒΡ β€” Ρ†Π΅Π½Π°. Если свСтодиодныС Ρ„Π°Ρ€Ρ‹ установлСны Π½Π΅ΠΏΡ€Π°Π²ΠΈΠ»ΡŒΠ½ΠΎ ΠΈΠ»ΠΈ ΠΎΠ½ΠΈ ΠΏΠ»ΠΎΡ…ΠΎ Π²Π΅Π½Ρ‚ΠΈΠ»ΠΈΡ€ΡƒΡŽΡ‚ΡΡ, срок ΠΈΡ… эксплуатации сокращаСтся.
  3. ΠšΡΠ΅Π½ΠΎΠ½ΠΎΠ²Ρ‹Π΅. ΠŸΠ»ΡŽΡΡ‹ β€” Ρ…ΠΎΡ€ΠΎΡˆΠΈΠ΅ тСхничСскиС характСристики. Π’Π°ΠΊΠΈΠ΅ ΠΌΠΎΠ΄Π΅Π»ΠΈ Π΄ΠΎΠ»Π³ΠΎΠ²Π΅Ρ‡Π½Ρ‹ ΠΈ эффСктивны. ΠœΠΈΠ½ΡƒΡ β€” высокая ΡΡ‚ΠΎΠΈΠΌΠΎΡΡ‚ΡŒ.

Π“Π΄Π΅ ΠΌΠΎΠΆΠ½ΠΎ Π²ΡΡ‚Ρ€Π΅Ρ‚ΠΈΡ‚ΡŒ обозначСния

Π‘Ρ‚ΠΎΠΈΡ‚ Π½Π°Ρ‡Π°Ρ‚ΡŒ с Ρ‚ΠΎΠ³ΠΎ, Ρ‡Ρ‚ΠΎ ΠΏΠΎΠ΄ΠΎΠ±Π½ΡƒΡŽ ΠΏΠΈΠΊΡ‚ΠΎΠ³Ρ€Π°ΠΌΠΌΡƒ ΠΌΠΎΠΆΠ½ΠΎ Π²ΡΡ‚Ρ€Π΅Ρ‚ΠΈΡ‚ΡŒ Π½Π΅ Π²ΠΎ всСх автомобилях. НапримСр, ΠΎΠ½Π° практичСски Π½Π΅ встрСчаСтся Π² Π±ΡŽΠ΄ΠΆΠ΅Ρ‚Π½Ρ‹Ρ… ΠΌΠ°ΡˆΠΈΠ½Π°Ρ…. ЗабСгая Π²ΠΏΠ΅Ρ€Ρ‘Π΄, ΡƒΡ‚ΠΎΡ‡Π½ΠΈΠΌ, Ρ‡Ρ‚ΠΎ эти символы ΠΎΠ±ΠΎΠ·Π½Π°Ρ‡Π°ΡŽΡ‚ Ρ€Π°Π±ΠΎΡ‚Ρƒ ΠΈΠ»ΠΈ Π²ΠΊΠ»ΡŽΡ‡Π΅Π½ΠΈΠ΅ ΠΏΡ€ΠΎΡ‚ΠΈΠ²ΠΎΡ‚ΡƒΠΌΠ°Π½Π½Ρ‹Ρ… Ρ„Π°Ρ€. Как ΠΏΠ΅Ρ€Π΅Π΄Π½ΠΈΡ…, Ρ‚Π°ΠΊ ΠΈ Π·Π°Π΄Π½ΠΈΡ….

Π˜Ρ… ΠΎΠ±ΠΎΠ·Π½Π°Ρ‡Π°ΡŽΡ‚ нСсколькими способами:

  • Π² Π²ΠΈΠ΄Π΅ ΠΏΠΎΠ»ΡƒΠΊΡ€ΡƒΠ³Π° с ΠΏΠ΅Ρ€Π΅Ρ‡Ρ‘Ρ€ΠΊΠ½ΡƒΡ‚Ρ‹ΠΌΠΈ прямыми линиями;
  • Π² Π²ΠΈΠ΄Π΅ ΠΏΠΎΠ»ΡƒΠΊΡ€ΡƒΠ³Π° с ΠΏΠ΅Ρ€Π΅Ρ‡Ρ‘Ρ€ΠΊΠ½ΡƒΡ‚Ρ‹ΠΌΠΈ косыми линиями;

Π§Π°Ρ‰Π΅ всСго ΠΏΠΎΠ΄ΠΎΠ±Π½ΡƒΡŽ ΠΎΠΏΡ†ΠΈΡŽ ΠΌΠΎΠΆΠ½ΠΎ Π²ΡΡ‚Ρ€Π΅Ρ‚ΠΈΡ‚ΡŒ Π½Π° ΠΌΠ°ΡˆΠΈΠ½Π°Ρ… Ρ‚ΠΎΠΏΠΎΠ²Ρ‹Ρ… ΠΈΠ»ΠΈ Π»ΡŽΠΊΡΠΎΠ²Ρ‹Ρ… вСрсий. Или Π² автомобилях, ΠΏΠΎΠ΄Π³ΠΎΡ‚ΠΎΠ²Π»Π΅Π½Π½Ρ‹Ρ… ΠΊ Π΅Π·Π΄Π΅ ΠΏΠΎ Π±Π΅Π·Π΄ΠΎΡ€ΠΎΠΆΡŒΡŽ β€” Π΄ΠΆΠΈΠΏΠ°Ρ… ΠΈ кроссовСрах.

ΠžΠ±ΠΎΠ·Π½Π°Ρ‡Π΅Π½ΠΈΠ΅ ΠΏΡ€ΠΎΡ‚ΠΈΠ²ΠΎΡ‚ΡƒΠΌΠ°Π½Π½Ρ‹Ρ… Ρ„Π°Ρ€ ΠΌΠΎΠΆΠ½ΠΎ ΡƒΠ²ΠΈΠ΄Π΅Ρ‚ΡŒ Π² качСствС свСтового сигнализатора Π½Π° ΠΏΡ€ΠΈΠ±ΠΎΡ€Π½ΠΎΠΉ доскС. Π˜Ρ… ΠΆΠ΅ наносят Π½Π° ΠΊΠ½ΠΎΠΏΠΊΠΈ управлСния, ΠΊΠΎΡ‚ΠΎΡ€Ρ‹Π΅ ΠΌΠΎΠ³ΡƒΡ‚ Π½Π°Ρ…ΠΎΠ΄ΠΈΡ‚ΡŒΡΡ Π½Π°:

  1. ΠŸΠΎΠ΄Ρ€ΡƒΠ»Π΅Π²ΠΎΠΌ ΠΏΠ΅Ρ€Π΅ΠΊΠ»ΡŽΡ‡Π°Ρ‚Π΅Π»Π΅. ΠšΠ½ΠΎΠΏΠΊΡƒ с ΠΏΠΈΠΊΡ‚ΠΎΠ³Ρ€Π°ΠΌΠΌΠΎΠΉ ΠΌΠΎΠ³ΡƒΡ‚ вынСсти Π½Π° Ρ€Ρ‹Ρ‡Π°Π³ управлСния свСтом, Ρ‡Ρ‚ΠΎΠ±Ρ‹ Π²ΠΎΠ΄ΠΈΡ‚Π΅Π»ΡŽ Π±Ρ‹Π»ΠΎ ΡƒΠ΄ΠΎΠ±Π½ΠΎ Π²ΠΊΠ»ΡŽΡ‡Π°Ρ‚ΡŒ ΠΈ Π²Ρ‹ΠΊΠ»ΡŽΡ‡Π°Ρ‚ΡŒ всю ΠΎΠΏΡ‚ΠΈΠΊΡƒ ΠΌΠ°ΡˆΠΈΠ½Ρ‹ ΠΈΠ· ΠΎΠ΄Π½ΠΎΠ³ΠΎ мСста.
  2. На Π±Π»ΠΎΠΊΠ΅ управлСния свСтом Π² области Ρ‚ΠΎΡ€ΠΏΠ΅Π΄ΠΎ. Если Π² машинС стоят Ρ‚ΠΎΠ»ΡŒΠΊΠΎ ΠΏΠ΅Ρ€Π΅Π΄Π½ΠΈΠ΅ ΠΈΠ»ΠΈ Π·Π°Π΄Π½ΠΈΠ΅ ΠΏΡ€ΠΎΡ‚ΠΈΠ²ΠΎΡ‚ΡƒΠΌΠ°Π½Π½Ρ‹Π΅ Ρ„ΠΎΠ½Π°Ρ€ΠΈ, Ρ‚ΠΎ Π΅Π΄ΠΈΠ½ΠΈΡ‡Π½ΡƒΡŽ ΠΊΠ½ΠΎΠΏΠΊΡƒ выносят Π½Π΅Π΄Π°Π»Π΅ΠΊΠΎ ΠΎΡ‚ водитСля. НапримСр, справа ΠΈΠ»ΠΈ слСва ΠΎΡ‚ Ρ€ΡƒΠ»Π΅Π²ΠΎΠΉ ΠΊΠΎΠ»ΠΎΠ½ΠΊΠΈ.

Иногда ΠΊΠ½ΠΎΠΏΠΊΠΈ ΡƒΡΡ‚Π°Π½Π°Π²Π»ΠΈΠ²Π°ΡŽΡ‚ Π½Π° Ρ†Π΅Π½Ρ‚Ρ€Π°Π»ΡŒΠ½ΡƒΡŽ панСль ΠΈΠ»ΠΈ Ρ‚ΠΎΠ½Π½Π΅Π»ΡŒ. Однако Ρ‡Π°Ρ‰Π΅ всСго ΠΈΡ… ΠΌΠΎΠΆΠ½ΠΎ ΡƒΠ²ΠΈΠ΄Π΅Ρ‚ΡŒ ΠΈΠΌΠ΅Π½Π½ΠΎ Π½Π° ΠΏΠΎΠ΄Ρ€ΡƒΠ»Π΅Π²ΠΎΠΌ лСпСсткС.

Π’Π°Ρ€ΠΈΠ°Π½Ρ‚Ρ‹ ΠΈ схСмы ΠΏΠΎΠ΄ΠΊΠ»ΡŽΡ‡Π΅Π½ΠΈΡ

Если Π² Π°Π²Ρ‚ΠΎ Π½Π΅Ρ‚ ΠΏΡ€ΠΎΠ²ΠΎΠ΄ΠΊΠΈ для ПВЀ, придСтся своими силами ΠΏΡ€ΠΎΠΊΠ»Π°Π΄Ρ‹Π²Π°Ρ‚ΡŒ Π½Π΅ΠΎΠ±Ρ…ΠΎΠ΄ΠΈΠΌΡ‹Π΅ ΠΏΡ€ΠΎΠ²ΠΎΠ΄Π°. Π•ΡΡ‚ΡŒ Π΄Π²Π° Π²Π°Ρ€ΠΈΠ°Π½Ρ‚Π° β€” Ρ‡Π΅Ρ€Π΅Π· Ρ€Π΅Π»Π΅ ΠΈ Ρ‡Π΅Ρ€Π΅Π· ΠΊΠ½ΠΎΠΏΠΊΡƒ.

Π§Π΅Ρ€Π΅Π· Ρ€Π΅Π»Π΅

Π§Ρ‚ΠΎΠ±Ρ‹ ΠΏΠΎΠ΄ΠΊΠ»ΡŽΡ‡ΠΈΡ‚ΡŒ ΠΏΡ€ΠΎΡ‚ΠΈΠ²ΠΎΡ‚ΡƒΠΌΠ°Π½ΠΊΠΈ Ρ‡Π΅Ρ€Π΅Π· Ρ€Π΅Π»Π΅, Π½Π΅ΠΎΠ±Ρ…ΠΎΠ΄ΠΈΠΌΠΎ Π·Π°ΠΏΠ°ΡΡ‚ΠΈΡΡŒ Ρ€Π΅ΠΆΡƒΡ‰ΠΈΠΌΠΈ инструмСнтами ΠΈ ΠΊΠ»Π΅ΠΌΠΌΠ½ΠΈΠΊΠΎΠΌ. Π•Ρ‰Π΅ понадобится Π³ΠΎΡ„Ρ€Π°, тСрмоусадка ΠΈ ΠΈΠ·ΠΎΠ»Π΅Π½Ρ‚Π°.

ΠŸΡ€Π°Π²ΠΈΠ»Π° установки Ρ‡Π΅Ρ€Π΅Π· Ρ€Π΅Π»Π΅:

  1. ΠžΡ‚Ρ‹Ρ‰ΠΈΡ‚Π΅ Π² Π°Π²Ρ‚ΠΎΠΌΠΎΠ±ΠΈΠ»Π΅ мСсто для ΠΌΠΎΠ½Ρ‚Π°ΠΆΠ° Ρ€Π΅Π»Π΅. ΠŸΡ€ΠΈΠΊΡ€Π΅ΠΏΠΈΡ‚Π΅ Π΅Π³ΠΎ Ρ‚Π°ΠΊ, Ρ‡Ρ‚ΠΎΠ±Ρ‹ ΠΊ Π½Π΅ΠΌΡƒ ΠΌΠΎΠΆΠ½ΠΎ Π±Ρ‹Π»ΠΎ Π»Π΅Π³ΠΊΠΎ Π΄ΠΎΠ±Ρ€Π°Ρ‚ΡŒΡΡ Π² случаС Π·Π°ΠΌΠ΅Π½Ρ‹ ΠΈΠ»ΠΈ Ρ€Π΅ΠΌΠΎΠ½Ρ‚Π°.
  2. НайдитС мСсто ΠΏΠΎΠ΄ ΠΊΠ½ΠΎΠΏΠΊΡƒ.
  3. ΠŸΡ€ΠΈΠ³ΠΎΡ‚ΠΎΠ²ΡŒΡ‚Π΅ ΠΏΡ€ΠΎΠ²ΠΎΠ΄Π° питания. Π”Π»ΠΈΠ½Π° Π΄ΠΎΠ»ΠΆΠ½Π° Π±Ρ‹Ρ‚ΡŒ Ρ‚Π°ΠΊΠΎΠΉ, Ρ‡Ρ‚ΠΎΠ±Ρ‹ ΠΏΡ€ΠΎΠ²ΠΎΠ΄Π° ΠΏΡ€ΠΎΠΊΠ»Π°Π΄Ρ‹Π²Π°Π»ΠΈΡΡŒ свободно, Π±Π΅Π· натяТСния.
  4. ΠŸΠΎΡΡ‚Π°Π²ΡŒΡ‚Π΅ ΠΏΡ€Π΅Π΄ΠΎΡ…Ρ€Π°Π½ΠΈΡ‚Π΅Π»ΡŒ Π½Π° 10 А. ΠžΡ‚ ΠΊΠΎΠ½Ρ‚Π°ΠΊΡ‚Π° β„– 30 ΠΏΡ€ΠΎΠ»ΠΎΠΆΠΈΡ‚Π΅ Π³ΠΎΡ‚ΠΎΠ²Ρ‹ΠΉ ΠΏΡ€ΠΎΠ²ΠΎΠ΄ ΠΊ ΠΊΠ»Π΅ΠΌΠΌΠ΅ Β«+Β» аккумулятора.
  5. ΠžΡ‚ ΠΊΠΎΠ½Ρ‚Π°ΠΊΡ‚Π° β„– 85 ΠΎΡ‚Π²Π΅Π΄ΠΈΡ‚Π΅ ΠΏΡ€ΠΎΠ²ΠΎΠ΄ ΠΊ ΠΊΠ½ΠΎΠΏΠΊΠ΅ Π²ΠΊΠ»ΡŽΡ‡Π΅Π½ΠΈΡ.
  6. ΠŸΠΎΠ΄ΡΠΎΠ΅Π΄ΠΈΠ½ΠΈΡ‚Π΅ массу с Ρ€Π΅Π»Π΅ с ΠΏΠΎΠΌΠΎΡ‰ΡŒΡŽ ΠΊΠΎΠ½Ρ‚Π°ΠΊΡ‚Π° β„– 86.
  7. ΠŸΠΎΡΡ‚Π°Π²ΡŒΡ‚Π΅ ПВЀ Π½Π° Π±Π°ΠΌΠΏΠ΅Ρ€.
  8. ΠŸΡ€ΠΈΡΠΎΠ΅Π΄ΠΈΠ½ΠΈΡ‚Π΅ ΠΏΡ€ΠΎΠ²ΠΎΠ΄ «–» с ΠΊΡƒΠ·ΠΎΠ²Π° ΠΈ Β«+Β» ΠΊΠΎΠ½Ρ‚Π°ΠΊΡ‚Π° β„– 87.
  9. Π’ΠΊΠ»ΡŽΡ‡ΠΈΡ‚Π΅ Ρ„Π°Ρ€Ρ‹ ΠΈ ΠΏΡ€ΠΎΠ²Π΅Ρ€ΡŒΡ‚Π΅ ΠΈΡ… Ρ€Π°Π±ΠΎΡ‚Ρƒ.

На Π·Π°ΠΌΠ΅Ρ‚ΠΊΡƒ! НС рСкомСндуСтся ΠΏΠΎΠ΄ΠΊΠ»ΡŽΡ‡Π°Ρ‚ΡŒ ПВЀ Π°Π²Ρ‚ΠΎΠ½ΠΎΠΌΠ½ΠΎ. Они посадят аккумулятор Π² считанныС Π΄Π½ΠΈ.

Π§Π΅Ρ€Π΅Π· ΠΊΠ½ΠΎΠΏΠΊΡƒ

Если ПВЀ ΠΏΠΎΠ΄ΠΊΠ»ΡŽΡ‡Π°Π΅Ρ‚ΡΡ Ρ‡Π΅Ρ€Π΅Π· ΠΊΠ½ΠΎΠΏΠΊΡƒ, достаточно любого ΠΏΡ€ΠΎΠ²ΠΎΠ΄Π° с напряТСниСм. Как ΠΏΡ€Π°Π²ΠΈΠ»ΠΎ, ΠΏΠΈΡ‚Π°Π½ΠΈΠ΅ Π±Π΅Ρ€ΡƒΡ‚ с Β«+Β» Π³Π°Π±Π°Ρ€ΠΈΡ‚ΠΎΠ² ΠΈΠ»ΠΈ с ΠΏΡ€ΠΎΠ²ΠΎΠ΄ΠΊΠΈ систСмы заТигания.

По красному ΠΏΡ€ΠΎΠ²ΠΎΠ΄Ρƒ подаСтся Β«+Β», ΠΏΠΎ Ρ‡Π΅Ρ€Π½ΠΎΠΌΡƒ β€” масса, подсоСдинСнная ΠΊ ΠΊΡƒΠ·ΠΎΠ²Ρƒ. НайдитС Π½Π° ΠΏΠ°Π½Π΅Π»ΠΈ мСсто, ΠΊ ΠΊΠΎΡ‚ΠΎΡ€ΠΎΠΌΡƒ Π±Ρ‹ Π»Π΅Π³ΠΊΠΎ Π΄ΠΎΡ‚ΡΠ³ΠΈΠ²Π°Π»Π°ΡΡŒ Ρ€ΡƒΠΊΠ°, Ρ‚Π°ΠΌ ΠΈ ΠΏΠΎΡΡ‚Π°Π²ΡŒΡ‚Π΅ ΠΊΠ½ΠΎΠΏΠΊΡƒ.

Π’Π°ΠΊΡƒΡŽ схСму ΠΏΠΎΠ΄ΠΊΠ»ΡŽΡ‡Π΅Π½ΠΈΡ для ПВЀ рСкомСндуСтся Π²Ρ‹Π±ΠΈΡ€Π°Ρ‚ΡŒ автомобилистам, ΠΊΠΎΡ‚ΠΎΡ€Ρ‹Π΅ склонны Π·Π°Π±Ρ‹Π²Π°Ρ‚ΡŒ Π²Ρ‹ΠΊΠ»ΡŽΡ‡Π°Ρ‚ΡŒ ΠΈΡ… ΠΏΡ€ΠΈ остановкС Π°Π²Ρ‚ΠΎ. Они Π²Ρ‹ΠΊΠ»ΡŽΡ‡Π°ΡŽΡ‚ΡΡ, ΠΊΠΎΠ³Π΄Π° Π²ΠΎΠ΄ΠΈΡ‚Π΅Π»ΡŒ, Π·Π°ΠΊΠΎΠ½Ρ‡ΠΈΠ² Π΄Π²ΠΈΠΆΠ΅Π½ΠΈΠ΅, достаСт ΠΊΠ»ΡŽΡ‡ заТигания.

Π‘Ρ…Π΅ΠΌΡ‹ ΠΏΠΎΠ΄ΠΊΠ»ΡŽΡ‡Π΅Π½ΠΈΡ

Π‘Ρ…Π΅ΠΌΠ° ΠΏΠΎΠ΄ΠΊΠ»ΡŽΡ‡Π΅Π½ΠΈΡ ΠΏΡ€ΠΎΡ‚ΠΈΠ²ΠΎΡ‚ΡƒΠΌΠ°Π½Π½Ρ‹Ρ… Ρ„Π°Ρ€ выбираСтся Π°Π²Ρ‚ΠΎΠ»ΡŽΠ±ΠΈΡ‚Π΅Π»Π΅ΠΌ Π½Π° своС усмотрСниС. Π•ΡΡ‚ΡŒ Π΄Π²Π° Π²Π°Ρ€ΠΈΠ°Π½Ρ‚Π° β€” ΠΏΠΎΡΠ»Π΅Π΄ΠΎΠ²Π°Ρ‚Π΅Π»ΡŒΠ½ΠΎΠ΅ ΠΈΠ»ΠΈ ΠΏΠ°Ρ€Π°Π»Π»Π΅Π»ΡŒΠ½ΠΎΠ΅ Π²ΠΊΠ»ΡŽΡ‡Π΅Π½ΠΈΠ΅.

ΠŸΠ°Ρ€Π°Π»Π»Π΅Π»ΡŒΠ½Π°Ρ схСма

ΠŸΡ€ΠΈ ΠΏΠ°Ρ€Π°Π»Π»Π΅Π»ΡŒΠ½ΠΎΠΉ схСмС ΠΏΠΎΠ΄Π°Ρ‡Π° Ρ€Π°Π·Π½ΠΈΡ†Ρ‹ ΠΏΠΎΡ‚Π΅Π½Ρ†ΠΈΠ°Π»ΠΎΠ² происходит Ρ‚ΠΎΠ»ΡŒΠΊΠΎ ΠΏΡ€ΠΈ запускС ΠΌΠΎΡ‚ΠΎΡ€Π° ΠΈΠ»ΠΈ Π²ΠΊΠ»ΡŽΡ‡Π΅Π½ΠΈΠΈ заТигания.

ΠŸΡ€ΠΎΠ²ΠΎΠ΄ Β«+Β» подсоСдинСн ΠΊ Ρ„Π°Ρ€Π°ΠΌ ΠΎΡ‚ ΠΊΠΎΠ½Ρ‚Π°ΠΊΡ‚Π° Ρ€Π΅Π»Π΅ΠΉΠ½ΠΎΠ³ΠΎ элСмСнта. ПослСдний записываСтся Ρ‡Π΅Ρ€Π΅Π· ΠΏΠ»Π°Π²ΠΊΠΈΠΉ ΠΏΡ€Π΅Π΄ΠΎΡ…Ρ€Π°Π½ΠΈΡ‚Π΅Π»ΡŒ Π½Π° 15 А β€” Π΅Π³ΠΎ ΠΏΡ€Π΅Π΄Π΅Π»Π° Π²ΠΏΠΎΠ»Π½Π΅ достаточно для большСй части ΠΏΡ€ΠΎΡ‚ΠΈΠ²ΠΎΡ‚ΡƒΠΌΠ°Π½ΠΎΠΊ. ΠŸΠ»Π°Π²ΠΊΡƒΡŽ вставку рСкомСндуСтся ΡΡ‚Π°Π²ΠΈΡ‚ΡŒ ΠΏΠΎΠ±Π»ΠΈΠΆΠ΅ ΠΊ аккумуляторной ΠΊΠ»Π΅ΠΌΠΌΠ΅.

Если подвСсти ΠΊ ПВЀ 12 Π’ ΠΎΡ‚ сигнального ΠΏΡ€ΠΎΠ²ΠΎΠ΄Π° контроля Π·Π° зарядкой, Ρ‚ΠΎ ΠΏΡ€ΠΈ Π²ΠΊΠ»ΡŽΡ‡Π΅Π½ΠΈΠΈ ΠΌΠΎΡ‚ΠΎΡ€Π° Ρ„Π°Ρ€Ρ‹ ΠΌΠΎΠ³ΡƒΡ‚ Π²ΠΊΠ»ΡŽΡ‡Π°Ρ‚ΡŒΡΡ автоматичСски.

ΠŸΠΎΡ€ΡΠ΄ΠΎΠΊ ΠΏΠΎΠ΄ΠΊΠ»ΡŽΡ‡Π΅Π½ΠΈΡ:

  1. ΠšΠΎΠ½Ρ‚Π°ΠΊΡ‚ «–» ΠΎΡ‚ Ρ„Π°Ρ€ ΠΏΠΎΠ΄ΡΠΎΠ΅Π΄ΠΈΠ½ΡΡŽΡ‚ ΠΊ ΠΊΡƒΠ·ΠΎΠ²Ρƒ.
  2. ΠšΠ»Π΅ΠΌΠΌΡ‹ Β«+Β» ΡΠΎΠ΅Π΄ΠΈΠ½ΡΡŽΡ‚ ΠΌΠ΅ΠΆΠ΄Ρƒ собой ΠΈ ΠΏΠΎΠ΄ΠΊΠ»ΡŽΡ‡Π°ΡŽΡ‚ ΠΊ ΠΊΠ»Π΅ΠΌΠΌΠ΅ Ρ€Π΅Π»Π΅ β„– 87.
  3. ΠšΠΎΠ½Ρ‚Π°ΠΊΡ‚ Ρ€Π΅Π»Π΅ Β«+Β» β„– 30 ΠΏΠΎΠ΄ΠΊΠ»ΡŽΡ‡Π°ΡŽΡ‚ ΠΊ аккумулятору.

ΠŸΡ€ΠΈ ΠΏΠ°Ρ€Π°Π»Π»Π΅Π»ΡŒΠ½ΠΎΠΌ Π²ΠΊΠ»ΡŽΡ‡Π΅Π½ΠΈΠΈ обСспСчиваСтся полноцСнная Π·Π°ΠΏΠΈΡ‚ΠΊΠ° ΠΊΠ°ΠΆΠ΄ΠΎΠΉ Ρ„Π°Ρ€Ρ‹, Ρ‡Ρ‚ΠΎ позволяСт Π½Π΅ Ρ‚Π΅Ρ€ΡΡ‚ΡŒ Π² яркости Π»Π°ΠΌΠΏ. ΠŸΠΎΡ‚Ρ€Π΅Π±Π»ΡΠ΅ΠΌΡ‹ΠΉ Ρ‚ΠΎΠΊ ΠΏΡ€ΠΈ этом возрастаСт.

ΠŸΠΎΡΠ»Π΅Π΄ΠΎΠ²Π°Ρ‚Π΅Π»ΡŒΠ½Π°Ρ схСма

Вакая схСма ΠΌΠΈΠ½ΠΈΠΌΠΈΠ·ΠΈΡ€ΡƒΠ΅Ρ‚ потрСбляСмый Ρ‚ΠΎΠΊ, Π° Π»Π°ΠΌΠΏΠΎΡ‡ΠΊΠΈ горят ΠΌΠ΅Π½Π΅Π΅ ярко, Ρ‡Π΅ΠΌ ΠΌΠΎΠ³Π»ΠΈ Π±Ρ‹. Π­Ρ‚Ρƒ схСму ΠΏΡ€ΠΈΠΌΠ΅Π½ΡΡŽΡ‚, Ссли ПВЀ Π·Π°ΠΌΠ΅Ρ‰Π°Π΅Ρ‚ Π³Π»Π°Π²Π½Ρ‹Π΅ Ρ„Π°Ρ€Ρ‹.

ΠŸΠΎΡ€ΡΠ΄ΠΎΠΊ дСйствий:

  1. ΠšΠΎΠ½Ρ‚Π°ΠΊΡ‚Ρ‹ «–» Ρ„Π°Ρ€ ΡΠΎΠ΅Π΄ΠΈΠ½ΡΡŽΡ‚ ΠΌΠ΅ΠΆΠ΄Ρƒ собой.
  2. ΠšΠ»Π΅ΠΌΠΌΡƒ Β«+Β» ΠΎΠ΄Π½ΠΎΠΉ Ρ„Π°Ρ€Ρ‹ ΠΏΡ€ΠΈΡΠΎΠ΅Π΄ΠΈΠ½ΡΡŽΡ‚ ΠΊ ΠΊΡƒΠ·ΠΎΠ²Ρƒ, Π΄Ρ€ΡƒΠ³ΡƒΡŽ Π²ΠΏΠ°ΠΈΠ²Π°ΡŽΡ‚ Π² Ρ€Π΅Π»Π΅.
  3. ΠšΠ½ΠΎΠΏΠΊΡƒ ΠΈ кабСль Β«+Β» ΠΎΡ‚ заТигания ΠΏΠΎΠ΄ΡΠΎΠ΅Π΄ΠΈΠ½ΡΡŽΡ‚ ΠΊ ΠΊΠ»Π΅ΠΌΠΌΠ°ΠΌ β„– 85 ΠΈ β„– 86.
  4. ΠšΠΎΠ½Ρ‚Π°ΠΊΡ‚ β„– 30 ΠΏΠΎΠ΄ΡΠΎΠ΅Π΄ΠΈΠ½ΡΡŽΡ‚ ΠΊ ΠΊΠ»Π΅ΠΌΠΌΠ΅ Β«+Β» аккумуляторной Π±Π°Ρ‚Π°Ρ€Π΅ΠΈ.

ΠŸΡ€ΠΎΡ‚ΠΈΠ²ΠΎΡ‚ΡƒΠΌΠ°Π½ΠΊΠΈ Π·Π°Π³ΠΎΡ€Π°ΡŽΡ‚ΡΡ ΠΏΡ€ΠΈ пускС ΠΌΠΎΡ‚ΠΎΡ€Π° ΠΈ Π²Ρ‹ΠΊΠ»ΡŽΡ‡Π°ΡŽΡ‚ΡΡ вмСстС с Π΅Π³ΠΎ остановкой.

На Π·Π°ΠΌΠ΅Ρ‚ΠΊΡƒ! Π’ соврСмСнных ΠΈΠ½ΠΎΠΌΠ°Ρ€ΠΊΠ°Ρ… прСдусмотрСна особая прСдмонтаТная элСктроподготовка β€” ΠΎ Π½Π΅ΠΉ судят ΠΏΠΎ ΠΊΠ»Π΅ΠΌΠΌΠ°ΠΌ Β«fog lamp relayΒ».

Π’ΠΈΠ΄Π΅ΠΎ

ΠœΠΎΠ½Ρ‚Π°ΠΆΠ½Π°Ρ схСма для установки ΠΏΡ€ΠΎΡ‚ΠΈΠ²ΠΎΡ‚ΡƒΠΌΠ°Π½Π½Ρ‹Ρ… Ρ„Π°Ρ€ ΠΏΠΎΠΊΠ°Π·Π°Π½Π° Π² Π²ΠΈΠ΄Π΅ΠΎΡ€ΠΎΠ»ΠΈΠΊΠ΅, снятом для ΠΊΠ°Π½Π°Π»Π° Β«Π›ΡŽΠ±ΠΈΡ‚Π΅Π»ΡŒ АвтомобилСй».

ΠΈ Ρ‚Π°ΠΊ Π²Ρ‹ ΡΠΎΠ±Ρ€Π°Π»ΠΈΡΡŒ ΠΏΠΎΡΡ‚Π°Π²ΠΈΡ‚ΡŒ ΠΏΡ€ΠΎΡ‚ΠΈΠ²ΠΎΡ‚ΡƒΠΌΠ°Π½Π½Ρ‹Π΅ Ρ„Π°Ρ€Ρ‹! Π½ΠΎ Π½Π΅ ΠΎΡ‡Π΅Π½ΡŒ ΠΏΠΎΠ½ΠΈΠΌΠ°Π΅Ρ‚Π΅ ΠΊΠ°ΠΊ, Π²Ρ€ΠΎΠ΄Π΅ Π² Π½Π΅Ρ‚Π΅ ΠΈΠ½Ρ„Ρ‹ ΠΏΠΎ ΠΏΠΎΠ²ΠΎΠ΄Ρƒ этого ΠΌΠ½ΠΎΠ³ΠΎ, ΠΈ схСм Ρ‚ΠΎΠΆΠ΅ ΠΊΡƒΡ‡Π°, Π½ΠΎ прСдстваим Ρ‡Ρ‚ΠΎ ППЦ Π²Ρ‹ Π½Π΅ ΠΏΠΎΠ½ΠΈΠΌΠ°Π΅Ρ‚Π΅ схСмы,

я ΠΏΠΎΡΡ‚Π°Ρ€Π°ΡŽΡΡŒ Π½Π°ΠΏΠΈΡΠ°Ρ‚ΡŒ ΠΎΡ‡Π΅Π½ΡŒ ΠΏΠΎΠ΄Ρ€ΠΎΠ±Π½Ρ‹ΠΉ ΠΎΡ‚Ρ‡Π΅Ρ‚, Ρ‡Ρ‚ΠΎ послС Π›ΡŽΠ±ΠΎΠΉ Ρ‡Π΅Π»ΠΎΠ²Π΅ΠΊ смог ΠΏΠΎΠ΄ΠΊΠ»ΡŽΡ‡ΠΈΡ‚ΡŒ ΠΈΡ… прям Π²ΠΎΠ·Π»Π΅ Π΄ΠΎΠΌΠ°, ΠΈ Π½Π΅ ΠΏΠ»Π°Ρ‚ΠΈΡ‚ Π½ΠΈΠΊΠΎΠΌΡƒ Π΄Π΅Π½Π³ΠΈ! самоС Π“Π»Π°Π²Π½ΠΎΠ΅ Π‘Π½ΠΈΠΌΠΈΡ‚Π΅ ΠΊΠ»Π΅ΠΌΡ‹ акумулятора!

Π±ΡƒΠ΄Ρƒ ΠΎΠΏΠΈΡΡ‹Π²Π°Ρ‚ΡŒ ΠΏΠΎ своСму ΠΌΠ°Ρ‚ΠΈΠ·Ρƒ! Π½ΠΎ ΠΏΠΎ этому Π‘Π– ΠΌΠΎΠΆΠ½ΠΎ ΠΏΠΎΠ΄ΠΊΠ»ΡŽΡ‡ΠΈΡ‚ΡŒ ΠΊΠΎΠΌΡƒ ΡƒΠ³ΠΎΠ΄Π½ΠΎ! ΠΈ Ρ‚Π°ΠΊ Π½Π°Ρ‡Π½Π΅ΠΌ! Π²Π½Π°Ρ‡Π°Π»Π΅ Ρ‡Ρ‚ΠΎ Π½Π°ΠΌ Π½ΡƒΠΆΠ½ΠΎ!

1) сами ΠŸΠ ΠžΠ’Π˜Π’ΠžΠ’Π£ΠœΠΠΠšΠ˜!

я Π½Π΅ Ρ„ΠΎΡ‚ΠΊΠ°Π» Π½ΠΈΡ‡Π΅Π³ΠΎ поэтому Π±ΡƒΠ΄Ρƒ ΠΎΠ±ΡŒΡΡΠ½ΡΡ‚ΡŒ Π½Π° Ρ‡ΡƒΠΆΠΈΡ… Ρ„ΠΎΡ‚ΠΊΠ°Ρ… ΠΈ Π½Π° листкС Π±ΡƒΠΌΠ°Π³ΠΈ! я Π½Π΅ Ρ…ΠΎΠ΄ΡƒΠΆΠ½ΠΈΠΊ, Π΄Π° это ΠΈ Π½Π΅ Π²Π°ΠΆΠ½ΠΎ Π³Π»Π°Π²Π½ΠΎΠ΅ Ρ‡Ρ‚ΠΎΠ± Π±Ρ‹Π»ΠΎ понятно!

ΠΈ Ρ‚Π°ΠΊ ΠΏΠΎΠ΅Ρ…Π°Π»ΠΈ! ΠΏΠΎ Π“ΠžΠ‘Π’Π£ Π½Π°Π΄ΠΎ Ρ‡Ρ‚ΠΎΠ± ΠΏΡ€ΠΎΡ‚ΠΈΠ²ΠΎΡ‚ΡƒΠΌΠ°Π½ΠΊΠΈ Π²ΠΊΠ»Ρ‡Π°Π»ΠΈΡΡŒ Ρ‚ΠΎΠΊΠ° с Π“Π°Π±Π°Ρ€ΠΈΡ‚Π°ΠΌΠΈ, (Π½ΠΎ это Π½Π΅ Π·Π½Π°Ρ‡ΠΈΡ‚ Ρ‡Ρ‚ΠΎ ΠΌΡ‹ Π½Π΅ смоТСм Π΅Π·Π΄ΠΈΡ‚ΡŒ с Π³Π°Π±Π°Ρ€ΠΈΡ‚Π°ΠΌΠΈ Π½ΠΎ Π±Π΅Π· ΠΏΡ€ΠΎΡ‚ΠΈΠ²ΠΎΡ‚ΡƒΠΌΠ°Π½ΠΎΠΊ) Ρ‚ΠΎΠ΅ΡΡ‚ΡŒ Ссли Π³Π°Π±Π°Ρ€ΠΈΡ‚Ρ‹ Π²Ρ‹ΠΊΠ»ΡŽΡ‡Π΅Π½Ρ‹ Ρ‚ΠΎ ΠΊΠ½ΠΎΠΏΠΊΠ° ΠΏΡ€ΠΎΡ‚ΠΈΠ²ΠΎΡ‚ΡƒΠΌΠ°Π½ΠΎΠΊ Π±ΡƒΠ΄Π΅Ρ‚ Ρ‰Π΅Π»ΠΊΠ°Ρ‚ΡŒ Π² ΠΏΡƒΡΡ‚ΡƒΡŽ! Π½ΠΎ ΠΊΠΎΠ³Π΄Π° Π²ΠΊΠ»ΡŽΡ‡Π΅Ρ‚Π΅ Π³Π°Π±Π°Ρ€ΠΈΡ‚Ρ‹ Ρ‚ΠΎ ΠΊΠ½ΠΎΠΏΠΊΠΎΠΉ ΠΏΡ€ΠΎΡ‚ΠΈΠ²ΠΎΡ‚ΡƒΠΌΠ°Π½ΠΎΠΊ ΠΌΠΎΠΆΠ½ΠΎ Π²ΠΊΠ»ΡŽΡ‡ΠΈΡ‚ΡŒ ΠΈΠ»ΠΈ Π²Ρ‹ΠΊΠ»ΡŽΡ‡ΠΈΡ‚ΡŒ эти самыС ΠΏΡ€ΠΎΡ‚ΠΈΠ²ΠΎΡ‚ΡƒΠΌΠ°Π½ΠΊΠΈ!

Π½Π°Ρ‡Π½Π΅ΠΌ с располоТСниС Π Π•Π›Π•! Π΅Π³ΠΎ ΠΌΠΎΠΆΠ½ΠΎ Π·Π°ΡΡƒΠ½ΡƒΡ‚ΡŒ ΠΊΡƒΠ΄Π° ΡƒΠ³ΠΎΠ΄Π½ΠΎ Ρ…ΠΎΡ‚ΡŒ Π² ΠΎΠ±Ρ‰ΠΈΠΉ Π±Π»ΠΎΠΊ, я Π½Π΅ стал ΠΌΡƒΡ‡Π°Ρ‚ΡŒΡΡ ΠΈ ΠΏΠΎΠ»ΠΎΠΆΠΈΠ» Π΅Π³ΠΎ Π·Π° ΠΏΡ€ΠΈΠ±ΠΎΡ€Π½ΡƒΡŽ понСль Π½Π°Π΄ ΠΌΠΎΠ³Π½ΠΈΡ‚ΠΎΠ»ΠΎΠΉ, поэтому ΠΏΠΈΡˆΡƒ ΠΊΠ°ΠΊ Π΄Π΅Π»Π°Π» я) снимаСм Ρ‡Π΅Ρ€Π½ΡƒΡŽ (ΠΊΡ‚ΠΎ Π½Π΅ красил) Ρ†Π΅Π½Ρ‚Ρ€Π°Π»ΡŒΠ½ΡƒΡŽ понСль, ΠΈ Π²ΠΈΠ΄ΠΈΠΌ Π²ΠΎΡ‚ Ρ‚Π°ΠΊΠΎΠ΅, Π²ΠΎΡ‚ Ρ‚ΡƒΠ΄Π° я ΠΈ ΠΊΠΈΠ½ΡƒΠ» Ρ€Π΅Π»Π΅

Ρ‚ΠΎΠ΅ΡΡ‚ΡŒ послС этих дСйствий Ρƒ нас ΡƒΠΆΠ΅ Π΅ΡΡ‚ΡŒ Ρ†Π΅ΠΏΡŒ 12 Π²ΠΎΠ»ΡŒΡ‚ ΠΎΡ‚ Π³Π°Π±Π°Ρ€ΠΈΡ‚ΠΎΠ² ΠΏΠΎΡ‚ΠΎΠΌ ΠΊΠ½ΠΎΠΏΠΊΠ° которая Ρ€Π°Π·Ρ€Ρ‹Π²Π°Π΅Ρ‚ Ρ†Π΅ΠΏΡŒ ΠΈ ΠΏΠΎΡ‚ΠΎΠΌ Ρ€Π΅Π»Π΅ 85 ΠΊΠΎΠ½Ρ‚Π°ΠΊΡ‚)

Π΄Π°Π»Π΅Π΅ ΠΊΠΎΠ½Ρ‚Π°ΠΊΡ‚ 87 протягиваСм Ρ‡Π΅Ρ€Π΅Π· Π·Π°Π³Π»ΡƒΡˆΠΊΡƒ ΠΏΠΎΠ΄ пСдалями ΠΈ тянСм ΠΊ АКУМУ! Π½Π΅Π·Π°Π±ΡƒΠ΄ΡŒΡ‚Π΅ ΠΏΠΎΡΡ‚Π°Π²ΠΈΡ‚ΡŒ ΠΏΡ€Π΅Π΄ΠΎΡ…Ρ€Π°Π½ΠΈΡ‚Π΅Π»ΡŒ, ΠΎΠ½ ΡΡ‚Π°Π²ΠΈΡ‚ΡŒΡΡ ΠΌΠ΅ΠΆΠ΄Ρƒ плюсом Π°ΠΊΡƒΠΌΠ° ΠΈ Ρ€Π΅Π»Π΅ 87 ΠΊΠΎΠ½Ρ‚Π°ΠΊΡ‚( ΠΏΡ€Π΅Π΄ΠΎΡ…Ρ€Π°Π½ΠΈΡ‚Π΅Π»ΡŒ ставтС Π±Π»ΠΈΠΆΠ΅ ΠΊ Π°ΠΊΡƒΠΌΡƒ) ΠΏΠΎΠ» ΠΏΡƒΡ‚ΠΈ ΠΏΡ€ΠΎΡˆΠ»ΠΈ ΠΈΠ΄Π΅ΠΌ Π΄Π°Π»Π΅Π΅ 86 ΠΊΠΎΠ½Ρ‚Π°ΠΊΡ‚ Ρ€Π΅Π»Π΅ ΠΊΠΈΠ΄Π°Π΅ΠΌ Π½Π° ΠΊΡƒΠ·ΠΎΠ², Π»ΠΈΠ±ΠΎ минус (Ρ‡Π°Ρ‰Π΅ всСго Ρ‡Π΅Ρ€Π½Ρ‹ΠΉ ΠΏΡ€ΠΎΠ²ΠΎΠ΄ ) Π±Ρ‹Π»ΠΎ Π±Ρ‹ Π½Π΅ΠΏΠ»ΠΎΡ…ΠΎ ΠΈ Π΅Π³ΠΎ Π½Π° Π°ΠΊΡƒΠΌ ΠΏΡ€ΠΎΡ‚ΡΠ½ΡƒΡ‚ΡŒ Π½ΠΎ это Π½Π΅ ΠΎΠ±ΡΠ·Π°Ρ‚Π΅Π»ΡŒΠ½ΠΎ!

всС ΠΎΡΡ‚Π°Π»ΠΎΡΡŒ послСднСС сами ΠΏΡ€ΠΎΡ‚ΠΈΠ²ΠΎΡ‚ΡƒΠΌΠ°Π½ΠΊΠΈ,

Π½Π°Ρ‡Π½Π΅ΠΌ с ΠΏΠ΅Ρ€Π΅Π΄ΠΊΠ° ΠΏΡ€ΠΎΡ‚ΠΈΠ²ΠΎΡ‚ΡƒΠΌΠ°Π½ΠΊΠΈ установлСны Π² ΡˆΡ‚Π°Ρ‚Π½ΠΎΠ΅ мСсто, ΠΈ ΠΈΠ· ΠΊΠ°ΠΆΠ΄ΠΎΠΉ Ρ„Π°Ρ€Ρ‹ ΠΈΠ΄Π΅Ρ‚ 2-Π° ΠΏΡ€ΠΎΠ²ΠΎΠ΄Π° плюс ΠΈ минус! с минусом ΠΊΠ°ΠΊ всСгда ΠΊΠΈΠ΄Π°ΠΉΡ‚Π΅ Π½Π° ΠΊΡƒΠ·ΠΎΠ², Π»ΠΈΠ±ΠΎ соСдинитС 2-Π° минуса ΠΎΡ‚ Π΄Π²ΡƒΡ… Ρ„Π°Ρ€ ΠΈ ΠΊΠΈΠ½Ρ‚Π΅ Π½Π° минус Π°ΠΊΡƒΠΌΠ°)(это Π±Π΅Π· Ρ€Π°Π·Π½ΠΈΡ†Ρ‹! просто Ссли ΠΊΠΈΠ΄Π°Ρ‚ΡŒ Π½Π° ΠΊΡƒΠ·ΠΎΠ² Ρ‚ΠΎ ΠΏΡ€ΠΎΠ²ΠΎΠ΄ΠΎΠ² мСньшС Ρ‚ΡΠ½ΡƒΡ‚ΡŒ Π½Π°Π΄ΠΎ! Π° Π²ΠΎΡ‚ ΠΏΠ»ΡŽΡΡ‹ ΠΎΡ‚ 2-ΡƒΡ… Ρ„Π°Ρ€ соСдСнитС ΠΈ тянитС Π² салон Ρ‡Π΅Ρ€Π΅Π· Ρ‚ΡƒΠΆΠ΅ Π·Π°Π³Π»ΡƒΡˆΠΊΡƒ Ρ‡Ρ‚ΠΎ ΠΈ плюс Π½Π° Π°ΠΊΡƒΠΌ, …вытянули? Ρ‚Π΅ΠΏΠ΅Ρ€ΡŒ ΠΏΠΎΠ΄Π½ΠΈΠΌΠΈΡ‚Π΅ Π΅Π³ΠΎ Π½Π°Π²Π΅Ρ€Ρ… ΠΊ Π½Π°ΡˆΠ΅ΠΌΡƒ Ρ€Π΅Π»Π΅ Ρ‚Π°ΠΊ Ρ‡Ρ‚ΠΎΠ± ΠΏΡ€ΠΎΠ²ΠΎΠ΄ΠΎΠ² Π½Π΅Π±Ρ‹Π»ΠΎ Π²ΠΈΠ΄Π½ΠΎ(Ρ‚Π°ΠΊ ΠΆΠ΅ красивСС) ΠΈ ΠΏΠΎΠ΄ΠΊΠ»ΡŽΡ‡ΠΈΡ‚Π΅ Π΅Π³ΠΎ ΠΊ Ρ€Π°Π·ΡŒΠ΅ΠΌΡƒ 30, Π½Ρƒ Π²Ρ€ΠΎΠ΄Π΅ ΠΈ всС ΠΏΠΎΠΊΠ° Π½Π΅ собирайтС, ΠΌΠ°Π»ΠΎ Π»ΠΈ Ρ‡Ρ‚ΠΎ Π½Π΅ Ρ‚Π°ΠΊ! Ρ‡Ρ‚ΠΎΠ± ΠΎΠΏΡΡ‚ΡŒ Π½Π΅ Ρ€Π°Π·Π±ΠΈΡ€Π°Ρ‚ΡŒ! ΠΈΠ΄ΠΈΡ‚Π΅ Π½Π°ΠΊΠΈΠ½Ρ‚Π΅ ΠΊΠ»Π΅ΠΌΡ‹ Π½Π° акум…я ΠΏΠΎΠΊΠ° Ρ‚ΡƒΡ‚ ΠΏΠΎΠ΄ΠΎΠΆΠ΄Ρƒ) Ρ‚Π΅ΠΏΠ΅Ρ€ΡŒ Π²ΠΊΠ»ΡŽΡ‡ΠΈΡ‚Π΅ Π³Π°Π±Π°Ρ€ΠΈΡ‚Ρ‹ ΠΈ ΠΏΡ€ΠΎΠ²Π΅Ρ€Ρ‚Π΅ Ρ€Π°Π±ΠΎΡ‚Π°ΡŽΡ‚ Π»ΠΈ ΠΎΠ½ΠΈ! (ΠΊΠΎΠ½Π΅Ρ‡Π½ΠΎ Ρ€Π°Π±ΠΎΡ‚Π°ΡŽΡ‚) Π½Ρƒ ΠΈ ΠΏΠΎΠΏΡ€ΠΎΠ±ΡƒΠΉΡ‚Π΅ ΠΏΡ€ΠΎΡ‚ΠΈΠ²ΠΎΡ‚ΡƒΠΌΠ°Π½ΠΊΠΈ, Ссли всС Ρ€Π°Π±ΠΎΡ‚Π°Π΅Ρ‚ Ρ‚ΠΎ ΠΌΠΎΠΆΠ΅Ρ‚Π΅ всС ΠΏΡ€ΡΡ‚Π°Ρ‚ΡŒ ΠΈ ΡΠΎΠ±ΠΈΡ€Π°Ρ‚ΡŒ ΠΎΠ±Ρ€Π°Ρ‚Π½ΠΎ, ΠΏΡ€ΠΎΠ²Π΅Ρ€ΡŒΡ‚Π΅ Ρ‡Ρ‚ΠΎΠ± Π½ΠΈΠ³Π΄Π΅ Π½Π΅ ΠΊΠΎΡ€ΠΎΡ‚ΠΈΠ»ΠΎ!ΠΈ Π·Π°ΠΈΠ·ΠΎΠ»ΠΈΡ€ΡƒΠΉΡ‚Π΅ всС Π³ΠΎΠ»Ρ‹Π΅ ΠΏΡ€ΠΎΠ²ΠΎΠ΄Π°)

Π’Π°Ρ€ΠΈΠ°Π½Ρ‚Ρ‹ ΠΌΠΎΠ½Ρ‚Π°ΠΆΠ°

ΠŸΠ΅Ρ€Π΅Π΄ ΠΌΠΎΠ½Ρ‚ΠΈΡ€ΠΎΠ²Π°Π½ΠΈΠ΅ΠΌ ПВЀ Ρ€Π°Π·Ρ€Π°Π±Π°Ρ‚Ρ‹Π²Π°ΡŽΡ‚ ΡΠ»Π΅ΠΊΡ‚Ρ€ΠΎΠΌΠΎΠ½Ρ‚Π°ΠΆΠ½ΡƒΡŽ схСму β€” ΠΎΠ½Π° ΠΏΠΎΠ·Π²ΠΎΠ»ΠΈΡ‚ ΠΎΠ±ΠΎΠΉΡ‚ΠΈΡΡŒ Π±Π΅Π· лишнСй ΠΏΡ€ΠΎΠ²ΠΎΠ΄ΠΊΠΈ ΠΈ Π³Π°Ρ€Π°Π½Ρ‚ΠΈΡ€ΡƒΠ΅Ρ‚ Ρ„ΡƒΠ½ΠΊΡ†ΠΈΠΎΠ½ΠΈΡ€ΠΎΠ²Π°Π½ΠΈΠ΅ элСктроцСпи.

ΠœΠΎΠ½Ρ‚Π°ΠΆ Π² Π±Π°ΠΌΠΏΠ΅Ρ€

ΠŸΡ€ΠΈΠΌΠ΅Π½ΡΡŽΡ‚ 3 Π²Π°Ρ€ΠΈΠ°Π½Ρ‚Π° ΠΌΠΎΠ½Ρ‚Π°ΠΆΠ° β€” Π² ΠΏΡ€ΠΎΠ΅ΠΌΡ‹, прСдусмотрСнныС ΠΏΡ€ΠΎΠΈΠ·Π²ΠΎΠ΄ΠΈΡ‚Π΅Π»Π΅ΠΌ, Π½Π° ΠΊΡ€ΠΎΠ½ΡˆΡ‚Π΅ΠΉΠ½ ΠΈΠ»ΠΈ Π² Π³Π»ΡƒΡ…ΠΎΠΉ Π±Π°ΠΌΠΏΠ΅Ρ€, Π³Π΄Π΅ Π½Π΅Ρ‚ ниши ΠΏΠΎΠ΄ ПВЀ.

Для настройки ΠΈΡΠΏΠΎΠ»ΡŒΠ·ΡƒΡŽΡ‚ ΡΠ°ΠΌΠΎΠ΄Π΅Π»ΡŒΠ½Ρ‹ΠΉ шаблон. Π•Π³ΠΎ ΡƒΡΡ‚Π°Π½Π°Π²Π»ΠΈΠ²Π°ΡŽΡ‚ Π½Π° расстоянии 5 см ΠΎΡ‚ ПВЀ. Π’ΠΎ врСмя настройки линию совпадСния ΡƒΡΡ‚Π°Π½Π°Π²Π»ΠΈΠ²Π°ΡŽΡ‚ Π½Π° расстоянии 10 см Π½ΠΈΠΆΠ΅ Ρ†Π΅Π½Ρ‚Ρ€Π° Π»Π°ΠΌΠΏ. Благодаря настройкС Ρ„Π°Ρ€Ρ‹ ΠΎΠ±Π΅ΡΠΏΠ΅Ρ‡ΠΈΠ²Π°ΡŽΡ‚ ΠΏΠΎΠ²Ρ‹ΡˆΠ΅Π½Π½ΡƒΡŽ Π²ΠΈΠ΄ΠΈΠΌΠΎΡΡ‚ΡŒ ΠΈ ΠΌΠΈΠ½ΠΈΠΌΠΈΠ·ΠΈΡ€ΡƒΡŽΡ‚ Π²Π΅Ρ€ΠΎΡΡ‚Π½ΠΎΡΡ‚ΡŒ ослСплСния Π°Π²Ρ‚ΠΎ, Π΅Π΄ΡƒΡ‰ΠΈΠΌΠΈ навстрСчу.

ΠœΠΎΠ½Ρ‚Π°ΠΆ Π² Π³Π»ΡƒΡ…ΠΎΠΉ Π±Π°ΠΌΠΏΠ΅Ρ€

Π­Ρ‚ΠΎΡ‚ Π²Π°Ρ€ΠΈΠ°Π½Ρ‚ отличаСтся Ρ‚Ρ€ΡƒΠ΄ΠΎΠ΅ΠΌΠΊΠΎΡΡ‚ΡŒΡŽ, приходится Π²Ρ‹ΠΏΠΈΠ»ΠΈΠ²Π°Ρ‚ΡŒ отвСрстия ΠΏΠΎΠ΄ ПВЀ.

ΠŸΠΎΡ€ΡΠ΄ΠΎΠΊ дСйствий:

  1. Π‘Π½ΠΈΠΌΠΈΡ‚Π΅ Π±Π°ΠΌΠΏΠ΅Ρ€ ΠΈ ΠΏΠΎΡΡ‚Π°Π²ΡŒΡ‚Π΅ ΠΌΠ΅Ρ‚ΠΊΠΈ, Π³Π΄Π΅ Π±ΡƒΠ΄ΡƒΡ‚ Ρ„Π°Ρ€Ρ‹.
  2. ΠŸΡ€ΠΎΡΠ²Π΅Ρ€Π»ΠΈΡ‚Π΅ отвСрстия ΠΏΠΎ ΠΏΠ΅Ρ€ΠΈΠΌΠ΅Ρ‚Ρ€Ρƒ, Π° ΠΏΠΎΡ‚ΠΎΠΌ Π²Ρ‹Ρ€Π΅ΠΆΡŒΡ‚Π΅ острым Π½ΠΎΠΆΠΎΠΌ Ρ„Ρ€Π°Π³ΠΌΠ΅Π½Ρ‚ Π±Π°ΠΌΠΏΠ΅Ρ€Π°.
  3. ΠŸΡ€ΠΈΠΊΡ€Π΅ΠΏΠΈΡ‚Π΅ Ρ„Π°Ρ€Ρ‹ подходящими ΠΏΠΎ Ρ€Π°Π·ΠΌΠ΅Ρ€Ρƒ Π±ΠΎΠ»Ρ‚Π°ΠΌΠΈ. Π‘ΠΌΠΎΠ½Ρ‚ΠΈΡ€ΡƒΠΉΡ‚Π΅ Π·Π°Ρ‰ΠΈΡ‚Π½ΡƒΡŽ Π½Π°ΠΊΠ»Π°Π΄ΠΊΡƒ β€” ΠΏΡ€ΠΈΡ†Π΅ΠΏΠΈΡ‚Π΅ Π΅Π΅ Π·Π° Π±Π°ΠΌΠΏΠ΅Ρ€.

Настройку Ρ„Π°Ρ€ производят посрСдством ΡΠΏΠ΅Ρ†ΠΈΠ°Π»ΡŒΠ½ΠΎΠ³ΠΎ ΠΊΠ»ΡŽΡ‡Π° β€” ΠΎΠ½ Π²Ρ…ΠΎΠ΄ΠΈΡ‚ Π² ΠΊΠΎΠΌΠΏΠ»Π΅ΠΊΡ‚Π°Ρ†ΠΈΡŽ.

Π”Ρ€ΡƒΠ³ΠΈΠ΅ способы установки

ПВЀ ΠΌΠΎΠΆΠ½ΠΎ ΠΏΠΎΡΡ‚Π°Π²ΠΈΡ‚ΡŒ Π½Π° ΠΊΡ€ΠΎΠ½ΡˆΡ‚Π΅ΠΉΠ½, ΠΈΠ·Π³ΠΎΡ‚ΠΎΠ²Π»Π΅Π½Π½Ρ‹ΠΉ ΠΈΠ· ΠΏΡ€ΠΎΡ‡Π½ΠΎΠΉ стали. ΠšΡ€Π΅ΠΏΡΡ‚ ΠΊΡ€ΠΎΠ½ΡˆΡ‚Π΅ΠΉΠ½ согласно Π½ΠΎΡ€ΠΌΠ°ΠΌ ΠΌΠΎΠ½Ρ‚Π°ΠΆΠ° ПВЀ. Π”Π΅Ρ‚Π°Π»ΡŒ Π·Π°ΠΊΡ€Π΅ΠΏΠ»ΡΡŽΡ‚ с ΠΏΠΎΠΌΠΎΡ‰ΡŒΡŽ саморСзов. К Ρ‚ΠΎΡ‡ΠΊΠ°ΠΌ крСплСния ставят ПВЀ ΠΈ ΠΏΠΎΠ΄ΠΊΠ»ΡŽΡ‡Π°ΡŽΡ‚ ΠΏΡ€ΠΎΠ²ΠΎΠ΄ΠΊΡƒ.

ΠœΠΎΠ½Ρ‚Π°ΠΆ Π² ΠΏΡ€ΠΎΠ΅ΠΌ, прСдусмотрСнный ΠΏΡ€ΠΎΠΈΠ·Π²ΠΎΠ΄ΠΈΡ‚Π΅Π»Π΅ΠΌ:

  1. ΠŸΡ€ΠΈΠΏΠΎΠ΄Π½ΠΈΠΌΠΈΡ‚Π΅ Π°Π²Ρ‚ΠΎΠΌΠΎΠ±ΠΈΠ»ΡŒ, ΠΏΠΎΠ΄Π»ΠΎΠΆΠΈΠ² ΠΏΠΎΠ΄ ΠΏΠ΅Ρ€Π΅Π΄Π½ΠΈΠ΅ колСса дСрСвянный брус Ρ‚ΠΎΠ»Ρ‰ΠΈΠ½ΠΎΠΉ 20 см.
  2. Π‘Π½ΠΈΠΌΠΈΡ‚Π΅ Π·Π°Ρ‰ΠΈΡ‚Π½Ρ‹Π΅ Ρ‰ΠΈΡ‚ΠΊΠΈ, ΠΊΠΎΡ‚ΠΎΡ€Ρ‹Π΅ Π·Π°ΠΊΡ€Ρ‹Π²Π°ΡŽΡ‚ Π½ΠΈΠ· Π±Π°ΠΌΠΏΠ΅Ρ€Π°.
  3. ΠžΡ‚Π²ΠΈΠ½Ρ‚ΠΈΡ‚Π΅ фиксаторы ΠΎΡ‚ Π·Π°Π³Π»ΡƒΡˆΠ΅ΠΊ, прСдусмотрСнных ΠΏΡ€ΠΎΠΈΠ·Π²ΠΎΠ΄ΠΈΡ‚Π΅Π»Π΅ΠΌ.
  4. Вставив ПВЀ Π² Π½Π°ΠΏΡ€Π°Π²Π»ΡΡŽΡ‰ΠΈΠ΅, ΠΏΡ€ΠΈΠΊΡ€Π΅ΠΏΠΈΡ‚Π΅ ΠΈΡ… саморСзами.

Если ΡƒΡΡ‚Π°Π½Π°Π²Π»ΠΈΠ²Π°ΡŽΡ‚ΡΡ Π½Π΅ΠΎΡ€ΠΈΠ³ΠΈΠ½Π°Π»ΡŒΠ½Ρ‹Π΅ Ρ„Π°Ρ€Ρ‹, Ρ€Π°Π·ΠΌΠ΅Ρ€Ρ‹ ΠΈ Ρ‚ΠΎΡ‡ΠΊΠΈ крСплСния ΠΌΠΎΠ³ΡƒΡ‚ Π½Π΅ ΡΠΎΠ²ΠΏΠ°ΡΡ‚ΡŒ. Π’ΠΎΠ³Π΄Π° элСмСнты приходится ΠΏΠΎΠ΄Π³ΠΎΠ½ΡΡ‚ΡŒ.

Как ΡƒΡΡ‚Π°Π½ΠΎΠ²ΠΈΡ‚ΡŒ ΠΈ ΠΏΠΎΠ΄ΠΊΠ»ΡŽΡ‡ΠΈΡ‚ΡŒ своими Ρ€ΡƒΠΊΠ°ΠΌΠΈ?

Установка ΠΏΡ€ΠΎΡ‚ΠΈΠ²ΠΎΡ‚ΡƒΠΌΠ°Π½Π½Ρ‹Ρ… Ρ„Π°Ρ€ собствСнными силами начинаСтся с Ρ€Π°Π·Ρ€Π°Π±ΠΎΡ‚ΠΊΠΈ схСмы ΠΏΠΎΠ΄ΠΊΠ»ΡŽΡ‡Π΅Π½ΠΈΡ. ΠŸΡ€Π°Π²ΠΈΠ»ΡŒΠ½ΠΎ разработанная схСма позволяСт ΠΈΠ·Π±Π΅ΠΆΠ°Ρ‚ΡŒ Π»ΠΈΡˆΠ½ΠΈΡ… ΠΏΡ€ΠΎΠ²ΠΎΠ΄ΠΎΠ² ΠΈ обСспСчиваСт Π½Π°Π΄Π΅ΠΆΠ½ΡƒΡŽ Ρ€Π°Π±ΠΎΡ‚Ρƒ элСктричСской Ρ†Π΅ΠΏΠΈ. Π Π°Π±ΠΎΡ‚Ρ‹ ΠΏΠΎ ΠΌΠΎΠ½Ρ‚Π°ΠΆΡƒ рСкомСндуСтся Π²Ρ‹ΠΏΠΎΠ»Π½ΡΡ‚ΡŒ Π² Π³Π°Ρ€Π°ΠΆΠ΅, хотя ΠΌΠ½ΠΎΠ³ΠΈΠ΅ Π²Π»Π°Π΄Π΅Π»ΡŒΡ†Ρ‹ производят установку ΠΏΠΎΠ΄ ΠΎΡ‚ΠΊΡ€Ρ‹Ρ‚Ρ‹ΠΌ Π½Π΅Π±ΠΎΠΌ.

Установка ПВЀ Π² ΠΏΠ΅Ρ€Π΅Π΄Π½ΠΈΠΉ Π±Π°ΠΌΠΏΠ΅Ρ€ ΠΈ настройка

ВстрСчаСтся Ρ‚Ρ€ΠΈ способа установки:

  • Π² ΡˆΡ‚Π°Ρ‚Π½Ρ‹Π΅ ΠΏΡ€ΠΎΠ΅ΠΌΡ‹;
  • Π½Π° внСшнСм ΠΊΡ€ΠΎΠ½ΡˆΡ‚Π΅ΠΉΠ½Π΅;
  • Π² Π³Π»ΡƒΡ…ΠΎΠΉ Π±Π°ΠΌΠΏΠ΅Ρ€, Π½Π΅ ΠΏΡ€Π΅Π΄Π½Π°Π·Π½Π°Ρ‡Π΅Π½Π½Ρ‹ΠΉ для установки Ρ„Π°Ρ€.

РаспространСнныС ошибки

Если Ρƒ Π°Π²Ρ‚ΠΎΠ»ΡŽΠ±ΠΈΡ‚Π΅Π»Ρ Π½Π΅Ρ‚ ΠΎΠΏΡ‹Ρ‚Π° элСктромонтаТных Ρ€Π°Π±ΠΎΡ‚, устанавливая ПВЀ, Π»Π΅Π³ΠΊΠΎ ΠΎΡˆΠΈΠ±ΠΈΡ‚ΡŒΡΡ. Π§Ρ‚ΠΎΠ±Ρ‹ ΠΏΡ€Π°Π²ΠΈΠ»ΡŒΠ½ΠΎ ΠΏΠΎΠ΄ΡΠΎΠ΅Π΄ΠΈΠ½ΠΈΡ‚ΡŒ Ρ„Π°Ρ€Ρ‹, рСкомСндуСтся Π·Π°Π±Π»Π°Π³ΠΎΠ²Ρ€Π΅ΠΌΠ΅Π½Π½ΠΎ ΡƒΠ·Π½Π°Ρ‚ΡŒ ΠΎ Ρ‚ΠΈΠΏΠΈΡ‡Π½Ρ‹Ρ… ΠΏΡ€ΠΎΠΌΠ°Ρ…Π°Ρ… Π½ΠΎΠ²ΠΈΡ‡ΠΊΠΎΠ².

НСдочСты ΠΈ ΠΏΡ€ΠΎΠ±Π»Π΅ΠΌΡ‹:

  1. ΠŸΡ€ΠΎΠ±Π»Π΅ΠΌΠ° Π²ΠΎΠ·Π½ΠΈΠΊΠ°Π΅Ρ‚ ΠΏΡ€ΠΈ подсоСдинСнии ПВЀ ΠΊ Π³Π°Π±Π°Ρ€ΠΈΡ‚Π°ΠΌ. Π­Ρ‚Π° систСма Π½Π΅ рассчитана Π½Π° Ρ‚ΠΎΠΊ, ΠΊΠΎΡ‚ΠΎΡ€Ρ‹ΠΉ Π΅ΠΉ приходится Π²Ρ‹Π΄Π΅Ρ€ΠΆΠΈΠ²Π°Ρ‚ΡŒ ΠΈΠ·-Π·Π° внСдрСния Π΄ΠΎΠΏΠΎΠ»Π½ΠΈΡ‚Π΅Π»ΡŒΠ½ΠΎΠΉ Π½Π°Π³Ρ€ΡƒΠ·ΠΊΠΈ. Π’ ΠΈΡ‚ΠΎΠ³Π΅ Π½Π°Π±Π»ΡŽΠ΄Π°Π΅Ρ‚ΡΡ ΠΏΠ΅Ρ€Π΅Π³Ρ€Π΅Π² ΠΈ ΠΏΠ»Π°Π²Π»Π΅Π½ΠΈΠ΅ ΠΏΡ€ΠΎΠ²ΠΎΠ΄ΠΎΠ². Если ПВЀ Π·Π°Π³ΠΎΡ€Π°ΡŽΡ‚ΡΡ вмСстС с Π³Π°Π±Π°Ρ€ΠΈΡ‚Π°ΠΌΠΈ, ΠΎΠ±ΠΎΡ€ΡƒΠ΄ΠΎΠ²Π°Π½ΠΈΠ΅ быстро Π²Ρ‹ΠΉΠ΄Π΅Ρ‚ ΠΈΠ· строя.
  2. Если ΠΏΠ°Ρ€Π°ΠΌΠ΅Ρ‚Ρ€Ρ‹ ΠΌΠΎΠ½Ρ‚Π°ΠΆΠ° ПВЀ Π½Π΅ ΠΎΡ‚Π²Π΅Ρ‡Π°Π΅Ρ‚ Π½ΠΎΡ€ΠΌΠ°ΠΌ ΠŸΠ”Π”, ΠΊ Π²ΠΎΠ΄ΠΈΡ‚Π΅Π»ΡŽ ΠΌΠΎΠ³ΡƒΡ‚ ΠΏΡ€Π΅Π΄ΡŠΡΠ²ΠΈΡ‚ΡŒ ΠΏΡ€Π΅Ρ‚Π΅Π½Π·ΠΈΠΈ Ρ€Π°Π±ΠΎΡ‚Π½ΠΈΠΊΠΈ Π“Π˜Π‘Π”Π”. Если ΠΎΠ½ΠΈ ΠΎΠ±Π½Π°Ρ€ΡƒΠΆΠ°Ρ‚ Π½Π°Ρ€ΡƒΡˆΠ΅Π½ΠΈΠ΅ установлСнных Π½ΠΎΡ€ΠΌ, Ρ‚ΠΎ Π²ΠΎΠ΄ΠΈΡ‚Π΅Π»ΡŒ Π±ΡƒΠ΄Π΅Ρ‚ ΠΎΡˆΡ‚Ρ€Π°Ρ„ΠΎΠ²Π°Π½.
  3. Π‘Ρ‹Π²Π°Π΅Ρ‚, Ρ‡Ρ‚ΠΎ Π°Π²Ρ‚ΠΎΠ»ΡŽΠ±ΠΈΡ‚Π΅Π»ΡŒ Π²Ρ‹Π±ΠΈΡ€Π°Π΅Ρ‚ Π½Π΅ΡƒΠ΄Π°Ρ‡Π½ΠΎΠ΅ мСсто ΠΏΠΎΠ΄ ΠΊΠ½ΠΎΠΏΠΊΡƒ. Если ΠΎΠ½Π° располоТСна слишком Π΄Π°Π»Π΅ΠΊΠΎ, Π²ΠΎΠ΄ΠΈΡ‚Π΅Π»ΡŽ слоТно Π΄ΠΎΡ‚ΡΠ½ΡƒΡ‚ΡŒΡΡ Π΄ΠΎ Π½Π΅Π΅ Π­Ρ‚ΠΎ ΠΌΠΎΠΆΠ΅Ρ‚ ΡΠΏΡ€ΠΎΠ²ΠΎΡ†ΠΈΡ€ΠΎΠ²Π°Ρ‚ΡŒ Π”Π’ΠŸ.

Если Π²Ρ‹ часто Π΅Π·Π΄ΠΈΡ‚Π΅ Π² условиях ΠΏΡ€ΠΎΠ±Π»Π΅ΠΌΠ½ΠΎΠΉ видимости, Π²Π°ΠΌ стоит ΠΏΠΎΡΡ‚Π°Π²ΠΈΡ‚ΡŒ ПВЀ. Благодаря ΠΈΠΌ Π²Ρ‹ смоТСтС Π»ΡƒΡ‡ΡˆΠ΅ ΠΊΠΎΠ½Ρ‚Ρ€ΠΎΠ»ΠΈΡ€ΠΎΠ²Π°Ρ‚ΡŒ Π°Π²Ρ‚ΠΎ Π² слоТных условиях, Ρ‡Ρ‚ΠΎ обСспСчит Π²Π°ΠΌ Π±ΠΎΠ»Π΅Π΅ высокий ΡƒΡ€ΠΎΠ²Π΅Π½ΡŒ бСзопасности ΠΏΡ€ΠΈ Π΅Π·Π΄Π΅.

ΠŸΠΎΠ΄ΠΊΠ»ΡŽΡ‡Π΅Π½ΠΈΠ΅ Ρ‚ΡƒΠΌΠ°Π½ΠΎΠΊ Π½Π° Π³Π°Π·Π΅Π»ΠΈ | Π₯итрости Π–ΠΈΠ·Π½ΠΈ

Π‘ΠΎΠ΄Π΅Ρ€ΠΆΠ°Π½ΠΈΠ΅

Когда собирался ΡΡ‚Π°Π²ΠΈΡ‚ΡŒ Π±Π°ΠΌΠΏΠ΅Ρ€ «Π±ΠΈΠ·Π½Π΅Ρ», Ρ‚ΠΎ Ρ€Π΅ΡˆΠΈΠ» Π½Π΅ΠΏΡ€Π΅ΠΌΠ΅Π½Π½ΠΎ ΡƒΡΡ‚Π°Π½ΠΎΠ²ΠΈΡ‚ΡŒ ΠΈ ΠΏΠΎΠ΄ΠΊΠ»ΡŽΡ‡ΠΈΡ‚ΡŒ Ρ‚ΡƒΠΌΠ°Π½ΠΊΠΈ. На ΠΏΡ€Π΅ΠΆΠ½Π΅ΠΌ Π±Π°ΠΌΠΏΠ΅Ρ€Π΅ Ρ‚ΡƒΠΌΠ°Π½ΠΊΠΈ стояли, Ρ‡Ρ‚ΠΎ называСтся, для ΠΌΠ΅Π±Π΅Π»ΠΈ (Π±Π΅Π· ΠΏΠΎΠ΄ΠΊΠ»ΡŽΡ‡Π΅Π½ΠΈΡ). Π”Π° ΠΈ Ρ‚ΠΎ Ρ‚ΠΎΠ»ΡŒΠΊΠΎ слСва. Π’ΡƒΠΌΠ°Π½ΠΊΠΈ ΠΏΡ€ΠΈΠΊΡ€ΡƒΡ‚ΠΈΠ» ΠΏΡ€ΠΈ установкС Π±Π°ΠΌΠΏΠ΅Ρ€Π°, Ρ‚ΠΎΠ³Π΄Π° ΠΆΠ΅ собрал схСму. Π’ΠΊΠ»ΡŽΡ‡Π΅Π½ΠΈΠ΅ Ρ€Π΅ΡˆΠΈΠ» ΡΠ΄Π΅Π»Π°Ρ‚ΡŒ ΠΎΡ‚ Ρ†Π΅Π½Ρ‚Ρ€Π°Π»ΡŒΠ½ΠΎΠ³ΠΎ ΠΏΠ΅Ρ€Π΅ΠΊΠ»ΡŽΡ‡Π°Ρ‚Π΅Π»Ρ свСта (Π½Π° панСлях ΠΎΠ±Ρ€Π°Π·Ρ†Π° 2003 Π³ΠΎΠ΄Π° Π΅ΡΡ‚ΡŒ ΡΠΎΠΎΡ‚Π²Π΅Ρ‚ΡΡ‚Π²ΡƒΡŽΡ‰Π΅Π΅ ΠΏΠΎΠ»ΠΎΠΆΠ΅Π½ΠΈΠ΅). ΠŸΡ€ΠΎΠ±Π»Π΅ΠΌΠ° Π²ΠΎΠ·Π½ΠΈΠΊΠ»Π° ΠΈΠ·-Π·Π° отсутствия Π² ΠΏΡ€ΠΎΠ΄Π°ΠΆΠ΅ ΠΊΠΎΠ½Ρ‚Π°ΠΊΡ‚ΠΎΠ², примСняСмых Π² ΠΊΠΎΠ»ΠΎΠ΄ΠΊΠ΅. Π’ ΠΈΡ‚ΠΎΠ³Π΅ ΠΎΠ±ΠΎΡˆΡ‘Π»ΡΡ ΠΎΠ±Ρ‹Ρ‡Π½Ρ‹ΠΌΠΈ «ΠΌΠ°ΠΌΠ°ΠΌΠΈ» подходящСго Ρ€Π°Π·ΠΌΠ΅Ρ€Π° (ΠΎΠ΄Π΅Π²Π°Π΅ΠΌ сначала ΠΊΠΎΠ½Ρ‚Π°ΠΊΡ‚, Π° свСрху Π½Π° Π½Π΅Π³ΠΎ ΠΊΠΎΠ»ΠΎΠ΄ΠΊΡƒ). Π˜Ρ‚Π°ΠΊ, Π² ΠΏΠ΅Ρ€Π΅ΠΊΠ»ΡŽΡ‡Π°Ρ‚Π΅Π»Π΅ Π΅ΡΡ‚ΡŒ ΠΎΠ΄ΠΈΠ½ свободный ΠΊΠΎΠ½Ρ‚Π°ΠΊΡ‚ β„–10. К Π½Π΅ΠΌΡƒ-Ρ‚ΠΎ ΠΈ ΠΏΠΎΠ΄ΠΊΠ»ΡŽΡ‡Π°Π΅ΠΌ ΠΏΡ€ΠΎΠ²ΠΎΠ΄, ΠΈΠ΄ΡƒΡ‰ΠΈΠΉ ΠΎΡ‚ Ρ€Π΅Π»Π΅. ΠŸΡ€ΠΈ Ρ‚Π°ΠΊΠΎΠΉ схСмС Ρ‚ΡƒΠΌΠ°Π½ΠΊΠΈ Π²ΠΊΠ»ΡŽΡ‡Π°ΡŽΡ‚ΡΡ вмСстС с Π±Π»ΠΈΠΆΠ½ΠΈΠΌ/дальним свСтом. Π£ΠΆΠ΅ Ρ…ΠΎΡ€ΠΎΡˆΠΎ, Π½ΠΎ Π½Π΅ совсСм. НуТно ΠΎΡ‚Π΄Π΅Π»ΡŒΠ½ΠΎΠ΅ Π²ΠΊΠ»ΡŽΡ‡Π΅Π½ΠΈΠ΅. Для этого Π²Ρ‹Ρ‚Π°Ρ‰ΠΈΠ» ΠΈΠ· ΠΊΠΎΠ»ΠΎΠ΄ΠΊΠΈ ΠΆΡ‘Π»Ρ‚Ρ‹ΠΉ ΠΏΡ€ΠΎΠ²ΠΎΠ΄ β„–8, Π·Π°ΠΈΠ·ΠΎΠ»ΠΈΡ€ΠΎΠ²Π°Π» ΠΊΠΎΠ½Ρ‚Π°ΠΊΡ‚ ΠΈ оставил Π² ΠΆΠ³ΡƒΡ‚Π΅. ВмСсто Π½Π΅Π³ΠΎ вставил Π² ΠΊΠΎΠ»ΠΎΠ΄ΠΊΡƒ кусочСк ΠΏΡ€ΠΎΠ²ΠΎΠ΄Π° Π΄Π»ΠΈΠ½ΠΎΠΉ 8 β€” 10 см. Π•Π³ΠΎ соСдинил с Π·Π΅Π»Ρ‘Π½Ρ‹ΠΌ ΠΏΡ€ΠΎΠ²ΠΎΠ΄ΠΎΠΌ β„–9. ΠŸΠΎΠ»ΡƒΡ‡ΠΈΠ»ΠΎΡΡŒ Ρ‚ΠΎ, Ρ‡Ρ‚ΠΎ Π½ΡƒΠΆΠ½ΠΎ β€” Ρ‚ΡƒΠΌΠ°Π½ΠΊΠΈ Π²ΠΊΠ»ΡŽΡ‡Π°ΡŽΡ‚ΡΡ вмСсто Π±Π»ΠΈΠΆΠ½Π΅Π³ΠΎ свСта.

Π˜Π½Ρ„ΠΎΡ€ΠΌΠ°Ρ†ΠΈΡ ΠΏΠΎ ΠΏΠΎΠ΄ΠΊΠ»ΡŽΡ‡Π΅Π½ΠΈΡŽ взята Π½Π° Ρ„ΠΎΡ€ΡƒΠΌΠ΅ Π“Π°Π·Π΅Π»ΡŒΠΊΠ»ΡƒΠ±Π°.

Π’ΠΎ врСмя Π½Π΅Π½Π°ΡΡ‚ΡŒΡ ΡΡ„Ρ„Π΅ΠΊΡ‚ΠΈΠ²Π½ΠΎΡΡ‚ΡŒ Ρ„Π°Ρ€ Π³ΠΎΠ»ΠΎΠ²Π½ΠΎΠ³ΠΎ свСта Π·Π°ΠΌΠ΅Ρ‚Π½ΠΎ сниТаСтся, Ρ‡Ρ‚ΠΎ Π½Π΅ Ρ‚ΠΎΠ»ΡŒΠΊΠΎ Π΄Π΅Π»Π°Π΅Ρ‚ ΡƒΠΏΡ€Π°Π²Π»Π΅Π½ΠΈΠ΅ Π°Π²Ρ‚ΠΎΠΌΠΎΠ±ΠΈΠ»Π΅ΠΌ ΠΌΠ΅Π½Π΅Π΅ ΠΊΠΎΠΌΡ„ΠΎΡ€Ρ‚Π½Ρ‹ΠΌ, Π½ΠΎ ΠΈ сказываСтся Π½Π° бСзопасности ΠΏΠΎΠ΅Π·Π΄ΠΊΠΈ.

Π‘Π²Π΅Ρ‚ΠΎΠ²ΠΎΠΉ Π»ΡƒΡ‡ Ρ„Π°Ρ€ Π±Π»ΠΈΠΆΠ½Π΅Π³ΠΎ ΠΈ дальнСго свСта отраТаСтся ΠΎΡ‚ капСль доТдя ΠΈ частичСк водяного ΠΏΠ°Ρ€Π° Π²ΠΎ врСмя Ρ‚ΡƒΠΌΠ°Π½Π°, образуя ΠΏΠ»ΠΎΡ‚Π½ΡƒΡŽ Π±Π΅Π»ΡƒΡŽ ΠΏΠ΅Π»Π΅Π½Ρƒ.

Настоящим спасСниСм для водитСля Π² Ρ‚Π°ΠΊΠΈΡ… ΠΏΠΎΠ³ΠΎΠ΄Π½Ρ‹Ρ… условиях станут ΠΏΡ€ΠΎΡ‚ΠΈΠ²ΠΎΡ‚ΡƒΠΌΠ°Π½Π½Ρ‹Π΅ Ρ„Π°Ρ€Ρ‹ (ПВЀ), ΠΊΠΎΡ‚ΠΎΡ€Ρ‹Π΅ Π΄Π°ΡŽΡ‚ плоский ΠΈ ΡˆΠΈΡ€ΠΎΠΊΠΈΠΉ Π³ΠΎΡ€ΠΈΠ·ΠΎΠ½Ρ‚Π°Π»ΡŒΠ½Ρ‹ΠΉ Π»ΡƒΡ‡ свСта.

Он стСлСтся Π½Π°Π΄ Π΄ΠΎΡ€ΠΎΠ³ΠΎΠΉ, Ρ…ΠΎΡ€ΠΎΡˆΠΎ подсвСчивая ΠΎΠ±ΠΎΡ‡ΠΈΠ½Ρƒ, ΡƒΠ»ΡƒΡ‡ΡˆΠ°Ρ Π²ΠΈΠ΄ΠΈΠΌΠΎΡΡ‚ΡŒ самого автомобиля для Π²ΠΎΠ΄ΠΈΡ‚Π΅Π»Π΅ΠΉ встрСчных Π°Π²Ρ‚ΠΎ.

Π¨Ρ‚Π°Ρ‚Π½Ρ‹Π΅ ПВЀ Ρ€Π΅Π΄ΠΊΠΎ Π²ΡΡ‚Ρ€Π΅Ρ‡Π°ΡŽΡ‚ΡΡ Π² Π±Π°Π·ΠΎΠ²Ρ‹Ρ… комплСктациях Π°Π²Ρ‚ΠΎΠΌΠΎΠ±ΠΈΠ»Π΅ΠΉ, ΠΎΠ΄Π½Π°ΠΊΠΎ ΠΏΡ€ΠΈ ΠΆΠ΅Π»Π°Π½ΠΈΠΈ ΠΏΡ€ΠΎΡ‚ΠΈΠ²ΠΎΡ‚ΡƒΠΌΠ°Π½ΠΊΠΈ всСгда ΠΌΠΎΠΆΠ½ΠΎ приобрСсти ΠΎΡ‚Π΄Π΅Π»ΡŒΠ½ΠΎ ΠΈ ΡƒΡΡ‚Π°Π½ΠΎΠ²ΠΈΡ‚ΡŒ ΡΠ°ΠΌΠΎΡΡ‚ΠΎΡΡ‚Π΅Π»ΡŒΠ½ΠΎ. Для этого достаточно ΠΈΠΌΠ΅Ρ‚ΡŒ Π±Π°Π·ΠΎΠ²Ρ‹Π΅ знания Π² области элСктротСхники ΠΈ элСктроники.

Π’ΠΎΡ‚ Ρ‚Π°ΠΊ выглядит схСма ΠΏΠΎΠ΄ΠΊΠ»ΡŽΡ‡Π΅Π½ΠΈΡ ΠΊΠ½ΠΎΠΏΠΊΠΈ ПВЀ Ρ‡Π΅Ρ€Π΅Π· Ρ€Π΅Π»Π΅:

На Π½Π΅ΠΉ всС ΠΏΡ€Π΅Π΄Π΅Π»ΡŒΠ½ΠΎ просто ΠΈ понятно.

  1. Π‘ аккумуляторной Π±Π°Ρ‚Π°Ρ€Π΅ΠΈ плюс Ρ‡Π΅Ρ€Π΅Π· ΠΏΡ€Π΅Π΄ΠΎΡ…Ρ€Π°Π½ΠΈΡ‚Π΅Π»ΡŒ ΠΈΠ΄Π΅Ρ‚ Π½Π° ΠΊΠΎΠ½Ρ‚Π°ΠΊΡ‚ Ρ€Π΅Π»Π΅ (30) ΠΈ Π΄Π°Π»Π΅Π΅ с ΠΊΠΎΠ½Ρ‚Π°ΠΊΡ‚Π° (85) ΠΈΠ΄Π΅Ρ‚ Π½Π° ΠΎΠ΄ΠΈΠ½ ΠΊΠΎΠ½Ρ‚Π°ΠΊΡ‚ ΠΊΠ½ΠΎΠΏΠΊΠΈ Π²ΠΊΠ»ΡŽΡ‡Π΅Π½ΠΈΡ/Π²Ρ‹ΠΊΠ»ΡŽΡ‡Π΅Π½ΠΈΡ ПВЀ.
  2. На Π²Ρ‚ΠΎΡ€ΠΎΠΉ ΠΊΠΎΠ½Ρ‚Π°ΠΊΡ‚ ΠΊΠ½ΠΎΠΏΠΊΠΈ подаСтся минус.
  3. Π”Π°Π»Π΅Π΅ с ΠΊΠΎΠ½Ρ‚Π°ΠΊΡ‚Π° Ρ€Π΅Π»Π΅ (87) плюс ΠΈΠ΄Π΅Ρ‚ Π½Π° Π»Π°ΠΌΠΏΡ‹ ΠΏΡ€ΠΎΡ‚ΠΈΠ²ΠΎΡ‚ΡƒΠΌΠ°Π½Π½Ρ‹Ρ… Ρ„Π°Ρ€.
  4. К ΠΊΠΎΠ½Ρ‚Π°ΠΊΡ‚Ρƒ (86) ΠΏΠΎΠ΄ΠΊΠ»ΡŽΡ‡Π°Π΅Ρ‚ΡΡ минус.

ΠžΠ±Ρ€Π°Ρ‚ΠΈΡ‚Π΅ Π²Π½ΠΈΠΌΠ°Π½ΠΈΠ΅ Π½Π° Ρ‚ΠΎ, Ρ‡Ρ‚ΠΎ каТдая модСль автомобиля ΠΈΠΌΠ΅Π΅Ρ‚ свои особСнности, поэтому схСма ΠΌΠΎΠΆΠ΅Ρ‚ ΠΊΠΎΡ€Ρ€Π΅ΠΊΡ‚ΠΈΡ€ΠΎΠ²Π°Ρ‚ΡŒΡΡ.

Π˜Π½ΡΡ‚Ρ€ΡƒΠΊΡ†ΠΈΡ β€” ΠΊΠ°ΠΊ ΠΏΠΎΠ΄ΠΊΠ»ΡŽΡ‡ΠΈΡ‚ΡŒ ΠΏΡ€ΠΎΡ‚ΠΈΠ²ΠΎΡ‚ΡƒΠΌΠ°Π½ΠΊΠΈ Ρ‡Π΅Ρ€Π΅Π· Ρ€Π΅Π»Π΅ ΠΈ ΠΊΠ½ΠΎΠΏΠΊΡƒ Π½Π° ΠΏΡ€ΠΈΠΌΠ΅Ρ€Π΅ Π¨Π΅Π²Ρ€ΠΎΠ»Π΅ Нива

Как Π²ΠΈΠ΄Π½ΠΎ ΠΈΠ· прСдставлСнной Π²Ρ‹ΡˆΠ΅ схСмы, для ΠΏΠΎΠ΄ΠΊΠ»ΡŽΡ‡Π΅Π½ΠΈΡ ΠΏΡ€ΠΎΡ‚ΠΈΠ²ΠΎΡ‚ΡƒΠΌΠ°Π½ΠΎΠΊ понадобятся:

  • сами ПВЀ;
  • Ρ€Π΅Π»Π΅ ΠΏΡ€ΠΎΡ‚ΠΈΠ²ΠΎΡ‚ΡƒΠΌΠ°Π½Π½Ρ‹Ρ… Ρ„Π°Ρ€;
  • ΠΊΠ½ΠΎΠΏΠΊΠ° Π²ΠΊΠ»ΡŽΡ‡Π΅Π½ΠΈΡ/Π²Ρ‹ΠΊΠ»ΡŽΡ‡Π΅Π½ΠΈΡ Ρ„Π°Ρ€;
  • ΠΏΡ€Π΅Π΄ΠΎΡ…Ρ€Π°Π½ΠΈΡ‚Π΅Π»ΡŒ 10 А;
  • ΠΏΡ€ΠΎΠ²ΠΎΠ΄Π° ΠΈ ΡΠΎΠ΅Π΄ΠΈΠ½ΠΈΡ‚Π΅Π»ΡŒΠ½Ρ‹Π΅ ΠΊΠ»Π΅ΠΌΠΌΡ‹.

Для Ρ€Π°Π±ΠΎΡ‚Ρ‹ Ρ‚Π°ΠΊΠΆΠ΅ понадобятся Π½Π΅ΠΊΠΎΡ‚ΠΎΡ€Ρ‹Π΅ инструмСнты: ΠΎΡ‚Π²Π΅Ρ€Ρ‚ΠΊΠΈ, острый Π½ΠΎΠΆ для зачистки ΠΏΡ€ΠΎΠ²ΠΎΠ΄ΠΎΠ² ΠΈ ΠΈΠ·ΠΎΠ»Π΅Π½Ρ‚Π°.


Π Π°Π±ΠΎΡ‚Ρƒ ΠΏΠΎ установкС ΠΈ ΠΏΠΎΠ΄ΠΊΠ»ΡŽΡ‡Π΅Π½ΠΈΡŽ ΠΏΡ€ΠΎΡ‚ΠΈΠ²ΠΎΡ‚ΡƒΠΌΠ°Π½ΠΎΠΊ Ρ‡Π΅Ρ€Π΅Π· Ρ€Π΅Π»Π΅ ΠΈ ΠΊΠ½ΠΎΠΏΠΊΡƒ выполняСм Π² ΡΠ»Π΅Π΄ΡƒΡŽΡ‰Π΅ΠΉ ΠΏΠΎΡΠ»Π΅Π΄ΠΎΠ²Π°Ρ‚Π΅Π»ΡŒΠ½ΠΎΡΡ‚ΠΈ (рассмотрим Π½Π° ΠΏΡ€ΠΈΠΌΠ΅Ρ€Π΅ Π¨Π΅Π²Ρ€ΠΎΠ»Π΅ Нива):

  1. ΠžΠΏΡ€Π΅Π΄Π΅Π»ΡΠ΅ΠΌ мСсто располоТСния Ρ€Π΅Π»Π΅. ΠŸΠΎΡΠΊΠΎΠ»ΡŒΠΊΡƒ Π΄Π°Π½Π½Ρ‹ΠΉ ΠΊΠΎΠΌΠΏΠΎΠ½Π΅Π½Ρ‚ ΠΈΠΌΠ΅Π΅Ρ‚ нСбольшиС Ρ€Π°Π·ΠΌΠ΅Ρ€Ρ‹, Π΅Π³ΠΎ ΠΌΠΎΠΆΠ½ΠΎ Π±Π΅Π· Ρ‚Ρ€ΡƒΠ΄Π° ΡΠΏΡ€ΡΡ‚Π°Ρ‚ΡŒ Π·Π° ΠΏΡ€ΠΈΠ±ΠΎΡ€Π½ΠΎΠΉ панСлью.
  2. ΠžΠΏΡ€Π΅Π΄Π΅Π»ΡΠ΅ΠΌ мСсто располоТСния ΠΊΠ½ΠΎΠΏΠΊΠΈ Π²ΠΊΠ»ΡŽΡ‡Π΅Π½ΠΈΡ/Π²Ρ‹ΠΊΠ»ΡŽΡ‡Π΅Π½ΠΈΡ ПВЀ;
  3. ΠžΡ‚ΠΌΠ΅Ρ€ΡΠ΅ΠΌ Π½Π΅ΠΎΠ±Ρ…ΠΎΠ΄ΠΈΠΌΡƒΡŽ для ΠΏΠΎΠ΄ΠΊΠ»ΡŽΡ‡Π΅Π½ΠΈΡ Π΄Π»ΠΈΠ½Ρƒ ΠΏΡ€ΠΎΠ²ΠΎΠ΄ΠΎΠ²;
  4. Π”Π°Π»Π΅Π΅ протягиваСм ΠΌΠ΅Π΄Π½Ρ‹ΠΉ ΠΏΡ€ΠΎΠ²ΠΎΠ΄ ΠΎΡ‚ ΠΊΠΎΠ½Ρ‚Π°ΠΊΡ‚Π° 30 ΠΊ аккумулятору (+), ΠΏΡ€Π΅Π΄Π²Π°Ρ€ΠΈΡ‚Π΅Π»ΡŒΠ½ΠΎ установив Π² ΡƒΠ΄ΠΎΠ±Π½ΠΎΠΌ мСстС ΠΏΡ€Π΅Π΄ΠΎΡ…Ρ€Π°Π½ΠΈΡ‚Π΅Π»ΡŒ Π½Π° 10 АмпСр;
  5. ΠŸΠΎΠ΄ΠΊΠ»ΡŽΡ‡Π°Π΅ΠΌ ΠΊΠ½ΠΎΠΏΠΊΡƒ ΠΏΡ€ΠΎΡ‚ΠΈΠ²ΠΎΡ‚ΡƒΠΌΠ°Π½Π½Ρ‹Ρ… Ρ„Π°Ρ€ ΠΊ ΠΊΠΎΠ½Ρ‚Π°ΠΊΡ‚Ρƒ 85 ΠΈ устанавливаСм Π² Π²Ρ‹Π±Ρ€Π°Π½Π½ΠΎΠ΅ мСсто. Как ΠΏΡ€Π°Π²ΠΈΠ»ΠΎ, ΠΊΠ½ΠΎΠΏΠΊΠ° ΠΏΠ΅Ρ€Π΅Π΄Π½ΠΈΡ… ПВЀ устанавливаСтся Π½Π° мСсто ΡˆΡ‚Π°Ρ‚Π½Ρ‹Ρ… ΠΊΠ½ΠΎΠΏΠΎΠΊ, Π³Π΄Π΅ стоит Π·Π°Π³Π»ΡƒΡˆΠΊΠ°;
  6. ΠšΠΎΠ½Ρ‚Π°ΠΊΡ‚ Ρ€Π΅Π»Π΅ 86 Π² любом ΡƒΠ΄ΠΎΠ±Π½ΠΎΠΌ мСстС соСдиняСм с массой;
  7. УстанавливаСм ΠΏΡ€ΠΎΡ‚ΠΈΠ²ΠΎΡ‚ΡƒΠΌΠ°Π½Π½Ρ‹Π΅ Ρ„Π°Ρ€Ρ‹. ΠŸΡ€ΠΎΠΈΠ·Π²ΠΎΠ΄ΠΈΡ‚Π΅Π»ΠΈ ΠΏΠΎΠ·Π°Π±ΠΎΡ‚ΠΈΠ»ΠΈΡΡŒ ΠΎ Ρ‚ΠΎΠΌ, Ρ‡Ρ‚ΠΎΠ±Ρ‹ Ρ‚Π΅ Π²Π»Π°Π΄Π΅Π»ΡŒΡ†Ρ‹, ΠΊΠΎΡ‚ΠΎΡ€Ρ‹Π΅ ΠΏΠΎΠΆΠ΅Π»Π°ΡŽΡ‚ ΡƒΡΡ‚Π°Π½ΠΎΠ²ΠΈΡ‚ΡŒ ΠΏΡ€ΠΎΡ‚ΠΈΠ²ΠΎΡ‚ΡƒΠΌΠ°Π½Π½Ρ‹Π΅ Ρ„Π°Ρ€Ρ‹, Π½Π΅ Π²Ρ‹Π΄ΡƒΠΌΡ‹Π²Π°Π»ΠΈ «вСлосипСд». Π’ ΠΏΠ΅Ρ€Π΅Π΄Π½ΠΈΡ… Π±Π°ΠΌΠΏΠ΅Ρ€Π°Ρ… Π¨Π΅Π²Ρ€ΠΎΠ»Π΅ Нива для этого прСдусмотрСны ΡΠΏΠ΅Ρ†ΠΈΠ°Π»ΡŒΠ½Ρ‹Π΅ ниши, ΠΊΠΎΡ‚ΠΎΡ€Ρ‹Π΅ Π·Π°ΠΊΡ€Ρ‹Ρ‚Ρ‹ Π·Π°Π³Π»ΡƒΡˆΠΊΠ°ΠΌΠΈ. Для Ρ‚ΠΎΠ³ΠΎ Ρ‡Ρ‚ΠΎΠ±Ρ‹ ΠΈΡ… ΡΠ½ΡΡ‚ΡŒ Π½Π΅ΠΎΠ±Ρ…ΠΎΠ΄ΠΈΠΌΠΎ с Π²Π½ΡƒΡ‚Ρ€Π΅Π½Π½Π΅ΠΉ стороны Π±Π°ΠΌΠΏΠ΅Ρ€Π° ΠΎΡ‚ΠΊΡ€ΡƒΡ‚ΠΈΡ‚ΡŒ саморСзы. ВставляСм Ρ„Π°Ρ€Ρ‹ ΠΈ смотрим Π½Π° обозначСния ΠΏΡ€ΠΎΠ²ΠΎΠ΄ΠΎΠ².
  8. Плюсовой ΠΏΡ€ΠΎΠ²ΠΎΠ΄ Π½Π° Π½ΠΈΡ… ΠΏΠΎΠ΄Π°Π΅ΠΌ с ΠΊΠΎΠ½Ρ‚Π°ΠΊΡ‚Π° 87, минус β€” с ΠΊΡƒΠ·ΠΎΠ²Π° Π°Π²Ρ‚ΠΎ;
  9. ΠŸΡ€ΠΎΠ²ΠΎΠ΄ΠΈΠΌ тСстированиС.

Π’Π˜Π”Π•Πž ИНБВРУКЦИЯ
» alt=»»>

ΠŸΠΎΠ»Π΅Π·Π½Ρ‹Π΅ ΠΏΡ€Π°Π²ΠΈΠ»Π° β€” Π½Π°Π΄ΠΎ Π·Π½Π°Ρ‚ΡŒ

ΠŸΡ€ΠΈ Π²Ρ‹Π±ΠΎΡ€Π΅ самих ΠΏΡ€ΠΎΡ‚ΠΈΠ²ΠΎΡ‚ΡƒΠΌΠ°Π½ΠΎΠΊ ΠΈ мСста для ΠΈΡ… располоТСния слСдуСт Π²Π½ΠΈΠΌΠ°Ρ‚Π΅Π»ΡŒΠ½ΠΎ ΠΎΠ·Π½Π°ΠΊΠΎΠΌΠΈΡ‚ΡŒΡΡ с принятыми Π½ΠΎΡ€ΠΌΠ°ΠΌΠΈ, ΠΊΠΎΡ‚ΠΎΡ€Ρ‹Π΅ строго прописаны Π² ΠŸΠ”Π”.

Π’Π°ΠΊ, Π½Π° Ρ‚Π΅Ρ€Ρ€ΠΈΡ‚ΠΎΡ€ΠΈΠΈ России допускаСтся установка Ρ‚ΠΎΠ»ΡŒΠΊΠΎ заводских ПВЀ, ΠΏΡ€ΠΎΡˆΠ΅Π΄ΡˆΠΈΡ… ΡΠ΅Ρ€Ρ‚ΠΈΡ„ΠΈΠΊΠ°Ρ†ΠΈΡŽ. ΠŸΠΎΠ΄Ρ‚Π²Π΅Ρ€ΠΆΠ΄Π΅Π½ΠΈΠ΅ΠΌ этого являСтся Π·Π½Π°ΠΊ Π•22 Π² ΠΊΡ€ΡƒΠ³Π΅, ΠΊΠΎΡ‚ΠΎΡ€Ρ‹ΠΉ наносится Π½Π° корпус ПВЀ.

  1. Π‘Π°ΠΌΠΈΡ… ΠΏΡ€ΠΎΡ‚ΠΈΠ²ΠΎΡ‚ΡƒΠΌΠ°Π½ΠΎΠΊ Π΄ΠΎΠ»ΠΆΠ½ΠΎ Π±Ρ‹Ρ‚ΡŒ Π΄Π²Π΅ ΡˆΡ‚ΡƒΠΊΠΈ β€” Π½Π΅ большС ΠΈ Π½Π΅ мСньшС.
  2. УстановлСны ΠΎΠ½ΠΈ Π΄ΠΎΠ»ΠΆΠ½Ρ‹ Π±Ρ‹Ρ‚ΡŒ Π½Π° расстоянии Π½Π΅ Π±ΠΎΠ»Π΅Π΅ 40 см ΠΎΡ‚ ΠΊΡ€ΠΎΠΌΠΊΠΈ рассСиватСля Π±ΠΎΠΊΠΎΠ²ΠΎΠ³ΠΎ Π³Π°Π±Π°Ρ€ΠΈΡ‚Π° ΠΈ Π½Π΅ Π½ΠΈΠΆΠ΅ 25 см ΠΎΡ‚ уровня повСрхности Π΄ΠΎΡ€ΠΎΠΆΠ½ΠΎΠ³ΠΎ покрытия. Π£ Π±ΠΎΠ»ΡŒΡˆΠΈΠ½ΡΡ‚Π²Π° соврСмСнных Π°Π²Ρ‚ΠΎΠΌΠΎΠ±ΠΈΠ»Π΅ΠΉ Π½Π° Π±Π°ΠΌΠΏΠ΅Ρ€Π΅ прСдусмотрСны ΡˆΡ‚Π°Ρ‚Π½Ρ‹Π΅ мСста для установки ПВЀ.
  3. Если Π΄Π°Π½Π½ΠΎΠΉ ΠΊΠΎΠΌΠΏΠ»Π΅ΠΊΡ‚Π°Ρ†ΠΈΠ΅ΠΉ ΠΏΡ€ΠΎΡ‚ΠΈΠ²ΠΎΡ‚ΡƒΠΌΠ°Π½ΠΊΠΈ Π½Π΅ прСдусмотрСны, Π½Π° ΠΈΡ… мСсто ставятся Π·Π°Π³Π»ΡƒΡˆΠΊΠΈ, ΠΊΠΎΡ‚ΠΎΡ€Ρ‹Π΅ Π»Π΅Π³ΠΊΠΎ ΠΈΠ·Π²Π»Π΅ΠΊΠ°ΡŽΡ‚ΡΡ ΠΈ Π½Π° ΠΈΡ… мСсто ΠΌΠΎΠ½Ρ‚ΠΈΡ€ΡƒΡŽΡ‚ΡΡ ПВЀ.

Π’ Π±ΠΎΠ»ΡŒΡˆΠΈΠ½ΡΡ‚Π²Π΅ ΠΏΡ€ΠΎΡ‚ΠΈΠ²ΠΎΡ‚ΡƒΠΌΠ°Π½Π½Ρ‹Ρ… Ρ„Π°Ρ€, прСдставлСнных Π½Π° Ρ€Ρ‹Π½ΠΊΠ΅, примСняСтся стандартная однонитСвая Π»Π°ΠΌΠΏΠ° ΠΊΠ°Ρ‚Π΅Π³ΠΎΡ€ΠΈΠΈ Н1. Π’ΠΊΠ»ΡŽΡ‡Π°Ρ‚ΡŒΡΡ ПВЀ Π΄ΠΎΠ»ΠΆΠ½Ρ‹ Ρ‚ΠΎΠ»ΡŒΠΊΠΎ вмСстС с Π³Π°Π±Π°Ρ€ΠΈΡ‚Π½Ρ‹ΠΌΠΈ огнями.

ΠΠ΅ΠΏΡ€Π°Π²ΠΈΠ»ΡŒΠ½Π°Ρ установка ΠΏΡ€ΠΎΡ‚ΠΈΠ²ΠΎΡ‚ΡƒΠΌΠ°Π½Π½Ρ‹Π΅ Ρ„Π°Ρ€ ΠΌΠΎΠΆΠ΅Ρ‚ привСсти ΠΊ возникновСнию Π”Π’ΠŸ ΠΈΠ»ΠΈ Π±ΡƒΠ΄Π΅Ρ‚ Π²Ρ‹Π·Ρ‹Π²Π°Ρ‚ΡŒ дискомфорт Ρƒ Π²ΠΎΠ΄ΠΈΡ‚Π΅Π»Π΅ΠΉ встрСчных Π°Π²Ρ‚ΠΎΠΌΠΎΠ±ΠΈΠ»Π΅ΠΉ. Π‘ΠΎΠ±Π»ΡŽΠ΄Π°ΠΉΡ‚Π΅ всС ΠΏΡ€Π°Π²ΠΈΠ»Π°, ΠΈ Π²Ρ‹ Π½ΠΈΠΊΠΎΠ³Π΄Π° Π½Π΅ ΠΏΠΎΠΏΠ°Π΄Π΅Ρ‚Π΅ Π² Π½Π΅ΠΏΡ€ΠΈΡΡ‚Π½ΡƒΡŽ ΡΠΈΡ‚ΡƒΠ°Ρ†ΠΈΡŽ.

Π€Π°Ρ€Ρ‹ ΠΏΡ€ΠΎΡ‚ΠΈΠ²ΠΎΡ‚ΡƒΠΌΠ°Π½Π½Ρ‹Π΅ Π½Π° «Π“Π°Π·Π΅Π»ΡŒ» ΡƒΡΡ‚Π°Π½Π°Π²Π»ΠΈΠ²Π°ΡŽΡ‚ΡΡ Π½Π΅ для красоты, Π° ΠΈΠ· нСобходимости ΡƒΠ»ΡƒΡ‡ΡˆΠΈΡ‚ΡŒ Π²ΠΈΠ΄ΠΈΠΌΠΎΡΡ‚ΡŒ Π½Π° Π΄ΠΎΡ€ΠΎΠ³Π΅ Π²ΠΎ врСмя Ρ‚ΡƒΠΌΠ°Π½Π° ΠΈΠ»ΠΈ доТдя ΠΈ снСга. Однако Π½Π΅ΠΊΠΎΡ‚ΠΎΡ€Ρ‹Π΅ ΠΌΠΎΠ΄Π΅Π»ΠΈ Π½Π΅ ΠΊΠΎΠΌΠΏΠ»Π΅ΠΊΡ‚ΡƒΡŽΡ‚ΡΡ ΠΈΠΌΠΈ Π·Π°Π²ΠΎΠ΄ΠΎΠΌ-ΠΈΠ·Π³ΠΎΡ‚ΠΎΠ²ΠΈΡ‚Π΅Π»Π΅ΠΌ. О Ρ‚ΠΎΠΌ, ΠΊΠ°ΠΊ самому ΠΏΡ€Π°Π²ΠΈΠ»ΡŒΠ½ΠΎ Π²Ρ‹Π±Ρ€Π°Ρ‚ΡŒ, ΡƒΡΡ‚Π°Π½ΠΎΠ²ΠΈΡ‚ΡŒ ΠΈ ΠΏΠΎΠ΄ΠΊΠ»ΡŽΡ‡ΠΈΡ‚ΡŒ Ρ„Π°Ρ€Ρ‹, ΠΈ ΠΏΠΎΠΉΠ΄Π΅Ρ‚ Ρ€Π΅Ρ‡ΡŒ Π½ΠΈΠΆΠ΅.

О ΠΏΡ€ΠΎΡ‚ΠΈΠ²ΠΎΡ‚ΡƒΠΌΠ°Π½Π½Ρ‹Ρ… Ρ„Π°Ρ€Π°Ρ…

ΠžΠ±Ρ‹Ρ‡Π½Ρ‹Π΅ ΠΏΠ΅Ρ€Π΅Π΄Π½ΠΈΠ΅ Ρ„Π°Ρ€Ρ‹ Π² Ρ‚ΡƒΠΌΠ°Π½, доТдь, ΠΎΠ±ΠΈΠ»ΡŒΠ½Ρ‹ΠΉ снСгопад Π΄Π°ΡŽΡ‚ ΠΌΠ°Π»ΠΎ ΠΏΠΎΠ»ΡŒΠ·Ρ‹: ΠΎΡ‚ Π½ΠΈΡ… пространство ΠΏΠ΅Ρ€Π΅Π΄ машиной Π½Π΅ просматриваСтся, ΠΎΠ³Ρ€Π°Π½ΠΈΡ‡Π΅Π½ΠΎ бСлСсой ΠΏΠ΅Π»Π΅Π½ΠΎΠΉ, особСнно Π² дальнСм Ρ€Π΅ΠΆΠΈΠΌΠ΅. ΠŸΡ€ΠΎΠΈΡΡ…ΠΎΠ΄ΠΈΡ‚ это ΠΈΠ·-Π·Π° отраТСния свСта ΠΎΡ‚ капСль Ρ‚ΡƒΠΌΠ°Π½Π°, доТдя, снСТинок.

Π’ Ρ‚Π°ΠΊΠΎΠΉ ситуации Π½Π΅Π·Π°ΠΌΠ΅Π½ΠΈΠΌΡ‹ΠΌ ΠΏΠΎΠΌΠΎΡ‰Π½ΠΈΠΊΠΎΠΌ слуТит Π±Π»ΠΎΠΊ ΠΏΡ€ΠΎΡ‚ΠΈΠ²ΠΎΡ‚ΡƒΠΌΠ°Π½Π½Ρ‹Ρ… Ρ„Π°Ρ€ (ПВЀ), установка ΠΊΠΎΡ‚ΠΎΡ€ΠΎΠ³ΠΎ Π½Π° любой Π°Π²Ρ‚ΠΎΠΌΠΎΠ±ΠΈΠ»ΡŒ ΠΏΠΎΠ·Π²ΠΎΠ»ΠΈΡ‚ Ρ€Π°Π²Π½ΠΎΠΌΠ΅Ρ€Π½ΠΎ ΠΎΡΠ²Π΅Ρ‚ΠΈΡ‚ΡŒ Π΄ΠΎΡ€ΠΎΠ³Ρƒ ΠΈ ΠΈΠ·Π±Π΅ΠΆΠ°Ρ‚ΡŒ опасных ситуаций.

Но Π²Π°ΠΆΠ½ΠΎ Π½Π΅ Π·Π°Π±Ρ‹Π²Π°Ρ‚ΡŒ, Ρ‡Ρ‚ΠΎ Ρ„Π°Ρ€Π° Ρ„Π°Ρ€Π΅ Ρ€ΠΎΠ·Π½ΡŒ, ΠΈ Π·Π½Π°Ρ‚ΡŒ ΠΊΠΎΠ΅-Ρ‡Ρ‚ΠΎ ΠΎΠ± этом Π½Π΅ ΠΏΠΎΠΌΠ΅ΡˆΠ°Π΅Ρ‚ Π»ΡŽΠ±ΠΎΠΌΡƒ Π²ΠΎΠ΄ΠΈΡ‚Π΅Π»ΡŽ.

НСплохими характСристиками ΠΎΠ±Π»Π°Π΄Π°ΡŽΡ‚ российскиС ΠΏΡ€ΠΎΡ‚ΠΈΠ²ΠΎΡ‚ΡƒΠΌΠ°Π½Π½Ρ‹Π΅ Ρ„Π°Ρ€Ρ‹, ΠΏΡ€ΠΎΠΈΠ·Π²ΠΎΠ΄ΠΈΠΌΡ‹Π΅ Π½Π° прСдприятиях Β«Π‘ΠΎΡˆ-Рязань», «АвтосвСт». Π’ΠΏΠΎΠ»Π½Π΅ Π±ΡŽΠ΄ΠΆΠ΅Ρ‚Π½Π°Ρ ΡΡ‚ΠΎΠΈΠΌΠΎΡΡ‚ΡŒ – Π΅Ρ‰Π΅ ΠΎΠ΄ΠΈΠ½ плюс Ρ‚Π°ΠΊΠΈΡ… ΠΈΠ·Π΄Π΅Π»ΠΈΠΉ.

Π€Π°Ρ€Ρ‹ азиатских Ρ„ΠΈΡ€ΠΌ Al Khateeb ΠΈ Saca Π±ΠΎΠ»Π΅Π΅ ΠΏΡ€ΠΈΠ²Π»Π΅ΠΊΠ°Ρ‚Π΅Π»ΡŒΠ½Ρ‹ своим внСшним Π²ΠΈΠ΄ΠΎΠΌ, Π½ΠΎ ΠΈΡ… ΡΡ‚ΠΎΠΈΠΌΠΎΡΡ‚ΡŒ Π²Π΄Π²ΠΎΠ΅ Π²Ρ‹ΡˆΠ΅ российских, Π° большая ΠΌΠΎΡ‰Π½ΠΎΡΡ‚ΡŒ часто Π²Π΅Π΄Π΅Ρ‚ ΠΊ ΠΏΠ΅Ρ€Π΅Π³Ρ€Π΅Π²Ρƒ ΠΈ Π΄Π΅Ρ„ΠΎΡ€ΠΌΠ°Ρ†ΠΈΠΈ пластмассовых Π΄Π΅Ρ‚Π°Π»Π΅ΠΉ.

Π‘Π°ΠΌΠΎΠΉ качСствСнной ΠΏΡ€ΠΎΡ‚ΠΈΠ²ΠΎΡ‚ΡƒΠΌΠ°Π½Π½ΠΎΠΉ ΠΎΠΏΡ‚ΠΈΠΊΠΎΠΉ ΡΡ‡ΠΈΡ‚Π°ΡŽΡ‚ΡΡ ΠΏΡ€ΠΈΠ±ΠΎΡ€Ρ‹ Π½Π΅ΠΌΠ΅Ρ†ΠΊΠΎΠΉ ΠΊΠΎΠΌΠΏΠ°Π½ΠΈΠΈ Hella, Π½ΠΎ это ΡƒΠ΄ΠΎΠ²ΠΎΠ»ΡŒΡΡ‚Π²ΠΈΠ΅ ΠΈΠ·-Π·Π° стоимости доступно Π½Π΅ всякому Π²ΠΎΠ΄ΠΈΡ‚Π΅Π»ΡŽ.

ΠŸΡ€ΠΈ ΠΏΠΎΠΊΡƒΠΏΠΊΠ΅ ΠΏΡ€Π΅ΠΆΠ΄Π΅ всСго ΠΎΠ±Ρ€Π°Ρ‰Π°ΠΉΡ‚Π΅ Π²Π½ΠΈΠΌΠ°Π½ΠΈΠ΅ Π½Π° внСшний Π²ΠΈΠ΄ Ρ„Π°Ρ€Ρ‹ (ΠΊΠΎΠΌΠΏΠ»Π΅ΠΊΡ‚Π°): Β«ΠΏΡ€ΠΎΡ‚ΠΈΠ²ΠΎΡ‚ΡƒΠΌΠ°Π½ΠΊΠΈΒ» ΠΈΠΌΠ΅ΡŽΡ‚ ΠΌΠ°Ρ€ΠΊΠ΅Ρ€ Π² Π²ΠΈΠ΄Π΅ Π±ΡƒΠΊΠ²Ρ‹ Β«Π’Β». Π”Π°Π»Π΅Π΅ провСряСтся Π³Π΅Ρ€ΠΌΠ΅Ρ‚ΠΈΡ‡Π½ΠΎΡΡ‚ΡŒ ΠΌΠ΅ΠΆΠ΄Ρƒ рассСиватСлСм ΠΈ ΠΎΡ‚Ρ€Π°ΠΆΠ°Ρ‚Π΅Π»Π΅ΠΌ (корпусом). ΠΠ°Ρ€ΡƒΡˆΠ΅Π½ΠΈΠ΅ нСпроницаСмости этого соСдинСния ΠΏΡ€ΠΈΠ²Π΅Π΄Π΅Ρ‚ ΠΊ ΡΠΎΠΊΡ€Π°Ρ‰Π΅Π½ΠΈΡŽ срока слуТбы Ρ„Π°Ρ€Ρ‹.

Π’ΠΈΠ΄Ρ‹ Β«ΠΏΡ€ΠΎΡ‚ΠΈΠ²ΠΎΡ‚ΡƒΠΌΠ°Π½ΠΎΠΊΒ»

По Ρ„ΠΎΡ€ΠΌΠ΅ исполнСния Ρ„Π°Ρ€Ρ‹ ΠΏΡ€ΠΎΡ‚ΠΈΠ²ΠΎΡ‚ΡƒΠΌΠ°Π½Π½Ρ‹Π΅ Π½Π° Β«Π“Π°Π·Π΅Π»ΡŒΒ» дСлятся Π½Π° ΠΏΡ€ΡΠΌΠΎΡƒΠ³ΠΎΠ»ΡŒΠ½Ρ‹Π΅, ΠΊΠ²Π°Π΄Ρ€Π°Ρ‚Π½Ρ‹Π΅ ΠΈ ΠΎΠ²Π°Π»ΡŒΠ½Ρ‹Π΅, Π½ΠΎ Ρ„ΠΎΡ€ΠΌΠ° корпуса Π½Π΅ ΠΈΠ³Ρ€Π°Π΅Ρ‚ особой Ρ€ΠΎΠ»ΠΈ ΠΈ Π½Π΅ Π΄Π°Π΅Ρ‚ ΠΊΠ°ΠΊΠΈΡ…-Ρ‚ΠΎ особых прСимущСств ΠΎΠ΄Π½ΠΎΠΉ ΠΏΠ΅Ρ€Π΅Π΄ Π΄Ρ€ΡƒΠ³ΠΈΠΌΠΈ.

Β«ΠŸΡ€ΠΎΡ‚ΠΈΠ²ΠΎΡ‚ΡƒΠΌΠ°Π½ΠΊΠΈΒ» ΠΌΠΎΠ³ΡƒΡ‚ Ρ€Π°Π·Π»ΠΈΡ‡Π°Ρ‚ΡŒΡΡ ΠΏΠΎ оптичСским схСмам. Благодаря использованию Ρ€Π°Π·Π½Ρ‹Ρ… Π²Π°Ρ€ΠΈΠ°Π½Ρ‚ΠΎΠ² Ρ€Π΅ΡˆΠ°Π΅Ρ‚ΡΡ Π² Ρ€Π°Π·Π»ΠΈΡ‡Π½ΠΎΠΉ стСпСни вопрос ΠΎΠΏΡ‚ΠΈΠΌΠ°Π»ΡŒΠ½ΠΎΠ³ΠΎ ΡΠΎΠΎΡ‚Π½ΠΎΡˆΠ΅Π½ΠΈΡ максимального свСтового ΠΏΠΎΡ‚ΠΎΠΊΠ° ΠΈ минимально расходуСмой мощности Π½Π° Π΅Π³ΠΎ обСспСчСниС.

НаибольшСС распространСниС ΠΏΠΎΠ»ΡƒΡ‡ΠΈΠ»ΠΈ Ρ„Π°Ρ€Ρ‹, ΠΈΠΌΠ΅ΡŽΡ‰ΠΈΠ΅ параболичСский ΠΎΡ‚Ρ€Π°ΠΆΠ°Ρ‚Π΅Π»ΡŒ. ΠŸΡ€ΠΈ этой схСмС мСсто установки Π»Π°ΠΌΠΏΠΎΡ‡ΠΊΠΈ совмСщаСтся с Ρ„ΠΎΠΊΠ°Π»ΡŒΠ½ΠΎΠΉ Ρ‚ΠΎΡ‡ΠΊΠΎΠΉ, Ρ‡Ρ‚ΠΎ позволяСт Ρ„ΠΎΡ€ΠΌΠΈΡ€ΠΎΠ²Π°Ρ‚ΡŒ свСтовой ΠΏΠΎΡ‚ΠΎΠΊ Π² Π²ΠΈΠ΄Π΅ Π³ΠΎΡ€ΠΈΠ·ΠΎΠ½Ρ‚Π°Π»ΡŒΠ½ΠΎΠΉ полосы. ΠžΡΠΎΠ±Ρ‹ΠΉ экран ΠΎΠ³Ρ€Π°Π½ΠΈΡ‡ΠΈΠ²Π°Π΅Ρ‚ Π΅Π³ΠΎ распространСниС Π²Π²Π΅Ρ€Ρ…. ΠšΠŸΠ” Ρ„Π°Ρ€ составляСт 27%.

Π‘ΠΎΠ»Π΅Π΅ высокий ΠšΠŸΠ” (Π΄ΠΎ 45%) ΠΈΠΌΠ΅ΡŽΡ‚ ΠΏΡ€ΠΎΡ‚ΠΈΠ²ΠΎΡ‚ΡƒΠΌΠ°Π½Π½Ρ‹Π΅ Ρ„Π°Ρ€Ρ‹ с ΠΎΡ‚Ρ€Π°ΠΆΠ°Ρ‚Π΅Π»Π΅ΠΌ свободной Ρ„ΠΎΡ€ΠΌΡ‹. ΠšΠΎΠ½ΡΡ‚Ρ€ΡƒΠΊΡ†ΠΈΡ позволяСт ΠΈΡΠΏΠΎΠ»ΡŒΠ·ΠΎΠ²Π°Ρ‚ΡŒ Π·Π½Π°Ρ‡ΠΈΡ‚Π΅Π»ΡŒΠ½Ρ‹ΠΉ объСм свСта, обСспСчивая ΠΌΠ°ΠΊΡΠΈΠΌΠ°Π»ΡŒΠ½ΡƒΡŽ ΠΎΠ±Π»Π°ΡΡ‚ΡŒ Π΅Π³ΠΎ распрСдСлСния. Однако Ρ„Π°Ρ€Ρ‹ с этой оптичСской схСмой ΠΈΠΌΠ΅ΡŽΡ‚ Π²Ρ‹ΡΠΎΠΊΡƒΡŽ ΡΡ‚ΠΎΠΈΠΌΠΎΡΡ‚ΡŒ ΠΈ Π½Π΅ ΠΏΠΎΠ»ΡƒΡ‡ΠΈΠ»ΠΈ ΠΏΠΎΠΊΠ° ΡˆΠΈΡ€ΠΎΠΊΠΎΠ³ΠΎ распространСния.

Π’Ρ‹Π±ΠΎΡ€ мСста установки Ρ„Π°Ρ€

Π’Π°ΠΆΠ½Ρ‹ΠΌ ΠΌΠΎΠΌΠ΅Π½Ρ‚ΠΎΠΌ Π² ΠΊΠΎΠΌΠΏΠ»Π΅ΠΊΡ‚Π°Ρ†ΠΈΠΈ любого автомобиля являСтся установка ΠΏΡ€ΠΎΡ‚ΠΈΠ²ΠΎΡ‚ΡƒΠΌΠ°Π½Π½Ρ‹Ρ… Ρ„Π°Ρ€. Β«Π“Π°Π·Π΅Π»ΡŒΒ» Π² этом вопросС Π½Π΅ являСтся ΠΈΡΠΊΠ»ΡŽΡ‡Π΅Π½ΠΈΠ΅ΠΌ, принимая Π²ΠΎ Π²Π½ΠΈΠΌΠ°Π½ΠΈΠ΅ коммСрчСский всСпогодный Ρ…Π°Ρ€Π°ΠΊΡ‚Π΅Ρ€ Π΅Π΅ эксплуатации.

БущСствуСт Π΄Π²Π° Π²Π°Ρ€ΠΈΠ°Π½Ρ‚Π° установки Ρ„Π°Ρ€ Π½Π° Β«Π“Π°Π·Π΅Π»ΠΈΒ»: Π½Π° Π±Π°ΠΌΠΏΠ΅Ρ€Π΅ ΠΈ Π½Π° ΠΊΡ€Ρ‹ΡˆΠ΅. Π’Ρ€Π°Π΄ΠΈΡ†ΠΈΠΎΠ½Π½ΠΎ ΠΈΡΠΏΠΎΠ»ΡŒΠ·ΡƒΠ΅Ρ‚ΡΡ ΠΏΠ΅Ρ€Π²Ρ‹ΠΉ.

Π’Ρ‚ΠΎΡ€ΠΎΠΉ Π²Π°Ρ€ΠΈΠ°Π½Ρ‚ ΠΏΠ»ΠΎΡ…ΠΎ согласуСтся с ΠΏΡ€Π°Π²ΠΈΠ»Π°ΠΌΠΈ Π΄ΠΎΡ€ΠΎΠΆΠ½ΠΎΠ³ΠΎ двиТСния, Ρ‚Π°ΠΊ ΠΊΠ°ΠΊ этот Π΄ΠΎΠΊΡƒΠΌΠ΅Π½Ρ‚ устанавливаСт ограничСния ΠΏΠΎ Ρ€Π°Π·ΠΌΠ΅Ρ‰Π΅Π½ΠΈΡŽ Π½Π° Π°Π²Ρ‚ΠΎΠΌΠΎΠ±ΠΈΠ»Π΅ Π½Π΅ прСдусмотрСнного Π·Π°Π²ΠΎΠ΄ΠΎΠΌ ΠΎΡΠ²Π΅Ρ‚ΠΈΡ‚Π΅Π»ΡŒΠ½ΠΎΠ³ΠΎ оборудования. Установка Ρ„Π°Ρ€ Π½Π° ΠΊΡ€Ρ‹ΡˆΠ΅ Ρ€Π°Π·Ρ€Π΅ΡˆΠ΅Π½Π° Ρ‚ΠΎΠ»ΡŒΠΊΠΎ Π² случаС эксплуатации транспортного срСдства Π² слоТных условиях: Π³Ρ€Π°Π²ΠΈΠΉΠ½ΠΎΠ΅ Π΄ΠΎΡ€ΠΎΠΆΠ½ΠΎΠ΅ ΠΏΠΎΠΊΡ€Ρ‹Ρ‚ΠΈΠ΅, Π±Π΅Π·Π΄ΠΎΡ€ΠΎΠΆΡŒΠ΅.

ΠŸΠ΅Ρ€Π²Ρ‹ΠΉ Π²Π°Ρ€ΠΈΠ°Π½Ρ‚ Ρ‚ΠΎΠΆΠ΅ ΠΈΠΌΠ΅Π΅Ρ‚ ограничСния: ΡƒΡΡ‚Π°Π½ΠΎΠ²ΠΈΡ‚ΡŒ ΠΏΡ€ΠΎΡ‚ΠΈΠ²ΠΎΡ‚ΡƒΠΌΠ°Π½Π½Ρ‹Π΅ Ρ„Π°Ρ€Ρ‹ Π½Π° Β«Π“Π°Π·Π΅Π»ΡŒΒ» ΠΌΠΎΠΆΠ½ΠΎ Π½Π΅ Π²Ρ‹ΡˆΠ΅ уровня ΠΎΠ±Ρ‹Ρ‡Π½Ρ‹Ρ… Ρ„Π°Ρ€, ΠΏΡ€ΠΈ этом Π³Π°Π±Π°Ρ€ΠΈΡ‚Ρ‹ автомобиля Π½Π΅ Π½Π°Ρ€ΡƒΡˆΠ°ΡŽΡ‚ΡΡ Π±ΠΎΠ»Π΅Π΅ 400 ΠΌΠΌ, Π° расстояниС ΠΎΡ‚ Ρ„Π°Ρ€ Π΄ΠΎ Π΄ΠΎΡ€ΠΎΠΆΠ½ΠΎΠ³ΠΎ покрытия Π½Π΅ Π΄ΠΎΠ»ΠΆΠ½ΠΎ ΡΠΎΡΡ‚Π°Π²Π»ΡΡ‚ΡŒ ΠΌΠ΅Π½Π΅Π΅ 250 ΠΌΠΌ.

Нормативы ΠΏΠΎ установкС

Π€Π°Ρ€Ρ‹ ΠΏΡ€ΠΎΡ‚ΠΈΠ²ΠΎΡ‚ΡƒΠΌΠ°Π½Π½Ρ‹Π΅ Π½Π° Β«Π“Π°Π·Π΅Π»ΡŒΒ» ΡƒΡΡ‚Π°Π½Π°Π²Π»ΠΈΠ²Π°ΡŽΡ‚ΡΡ с ΠΎΠ±ΡΠ·Π°Ρ‚Π΅Π»ΡŒΠ½Ρ‹ΠΌ ΡƒΡ‡Π΅Ρ‚ΠΎΠΌ мощности ΠΈΡ… ΠΏΠΎΡ‚ΠΎΠΊΠ°. ЗапрСщаСтся использованиС ПВЀ, Ссли этот ΠΏΠΎΠΊΠ°Π·Π°Ρ‚Π΅Π»ΡŒ ΠΏΡ€Π΅Π²Ρ‹ΡˆΠ°Π΅Ρ‚ ΠΌΠΎΡ‰Π½ΠΎΡΡ‚ΡŒ основных источников свСта.

Π€Π°Ρ€Ρ‹ Π΄ΠΎΠ»ΠΆΠ½Ρ‹ Π±Ρ‹Ρ‚ΡŒ установлСны симмСтрично ΠΎΡ‚Π½ΠΎΡΠΈΡ‚Π΅Π»ΡŒΠ½ΠΎ оси автомобиля.

Если Π·Π°Π²ΠΎΠ΄ΠΎΠΌ ΡƒΠΆΠ΅ прСдусмотрСны ΠΈ ΠΎΠ±ΠΎΠ·Π½Π°Ρ‡Π΅Π½Ρ‹ Π½Π° Π±Π°ΠΌΠΏΠ΅Ρ€Π΅ мСста установки Β«ΠΏΡ€ΠΎΡ‚ΠΈΠ²ΠΎΡ‚ΡƒΠΌΠ°Π½ΠΎΠΊΒ», Ρ‚ΠΎ слСдуСт ΠΏΡ€ΠΈΠ΄Π΅Ρ€ΠΆΠΈΠ²Π°Ρ‚ΡŒΡΡ заводских Ρ€Π΅ΠΊΠΎΠΌΠ΅Π½Π΄Π°Ρ†ΠΈΠΉ ΠΈ ΡƒΡΡ‚Π°Π½Π°Π²Π»ΠΈΠ²Π°Ρ‚ΡŒ Ρ„Π°Ρ€Ρ‹ ΠΈΠΌΠ΅Π½Π½ΠΎ Π² этих Π·ΠΎΠ½Π°Ρ….

ПослС фиксирования ПВЀ ΠΎΠ±ΡΠ·Π°Ρ‚Π΅Π»ΡŒΠ½ΠΎ провСряСтся ΠΏΡ€Π°Π²ΠΈΠ»ΡŒΠ½ΠΎΡΡ‚ΡŒ ΠΈΡ… Ρ€Π΅Π³ΡƒΠ»ΠΈΡ€ΠΎΠ²ΠΊΠΈ. Π–Π΅Π»Π°Ρ‚Π΅Π»ΡŒΠ½ΠΎ, Ρ‡Ρ‚ΠΎΠ±Ρ‹ это дСлалось Π½Π° станции тСхобслуТивания спСциалистом-элСктриком. Π­Ρ‚ΠΎΡ‚ шаг, ΠΏΠΎΠΆΠ°Π»ΡƒΠΉ, самый Π²Π°ΠΆΠ½Ρ‹ΠΉ ΠΈΠ· ΠΏΡ€Π΅Π΄Ρ‹Π΄ΡƒΡ‰ΠΈΡ…, Ρ‚Π°ΠΊ ΠΊΠ°ΠΊ ΠΎΡ‚ Π½Π΅Π³ΠΎ зависит качСство Ρ€Π°Π±ΠΎΡ‚Ρ‹ ΠΏΡ€ΠΎΡ‚ΠΈΠ²ΠΎΡ‚ΡƒΠΌΠ°Π½Π½Ρ‹Ρ… Ρ„Π°Ρ€. Волько Π² этом случаС ΠΎΠ½ΠΈ Π±ΡƒΠ΄ΡƒΡ‚ ΠΏΠΎΠ»Π΅Π·Π½Ρ‹ΠΌ ΠΎΠ±ΠΎΡ€ΡƒΠ΄ΠΎΠ²Π°Π½ΠΈΠ΅ΠΌ.

ΠŸΠΎΠ΄ΠΊΠ»ΡŽΡ‡Π΅Π½ΠΈΠ΅ Β«ΠΏΡ€ΠΎΡ‚ΠΈΠ²ΠΎΡ‚ΡƒΠΌΠ°Π½ΠΎΠΊΒ»

ПослС Ρ‚ΠΎΠ³ΠΎ ΠΊΠ°ΠΊ Ρ„Π°Ρ€Ρ‹ ΠΏΡ€ΠΎΡ‚ΠΈΠ²ΠΎΡ‚ΡƒΠΌΠ°Π½Π½Ρ‹Π΅ Π½Π° Β«Π“Π°Π·Π΅Π»ΡŒΒ» Π±ΡƒΠ΄ΡƒΡ‚ установлСны, трСбуСтся произвСсти ΠΈΡ… ΠΏΠΎΠ΄ΠΊΠ»ΡŽΡ‡Π΅Π½ΠΈΠ΅ ΠΊ Π±ΠΎΡ€Ρ‚ΠΎΠ²ΠΎΠΉ элСктросСти. Π­Ρ‚Π° Ρ€Π°Π±ΠΎΡ‚Π° Ρ‚Ρ€Π΅Π±ΡƒΠ΅Ρ‚ ΠΎΠΏΡ€Π΅Π΄Π΅Π»Π΅Π½Π½Ρ‹Ρ… Π·Π½Π°Π½ΠΈΠΉ, Π² ΠΏΡ€ΠΎΡ‚ΠΈΠ²Π½ΠΎΠΌ случаС ΠΌΠΎΠ³ΡƒΡ‚ Π±Ρ‹Ρ‚ΡŒ Π½Π°Ρ€ΡƒΡˆΠ΅Π½Ρ‹ Π½ΠΎΡ€ΠΌΠ°Ρ‚ΠΈΠ²Ρ‹ Π½Π΅ Ρ‚ΠΎΠ»ΡŒΠΊΠΎ элСктро-, Π½ΠΎ ΠΈ ΠΏΠΎΠΆΠ°Ρ€Π½ΠΎΠΉ бСзопасности. Π§Ρ‚ΠΎΠ±Ρ‹ ΠΏΡ€Π°Π²ΠΈΠ»ΡŒΠ½ΠΎ Π²Ρ‹ΠΏΠΎΠ»Π½ΠΈΡ‚ΡŒ этот этап, потрСбуСтся схСма ΠΏΠΎΠ΄ΠΊΠ»ΡŽΡ‡Π΅Π½ΠΈΡ ΠΏΡ€ΠΎΡ‚ΠΈΠ²ΠΎΡ‚ΡƒΠΌΠ°Π½Π½Ρ‹Ρ… Ρ„Π°Ρ€ (Β«Π“Π°Π·Π΅Π»ΡŒΒ» Π΄ΠΎΠ»ΠΆΠ½Π° Π±Ρ‹Ρ‚ΡŒ ΡƒΠΊΠΎΠΌΠΏΠ»Π΅ΠΊΡ‚ΠΎΠ²Π°Π½Π° схСмой Π±ΠΎΡ€Ρ‚ΠΎΠ²ΠΎΠΉ сСти).

ΠŸΡ€Π΅ΠΆΠ΄Π΅ всСго, слСдуСт ΠΏΡ€ΠΎΠ²Π΅Ρ€ΠΈΡ‚ΡŒ ΡΠΎΡ€Π°Π·ΠΌΠ΅Ρ€Π½ΠΎΡΡ‚ΡŒ сСчСний ΠΏΡ€ΠΎΠ²ΠΎΠ΄ΠΎΠ² Π±ΠΎΡ€Ρ‚ΠΎΠ²ΠΎΠΉ сСти ΠΈ ΠΏΡ€ΠΎΠ²ΠΎΠ΄ΠΎΠ² ΠΈΠ· ΠΊΠΎΠΌΠΏΠ»Π΅ΠΊΡ‚Π°Ρ†ΠΈΠΈ Ρ„Π°Ρ€. ДопускаСтся минимальноС сСчСниС Π΄ΠΎ 0,75 ΠΌΠΌ. Если ΠΈΡΠΏΠΎΠ»ΡŒΠ·ΠΎΠ²Π°Ρ‚ΡŒ ΠΏΡ€ΠΎΠ²ΠΎΠ΄Π° мСньшСго сСчСния, Ρ‚ΠΎ ΠΏΡ€ΠΎΠΈΠ·ΠΎΠΉΠ΄Π΅Ρ‚ ΠΈΡ… ΠΏΠ΅Ρ€Π΅Π³Ρ€Π΅Π², Π° ΠΏΠΎΡ‚ΠΎΠΌ ΠΈ Π²ΠΎΠ·Π³ΠΎΡ€Π°Π½ΠΈΠ΅.

Π’ ΠΎΠ±ΡΠ·Π°Ρ‚Π΅Π»ΡŒΠ½ΠΎΠΌ порядкС производится ΠΎΡ‚ΠΊΠ»ΡŽΡ‡Π΅Π½ΠΈΠ΅ Π±ΠΎΡ€Ρ‚ΠΎΠ²ΠΎΠ³ΠΎ аккумулятора. Вакая ΠΌΠ΅Ρ€Π° Π½Π΅ΠΎΠ±Ρ…ΠΎΠ΄ΠΈΠΌΠ° Π½Π° случай ΠΎΡˆΠΈΠ±ΠΎΡ‡Π½ΠΎΠ³ΠΎ ΠΏΠΎΠ΄ΠΊΠ»ΡŽΡ‡Π΅Π½ΠΈΡ ПВЀ ΠΈ ΠΊΠΎΡ€ΠΎΡ‚ΠΊΠΎΠ³ΠΎ замыкания Π² Ρ†Π΅ΠΏΠΈ.

Если ΠΊΠΎΠΌΠΏΠ»Π΅ΠΊΡ‚ΠΎΠΌ Ρ„Π°Ρ€ Π½Π΅ прСдусмотрСно Ρ€Π΅Π»Π΅ ΠΈΡ… Π²ΠΊΠ»ΡŽΡ‡Π΅Π½ΠΈΡ, слСдуСт Π΅Π³ΠΎ приобрСсти ΠΈ ΡƒΡΡ‚Π°Π½ΠΎΠ²ΠΈΡ‚ΡŒ, Ρ‚Π°ΠΊ ΠΊΠ°ΠΊ обычная ΠΊΠ½ΠΎΠΏΠΊΠ° ΠΌΠΎΠΆΠ΅Ρ‚ Π½Π΅ ΠΎΠ±Π΅ΡΠΏΠ΅Ρ‡ΠΈΡ‚ΡŒ Π½ΠΎΡ€ΠΌΠ°Π»ΡŒΠ½ΡƒΡŽ Ρ€Π°Π±ΠΎΡ‚Ρƒ ПВЀ ΠΈΠ·-Π·Π° большого Ρ‚ΠΎΠΊΠ° ΠΈΡ… питания ΠΈ подгорания Π΅Π΅ ΠΊΠΎΠ½Ρ‚Π°ΠΊΡ‚ΠΎΠ².

ΠŸΠΈΡ‚Π°Π½ΠΈΠ΅ ПВЀ подаСтся Ρ‚ΠΎΠ»ΡŒΠΊΠΎ Ρ‡Π΅Ρ€Π΅Π· Π²Ρ‹Π΄Π΅Π»Π΅Π½Π½Ρ‹Π΅ ΠΏΡ€Π΅Π΄ΠΎΡ…Ρ€Π°Π½ΠΈΡ‚Π΅Π»ΠΈ, Π² ΠΏΡ€ΠΎΡ‚ΠΈΠ²Π½ΠΎΠΌ случаС Π½Π΅ΠΈΡΠΏΡ€Π°Π²Π½ΠΎΡΡ‚ΡŒ Ρ„Π°Ρ€ ΠΈ ΠΈΡ… ΠΏΡ€ΠΎΠ²ΠΎΠ΄ΠΊΠΈ ΠΏΡ€ΠΈΠ²Π΅Π΄Π΅Ρ‚ ΠΊ ΠΎΡ‚ΠΊΠ»ΡŽΡ‡Π΅Π½ΠΈΡŽ всСй Π±ΠΎΡ€Ρ‚ΠΎΠ²ΠΎΠΉ элСктросСти Β«Π“Π°Π·Π΅Π»ΠΈΒ».

Π€Π°Ρ€Ρ‹ ΠΌΠΎΠΆΠ½ΠΎ ΡΡ‡ΠΈΡ‚Π°Ρ‚ΡŒ ΠΏΠΎΠ»Π½ΠΎΡΡ‚ΡŒΡŽ установлСнными, Ссли послС Π·Π°Π²Π΅Ρ€ΡˆΠ΅Π½ΠΈΡ ΠΈΡ… ΠΌΠΎΠ½Ρ‚Π°ΠΆΠ° ΠΈ ΠΏΠΎΠ΄ΠΊΠ»ΡŽΡ‡Π΅Π½ΠΈΡ аккумулятора всС устройства автомобиля, Π² Ρ‚ΠΎΠΌ числС ΠΈ ΠΎΡΠ²Π΅Ρ‚ΠΈΡ‚Π΅Π»ΡŒΠ½Ρ‹Π΅ ΠΏΡ€ΠΈΠ±ΠΎΡ€Ρ‹, Ρ€Π°Π±ΠΎΡ‚Π°ΡŽΡ‚ Π½ΠΎΡ€ΠΌΠ°Π»ΡŒΠ½ΠΎ.

Π Π΅Π³ΡƒΠ»ΠΈΡ€ΠΎΠ²ΠΊΠ°

Когда Π·Π°ΠΊΠΎΠ½Ρ‡Π΅Π½Π° установка ΠΏΡ€ΠΎΡ‚ΠΈΠ²ΠΎΡ‚ΡƒΠΌΠ°Π½Π½Ρ‹Ρ… Ρ„Π°Ρ€, Β«Π“Π°Π·Π΅Π»ΡŒΒ» загоняСтся Π½Π° Π³ΠΎΡ€ΠΈΠ·ΠΎΠ½Ρ‚Π°Π»ΡŒΠ½ΡƒΡŽ ΠΏΠ»ΠΎΡ‰Π°Π΄ΠΊΡƒ для ΠΈΡ… Ρ€Π΅Π³ΡƒΠ»ΠΈΡ€ΠΎΠ²ΠΊΠΈ. Для качСствСнного выполнСния этой Ρ€Π°Π±ΠΎΡ‚Ρ‹ слСдуСт:

  • довСсти Π΄ΠΎ Π½ΠΎΡ€ΠΌΡ‹ Π΄Π°Π²Π»Π΅Π½ΠΈΠ΅ Π² колСсах;
  • Π·Π°ΠΏΡ€Π°Π²ΠΈΡ‚ΡŒ ΠΏΠΎΠ»Π½ΠΎΡΡ‚ΡŒΡŽ Ρ‚ΠΎΠΏΠ»ΠΈΠ²Π½Ρ‹ΠΉ Π±Π°ΠΊ;
  • Π·Π°Π³Ρ€ΡƒΠ·ΠΈΡ‚ΡŒ Π°Π²Ρ‚ΠΎΠΌΠΎΠ±ΠΈΠ»ΡŒ Π΄ΠΎ ΠΎΠ±Ρ‹Ρ‡Π½ΠΎΠ³ΠΎ состояния;
  • Π½Π° Π²Π΅Ρ€Ρ‚ΠΈΠΊΠ°Π»ΡŒΠ½ΠΎΠΌ экранС (размСщаСтся Π² 10 ΠΌ ΠΎΡ‚ ΠΌΠ°ΡˆΠΈΠ½Ρ‹) Ρ€Π°Π·ΠΌΠ΅Ρ‡Π°ΡŽΡ‚ΡΡ Π²Π΅Ρ€Ρ‚ΠΈΠΊΠ°Π»ΡŒΠ½Π°Ρ (ось Π°Π²Ρ‚ΠΎ) ΠΈ Π΄Π²Π΅ ΠΏΠ°Ρ€Π°Π»Π»Π΅Π»ΡŒΠ½Ρ‹Π΅ Π»ΠΈΠ½ΠΈΠΈ (вСрхняя – расстояниС ΠΎΡ‚ Ρ†Π΅Π½Ρ‚Ρ€Π° Ρ„Π°Ρ€ Π΄ΠΎ повСрхности ΠΏΠ»ΠΎΡ‰Π°Π΄ΠΊΠΈ, ниТняя – 100 ΠΌΠΌ).

РСгулируСтся каТдая Ρ„Π°Ρ€Π° ΠΎΡ‚Π΄Π΅Π»ΡŒΠ½ΠΎ согласно Ρ€Π°Π·ΠΌΠ΅Ρ‡Π΅Π½Π½Ρ‹ΠΌ Π½Π° экранС линиям.

ПВЀ для Β«Π“Π°Π·Π΅Π»ΠΈΒ» сСрий «БизнСс» ΠΈ «НСкст»

ΠŸΡ€ΠΎΠ±Π»Π΅ΠΌΠ° бСзопасной Π΅Π·Π΄Ρ‹ Π² слоТных ΠΏΠΎΠ³ΠΎΠ΄Π½Ρ‹Ρ… условиях ΠΏΠΎΡ€ΠΎΠΆΠ΄Π°Π΅Ρ‚ Ρ‚Π°ΠΊΠΎΠΉ вопрос: «ΠšΠ°ΠΊΠΈΠ΅ ΠΏΡ€ΠΎΡ‚ΠΈΠ²ΠΎΡ‚ΡƒΠΌΠ°Π½Π½Ρ‹Π΅ Ρ„Π°Ρ€Ρ‹ («Π“Π°Π·Π΅Π»ΡŒ-БизнСс» – вашС Π°Π²Ρ‚ΠΎ) приобрСсти, Π²ΠΎΠ·ΠΌΠΎΠΆΠ΅Π½ Π»ΠΈ Π²Π°Ρ€ΠΈΠ°Π½Ρ‚ установки ПВЀ, ΠΏΡ€Π΅Π΄Π½Π°Π·Π½Π°Ρ‡Π΅Π½Π½Ρ‹Ρ… для Π΄Ρ€ΡƒΠ³ΠΈΡ… машин?»

Π‘Π»ΠΎΠΆΠ½ΠΎΡΡ‚ΡŒ Π·Π°ΠΊΠ»ΡŽΡ‡Π°Π΅Ρ‚ΡΡ Π² ΠΊΡ€Π΅ΠΏΠ»Π΅Π½ΠΈΠΈ Ρ„Π°Ρ€ ΠΈΠΌΠ΅Π½Π½ΠΎ для этого Π°Π²Ρ‚ΠΎ. Ошибка Π² Π²Ρ‹Π±ΠΎΡ€Π΅ ΠΌΠΎΠΆΠ΅Ρ‚ Π΄ΠΎΡ€ΠΎΠ³ΠΎ ΠΎΠ±ΠΎΠΉΡ‚ΠΈΡΡŒ Π²Π»Π°Π΄Π΅Π»ΡŒΡ†Ρƒ Β«Π“Π°Π·Π΅Π»ΠΈ-БизнСс». По мнСнию Π±ΠΎΠ»ΡŒΡˆΠΈΠ½ΡΡ‚Π²Π° Π²ΠΎΠ΄ΠΈΡ‚Π΅Π»Π΅ΠΉ ΠΈ спСциалистов автосСрвиса, ΠΎΠΏΡ‚ΠΈΠΌΠ°Π»ΡŒΠ½Ρ‹ΠΌ Π²Π°Ρ€ΠΈΠ°Π½Ρ‚ΠΎΠΌ ΠΌΠΎΠ³ΡƒΡ‚ Π±Ρ‹Ρ‚ΡŒ Β«ΠΏΡ€ΠΎΡ‚ΠΈΠ²ΠΎΡ‚ΡƒΠΌΠ°Π½ΠΊΠΈΒ» Β«Π›Π°Π΄Ρ‹-ΠŸΡ€ΠΈΠΎΡ€Ρ‹Β», хотя ΠΊΡ€Π΅ΠΏΠ»Π΅Π½ΠΈΠ΅ всС ΠΆΠ΅ придСтся Π΄ΠΎΡ€Π°Π±Π°Ρ‚Ρ‹Π²Π°Ρ‚ΡŒ: потрСбуСтся ΠΏΠ΅Ρ€Π΅Ρ…ΠΎΠ΄Π½ΠΈΠΊ.

ΠŸΡ€ΠΎΡ‚ΠΈΠ²ΠΎΡ‚ΡƒΠΌΠ°Π½Π½Ρ‹Π΅ Ρ„Π°Ρ€Ρ‹, Π½Π° Β«Π“Π°Π·Π΅Π»ΡŒ-НСкст» установлСнныС, Π½Π΅ создадут Π²ΠΎΠ΄ΠΈΡ‚Π΅Π»ΡŽ ΠΏΡ€ΠΎΠ±Π»Π΅ΠΌ, ΠΏΠΎΠ΄ΠΎΠ±Π½Ρ‹Ρ… Ρ‚Π΅ΠΌ, ΠΊΠΎΡ‚ΠΎΡ€Ρ‹Π΅ Π²ΡΡ‚Ρ€Π΅Ρ‡Π°ΡŽΡ‚ΡΡ Ρƒ Β«Π“Π°Π·Π΅Π»ΠΈ-БизнСс».

Данная модСль комплСктуСтся ΠΏΡ€ΠΎΡ‚ΠΈΠ²ΠΎΡ‚ΡƒΠΌΠ°Π½Π½Ρ‹ΠΌΠΈ срСдствами Π·Π°Π²ΠΎΠ΄ΠΎΠΌ-ΠΈΠ·Π³ΠΎΡ‚ΠΎΠ²ΠΈΡ‚Π΅Π»Π΅ΠΌ.

Π‘Ρ‚Π°Π²ΠΈΡ‚ΡŒ ΠΈΠ»ΠΈ Π½Π΅ ΡΡ‚Π°Π²ΠΈΡ‚ΡŒ? МнСниС Π²ΠΎΠ΄ΠΈΡ‚Π΅Π»Π΅ΠΉ

Будя ΠΏΠΎ ΠΎΡ‚Π·Ρ‹Π²Π°ΠΌ, Π²ΠΎΠ΄ΠΈΡ‚Π΅Π»ΠΈ Π΅Π΄ΠΈΠ½ΠΎΠ΄ΡƒΡˆΠ½ΠΎ ΠΏΡ€ΠΈΠ²Π΅Ρ‚ΡΡ‚Π²ΡƒΡŽΡ‚ установку «ΠΏΡ€ΠΎΡ‚ΠΈΠ²ΠΎΡ‚ΡƒΠΌΠ°Π½ΠΎΠΊ». Вопрос лишь Π² Ρ‚ΠΎΠΌ, ΠΊΠ°ΠΊΠΈΠ΅ Ρ„Π°Ρ€Ρ‹ ΡΡ‚Π°Π²ΠΈΡ‚ΡŒ Π½Π° «Π“Π°Π·Π΅Π»ΡŒ». ΠΠ²Ρ‚ΠΎΠΌΠΎΠ±ΠΈΠ»ΡŒΠ½Π°Ρ ΠΏΡ€ΠΎΠΌΡ‹ΡˆΠ»Π΅Π½Π½ΠΎΡΡ‚ΡŒ Ρ‚ΠΎΠ»ΡŒΠΊΠΎ сСйчас «Π΄ΠΎΡ€ΠΎΡΠ»Π°» Π΄ΠΎ ΠΈΡ… установки, Π° ΠΏΠ΅Ρ€Π²Ρ‹Π΅ ΠΌΠΎΠ΄Π΅Π»ΠΈ этого Π±Ρ‹Π»ΠΈ Π»ΠΈΡˆΠ΅Π½Ρ‹. ΠŸΠΎΡΡ‚ΠΎΠΌΡƒ водитСлями Ρ‡Π°Ρ‰Π΅ всСго ΠΈΠΌΠ΅Π½Π½ΠΎ эта ΠΏΡ€ΠΎΠ±Π»Π΅ΠΌΠ° затрагиваСтся Π½Π° спСциализированных Ρ„ΠΎΡ€ΡƒΠΌΠ°Ρ….

Π’Π°ΠΊΠΆΠ΅ Π²ΠΎΠ΄ΠΈΡ‚Π΅Π»ΠΈ ΡΡ‡ΠΈΡ‚Π°ΡŽΡ‚, Ρ‡Ρ‚ΠΎ Π»ΡƒΡ‡ΡˆΠ΅ ΠΈΡΠΏΠΎΠ»ΡŒΠ·ΠΎΠ²Π°Ρ‚ΡŒ ΠΆΠ΅Π»Ρ‚Ρ‹Π΅ «ΠΏΡ€ΠΎΡ‚ΠΈΠ²ΠΎΡ‚ΡƒΠΌΠ°Π½ΠΊΠΈ», ΠΏΡ€ΠΈ этом Π½Π΅ стоит ΡΠΊΡƒΠΏΠΈΡ‚ΡŒΡΡ Π½Π° Ρ…ΠΎΡ€ΠΎΡˆΠΈΠ΅ ПВЀ.

Π Π΅ΠΊΠΎΠΌΠ΅Π½Π΄ΡƒΡŽΡ‚ для Ρ‚ΡƒΠΌΠ°Π½Π° Π»Π°ΠΌΠΏΡ‹ «ΠžΡΡ€Π°ΠΌ Алл», для снСга – «ΠΠ°ΠΉΡ‚ Π‘Ρ€Π΅ΠΊΠ΅Ρ€» ΠΈΠ»ΠΈ «Π‘ΠΎΡˆ».

Π‘Ρ…Π΅ΠΌΠ° ΠΊΠ½ΠΎΠΏΠΊΠΈ Π²ΠΊΠ»ΡŽΡ‡Π΅Π½ΠΈΡ ΠΏΡ€ΠΎΡ‚ΠΈΠ²ΠΎΡ‚ΡƒΠΌΠ°Π½Π½Ρ‹Ρ… Ρ„Π°Ρ€

ΠŸΡ€ΠΈ установкС ΠΏΡ€ΠΎΡ‚ΠΈΠ²ΠΎΡ‚ΡƒΠΌΠ°Π½Π½Ρ‹Ρ… Ρ„Π°Ρ€ Ρ‡Π΅Ρ€Π΅Π· Ρ€Π΅Π»Π΅ Π°Π²Ρ‚ΠΎΠ»ΡŽΠ±ΠΈΡ‚Π΅Π»ΡŒ ΠΌΠΎΠΆΠ΅Ρ‚ ΠΈΡΠΏΡ‹Ρ‚Π°Ρ‚ΡŒ Π½Π΅ΠΊΠΎΡ‚ΠΎΡ€Ρ‹Π΅ слоТности. Π’Ρ€ΠΎΠ΄Π΅ с ΠΌΠΎΠ½Ρ‚Π°ΠΆΠΎΠΌ самих Ρ„Π°Ρ€ ΠΈ ΠΏΡ€ΠΎΠΊΠ»Π°Π΄ΠΊΠΎΠΉ ΠΏΡ€ΠΎΠ²ΠΎΠ΄ΠΎΠ² всС понятно, Π½ΠΎ ΠΊΠ°ΠΊ ΠΏΠΎΠ΄ΠΊΠ»ΡŽΡ‡ΠΈΡ‚ΡŒ Ρ€Π΅Π»Π΅ ΠΈ ΠΏΠΎΠ΄ΠΊΠ»ΡŽΡ‡ΠΈΡ‚ΡŒ всС вмСстС с прСдохранитСлями. Для этого ΠΆΠ΅Π»Π°Ρ‚Π΅Π»ΡŒΠ½ΠΎ ΠΈΠΌΠ΅Ρ‚ΡŒ ΠΌΠΈΠ½ΠΈΠΌΠ°Π»ΡŒΠ½Ρ‹Π΅ знания Π² элСктротСхникС.

ΠŸΠΎΠ΄Π³ΠΎΡ‚ΠΎΠ²ΠΊΠ° ΠΊ ΠΏΠΎΠ΄ΠΊΠ»ΡŽΡ‡Π΅Π½ΠΈΡŽ ΠΏΡ€ΠΎΡ‚ΠΈΠ²ΠΎΡ‚ΡƒΠΌΠ°Π½Π½Ρ‹Ρ… Ρ„Π°Ρ€ Ρ€Π΅Π»Π΅

Π˜Π½ΡΡ‚Ρ€ΡƒΠΌΠ΅Π½Ρ‚Ρ‹ Π½Π΅ΠΎΠ±Ρ…ΠΎΠ΄ΠΈΠΌΡ‹Π΅ для ΠΏΠΎΠ΄ΠΊΠ»ΡŽΡ‡Π΅Π½ΠΈΡ Ρ€Π΅Π»Π΅:

Π’Π°ΠΊΠΆΠ΅ понадобится большоС количСство расходных ΠΌΠ°Ρ‚Π΅Ρ€ΠΈΠ°Π»ΠΎΠ², Ρ‚Π°ΠΊΠΈΡ… ΠΊΠ°ΠΊ ΠΈΠ·ΠΎΠ»Π΅Π½Ρ‚Π°, Ρ…ΠΎΠΌΡƒΡ‚Ρ‹, ΠΊΠ»Π΅ΠΌΠΌΡ‹, тСрмоусадки, Π³ΠΎΡ„Ρ€Π°. Π­Ρ‚ΠΈ ΠΌΠ°Ρ‚Π΅Ρ€ΠΈΠ°Π»Ρ‹ обСспСчат соСдинСниС ΠΏΡ€ΠΎΠ²ΠΎΠ΄ΠΎΠ² ΠΈ ΠΈΡ… Π΄ΠΎΠ»Π³ΠΎΠ²Π΅Ρ‡Π½ΠΎΡΡ‚ΡŒ Π²ΠΎ врСмя использования.

Π‘Ρ…Π΅ΠΌΠ° ΠΏΠΎΠ΄ΠΊΠ»ΡŽΡ‡Π΅Π½ΠΈΡ ΠΏΡ€ΠΎΡ‚ΠΈΠ²ΠΎΡ‚ΡƒΠΌΠ°Π½ΠΎΠΊ Ρ‡Π΅Ρ€Π΅Π· Ρ€Π΅Π»Π΅

ΠŸΠ΅Ρ€Π΅Π΄ Π½Π°Ρ‡Π°Π»ΠΎΠΌ ΠΌΠΎΠ½Ρ‚Π°ΠΆΠ° Π½Π΅ΠΎΠ±Ρ…ΠΎΠ΄ΠΈΠΌΠΎ ΠΎΠΏΡ€Π΅Π΄Π΅Π»ΠΈΡ‚ΡŒΡΡ со схСмой Π²ΠΊΠ»ΡŽΡ‡Π΅Π½ΠΈΡ:

Автономная, ΠΏΡ€ΠΎΡ‚ΠΈΠ²ΠΎΡ‚ΡƒΠΌΠ°Π½ΠΊΠΈ Π±ΡƒΠ΄ΡƒΡ‚ Π²ΠΊΠ»ΡŽΡ‡Π°Ρ‚ΡŒΡΡ нСзависимо, ΠΏΡ€Π°Π²Π΄Π° Ρ‚ΠΎΠ³Π΄Π° Ρ„Π°Ρ€Ρ‹ ΠΌΠΎΠ³ΡƒΡ‚ ΠΏΠΎΡΠ°Π΄ΠΈΡ‚ΡŒ аккумулятор ΠΏΠΎΠ»Π½ΠΎΡΡ‚ΡŒΡŽ, Ρ‡Ρ‚ΠΎ сопряТСно с ΠΏΡ€ΠΎΠ±Π»Π΅ΠΌΠ°ΠΌΠΈ. ΠŸΠΎΠ΄ΠΊΠ»ΡŽΡ‡Π΅Π½ΠΈΠ΅ происходит ΠΊ ΠΏΠ»ΡŽΡΡƒ Π³Π°Π±Π°Ρ€ΠΈΡ‚ΠΎΠ² ΠΈΠ»ΠΈ ΠΏΡ€ΠΎΠ²ΠΎΠ΄Ρƒ заТигания +ACC

Π‘ΠΌΠΎΡ‚Ρ€ΠΈΡ‚Π΅ Π² Π²ΠΈΠ΄Π΅ΠΎ Π½ΠΈΠΆΠ΅, ΠΊΠ°ΠΊ ΠΏΡ€Π°Π²ΠΈΠ»ΡŒΠ½ΠΎ ΠΏΠΎΠ΄ΠΊΠ»ΡŽΡ‡ΠΈΡ‚ΡŒ Ρ€Π΅Π»Π΅.

Π‘ Π·Π°ΠΆΠΈΠ³Π°Π½ΠΈΠ΅ΠΌ, Π² этом случаС Π±Π΅Π· Π·Π°Π²Π΅Π΄Π΅Π½Π½ΠΎΠ³ΠΎ двигатСля Π½Π΅ Π²ΠΊΠ»ΡŽΡ‡ΠΈΡ‚ΡŒ ΠΏΡ€ΠΎΡ‚ΠΈΠ²ΠΎΡ‚ΡƒΠΌΠ°Π½Π½Ρ‹Π΅ Ρ„Π°Ρ€Ρ‹, ΠΎΠ±Ρ‹Ρ‡Π½ΠΎ ΠΈΡΠΏΠΎΠ»ΡŒΠ·ΡƒΠ΅Ρ‚ΡΡ плюс с Π·Π°ΠΌΠΊΠ° заТигания ΠΈΠ»ΠΈ IGN2, ΠΊΠΎΡ‚ΠΎΡ€Ρ‹ΠΉ Π»ΡƒΡ‡ΡˆΠ΅ всСго ΠΈΡΠΊΠ°Ρ‚ΡŒ с ΠΏΠΎΠΌΠΎΡ‰ΡŒΡŽ Π²ΠΎΠ»ΡŒΡ‚ΠΌΠ΅Ρ‚Ρ€Π°, Ρ‚Π°ΠΊ ΠΊΠ°ΠΊ Ссли ΠΈΡΠΏΠΎΠ»ΡŒΠ·ΠΎΠ²Π°Ρ‚ΡŒ Π»Π°ΠΌΠΏΠΎΠ²Ρ‹ΠΉ ΠΏΡ€ΠΎΠ±Π½ΠΈΠΊ, Π΅ΡΡ‚ΡŒ Π²Π΅Ρ€ΠΎΡΡ‚Π½ΠΎΡΡ‚ΡŒ поврСТдСния элСктроники автомобиля.

Π§Ρ‚ΠΎΠ±Ρ‹ ΠΏΠΎΠ΄ΠΊΠ»ΡŽΡ‡ΠΈΡ‚ΡŒ ΠΏΡ€ΠΎΡ‚ΠΈΠ²ΠΎΡ‚ΡƒΠΌΠ°Π½ΠΊΠΈ Ρ‡Π΅Ρ€Π΅Π· Ρ€Π΅Π»Π΅ Π½Π΅ ΠΎΠ±ΡΠ·Π°Ρ‚Π΅Π»ΡŒΠ½ΠΎ Ρ€Π°Π·Π±ΠΈΡ€Π°Ρ‚ΡŒ ΠΏΠΎΠ»ΠΎΠ²ΠΈΠ½Ρƒ ΠΊΡƒΠ·ΠΎΠ²Π°. Основная Ρ‡Π°ΡΡ‚ΡŒ Ρ€Π°Π±ΠΎΡ‚ проводится Π² салонС, Π³Π΄Π΅ Π½Π΅ΠΎΠ±Ρ…ΠΎΠ΄ΠΈΠΌΠΎ ΠΎΠ±Π΅ΡΠΏΠ΅Ρ‡ΠΈΡ‚ΡŒ доступ ΠΊ элСктрикС автомобиля, сняв Ρ‡Π°ΡΡ‚ΡŒ ΠΏΠ΅Ρ€Π΅Π΄Π½Π΅ΠΉ ΠΏΠ°Π½Π΅Π»ΠΈ.

Роль Ρ€Π΅Π»Π΅

Π Π΅Π»Π΅ Π½Π΅ΠΎΠ±Ρ…ΠΎΠ΄ΠΈΠΌΠΎ для сниТСния Ρ‚ΠΎΠΊΠ° Π² ΠΏΡ€ΠΎΠ²ΠΎΠ΄Π°Ρ…, ΠΏΠΈΡ‚Π°ΡŽΡ‰ΠΈΡ… ΠΏΡ€ΠΎΡ‚ΠΈΠ²ΠΎΡ‚ΡƒΠΌΠ°Π½Π½Ρ‹Π΅ Ρ„Π°Ρ€Ρ‹, Π²ΠΎ ΠΈΠ·Π±Π΅ΠΆΠ°Π½ΠΈΠ΅ ΠΈΡ… расплавлСния. Оно ΠΈΠΌΠ΅Π΅Ρ‚ 4 ΠΊΠΎΠ½Ρ‚Π°ΠΊΡ‚Π°, Π΄Π²Π° ΠΊΠΎΠΌΠΌΡƒΡ‚ΠΈΡ€ΡƒΡŽΡ‰ΠΈΡ… ΠΈ Π΄Π²Π° ΡƒΠΏΡ€Π°Π²Π»ΡΡŽΡ‰ΠΈΡ…, ΠΈΠ½ΠΎΠ³Π΄Π° ΠΈΡΠΏΠΎΠ»ΡŒΠ·ΡƒΡŽΡ‚ пятиконтактноС устройство. ΠžΠ±ΠΌΠΎΡ‚ΠΊΠ° возбуТдСния Π΄ΠΎΠ»ΠΆΠ½Π° ΠΈΠΌΠ΅Ρ‚ΡŒ сопротивлСниС Π½Π΅ ΠΌΠ΅Π½Π΅Π΅ 70 Ом, ΠΏΡ€ΠΈ мСньшСм Π·Π½Π°Ρ‡Π΅Π½ΠΈΠΈ Π²ΠΎΠ·ΠΌΠΎΠΆΠ½ΠΎ ΠΏΡ€Π΅Π²Ρ‹ΡˆΠ΅Π½ΠΈΠ΅ Ρ‚ΠΎΠΊΠ° ΠΈ Π²Ρ‹Ρ…ΠΎΠ΄ ΠΈΠ· строя прСдохранитСля. Π‘ΠΈΠ»Π° Ρ‚ΠΎΠΊΠ° Ρ€Π΅Π»Π΅ составляСт 30 А, Π²ΠΎΠ·ΠΌΠΎΠΆΠ½ΠΎ ΠΈΡΠΏΠΎΠ»ΡŒΠ·ΠΎΠ²Π°Ρ‚ΡŒ ΠΈ 70. ΠŸΠΎΠ΄ΠΊΠ»ΡŽΡ‡Π΅Π½ΠΈΠ΅ ΠΊΠΎΠ½Ρ‚Π°ΠΊΡ‚ΠΎΠ² происходит ΠΏΠΎ ΡΠ»Π΅Π΄ΡƒΡŽΡ‰Π΅ΠΉ схСмС:

  • 1-Ρ‹ΠΉ ΠΊΠΎΠΌΠΌΡƒΡ‚ΠΈΡ€ΡƒΡŽΡ‰ΠΈΠΉ β€” Π±Π»ΠΎΠΊ ΠΏΡ€Π΅Π΄ΠΎΡ…Ρ€Π°Π½ΠΈΡ‚Π΅Π»Π΅ΠΉ;
  • 2-ΠΎΠΉ ΠΊΠΎΠΌΠΌΡƒΡ‚ΠΈΡ€ΡƒΡŽΡ‰ΠΈΠΉ – ΠΏΡ€ΠΎΡ‚ΠΈΠ²ΠΎΡ‚ΡƒΠΌΠ°Π½ΠΊΠΈ;
  • 1-Ρ‹ΠΉ ΡƒΠΏΡ€Π°Π²Π»ΡΡŽΡ‰ΠΈΠΉ β€” ΠΊΠ½ΠΎΠΏΠΊΠ° Π²ΠΊΠ»ΡŽΡ‡Π΅Π½ΠΈΡ/Π²Ρ‹ΠΊΠ»ΡŽΡ‡Π΅Π½ΠΈΡ;
  • 2-ΠΎΠΉ ΡƒΠΏΡ€Π°Π²Π»ΡΡŽΡ‰ΠΈΠΉ – масса.

ΠŸΡ€Π΅Π΄ΠΎΡ…Ρ€Π°Π½ΠΈΡ‚Π΅Π»ΠΈ

ПослС выполнСния ΠΏΠΎΠ΄ΠΊΠ»ΡŽΡ‡Π΅Π½ΠΈΡ ΠΏΡ€ΠΎΠ²ΠΎΠ΄ΠΎΠ² ΠΊ Ρ€Π΅Π»Π΅ с ΠΏΠΎΠΌΠΎΡ‰ΡŒΡŽ ΠΊΠ»Π΅ΠΌΠΌΠ½ΠΈΠΊΠΎΠ² ΡΠ»Π΅Π΄ΡƒΡŽΡ‚ ΡƒΡΡ‚Π°Π½ΠΎΠ²ΠΈΡ‚ΡŒ ΠΏΡ€Π΅Π΄ΠΎΡ…Ρ€Π°Π½ΠΈΡ‚Π΅Π»ΠΈ, ΠΊΠΎΡ‚ΠΎΡ€Ρ‹Π΅ слуТат для Π·Π°Ρ‰ΠΈΡ‚Ρ‹ Ρ†Π΅ΠΏΠΈ ΠΎΡ‚ ΠΊΠΎΡ€ΠΎΡ‚ΠΊΠΎΠ³ΠΎ замыкания. ΠŸΡ€Π΅Π΄ΠΎΡ…Ρ€Π°Π½ΠΈΡ‚Π΅Π»ΡŒ Π½Π° 15 А Π½Π΅ΠΎΠ±Ρ…ΠΎΠ΄ΠΈΠΌΠΎ ΡƒΡΡ‚Π°Π½ΠΎΠ²ΠΈΡ‚ΡŒ Π² Π±Π»ΠΎΠΊΠ΅ ΠΏΡ€Π΅Π΄ΠΎΡ…Ρ€Π°Π½ΠΈΡ‚Π΅Π»Π΅ΠΉ ΠΈ Ρ‡Π΅Ρ€Π΅Π· Π½Π΅Π³ΠΎ Π·Π°ΠΏΠΈΡ‚Π°Ρ‚ΡŒ Ρ†Π΅ΠΏΡŒ ΠΎΡ‚ Π±Π°Ρ‚Π°Ρ€Π΅ΠΈ. Если Π½Π΅Ρ‚ возмоТности Ρ‚Π°ΠΊΠΎΠΉ установки, ΠΏΡ€Π΅Π΄ΠΎΡ…Ρ€Π°Π½ΠΈΡ‚Π΅Π»ΡŒ слСдуСт ΡƒΡΡ‚Π°Π½ΠΎΠ²ΠΈΡ‚ΡŒ ΠΎΡ‚Π΄Π΅Π»ΡŒΠ½ΠΎ, ΠΊΠ°ΠΊ ΠΌΠΎΠΆΠ½ΠΎ Π±Π»ΠΈΠΆΠ΅ ΠΊ аккумуляторной Π±Π°Ρ‚Π°Ρ€Π΅Π΅.

Как ΠΏΠΎΠ΄ΠΊΠ»ΡŽΡ‡ΠΈΡ‚ΡŒ ΠΊΠ½ΠΎΠΏΠΊΡƒ Π²ΠΊΠ»ΡŽΡ‡Π΅Π½ΠΈΡ?

Кнопка Π²ΠΊΠ»ΡŽΡ‡Π΅Π½ΠΈΡ Π²Ρ‹ΠΊΠ»ΡŽΡ‡Π΅Π½ΠΈΡ Π΄ΠΎΠ»ΠΆΠ½Π° Π±Ρ‹Ρ‚ΡŒ Π·Π°ΠΏΠΈΡ‚Π°Π½Π° любого ΠΏΡ€ΠΎΠ²ΠΎΠ΄Π° 12 Π’. Π’ зависимости ΠΎΡ‚ ΠΏΡ€Π΅Π΄ΠΏΠΎΡ‡Ρ‚Π΅Π½ΠΈΠΉ это ΠΌΠΎΠ³ΡƒΡ‚ Π±Ρ‹Ρ‚ΡŒ Π½Π΅ Ρ‚ΠΎΠ»ΡŒΠΊΠΎ ΡƒΠΏΡ€Π°Π²Π»ΡΡŽΡ‰ΠΈΠ΅ ΠΏΡ€ΠΎΠ²ΠΎΠ΄Π° ΠΎΡ‚ заТигания, Π½ΠΎ ΠΈ любой Π΄Ρ€ΡƒΠ³ΠΎΠΉ источник. Π§Π°Ρ‰Π΅ всСго Π΅Π³ΠΎ ΠΏΡ€ΠΈΡ†Π΅ΠΏΠ»ΡΡŽΡ‚ ΠΊ Π³Π°Π±Π°Ρ€ΠΈΡ‚Π½Ρ‹ΠΌ огням.

ΠŸΠΎΠ΄ΠΊΠ»ΡŽΡ‡Π΅Π½ΠΈΠ΅ самих Ρ„Π°Ρ€ Π½Π΅ отличаСтся ΡΠ»ΠΎΠΆΠ½ΠΎΡΡ‚ΡŒΡŽ. ΠšΡ€Π°ΡΠ½Ρ‹ΠΉ ΠΏΡ€ΠΎΠ²ΠΎΠ΄ – плюс, ΠΏΠΎΠ΄ΠΊΠ»ΡŽΡ‡Π°Π΅Ρ‚ΡΡ ΠΊ Ρ€Π΅Π»Π΅, Π° Ρ‡Π΅Ρ€Π½Ρ‹ΠΉ – масса, ΠΊ корпусу автомобиля.

ΠŸΡ€ΠΎΠ²Π΅Ρ€ΠΈΠ² ΠΏΡ€Π°Π²ΠΈΠ»ΡŒΠ½ΠΎΡΡ‚ΡŒ ΠΏΠΎΠ΄ΠΊΠ»ΡŽΡ‡Π΅Π½ΠΈΡ ΠΏΡ€ΠΎΡ‚ΠΈΠ²ΠΎΡ‚ΡƒΠΌΠ°Π½ΠΎΠΊ Ρ‡Π΅Ρ€Π΅Π· Ρ€Π΅Π»Π΅, ΠΌΠΎΠΆΠ½ΠΎ Π·Π°ΠΊΠΎΠ½Ρ‡ΠΈΡ‚ΡŒ Ρ€Π°Π±ΠΎΡ‚Ρ‹ ΠΏΠΎ ΠΌΠΎΠ½Ρ‚Π°ΠΆΡƒ ΠΏΡ€ΠΎΡ‚ΠΈΠ²ΠΎΡ‚ΡƒΠΌΠ°Π½Π½Ρ‹Ρ… Ρ„Π°Ρ€.

Одно ΠΈΠ· ΠΏΡ€ΠΈΡ€ΠΎΠ΄Π½Ρ‹Ρ… явлСний, Π΄ΠΎΡΡ‚Π°Π²Π»ΡΡŽΡ‰ΠΈΡ… наибольший дискомфорт Π°Π²Ρ‚ΠΎΠ»ΡŽΠ±ΠΈΡ‚Π΅Π»ΡŽ Π·Π° Ρ€ΡƒΠ»Π΅ΠΌ, являСтся Ρ‚ΡƒΠΌΠ°Π½. Π’ этом случаС СдинствСнным спасСниСм ΠΌΠΎΠ³ΡƒΡ‚ ΡΡ‚Π°Ρ‚ΡŒ ΠΏΡ€ΠΎΡ‚ΠΈΠ²ΠΎΡ‚ΡƒΠΌΠ°Π½Π½Ρ‹Π΅ ΠΎΠ³Π½ΠΈ, ΠΊΠΎΡ‚ΠΎΡ€Ρ‹Π΅ ΠΏΠΎΠ·Π²ΠΎΠ»ΡΡŽΡ‚ Π»ΡƒΡ‡ΡˆΠ΅ ΠΎΡ€ΠΈΠ΅Π½Ρ‚ΠΈΡ€ΠΎΠ²Π°Ρ‚ΡŒΡΡ Π² Π½Π΅ΠΏΠΎΠ³ΠΎΠ΄Ρƒ. НиТС ΠΌΡ‹ расскаТСм ΠΎ Ρ‚ΠΎΠΌ, ΠΊΠ°ΠΊ осущСствляСтся установка своими Ρ€ΡƒΠΊΠ°ΠΌΠΈ ΠΏΡ€ΠΎΡ‚ΠΈΠ²ΠΎΡ‚ΡƒΠΌΠ°Π½Π½Ρ‹Ρ… Ρ„Π°Ρ€ Π½Π° любоС Π°Π²Ρ‚ΠΎ.

ВрСбования ΠΊ установкС ПВЀ

ΠšΠΎΠΌΠΏΠ»Π΅ΠΊΡ‚ ΠΈΠ· ΠΏΡ€ΠΎΠ²ΠΎΠ΄ΠΎΠ², ΠΏΠ΅Ρ€Π΅ΠΊΠ»ΡŽΡ‡Π°Ρ‚Π΅Π»Ρ ΠΈ Ρ€Π΅Π»Π΅ для ΠΌΠΎΠ½Ρ‚Π°ΠΆΠ° ПВЀ

Π§Ρ‚ΠΎΠ±Ρ‹ ΠΏΡ€Π°Π²ΠΈΠ»ΡŒΠ½ΠΎ ΠΏΠΎΡΡ‚Π°Π²ΠΈΡ‚ΡŒ ΠΏΡ€ΠΎΡ‚ΠΈΠ²ΠΎΡ‚ΡƒΠΌΠ°Π½ΠΊΠΈ своими Ρ€ΡƒΠΊΠ°ΠΌΠΈ Π² Π±Π°ΠΌΠΏΠ΅Ρ€, Π½Π΅ΠΎΠ±Ρ…ΠΎΠ΄ΠΈΠΌΠΎ Π½Π΅ Ρ‚ΠΎΠ»ΡŒΠΊΠΎ Π·Π°Ρ€Π°Π½Π΅Π΅ ΠΊΡƒΠΏΠΈΡ‚ΡŒ ΡƒΠ½ΠΈΠ²Π΅Ρ€ΡΠ°Π»ΡŒΠ½Ρ‹ΠΉ Π½Π°Π±ΠΎΡ€ ΠΌΠΎΠ½Ρ‚Π°ΠΆΠ½Ρ‹Ρ… ΠΊΡ€Π΅ΠΏΠ»Π΅Π½ΠΈΠΉ, Π½ΠΎ ΠΈ ΡƒΡ‡ΠΈΡ‚Ρ‹Π²Π°Ρ‚ΡŒ всС ΠΏΡ€Π°Π²ΠΈΠ»Π° установки. РазумССтся, Π»ΡŽΠ±Ρ‹Π΅ ПВЀ Π΄ΠΎΠ»ΠΆΠ½Ρ‹ Π±Ρ‹Ρ‚ΡŒ ΠΌΠΎΠ½Ρ‚ΠΈΡ€ΠΎΠ²Π°Π½Ρ‹ ΠΏΡ€Π°Π²ΠΈΠ»ΡŒΠ½ΠΎ, с ΡƒΡ‡Π΅Ρ‚ΠΎΠΌ всСх Ρ‚Ρ€Π΅Π±ΠΎΠ²Π°Π½ΠΈΠΉ ΠΈ ΠΏΡ€Π°Π²ΠΈΠ». Как говорится Π² ΠŸΠ”Π”, Π½Π° ΠΎΠ΄Π½ΠΎ транспортноС срСдство Π² Π±Π°ΠΌΠΏΠ΅Ρ€ ΠΌΠΎΠΆΠ½ΠΎ ΠΏΠΎΡΡ‚Π°Π²ΠΈΡ‚ΡŒ Π½Π΅ Π±ΠΎΠ»Π΅Π΅ Π΄Π²ΡƒΡ… ΠΏΡ€ΠΎΡ‚ΠΈΠ²ΠΎΡ‚ΡƒΠΌΠ°Π½Π½Ρ‹Ρ… Ρ„ΠΎΠ½Π°Ρ€Π΅ΠΉ.

Π˜Ρ… ΠΌΠΎΠ½Ρ‚Π°ΠΆ осущСствляСтся Π² соотвСтствии с трСбованиями:

  1. ПВЀ Π΄ΠΎΠ»ΠΆΠ½Ρ‹ Π±Ρ‹Ρ‚ΡŒ Ρ€Π°Π·ΠΌΠ΅Ρ‰Π΅Π½Ρ‹ Π½Π° расстоянии Π½Π΅ Π±ΠΎΠ»Π΅Π΅ 40 см ΠΎΡ‚ Π±ΠΎΠΊΠΎΠ²ΠΎΠΉ части транспортного срСдства, которая опрСдСляСтся ΠΏΠΎ внСшнСй повСрхности фонаря.
  2. Π§Ρ‚ΠΎΠ±Ρ‹ всС ΡΠ΄Π΅Π»Π°Ρ‚ΡŒ ΠΏΡ€Π°Π²ΠΈΠ»ΡŒΠ½ΠΎ, устройства Π½Π΅ΠΎΠ±Ρ…ΠΎΠ΄ΠΈΠΌΠΎ ΡƒΡΡ‚Π°Π½ΠΎΠ²ΠΈΡ‚ΡŒ Π½Π° расстоянии Π½Π΅ Π½ΠΈΠΆΠ΅ 25 см ΠΎΡ‚ Π·Π΅ΠΌΠ»ΠΈ, Ρ‚ΠΎ Π΅ΡΡ‚ΡŒ ΠΏΠΎ Π½ΠΈΠΆΠ½Π΅ΠΉ части фонаря.
  3. Π‘Π²Π΅Ρ‚ΠΎΠ²Ρ‹Π΅ Ρ„ΠΎΠ½Π°Ρ€ΠΈ ПВЀ Π½Π΅ΠΎΠ±Ρ…ΠΎΠ΄ΠΈΠΌΠΎ ΡƒΡΡ‚Π°Π½ΠΎΠ²ΠΈΡ‚ΡŒ Π½ΠΈΠΆΠ΅ Π²Π΅Ρ€Ρ…Π½Π΅ΠΉ части ΡΠ²Π΅Ρ‚ΠΎΠΏΡ€ΠΎΠΏΡƒΡΠΊΠ°ΡŽΡ‰Π΅Π³ΠΎ ΠΏΡ€ΠΎΠ΅ΠΌΠ° Ρ„ΠΎΠ½Π°Ρ€Π΅ΠΉ Π±Π»ΠΈΠΆΠ½Π΅Π³ΠΎ свСта.
  4. Π§Ρ‚ΠΎ касаСтся ΡƒΠ³Π»ΠΎΠ² видимости, Ρ‚ΠΎ ΠΎΠ½ΠΈ Π½ΠΈ Π΄ΠΎΠ»ΠΆΠ½Ρ‹ Π±Ρ‹Ρ‚ΡŒ Π·Π°Π³Ρ€ΠΎΠΌΠΎΠΆΠ΄Π΅Π½Ρ‹. По Π²Π΅Ρ€Ρ‚ΠΈΠΊΠ°Π»ΠΈ ΡƒΠ³ΠΎΠ» видимости Π΄ΠΎΠ»ΠΆΠ΅Π½ ΡΠΎΡΡ‚Π°Π²Π»ΡΡ‚ΡŒ ΠΎΠΊΠΎΠ»ΠΎ 15 градусов, ΠΏΠΎ Π³ΠΎΡ€ΠΈΠ·ΠΎΠ½Ρ‚Π°Π»ΠΈ β€” Π² Ρ€Π°ΠΉΠΎΠ½Π΅ 45 градусов (Π°Π²Ρ‚ΠΎΡ€ Π²ΠΈΠ΄Π΅ΠΎ β€” Romanautoreview).

ΠΠ΅ΠΏΡ€Π°Π²ΠΈΠ»ΡŒΠ½ΠΎ Π±ΡƒΠ΄Π΅Ρ‚, Ссли ПВЀ Π²ΠΊΠ»ΡŽΡ‡Π°ΡŽΡ‚ΡΡ вмСстС с Π³Π°Π±Π°Ρ€ΠΈΡ‚Π°ΠΌΠΈ. БоотвСтствСнно, ΠΏΡ€ΠΈ ΠΌΠΎΠ½Ρ‚Π°ΠΆΠ΅ Π² Π±Π°ΠΌΠΏΠ΅Ρ€ ΠΈ ΠΏΠΎΠ΄ΠΊΠ»ΡŽΡ‡Π΅Π½ΠΈΠΈ Ρ€Π°Π·ΡŠΠ΅ΠΌΠΎΠ² ΠΈΡ… Π½Π΅ΠΎΠ±Ρ…ΠΎΠ΄ΠΈΠΌΠΎ ΠΏΠΎΡΡ‚Π°Π²ΠΈΡ‚ΡŒ Ρ‚Π°ΠΊΠΈΠΌ ΠΎΠ±Ρ€Π°Π·ΠΎΠΌ, Ρ‡Ρ‚ΠΎΠ±Ρ‹ ΠΎΠ½ΠΈ Π½Π΅ Π±Ρ‹Π»ΠΈ ΠΏΠΎΠ΄ΠΊΠ»ΡŽΡ‡Π΅Π½Ρ‹ ΠΊ Π³Π°Π±Π°Ρ€ΠΈΡ‚Π°ΠΌΠΈ. Ни для ΠΊΠΎΠ³ΠΎ Π½Π΅ сСкрСт, Ρ‡Ρ‚ΠΎ всС соврСмСнныС транспортныС срСдства ΠΎΡΠ½Π°Ρ‰Π°ΡŽΡ‚ΡΡ ΡΠΏΠ΅Ρ†ΠΈΠ°Π»ΡŒΠ½Ρ‹ΠΌ Ρ€Π°Π·ΡŠΠ΅ΠΌΠΎΠΌ для ΠΌΠΎΠ½Ρ‚Π°ΠΆΠ° ПВЀ, поэтому установка ΠΏΡ€ΠΎΡ‚ΠΈΠ²ΠΎΡ‚ΡƒΠΌΠ°Π½Π½Ρ‹Ρ… Ρ„Π°Ρ€ производится строго Π² Π½ΠΈΡ…. И Π΄Π°ΠΆΠ΅ Ссли самих Ρ„ΠΎΠ½Π°Ρ€Π΅ΠΉ Π½Π΅Ρ‚, Ρ‚ΠΎ Π΄ΠΎΠ»ΠΆΠ½Ρ‹ Π±Ρ‹Ρ‚ΡŒ ΡΠΎΠΎΡ‚Π²Π΅Ρ‚ΡΡ‚Π²ΡƒΡŽΡ‰ΠΈΠ΅ Π·Π°Π³Π»ΡƒΡˆΠΊΠΈ, Π² ΠΊΠΎΡ‚ΠΎΡ€Ρ‹Π΅ производится установка ΠΌΠΎΠ½Ρ‚Π°ΠΆΠ½Ρ‹Ρ… ΠΊΡ€Π΅ΠΏΠ»Π΅Π½ΠΈΠΉ ΠΈ самих Ρ„ΠΎΠ½Π°Ρ€Π΅ΠΉ.

Π’ Ρ‚ΠΎΠΌ случаС, Ссли Π½Π° вашСм Π°Π²Ρ‚ΠΎ Π½Π΅Ρ‚ Π·Π°Π³Π»ΡƒΡˆΠ΅ΠΊ ΠΈ посадочных мСст, ΠΊΡƒΠ΄Π° осущСствляСтся установка ΠΏΡ€ΠΎΡ‚ΠΈΠ²ΠΎΡ‚ΡƒΠΌΠ°Π½Π½Ρ‹Ρ… Ρ„Π°Ρ€, Ρ‚ΠΎ Π²Π°ΠΌ придСтся ΡΠ΄Π΅Π»Π°Ρ‚ΡŒ эти мСста самому. Если это Ρ‚Π°ΠΊ, Ρ‚ΠΎ ΠΏΡ€ΠΈ установкС ΠΊΠΎΠΌΠΏΠ»Π΅ΠΊΡ‚Π° Ρ„Π°Ρ€ Π² Π±Π°ΠΌΠΏΠ΅Ρ€ ΠΌΠΎΠΆΠ½ΠΎ ΠΈ Π΄Π°ΠΆΠ΅ Π½ΡƒΠΆΠ½ΠΎ Ρ€ΡƒΠΊΠΎΠ²ΠΎΠ΄ΡΡ‚Π²ΠΎΠ²Π°Ρ‚ΡŒΡΡ трСбованиями ΠΏΠΎ ΠΌΠΎΠ½Ρ‚Π°ΠΆΡƒ ΠΈ ΠΏΠΎΠ΄ΠΊΠ»ΡŽΡ‡Π΅Π½ΠΈΡŽ, Π² ΠΏΡ€ΠΎΡ‚ΠΈΠ²Π½ΠΎΠΌ случаС Ρƒ вас ΠΌΠΎΠ³ΡƒΡ‚ Π²ΠΎΠ·Π½ΠΈΠΊΠ½ΡƒΡ‚ΡŒ ΠΏΡ€ΠΎΠ±Π»Π΅ΠΌΡ‹ ΠΏΡ€ΠΈ ΠΏΡ€ΠΎΡ…ΠΎΠΆΠ΄Π΅Π½ΠΈΠΈ ΠΏΠ»Π°Π½ΠΎΠ²ΠΎΠ³ΠΎ ВО. Волько учитывая трСбования, Π²Ρ‹ смоТСтС Π΄ΠΎΠ±ΠΈΡ‚ΡŒΡΡ Ρ‚ΠΎΠ³ΠΎ, Ρ‡Ρ‚ΠΎ ΠΏΡ€ΠΈΠ±ΠΎΡ€Ρ‹ Π±ΡƒΠ΄ΡƒΡ‚ эффСктивно ΠΈΡΠΏΠΎΠ»ΡŒΠ·ΠΎΠ²Π°Ρ‚ΡŒΡΡ Π½Π° ΠΏΠΎΠ»Π½ΡƒΡŽ ΠΌΠΎΡ‰Π½ΠΎΡΡ‚ΡŒ. Π§Ρ‚ΠΎ касаСтся Ρ€Π΅Π³ΡƒΠ»ΠΈΡ€ΠΎΠ²ΠΊΠΈ, Ρ‚ΠΎ ΠΎΠ½Π° осущСствляСтся послС установки ПВЀ Π² Π±Π°ΠΌΠΏΠ΅Ρ€ вмСсто Π·Π°Π³Π»ΡƒΡˆΠ΅ΠΊ. Как Π²Ρ‹ ΠΏΠΎΠΌΠ½ΠΈΡ‚Π΅, Π²ΠΎ врСмя ΠΌΠΎΠ½Ρ‚Π°ΠΆΠ° Π½Π΅ΠΎΠ±Ρ…ΠΎΠ΄ΠΈΠΌΠΎ ΠΏΡ€ΠΎΠ΄ΡƒΠΌΠ°Ρ‚ΡŒ, ΠΊΠ°ΠΊ ΠΏΡ€Π°Π²ΠΈΠ»ΡŒΠ½ΠΎ ΡΠ΄Π΅Π»Π°Ρ‚ΡŒ ΠΌΠΎΠ½Ρ‚Π°ΠΆ фонаря, Ρ‡Ρ‚ΠΎΠ±Ρ‹ Π΅Π³ΠΎ ΡƒΠ³ΠΎΠ» свСтового ΠΏΠΎΡ‚ΠΎΠΊΠ° Π±Ρ‹Π» ΠΏΡ€Π°Π²ΠΈΠ»ΡŒΠ½Ρ‹ΠΌ.

Бпособы ΠΏΠΎΠ΄ΠΊΠ»ΡŽΡ‡Π΅Π½ΠΈΡ

ΠœΠΎΠ½Ρ‚Π°ΠΆ ΠΊΠ½ΠΎΠΏΠΊΠΈ Π²Ρ‹ΠΊΠ»ΡŽΡ‡Π΅Π½ΠΈΡ ПВЀ

Π˜Ρ‚Π°ΠΊ, Ссли установка ΠΏΡ€ΠΎΡ‚ΠΈΠ²ΠΎΡ‚ΡƒΠΌΠ°Π½Π½Ρ‹Ρ… Ρ„Π°Ρ€ осущСствляСтся Π² Π΄ΠΎΠΌΠ°ΡˆΠ½ΠΈΡ… условиях, Ρ‚ΠΎ Ρ‡Ρ‚ΠΎΠ±Ρ‹ всС ΡΠ΄Π΅Π»Π°Ρ‚ΡŒ ΠΏΡ€Π°Π²ΠΈΠ»ΡŒΠ½ΠΎ, ΠΏΠΎΡΡ‚Π°Π²ΠΈΡ‚ΡŒ ΠΊΠΎΠΌΠΏΠ»Π΅ΠΊΡ‚ Π² Π±Π°ΠΌΠΏΠ΅Ρ€ вмСсто Π·Π°Π³Π»ΡƒΡˆΠ΅ΠΊ ΠΈ ΠΏΠΎΠ΄ΠΊΠ»ΡŽΡ‡ΠΈΡ‚ΡŒ ΠΊ ΠΏΡ€ΠΎΠ²ΠΎΠ΄ΠΊΠ΅, ΠΌΠΎΠΆΠ½ΠΎ ΠΈΡΠΏΠΎΠ»ΡŒΠ·ΠΎΠ²Π°Ρ‚ΡŒ ΠΎΠ΄ΠΈΠ½ ΠΈΠ· Π΄Π²ΡƒΡ… Π²Π°Ρ€ΠΈΠ°Π½Ρ‚ΠΎΠ² ΠΏΠΎΠ΄ΠΊΠ»ΡŽΡ‡Π΅Π½ΠΈΡ. Как ΠΌΡ‹ ΠΏΠΎΠΌΠ½ΠΈΠΌ, ПВЀ Π² любом случаС Π½Π΅ Π²ΠΊΠ»ΡŽΡ‡Π°ΡŽΡ‚ΡΡ ΠΎΡ‚ Π³Π°Π±Π°Ρ€ΠΈΡ‚Π½Ρ‹Ρ… ΠΎΠ³Π½Π΅ΠΉ, это Π½Π΅ΠΎΠ±Ρ…ΠΎΠ΄ΠΈΠΌΠΎ ΠΏΠΎΠΌΠ½ΠΈΡ‚ΡŒ ΠΏΠ΅Ρ€Π΅Π΄ установкой ΠΊΠΎΠΌΠΏΠ»Π΅ΠΊΡ‚Π° Π² Π±Π°ΠΌΠΏΠ΅Ρ€ ΠΈ ΠΏΠΎΠ΄ΠΊΠ»ΡŽΡ‡Π΅Π½ΠΈΠ΅ΠΌ всСх Ρ€Π°Π·ΡŠΠ΅ΠΌΠΎΠ². Π’Π°Ρ€ΠΈΠ°Π½Ρ‚ΠΎΠ² ΠΌΠΎΠ½Ρ‚Π°ΠΆΠ° ΠΏΡ€ΠΈΠ±ΠΎΡ€ΠΎΠ² ΠΌΠΎΠΆΠ΅Ρ‚ Π±Ρ‹Ρ‚ΡŒ нСсколько. Один ΠΈΠ· самых простых Π² ΠΏΠ»Π°Π½Π΅ установки ΠΈ надСТности β€” это соСдинСниС с ΠΏΡ€ΠΎΠ²ΠΎΠ΄Π°ΠΌΠΈ транспортного срСдства, ΠΊΠΎΡ‚ΠΎΡ€Ρ‹Π΅ ΡƒΠΆΠ΅ ΠΏΡ€ΠΎΠ²Π΅Π΄Π΅Π½Ρ‹ ΠΈ ΠΏΡ€Π΅Π΄Π½Π°Π·Π½Π°Ρ‡Π΅Π½Ρ‹ для ΠΏΡ€ΠΎΡ‚ΠΈΠ²ΠΎΡ‚ΡƒΠΌΠ°Π½ΠΎΠΊ.

Π’ Π΄Π°Π½Π½ΠΎΠΌ случаС ΠΌΠΎΠΆΠ½ΠΎ всС ΡΠ΄Π΅Π»Π°Ρ‚ΡŒ ΠΏΡ€ΠΎΡ‰Π΅. Заводская ΠΏΡ€ΠΎΠ²ΠΎΠ΄ΠΊΠ° ΡƒΠΆΠ΅ оснащСна всСми Π½ΡƒΠΆΠ½Ρ‹ΠΌΠΈ ΠΊΠΎΠΌΠΏΠΎΠ½Π΅Π½Ρ‚Π°ΠΌΠΈ, Π² частности, ΠΏΡ€ΠΎΠ²ΠΎΠ΄Π°ΠΌΠΈ ΠΈ Ρ€Π°Π·ΡŠΠ΅ΠΌΠ°ΠΌΠΈ, ΠΊΠΎΡ‚ΠΎΡ€Ρ‹Π΅ Π½Π΅ΠΎΠ±Ρ…ΠΎΠ΄ΠΈΠΌΡ‹ для ΠΏΠΎΠ΄ΠΊΠ»ΡŽΡ‡Π΅Π½ΠΈΡ. Π’ ΠΏΡ€ΠΎΠ²ΠΎΠ΄ΠΊΠ΅ Π΅ΡΡ‚ΡŒ всС Ρ€Π΅Π»Π΅ ΠΈ ΠΏΡ€Π΅Π΄ΠΎΡ…Ρ€Π°Π½ΠΈΡ‚Π΅Π»ΠΈ, Π° Ρ‚Π°ΠΊΠΆΠ΅ Π²Ρ‹ΠΊΠ»ΡŽΡ‡Π°Ρ‚Π΅Π»ΡŒ, с ΠΏΠΎΠΌΠΎΡ‰ΡŒΡŽ ΠΊΠΎΡ‚ΠΎΡ€ΠΎΠ³ΠΎ ΠΌΠΎΠΆΠ½ΠΎ ΡƒΠΏΡ€Π°Π²Π»ΡΡ‚ΡŒ устройствами. По сути, Π²Π°ΠΌ остаСтся Ρ‚ΠΎΠ»ΡŒΠΊΠΎ Π΄Π΅ΠΌΠΎΠ½Ρ‚ΠΈΡ€ΠΎΠ²Π°Ρ‚ΡŒ Π·Π°Π³Π»ΡƒΡˆΠΊΠΈ ΠΈΠ· Π±Π°ΠΌΠΏΠ΅Ρ€Π° ΠΈ ΡƒΡΡ‚Π°Π½ΠΎΠ²ΠΈΡ‚ΡŒ ΠΊΠΎΠΌΠΏΠ»Π΅ΠΊΡ‚ ПВЀ, ΠΏΠΎΠ΄ΠΊΠ»ΡŽΡ‡ΠΈΠ² Π΅Π³ΠΎ ΠΊΠΎ всСм Ρ€Π°Π·ΡŠΠ΅ΠΌΠ°ΠΌ. Π”Π°Π½Π½Ρ‹ΠΉ Π²Π°Ρ€ΠΈΠ°Π½Ρ‚ Π°ΠΊΡ‚ΡƒΠ°Π»Π΅Π½ Π½Π΅ для всСх транспортных срСдства, Π° Ρ‚ΠΎΠ»ΡŒΠΊΠΎ для машин, оснащСнных Π·Π°Π³Π»ΡƒΡˆΠΊΠ°ΠΌΠΈ ΠΈΠ»ΠΈ Π°Π²Ρ‚ΠΎΠΌΠΎΠ±ΠΈΠ»Π΅ΠΉ, Π³Π΄Π΅ Π½ΡƒΠΆΠ½ΠΎ ΠΏΠΎΠΌΠ΅Π½ΡΡ‚ΡŒ Π²Ρ‹ΡˆΠ΅Π΄ΡˆΠΈΠ΅ ΠΈΠ· строя ΠΏΡ€ΠΎΡ‚ΠΈΠ²ΠΎΡ‚ΡƒΠΌΠ°Π½ΠΊΠΈ (Π°Π²Ρ‚ΠΎΡ€ Π²ΠΈΠ΄Π΅ΠΎ β€” АндрСй ΠšΡƒΠ²ΡˆΠΈΠ½ΠΎΠ²).

ΠŸΠΎΠΊΡƒΠΏΠ°Ρ ΠΊΠΎΠΌΠΏΠ»Π΅ΠΊΡ‚ ПВЀ Π½Π΅ΠΎΠ±Ρ…ΠΎΠ΄ΠΈΠΌΠΎ ΡƒΠ΄ΠΎΡΡ‚ΠΎΠ²Π΅Ρ€ΠΈΡ‚ΡŒΡΡ Π² Ρ‚ΠΎΠΌ, Ρ‡Ρ‚ΠΎ эту модСль ΠΌΠΎΠΆΠ½ΠΎ ΡƒΡΡ‚Π°Π½ΠΎΠ²ΠΈΡ‚ΡŒ Π½Π° Π²Π°ΡˆΡƒ ΠΌΠ°ΡˆΠΈΠ½Ρƒ. Π”ΠΎΠ»ΠΆΠ½Ρ‹ ΡΠΎΠΎΡ‚Π²Π΅Ρ‚ΡΡ‚Π²ΠΎΠ²Π°Ρ‚ΡŒ Π½Π΅ Ρ‚ΠΎΠ»ΡŒΠΊΠΎ Ρ€Π°Π·ΡŠΠ΅ΠΌΡ‹, Π½ΠΎ ΠΈ ΠΏΠ°Ρ€Π°ΠΌΠ΅Ρ‚Ρ€Ρ‹ установки. Π’ Ρ‚ΠΎΠΌ случаС, Ссли транспортноС срСдство Π½Π΅ оснащСно ΠΏΡ€ΠΎΠ²ΠΎΠ΄ΠΊΠΎΠΉ для ΠΌΠΎΠ½Ρ‚Π°ΠΆΠ° ΠΏΡ€ΠΎΡ‚ΠΈΠ²ΠΎΡ‚ΡƒΠΌΠ°Π½ΠΎΠΊ, Π²Π°ΠΌ Π½ΡƒΠΆΠ½ΠΎ Π±ΡƒΠ΄Π΅Ρ‚ Π΅Π΅ провСсти своими силами. Π§Ρ‚ΠΎΠ±Ρ‹ схСма ΠΏΠΎΠ΄ΠΊΠ»ΡŽΡ‡Π΅Π½ΠΈΡ ΠΏΡ€ΠΎΡ‚ΠΈΠ²ΠΎΡ‚ΡƒΠΌΠ°Π½Π½Ρ‹Ρ… Ρ„Π°Ρ€ Π±Ρ‹Π»Π° ΠΏΡ€Π°Π²ΠΈΠ»ΡŒΠ½ΠΎΠΉ, вмСстС с ΠΊΠΎΠΌΠΏΠ»Π΅ΠΊΡ‚ΠΎΠΌ слСдуСт ΠΊΡƒΠΏΠΈΡ‚ΡŒ Π΄ΠΎΠΏΠΎΠ»Π½ΠΈΡ‚Π΅Π»ΡŒΠ½Ρ‹Π΅ ΠΏΡ€ΠΎΠ²ΠΎΠ΄Π°, ΠΎΠ½ΠΈ Π΄ΠΎΠ»ΠΆΠ½Ρ‹ Π±Ρ‹Ρ‚ΡŒ ΠΌΠ΅Π΄Π½Ρ‹ΠΌΠΈ ΠΈ Π·Π°ΠΈΠ·ΠΎΠ»ΠΈΡ€ΠΎΠ²Π°Π½Π½Ρ‹ΠΌΠΈ. ΠšΡ€ΠΎΠΌΠ΅ Ρ‚ΠΎΠ³ΠΎ, Π½ΡƒΠΆΠ½ΠΎ Ρ‚Π°ΠΊΠΆΠ΅ приобрСсти Π²Ρ‹ΠΊΠ»ΡŽΡ‡Π°Ρ‚Π΅Π»ΡŒ ΠΏΡ€ΠΎΡ‚ΠΈΠ²ΠΎΡ‚ΡƒΠΌΠ°Π½Π½Ρ‹Ρ… Ρ„Π°Ρ€, ΠΏΡ€Π΅Π΄ΠΎΡ…Ρ€Π°Π½ΠΈΡ‚Π΅Π»ΡŒ ΠΈ Ρ€Π΅Π»Π΅, ΠΈΠ½ΠΎΠ³Π΄Π° эти ΠΊΠΎΠΌΠΏΠΎΠ½Π΅Π½Ρ‚Ρ‹ ΠΈΠ΄ΡƒΡ‚ Π² ΠΊΠΎΠΌΠΏΠ»Π΅ΠΊΡ‚Π΅ с ПВЀ.

ΠŸΠΎΠΌΠ½ΠΈΡ‚Π΅ ΠΎ Ρ‚ΠΎΠΌ, Ρ‡Ρ‚ΠΎ противотуманная ΠΎΠΏΡ‚ΠΈΠΊΠ° Π½ΠΈ Π² ΠΊΠΎΠ΅ΠΌ случаС Π½Π΅ Π΄ΠΎΠ»ΠΆΠ½Π° Π²ΠΊΠ»ΡŽΡ‡Π°Ρ‚ΡŒΡΡ вмСстС с Π³Π°Π±Π°Ρ€ΠΈΡ‚Π°ΠΌΠΈ. Бтандартная ΠΏΡ€ΠΎΠ²ΠΎΠ΄ΠΊΠ°, Π° Ρ‚Π°ΠΊΠΆΠ΅ Π²Ρ‹ΠΊΠ»ΡŽΡ‡Π°Ρ‚Π΅Π»ΡŒ Π½Π΅ рассчитаны Π½Π° ΡƒΡ€ΠΎΠ²Π΅Π½ΡŒ Ρ‚ΠΎΠΊΠ°, ΠΊΠΎΡ‚ΠΎΡ€Ρ‹Π΅ ΠΏΠΎΡ‚Ρ€Π΅Π±Π»ΡΡŽΡ‚ эта ΠΎΠΏΡ‚ΠΈΠΊΠ°. Π‘Π»Π΅Π΄ΡƒΠ΅Ρ‚ ΠΎΡ‚ΠΌΠ΅Ρ‚ΠΈΡ‚ΡŒ, Ρ‡Ρ‚ΠΎ элСктричСская Ρ†Π΅ΠΏΡŒ Π² любом случаС Π΄ΠΎΠ»ΠΆΠ½Π° Π±Ρ‹Ρ‚ΡŒ оснащСна ΠΏΡ€Π΅Π΄ΠΎΡ…Ρ€Π°Π½ΠΈΡ‚Π΅Π»Π΅ΠΌ, это ΠΎΡ‡Π΅Π½ΡŒ Π²Π°ΠΆΠ½ΠΎ. Волько ΠΏΡ€Π΅Π΄ΠΎΡ…Ρ€Π°Π½ΠΈΡ‚Π΅Π»ΡŒ ΠΏΠΎΠ·Π²ΠΎΠ»ΠΈΡ‚ спасти вашС транспортноС срСдство ΠΎΡ‚ возгорания, Ссли Π² Ρ†Π΅ΠΏΠΈ случайно ΠΏΡ€ΠΎΠΈΠ·ΠΎΠΉΠ΄Π΅Ρ‚ Π·Π°ΠΌΡ‹ΠΊΠ°Π½ΠΈΠ΅. Π‘Π°ΠΌ ΠΏΡ€Π΅Π΄ΠΎΡ…Ρ€Π°Π½ΠΈΡ‚Π΅Π»ΡŒ Π΄ΠΎΠ»ΠΆΠ΅Π½ Π±Ρ‹Ρ‚ΡŒ достаточно ΠΌΠΎΡ‰Π½Ρ‹ΠΌ β€” ΡƒΡ€ΠΎΠ²Π΅Π½ΡŒ мощности рассчитываСтся Π² соотвСтствии с ΠΌΠΎΡ‰Π½ΠΎΡΡ‚ΡŒΡŽ самой ΠΎΠΏΡ‚ΠΈΠΊΠΈ.

К ΠΏΡ€ΠΈΠΌΠ΅Ρ€Ρƒ, Ссли ΠΎΠΏΡ‚ΠΈΠΊΠ° рассчитана Π½Π° 60 Π²Π°Ρ‚Ρ‚, Ρ‚ΠΎ ΠΏΡ€ΠΈ ΠΏΠΎΠΊΡƒΠΏΠΊΠ΅ прСдохранитСля Π½Π΅ΠΎΠ±Ρ…ΠΎΠ΄ΠΈΠΌΠΎ Π·Π°Ρ€Π°Π½Π΅Π΅ Ρ€Π°ΡΡΡ‡ΠΈΡ‚Π°Ρ‚ΡŒ Π΅Π³ΠΎ ΠΌΠΎΡ‰Π½ΠΎΡΡ‚ΡŒ Π² соотвСтствии с Ρ„ΠΎΡ€ΠΌΡƒΠ»ΠΎΠΉ:

60 Π’Ρ‚ * 2 / 12 Π’ = 10 А

Π’ΠΎ Π΅ΡΡ‚ΡŒ само устройство Π΄ΠΎΠ»ΠΆΠ½ΠΎ Π±Ρ‹Ρ‚ΡŒ Π²Ρ‹Π±Ρ€Π°Π½ΠΎ с ΡƒΡ‡Π΅Ρ‚ΠΎΠΌ нСбольшого запаса. ΠžΠΏΡ‚ΠΈΠΌΠ°Π»ΡŒΠ½Ρ‹ΠΌ Π²Π°Ρ€ΠΈΠ°Π½Ρ‚ΠΎΠΌ Π±ΡƒΠ΄Π΅Ρ‚ ΠΏΠΎΠΊΡƒΠΏΠΊΠ° прСдохранитСля Π½ΠΎΠΌΠΈΠ½Π°Π»ΠΎΠΌ Π² 15 Π°ΠΌΠΏΠ΅Ρ€.

Π‘Ρ…Π΅ΠΌΡ‹ ΠΏΠΎΠ΄ΠΊΠ»ΡŽΡ‡Π΅Π½ΠΈΡ

Π‘Ρ…Π΅ΠΌΠ° подсоСдинСния ΠΏΡ€ΠΎΡ‚ΠΈΠ²ΠΎΡ‚ΡƒΠΌΠ°Π½Π½ΠΎΠΉ ΠΎΠΏΡ‚ΠΈΠΊΠΈ

Π’Π΅ΠΏΠ΅Ρ€ΡŒ ΠΏΠ΅Ρ€Π΅ΠΉΠ΄Π΅ΠΌ ΠΊ схСмам, Π² соотвСтствии с ΠΊΠΎΡ‚ΠΎΡ€Ρ‹ΠΌΠΈ ΠΊΠΎΠΌΠΏΠ»Π΅ΠΊΡ‚ ΠΏΡ€ΠΎΡ‚ΠΈΠ²ΠΎΡ‚ΡƒΠΌΠ°Π½ΠΎΠΊ Π΄ΠΎΠ»ΠΆΠ΅Π½ Π±Ρ‹Ρ‚ΡŒ ΠΏΠΎΠ΄ΠΊΠ»ΡŽΡ‡Π΅Π½ ΠΊΠΎ всСм Ρ€Π°Π·ΡŠΠ΅ΠΌΠ°ΠΌ. Один ΠΈΠ· самых простых Π²Π°Ρ€ΠΈΠ°Π½Ρ‚ΠΎΠ² β€” это ΠΏΠΎΠ΄ΠΊΠ»ΡŽΡ‡Π΅Π½ΠΈΠ΅ ΠΎΡ‚ аккумуляторной Π±Π°Ρ‚Π°Ρ€Π΅ΠΈ, для этого Π΄ΠΎΠΏΠΎΠ»Π½ΠΈΡ‚Π΅Π»ΡŒΠ½ΠΎ Π² Ρ†Π΅ΠΏΡŒ устанавливаСтся ΠΊΠ½ΠΎΠΏΠΊΠ° Π²ΠΊΠ»ΡŽΡ‡Π΅Π½ΠΈΡ ΠΏΡ€ΠΎΡ‚ΠΈΠ²ΠΎΡ‚ΡƒΠΌΠ°Π½Π½Ρ‹Ρ… Ρ„Π°Ρ€. КаТдая противотуманная Ρ„Π°Ρ€Π° оснащаСтся двумя ΠΊΠΎΠ½Ρ‚Π°ΠΊΡ‚Π°ΠΌΠΈ для питания. Один ΠΈΠ· этих ΠΊΠΎΠ½Ρ‚Π°ΠΊΡ‚ΠΎΠ² Π½Π° ΠΎΠ±ΠΎΠΈΡ… ΠΊΠΎΠΌΠΏΠΎΠ½Π΅Π½Ρ‚Π°Ρ… ΠΊΠΎΠΌΠΏΠ»Π΅ΠΊΡ‚Π° соСдиняСтся ΠΊΠ°Π±Π΅Π»Π΅ΠΌ Π² ΠΎΠ΄Π½Ρƒ Ρ†Π΅ΠΏΡŒ ΠΈ ΠΏΠΎΠ΄ΠΊΠ»ΡŽΡ‡Π°Π΅Ρ‚ΡΡ ΠΊ ΠΊΡƒΠ·ΠΎΠ²Ρƒ транспортного срСдства, Ρ‚ΠΎ Π΅ΡΡ‚ΡŒ массС. ΠœΠΎΠ½Ρ‚Π°ΠΆ ΠΊ массС Π΄ΠΎΠ»ΠΆΠ΅Π½ Π±Ρ‹Ρ‚ΡŒ осущСствлСн с ΠΏΠΎΠΌΠΎΡ‰ΡŒΡŽ ТСсткого фиксатора ΠΈΠ»ΠΈ крСплСния.

Как извСстно, ΠΊΡƒΠ·ΠΎΠ² автомобиля ΠΈΠ·Π½Π°Ρ‡Π°Π»ΡŒΠ½ΠΎ ΠΏΠΎΠ΄ΠΊΠ»ΡŽΡ‡Π°Π΅Ρ‚ΡΡ ΠΊ минусу ΠΠšΠ‘. ΠŸΠΎΡΡ‚ΠΎΠΌΡƒ для Ρ‚ΠΎΠ³ΠΎ, Ρ‡Ρ‚ΠΎΠ±Ρ‹ ΠΏΡ€Π΅Π΄ΠΎΡ‚Π²Ρ€Π°Ρ‚ΠΈΡ‚ΡŒ Π²ΠΎΠ·ΠΌΠΎΠΆΠ½Ρ‹Π΅ нСисправности Π² дальнСйшСм, ΠΎΠΏΡ‚ΠΈΠΌΠ°Π»ΡŒΠ½Ρ‹ΠΌ Π²Π°Ρ€ΠΈΠ°Π½Ρ‚ΠΎΠΌ Π±ΡƒΠ΄Π΅Ρ‚ ΠΏΠΎΠ΄ΠΊΠ»ΡŽΡ‡Π΅Π½ΠΈΠ΅ ΠΏΡ€ΠΎΠ²ΠΎΠ΄Π° нСпосрСдствСнно ΠΊ минусовой ΠΊΠ»Π΅ΠΌΠΌΠ΅ аккумулятора. Если Π² Π±ΡƒΠ΄ΡƒΡ‰Π΅ΠΌ Π½Π° ΠΊΡƒΠ·ΠΎΠ²Π΅ ΠΈΠ»ΠΈ элСмСнтС крСплСния Π½Π°Ρ‡Π½Π΅Ρ‚ ΠΎΠ±Ρ€Π°Π·ΠΎΠ²Ρ‹Π²Π°Ρ‚ΡŒΡΡ Ρ€ΠΆΠ°Π²Ρ‡ΠΈΠ½Π°, это Π±ΡƒΠ΄Π΅Ρ‚ ΡΠΏΠΎΡΠΎΠ±ΡΡ‚Π²ΠΎΠ²Π°Ρ‚ΡŒ ΠΏΠΎΡ‚Π΅Ρ€Π΅ ΠΊΠΎΠ½Ρ‚Π°ΠΊΡ‚Π° ΠΈ, соотвСтствСнно, сигнала. Π’Ρ‚ΠΎΡ€Ρ‹Π΅ ΠΊΠΎΠ½Ρ‚Π°ΠΊΡ‚Ρ‹ Π½Π΅ΠΎΠ±Ρ…ΠΎΠ΄ΠΈΠΌΠΎ ΠΏΠΎΠ΄ΡΠΎΠ΅Π΄ΠΈΠ½ΠΈΡ‚ΡŒ ΠΊ ΠΏΠ»ΡŽΡΡƒ ΠΠšΠ‘, ΠΎΠ½ΠΈ ΡΠΎΠ΅Π΄ΠΈΠ½ΡΡŽΡ‚ΡΡ ΠΌΠ΅ΠΆΠ΄Ρƒ собой, послС Ρ‡Π΅Π³ΠΎ кабСль выводится Π½Π° ΠΊΠΎΠ½Ρ‚Π°ΠΊΡ‚ ΠΏΠΎΠ΄ Π½ΠΎΠΌΠ΅Ρ€ΠΎΠΌ 87 Π² соотвСтствии со схСмой.

ПослС этих дСйствий ΠΎΡΡ‚Π°Π»ΡŒΠ½Ρ‹Π΅ ΠΊΠΎΠ½Ρ‚Π°ΠΊΡ‚Ρ‹ Ρ€Π΅Π»Π΅ ΡΠΎΠ΅Π΄ΠΈΠ½ΡΡŽΡ‚ΡΡ с Ρ€Π°Π·ΡŠΠ΅ΠΌΠ°ΠΌΠΈ:

  • ΠΏΠΎΠ΄ Π½ΠΎΠΌΠ΅Ρ€ΠΎΠΌ 30 β€” ΠΏΠΎΠ΄ΠΊΠ»ΡŽΡ‡Π°Π΅Ρ‚ΡΡ ΠΊ плюсовой ΠΊΠ»Π΅ΠΌΠΌΠ΅ ΠΠšΠ‘ Ρ‡Π΅Ρ€Π΅Π· ΠΏΡ€Π΅Π΄ΠΎΡ…Ρ€Π°Π½ΠΈΡ‚Π΅Π»ΡŒ;
  • Π½ΠΎΠΌΠ΅Ρ€ 86 β€” ΠΊ минусовой ΠΊΠ»Π΅ΠΌΠΌΠ΅ ΠΠšΠ‘ Π»ΠΈΠ±ΠΎ ΠΆΠ΅ ΠΊ ΠΊΡƒΠ·ΠΎΠ²Ρƒ Π°Π²Ρ‚ΠΎ, ΠΊΠ°ΠΊ сказано Π²Ρ‹ΡˆΠ΅;
  • ΠΏΠΎΠ΄ Π½ΠΎΠΌΠ΅Ρ€ΠΎΠΌ 85 β€” ΠΊ плюсовой ΠΊΠ»Π΅ΠΌΠΌΠ΅ ΠΠšΠ‘, для ΠΏΠΎΠ΄ΠΊΠ»ΡŽΡ‡Π΅Π½ΠΈΡ Π΄ΠΎΠΏΠΎΠ»Π½ΠΈΡ‚Π΅Π»ΡŒΠ½ΠΎ ΠΈΡΠΏΠΎΠ»ΡŒΠ·ΡƒΠ΅Ρ‚ΡΡ Π²Ρ‹ΠΊΠ»ΡŽΡ‡Π°Ρ‚Π΅Π»ΡŒ, Π° Ρ‚Π°ΠΊΠΆΠ΅ ΠΏΡ€Π΅Π΄ΠΎΡ…Ρ€Π°Π½ΠΈΡ‚Π΅Π»ΡŒ, ΠΏΡ€ΠΈΠΌΠ΅Π½ΡΡŽΡ‰ΠΈΠΉΡΡ Π² ΠΏΡ€ΠΎΠ²ΠΎΠ΄ΠΊΠ΅ ΠΊΠΎΠ½Ρ‚Π°ΠΊΡ‚Π° ΠΏΠΎΠ΄ Π½ΠΎΠΌΠ΅Ρ€ΠΎΠΌ 30.

Π‘Ρ…Π΅ΠΌΠ° для ΠΌΠΎΠ½Ρ‚Π°ΠΆΠ° ПВЀ

Π§Ρ‚ΠΎ касаСтся ΠΊΠ½ΠΎΠΏΠΊΠΈ Π²ΠΊΠ»ΡŽΡ‡Π΅Π½ΠΈΡ ΠΎΠΏΡ‚ΠΈΠΊΠΈ, Ρ‚ΠΎ ΠΎΠ½Π° монтируСтся Π² салонС транспортного срСдства, Ρ‚Π°ΠΌ, Π³Π΄Π΅ ΡƒΠ΄ΠΎΠ±Π½ΠΎ Π²ΠΎΠ΄ΠΈΡ‚Π΅Π»ΡŽ, ΠΎΠ±Ρ‹Ρ‡Π½ΠΎ это мСсто Π² Ρ€Π°ΠΉΠΎΠ½Π΅ ΠΏΡ€ΠΈΠ±ΠΎΡ€Π½ΠΎΠΉ ΠΏΠ°Π½Π΅Π»ΠΈ. Π‘Π°ΠΌΠΎ Ρ€Π΅Π»Π΅ слСдуСт Π²ΠΌΠΎΠ½Ρ‚ΠΈΡ€ΠΎΠ²Π°Ρ‚ΡŒ Π½Π° Π±Π»ΠΎΠΊΠ΅ ΠΏΡ€Π΅Π΄ΠΎΡ…Ρ€Π°Π½ΠΈΡ‚Π΅Π»Π΅ΠΉ, ΠΎΠ±Ρ‹Ρ‡Π½ΠΎ этот ΠΏΡ€ΠΈΠ±ΠΎΡ€ оснащаСтся Ρ€Π°Π·ΡŠΠ΅ΠΌΠ°ΠΌΠΈ для Ρ€Π΅Π»Π΅, ΠΊΠΎΡ‚ΠΎΡ€Ρ‹Π΅ ΠΌΠΎΠΆΠ½ΠΎ ΠΈΡΠΏΠΎΠ»ΡŒΠ·ΠΎΠ²Π°Ρ‚ΡŒ ΠΏΡ€ΠΈ ΠΏΠΎΠ΄ΠΊΠ»ΡŽΡ‡Π΅Π½ΠΈΠΈ Π΄ΠΎΠΏΠΎΠ»Π½ΠΈΡ‚Π΅Π»ΡŒΠ½ΠΎΠ³ΠΎ оборудования. ΠŸΡ€ΠΈ нСобходимости ΠΌΠΎΠΆΠ½ΠΎ ΠΏΠΎΠΏΡ€ΠΎΠ±ΠΎΠ²Π°Ρ‚ΡŒ ΠΏΠΎΠ΄ΠΊΠ»ΡŽΡ‡ΠΈΡ‚ΡŒΡΡ ΠΊ Ρ€Π΅Π»Π΅, ΠΊΠΎΡ‚ΠΎΡ€ΠΎΠ΅ ΡƒΠΆΠ΅ установлСно Π² Π±Π»ΠΎΠΊΠ΅.

МоТно ΠΈΡΠΏΠΎΠ»ΡŒΠ·ΠΎΠ²Π°Ρ‚ΡŒ Π±ΠΎΠ»Π΅Π΅ ΡΠ»ΠΎΠΆΠ½ΡƒΡŽ схСму подсоСдинСния ΠΊ ΠΏΠ»ΡŽΡΠΎΠ²ΠΎΠΌΡƒ Π²Ρ‹Ρ…ΠΎΠ΄Ρƒ ΠΠšΠ‘ Ρ‡Π΅Ρ€Π΅Π· Π·Π°ΠΌΠΎΠΊ заТигания. Π˜Π½Ρ‹ΠΌΠΈ словами, ΠΎΡ‚ ΠΊΠ½ΠΎΠΏΠΊΠΈ Π²ΠΊΠ»ΡŽΡ‡Π΅Π½ΠΈΡ ΠΎΠΏΡ‚ΠΈΠΊΠΈ ΠΏΡ€ΠΎΠ²ΠΎΠ΄ΠΊΠ° укладываСтся ΠΈ соСдиняСтся Π½Π΅ с аккумулятором, Π° с Π»ΡŽΠ±Ρ‹ΠΌ ΠΊΠ°Π±Π΅Π»Π΅ΠΌ, Π³Π΄Π΅ Π΅ΡΡ‚ΡŒ напряТСниС, ΠΏΠΎΡΠ²Π»ΡΡŽΡ‰Π΅Π΅ΡΡ Π² Ρ€Π΅Π·ΡƒΠ»ΡŒΡ‚Π°Ρ‚Π΅ Π²ΠΊΠ»ΡŽΡ‡Π΅Π½ΠΈΡ заТигания. ΠŸΠΎΡΠΊΠΎΠ»ΡŒΠΊΡƒ ΠΎΠΏΡ‚ΠΈΠΊΠ° Π² Π΄Π°Π½Π½ΠΎΠΌ случаС Π±ΡƒΠ΄Π΅Ρ‚ Π²ΠΊΠ»ΡŽΡ‡Π°Ρ‚ΡŒΡΡ Ρ‚ΠΎΠ»ΡŒΠΊΠΎ с Π²ΠΊΠ»ΡŽΡ‡Π΅Π½Π½Ρ‹ΠΌ Π·Π°ΠΆΠΈΠ³Π°Π½ΠΈΠ΅ΠΌ, Π² Π±ΡƒΠ΄ΡƒΡ‰Π΅ΠΌ это ΠΏΠΎΠ·Π²ΠΎΠ»ΠΈΡ‚ ΠΈΠ·Π±Π΅ΠΆΠ°Ρ‚ΡŒ Π²ΠΎΠ·ΠΌΠΎΠΆΠ½ΠΎΠΉ разрядки ΠΠšΠ‘, Ссли Π²ΠΎΠ΄ΠΈΡ‚Π΅Π»ΡŒ Π·Π°Π±ΡƒΠ΄Π΅Ρ‚ ΠΎΡ‚ΠΊΠ»ΡŽΡ‡ΠΈΡ‚ΡŒ ΠΏΡ€ΠΎΡ‚ΠΈΠ²ΠΎΡ‚ΡƒΠΌΠ°Π½ΠΊΠΈ. ΠžΠΏΠΈΡΠ°Π½Π½Ρ‹Π΅ Π²Ρ‹ΡˆΠ΅ схСмы Π΄Π°Π΄ΡƒΡ‚ Π²ΠΎΠ·ΠΌΠΎΠΆΠ½ΠΎΡΡ‚ΡŒ ΠΏΠΎΠ΄ΠΊΠ»ΡŽΡ‡ΠΈΡ‚ΡŒ ΠΎΠΏΡ‚ΠΈΠΊΡƒ Π² соотвСтствии с трСбованиями ΠŸΠ”Π” ΠΈ Π΄Π΅ΠΉΡΡ‚Π²ΡƒΡŽΡ‰Π΅Π³ΠΎ Π·Π°ΠΊΠΎΠ½ΠΎΠ΄Π°Ρ‚Π΅Π»ΡŒΡΡ‚Π²Π° Π Π€, для этого рСгулятор Π²ΠΊΠ»ΡŽΡ‡Π΅Π½ΠΈΡ ΠΏΡ€ΠΎΡ‚ΠΈΠ²ΠΎΡ‚ΡƒΠΌΠ°Π½ΠΎΠΊ Π΄ΠΎΠ»ΠΆΠ΅Π½ Π±Ρ‹Ρ‚ΡŒ ΠΏΠΎΠ΄ΠΊΠ»ΡŽΡ‡Π΅Π½ ΠΊ Π²Ρ‹ΠΊΠ»ΡŽΡ‡Π°Ρ‚Π΅Π»ΡŽ внСшнСго освСщСния ΠΌΠ°ΡˆΠΈΠ½Ρ‹.

Π’ΠΈΠ΄Π΅ΠΎ Β«ΠœΠΎΠ½Ρ‚Π°ΠΆ ΠΏΡ€ΠΎΡ‚ΠΈΠ²ΠΎΡ‚ΡƒΠΌΠ°Π½Π½ΠΎΠΉ ΠΎΠΏΡ‚ΠΈΠΊΠΈ Π½Π° Π›Π°Π΄Π° Π“Ρ€Π°Π½Ρ‚Π°Β»

На ΠΏΡ€ΠΈΠΌΠ΅Ρ€Π΅ Π›Π°Π΄Ρ‹ Π“Ρ€Π°Π½Ρ‚Ρ‹ Π²Ρ‹ ΠΌΠΎΠΆΠ΅Ρ‚Π΅ ΠΎΠ·Π½Π°ΠΊΠΎΠΌΠΈΡ‚ΡŒΡΡ с ΠΏΡ€ΠΎΡ†Π΅Π΄ΡƒΡ€ΠΎΠΉ ΠΌΠΎΠ½Ρ‚Π°ΠΆΠ° ПВЀ (Π°Π²Ρ‚ΠΎΡ€ Π²ΠΈΠ΄Π΅ΠΎ β€” NapoziTiV).

Π˜Π·Π²ΠΈΠ½ΠΈΡ‚Π΅, Π² настоящСС врСмя Π½Π΅Ρ‚ доступных опросов.

Π Π΅ΠΊΠΎΠΌΠ΅Π½Π΄ΡƒΠ΅ΠΌ ΠΊ ΠΏΡ€ΠΎΡ‡Ρ‚Π΅Π½ΠΈΡŽ

ΠšΠΎΠΌΠΌΠ΅Π½Ρ‚Π°Ρ€ΠΈΠΈ ΠΈ ΠΎΡ‚Π·Ρ‹Π²Ρ‹

Для ΠΏΡ‚Ρ„ с Π°Π²Ρ‚ΠΎ ΠΈΠ΄ΡƒΡ‚ 3 кабСля Π½Π° Ρ„Π°Ρ€Π°Ρ…, Ρ‚ΠΎΠ»ΡŒΠΊΠΎ 2 кабСля

Π’Ρ‹Π±Ρ€Π°Π»ΠΎΡΡŒ Π²Ρ€Π΅ΠΌΠ΅Ρ‡ΠΊΠΎ ΡΠΎΠ΄Ρ€Π°Ρ‚ΡŒ Π±Π°ΠΌΠΏΠ΅Ρ€ ΠΈ Π½Π°ΠΊΠΎΠ½Π΅Ρ† Π½Π°Ρ‡Π°Ρ‚ΡŒ. Бпасибо кстати ΠΊΠΎΠΌΡ€Π°Π΄Ρƒ Fremdling Π·Π° ΠΏΠΎΠΌΠΎΡ‰Πͺ Π² съСмС Π±Π°ΠΌΠΏΠ΅Ρ€Π°.

Π‘Ρ…Π΅ΠΌΠ° основываСтся Π½Π° 4Ρ… ΠΊΠΎΠ½Ρ‚Π°ΠΊΡ‚Π½ΠΎΠΌ Ρ€Π΅Π»Π΅ ΠΈ Ρ€Π΅Π»Π΅ Π·Π°Π΄Π½ΠΈΡ… ΠΏΡ‚Ρ„ ΠΎΡ‚ 14ΠΉ (23.3777 ΠΈΠ»ΠΈ 21.3777), ΠΊΠ½ΠΎΠΏΠΊΠΈ Π±Π΅Π· фиксации ΠΎΡ‚ ΠΎΠ±ΠΎΠ³Ρ€Π΅Π²Π° Π»ΠΎΠ±ΠΎΠ²ΠΎΠ³ΠΎ.

Если Ρ…ΠΎΡ‚ΠΈΡ‚Π΅ Ρ‡Ρ‚ΠΎΠ±Ρ‹ ПВЀ ΠΌΠΎΠΆΠ½ΠΎ Π±Ρ‹Π»ΠΎ Π²ΠΊΠ»ΡŽΡ‡ΠΈΡ‚ΡŒ всСгда Π±Π΅Π· зависимости ΠΎΡ‚ Π³Π°Π±Π°Ρ€ΠΈΡ‚ΠΎΠ², Ρ‚ΠΎ Π½Π° схСмС Ρ€Π΅Π»Π΅ 23.3777 ΠΊΠΎΠ½Ρ‚Π°ΠΊΡ‚ 6 Π·Π°ΠΌΡ‹ΠΊΠ°Π΅Ρ‚Π΅ с 1.

Π’ ΠΌΠΎΠ΅ΠΌ Π²Π°Ρ€ΠΈΠ°Π½Ρ‚Π΅ установки Π΅ΡΡ‚ΡŒ Ρ€Π°Π·ΡŠΠ΅ΠΌ, Ρ€Π°Π·Π΄Π΅Π»ΡΡŽΡ‰ΠΈΠΉ ΠΏΡ€ΠΎΠ²ΠΎΠ΄ΠΊΡƒ ПВЀ ΠΈ ΠΏΡ€ΠΎΠ²ΠΎΠ΄ΠΊΡƒ ΠΏΠΎΠ΄ΠΊΠ°ΠΏΠΎΡ‚Π½ΠΎΠ³ΠΎ пространства. Π’Π°ΠΊ-ΠΆΠ΅ ΠΈΠ½Π΄ΠΈΠΊΠ°Ρ†ΠΈΡŽ с ΡƒΠΏΡ€Π°Π²Π»ΡΡŽΡ‰Π΅Π³ΠΎ плюса я ΡƒΠ²Π΅Π» Π½Π΅ Π½Π° ΠΏΡ€ΠΈΠ±ΠΎΡ€ΠΊΡƒ Π° Π½Π° ΠΈΠ½Π΄ΠΈΠΊΠ°Ρ†ΠΈΡŽ ΠΊΠ½ΠΎΠΏΠΊΠΈ, Π²ΠΏΡ€ΠΈΠ½Ρ†ΠΈΠΏΠ΅ ΠΌΠΎΠΆΠ½ΠΎ этот ΠΏΡ€ΠΎΠ²ΠΎΠ΄ΠΎΠΊ Ρ€Π°Π·Π΄Π²ΠΎΠΈΡ‚ΡŒ ΠΈ Π½Π° ΠΊΠ½ΠΎΠΏΠΊΡƒ ΠΈ Π½Π° ΠΏΡ€ΠΈΠ±ΠΎΡ€ΠΊΡƒ.

Π’ΠΈΠ΄Π΅ΠΎ Ρ€Π°Π±ΠΎΡ‚Ρ‹ ΠΏΡ€ΠΎΠ²ΠΎΠ΄ΠΊΠΈ:

Π Π΅Π»Π΅ располоТил Π½Π°Π΄ Ρ„Π°Ρ€ΠΎΠΉ Π² ΠΊΡ€Ρ‹Π»Π΅:

+ взят с ΠΠšΠ‘ (шпилька 6ΠΌΠΌ, ΠΊΠ»ΡŽΡ‡ 10):
НС Π·Π°Π±Ρ‹Π²Π°Π΅ΠΌ ΠΏΡ€ΠΎ ΠΏΡ€Π΅Π΄ΠΎΡ…Ρ€Π°Π½!

Масса взята рядом Π½Π° стойку, Π³Π°ΠΉΠΊΠ° крСплСния ΠΊΠΎΡ€Π·ΠΈΠ½Ρ‹:

2 ΠΏΡ€ΠΎΠ²ΠΎΠ΄Π° (сигнал+индикация) тянСм Π² салон Ρ‡Π΅Ρ€Π΅Π· Π³Π΅Ρ€ΠΌΡƒ Ρ€Π΅Π·ΠΈΠ½ΠΎΠ²ΡƒΡŽ.
Π― взял ΠΏΡ€ΠΎΠ²ΠΎΠ»ΠΎΠΊΡƒ, ΠΏΡ€ΠΈΠΌΠΎΡ‚Π°Π» ΠΈΠ·ΠΎΠ»Π΅Π½Ρ‚ΠΎΠΉ ΠΊ Π½Π΅ΠΉ ΠΏΡ€ΠΎΠ²ΠΎΠ΄Π°, ΠΏΡ€Π΅Π΄Π²Π°Ρ€ΠΈΡ‚Π΅Π»ΡŒΠ½ΠΎ ΠΎΡ‚ΠΌΠ΅Ρ€ΠΈΠ² ΠΈ ΡƒΠ±Ρ€Π°Π² Π² Π³ΠΎΡ„Ρ€Ρƒ, ΠΈ Ρ‚ΡƒΠΏΠΎ ΠΏΡ€ΠΎΡ‚ΠΊΠ½ΡƒΠ» эту Ρ€Π΅Π·ΠΈΠ½ΠΊΡƒ, Ρ‚ΠΎΡ‡Π½Π΅Π΅ ΠΏΠ΅Ρ€Π²ΡƒΡŽ Π΅Π΅ Ρ‡Π°ΡΡ‚ΡŒ, Π΄Π°Π»Π΅Π΅ ΠΈΠ· салона ΠΏΠΎΠ΄Π»Π΅Π·, просунул ΠΏΠ°Π»Π΅Ρ† Π²Π½ΡƒΡ‚Ρ€ΡŒ это Ρ€Π΅Π·ΠΈΠ½ΠΎΠ²ΠΎΠΉ Ρ…Ρ€Π΅Π½ΠΎΠ²ΠΈΠ½Ρ‹ ΠΈ Π²Ρ‹Ρ‚Π°Ρ‰ΠΈΠ» ΠΏΡ€ΠΎΠ²ΠΎΠ΄ΠΊΡƒ Π½Π°Ρ‰ΡŽΠΏΠ°Π² ΠΏΡ€ΠΎΠ²ΠΎΠ»ΠΎΠΊΡƒ. Π•Ρ‰Π΅ Ρ‚ΠΎΡ‚ изврат… =)

ΠŸΡ€ΠΈ ΠΏΠΎΠ΄ΠΊΠ»ΡŽΡ‡Π΅Π½ΠΈΠΈ ΠΊΠ½ΠΎΠΏΠΊΠΈ ΠΌΡ‹ Π½Π° Π½Π΅Π΅ ΠΏΡ€ΠΈΠ²ΠΎΠ΄ΠΈΠΌ ΠΈΠ· ΠΏΠΎΠ΄ΠΊΠ°ΠΏΠΎΡ‚Π½ΠΎΠ³ΠΎ пространства 2 ΠΏΡ€ΠΎΠ²ΠΎΠ΄Π°:
-Π‘ΠΈΠ³Π½Π°Π»ΡŒΠ½Ρ‹ΠΉ (ΠΊΠΎΡ‚ΠΎΡ€Ρ‹ΠΉ замыкаСтся Π½Π° массу ΠΈ Π²ΠΊΠ»ΡŽΡ‡Π°Π΅Ρ‚ Ρ€Π΅Π»Π΅)
-Π˜Π½Π΄ΠΈΠΊΠ°Ρ†ΠΈΡ (Π»Π°ΠΌΠΏΠΎΡ‡ΠΊΠ° ΡΠΈΠ³Π½Π°Π»ΠΈΠ·ΠΈΡ€ΡƒΡŽΡ‰Π°Ρ Π²ΠΊΠ»/Π²Ρ‹ΠΊΠ»)
(Π’Ρ‹Π²ΠΎΠ΄ ΠΈΠ½Π΄ΠΈΠΊΠ°Ρ†ΠΈΠΈ ПВЀ Π½Π° ΠΏΡ€ΠΈΠ±ΠΎΡ€Π½ΡƒΡŽ панСль Ρ‡ΠΈΡ‚Π°Ρ‚ΡŒ Ρ‚ΡƒΡ‚)

ΠŸΠΎΠ΄ΡΠ²Π΅Ρ‚ΠΊΡƒ ΠΈ массу ΠΌΡ‹ Π±Π΅Ρ€Π΅ΠΌ с сосСда β€” рСостата управлСния ΠΊΠΎΡ€Ρ€Π΅ΠΊΡ‚ΠΎΡ€ΠΎΠΌ Ρ„Π°Ρ€.
Распиновки ΠΊΠ½ΠΎΠΏΠΊΠΈ ΠΈ рСостата.

Выглядит это ΠΏΡ€ΠΈΠΌΠ΅Ρ€Π½ΠΎ Ρ‚Π°ΠΊ:

По ΡΠ½ΡΡ‚ΠΈΡŽ Π±Π°ΠΌΠΏΠ΅Ρ€Π°:

Π‘ΠΎΠ²Π΅Ρ‚(!)
Π’ ΠΏΡƒΠ½ΠΊΡ‚Π΅ 2 послС Ρ‚ΠΎΠ³ΠΎ ΠΊΠ°ΠΊ Π²Ρ‹ΠΊΡ€ΡƒΡ‚ΠΈΡ‚Π΅ ΡˆΡƒΡ€ΡƒΠΏ с Π³ΠΎΠ»ΠΎΠ²ΠΎΠΉ 10, ΠΎΡ‚ΠΎΠ³Π½ΠΈΡ‚Π΅ Π»ΠΎΠΊΠ΅Ρ€ ΠΈ ΡƒΠ²ΠΈΠ΄ΠΈΡ‚Π΅ Ρ‚Π°ΠΌ ΠΊΡ€ΠΎΠ½ΡˆΡ‚Π΅ΠΉΠ½ Π·Π°Ρ‰Π΅Π»ΠΎΠΊ Π±Π°ΠΌΠΏΠ΅Ρ€Π°. ЧСрная Ρ„ΠΈΠ³ΠΎΠ²ΠΈΠ½Π° с двумя Π΄Ρ‹Ρ€ΠΊΠ°ΠΌΠΈ. Π’ этих Π΄Ρ‹Ρ€ΠΊΠ°Ρ… Π΅ΠΆΠ΅ 2 ΡˆΡƒΡ€ΡƒΠΏΠ° с Π³ΠΎΠ»ΠΎΠ²ΠΎΠΉ 10. Π‘Π»ΠΈΠΆΠ½ΠΈΠΉ ΠΊ колСсу ΡˆΡƒΡ€ΡƒΠΏ ΠΈΠ· ΠΊΡ€ΠΎΠ½ΡˆΡ‚Π΅ΠΉΠ½Π° Π²Ρ‹ΠΊΡ€ΡƒΡ‚ΠΈΡ‚Π΅ Π½Π° ΠΏΠΎΠ»ΠΎΠ²ΠΈΠ½Ρƒ Ρ‚Π΅ΠΌ самым ослабив Π·Π°ΠΆΠΈΠΌ. Π­Ρ‚ΠΎ упростит снятиС Π±Π°ΠΌΠΏΠ΅Ρ€Π° с Π·Π°Ρ‰Π΅Π»ΠΎΠΊ. ПослС снятия затянитС Π΅Π³ΠΎ Π½Π° мСсто.

Π”Π°Π»Π΅Π΅ скручиваСм Π·Π°Π³Π»ΡƒΡˆΠΊΠΈ ΠΈ Π½Π° Ρ‚Π΅-ΠΆΠ΅ мСста ставим ПВЀ. ΠœΠΎΠ½Ρ‚ΠΈΡ€ΡƒΠ΅ΠΌ ΠΏΡ€ΠΎΠ²ΠΎΠ΄ΠΊΡƒ.

Распиновка ΠΊΠ½ΠΎΠΏΠΊΠΈ ΠΏΡ‚Ρ„ Π½Π° Π›Π°Π΄Π° ΠŸΡ€ΠΈΠΎΡ€Π°

Π’ΠΠ–ΠΠž! ΠŸΠ΅Ρ€Π΅Π΄ Ρ‚Π΅ΠΌ, ΠΊΠ°ΠΊ Π·Π°Π΄Π°Ρ‚ΡŒ вопрос ΠΎ ΠΌΠΎΠΌΠ΅Π½Ρ‚Π°Ρ… ΠΏΠΎΠ΄ΠΊΠ»ΡŽΡ‡Π΅Π½ΠΈΡ ΠΏΡ€ΠΎΡ‚ΠΈΠ²ΠΎΡ‚ΡƒΠΌΠ°Π½ΠΎΠΊ, Π²Π½ΠΈΠΌΠ°Ρ‚Π΅Π»ΡŒΠ½ΠΎ ΠΏΡ€ΠΎΡ‡ΠΈΡ‚Π°ΠΉΡ‚Π΅ ΠΊΠΎΠΌΠΌΠ΅Π½Ρ‚Π°Ρ€ΠΈΠΈ ΠΊ этой ΡΡ‚Π°Ρ‚ΡŒΠ΅ ΠΈ сам тСкст ΡΡ‚Π°Ρ‚ΡŒΠΈ, скорСС всСго ΠΈΠΌΠ΅Π½Π½ΠΎ ВАШ вопрос ΡƒΠΆΠ΅ рассматривался, ΠΏΡ€ΠΈΡ‡Ρ‘ΠΌ нСсколько Ρ€Π°Π·. Π’Π°ΠΊ ΠΆΠ΅ ΠΏΠΎΠΌΠ½ΠΈΡ‚Π΅, ΡΡ‚Π°Ρ‚ΡŒΡ Π±Ρ‹Π»Π° написана Π±ΠΎΠ»Π΅Π΅ 2-Ρ… Π»Π΅Ρ‚ Π½Π°Π·Π°Π΄, ΠΊΠΎΠ½ΠΊΡ€Π΅Ρ‚Π½ΠΎ ΠΊΠ°ΠΊ ΠΈ Ρ‡Ρ‚ΠΎ ΠΏΠΎΠ΄ΠΊΠ»ΡŽΡ‡Π°Π» я Π½Π΅ помню, поэтому вопросы этого ΠΏΠ»Π°Π½Π° Ρ‚Π°ΠΊ ΠΆΠ΅ Π±ΡƒΠ΄ΡƒΡ‚ оставлСны Π±Π΅Π· ΠΎΡ‚Π²Π΅Ρ‚Π° Ρ…ΠΎΡ‚ΡŒ Π² ΠΊΠΎΠΌΠΌΠ΅Π½Ρ‚Π°Ρ…, Ρ…ΠΎΡ‚ΡŒ Π² Π»ΠΈΡ‡ΠΊΠ΅. Бпасибо Π·Π° Π²Π½ΠΈΠΌΠ°Π½ΠΈΠ΅.

ИдСя установки ΠΏΡ€ΠΎΡ‚ΠΈΠ²ΠΎΡ‚ΡƒΠΌΠ°Π½ΠΎΠΊ Π±Ρ‹Π»Π° с самой ΠΏΠΎΠΊΡƒΠΏΠΊΠΈ Π°Π²Ρ‚ΠΎ, Π½ΠΎ Ρ‚Π°ΠΊ ΠΊΠ°ΠΊ Ρ€Π°Π½ΡŒΡˆΠ΅ Π½Π° ΠΏΡ€ΠΈΠΎΡ€Ρ‹ я ΠΈΡ… Π½Π΅ ставил, Π±ΠΎΠ»Π΅Π΅ Ρ‚ΠΎΠ³ΠΎ скаТу, Π²ΠΎΠΎΠ±Ρ‰Π΅ Π½ΠΈΠΊΡƒΠ΄Π° Π½Π΅ ставил Ρ‚ΡƒΠΌΠ°Π½ΠΊΠΈ, Π±Ρ‹Π»ΠΈ сомнСния. Π‘Π½Π°Ρ‡Π°Π»Π° Ρ…ΠΎΡ‚Π΅Π» ΠΎΡ‚ΠΎΠ³Π½Π°Ρ‚ΡŒ Π°Π²Ρ‚ΠΎ Π½Π° сСрвис, Π΄ΡƒΠΌΠ°ΡŽ, ΠΏΡƒΡΡ‚ΡŒ Ρ‚Π°ΠΌ поставят, ΡƒΠ·Π½Π°Π» Ρ†Π΅Π½Ρ‹ ΠΈ Ρ‚Π΄ ΠΈ Ρ‚ΠΏ, Ρ‚ΠΎΠ»ΡŒΠΊΠΎ Π·Π° Ρ€Π°Π±ΠΎΡ‚Ρƒ ΠΏΡ€ΠΈΡˆΠ»ΠΎΡΡŒ Π±Ρ‹ ΠΎΡ‚Π΄Π°Ρ‚ΡŒ ΠΎΠΊΠΎΠ»ΠΎ 2-Ρ… тысяч, Π΄ΡƒΠΌΠ°ΡŽ, Π½Π΅, Π΄ΠΎΡ€ΠΎΠ³ΠΎΠ²Π°Ρ‚ΠΎ. Но идСя мСня Π½Π΅ ΠΏΠΎΠΊΠΈΠ΄Π°Π»Π°.

Π‘Π½Π°Ρ‡Π°Π»Π° я ΠΏΡ€ΠΈΠΎΠ±Ρ€Ρ‘Π» ΠΊΠΎΠΌΠΏΠ»Π΅ΠΊΡ‚ ΠΏΡ€ΠΎΠ²ΠΎΠ΄ΠΎΠ² для установки Ρ‚ΡƒΠΌΠ°Π½ΠΎΠΊ для ΠΏΡ€ΠΈΠΎΡ€Ρ‹ с ΠΊΠ½ΠΎΠΏΠΊΠΎΠΉ, катался со ΠΌΠ½ΠΎΠΉ этот ΠΊΠΎΠΌΠΏΠ»Π΅ΠΊΡ‚ ΠΎΠΊΠΎΠ»ΠΎ Π½Π΅Π΄Π΅Π»ΠΈ, Π½ΠΎ свободная ΠΌΠΈΠ½ΡƒΡ‚ΠΊΠ° всё Ρ‚Π°ΠΊΠΈ ΠΏΠΎΠ΄Π²Π΅Ρ€Π½ΡƒΠ»Π°ΡΡŒ ΠΈ я Ρ€Π΅ΡˆΠΈΠ»ΡΡ Π½Π° установку ΠΊΠ½ΠΎΠΏΠΊΠΈ (для мСня самым слоТным Π±Ρ‹Π»ΠΎ ΠΈΠ½ΡΡ‚Π°Π»ΠΈΡ€ΠΎΠ²Π°Ρ‚ΡŒ ΠΊΠ½ΠΎΠΏΠΊΡƒ Π² салон ΠΈ вывСсти ΠΏΡ€ΠΎΠ²ΠΎΠ΄ Π½Π° Ρ€Π΅Π»Π΅ ΠΏΠΎΡ‚ΠΎΠΌΡƒ ΠΊΠ°ΠΊ ΠΌΠΎΠΆΠ½ΠΎ Π±Ρ‹Π»ΠΎ Π½Π΅ слабо Ρ‚Π°ΠΊ Ρ€Π°ΡΠΊΠΎΡ†Π°Ρ‚ΡŒ Ρ‚Π°ΠΊ Π½Π°Π·Ρ‹Π²Π°Π΅ΠΌΡƒΡŽ Π±ΠΎΡ€ΠΎΠ΄Ρƒ), кстати, ΠΎ Π±ΠΎΡ€ΠΎΠ΄Π΅, ΠΎΠ½Π° дСрТится 6-ю саморСзами, 2 Π²ΠΊΡ€ΡƒΡ‡ΠΈΠ²Π°ΡŽΡ‚ΡΡ Ρ‚ΡƒΠ΄Π°, Π³Π΄Π΅ Ρ€Π°Π·ΡŠΡ‘ΠΌ ΠΌΠ°Π³Π½ΠΈΡ‚ΠΎΠ»Ρ‹, 2 Ρ‚Π°ΠΌ Π³Π΄Π΅ отсСк с надписью Β«OPENΒ» ΠΈΠ»ΠΈ Π±Π΅Π· этой надписи ΠΊΠ°ΠΊ Ρƒ мСня ΠΈ Π΅Ρ‰Ρ‘ 2 спрятанны ΠΏΠΎΠ΄ ΠΏΠ΅ΠΏΠ΅Π»ΡŒΠ½ΠΈΡ†Π΅ΠΉ, ΠΊΠΎΡ‚ΠΎΡ€ΡƒΡŽ Π½Π°Π΄ΠΎ Π±ΡƒΠ΄Π΅Ρ‚ Π²Ρ‹Π½ΠΈΠΌΠ°Ρ‚ΡŒ, послС Ρ‚ΠΎΠ³ΠΎ ΠΊΠ°ΠΊ ΠΎΠ½ΠΈ ΠΎΡ‚ΠΊΡ€ΡƒΡ‡Π΅Π½Ρ‹ лицСвая панСлька Π»Π΅Π³ΠΊΠΎ снимаСтся ΠΈ Ρ‚ΡƒΠ΄Π° инсталируСтся ΠΊΠ½ΠΎΠΏΠΊΠ° ΠΏΡ€ΠΎΡ‚ΠΈΠ²ΠΎΡ‚ΡƒΠΌΠ°Π½ΠΎΠΊ согласно инструкции, которая ΠΈΠ΄Ρ‘Ρ‚ ΠΊ ΠΊΠΎΠΌΠΏΠ»Π΅ΠΊΡ‚Π΅(я Π½Π° всякий случай провСрял всС ΠΏΡ€ΠΎΠ²ΠΎΠ΄Π° ΠΌΡƒΠ»ΡŒΡ‚ΠΈΠΌΠ΅Ρ‚Ρ€ΠΎΠΌ, Ρ‡Ρ‚ΠΎΠ±Ρ‹ Π½Π΅ Π½Π°ΠΊΠΎΡΡΡ‡ΠΈΡ‚ΡŒ). ПослС ΠΏΠΎΠ΄ΠΊΠ»ΡŽΡ‡Π΅Π½ΠΈΡ ΠΊΠ½ΠΎΠΏΠΊΠΈ выводится 1 СдинствСнный ΠΏΡ€ΠΎΠ²ΠΎΠ΄ Π² ΠΏΠΎΠ΄ΠΊΠ°ΠΏΠΎΡ‚Π½ΠΎΠ΅ пространство Π½Π° Ρ€Π΅Π»Π΅, ΠΊΠΎΡ‚ΠΎΡ€ΠΎΠ΅ устанавливаСтся Ρ‡ΡƒΡ‚ΡŒ ΠΏΠΎΠ·ΠΆΠ΅.

Π”Π°Π»Π΅Π΅, для установки самих Ρ‚ΡƒΠΌΠ°Π½ΠΎΠΊ Π½Π΅ΠΎΠ±Ρ…ΠΎΠ΄ΠΈΠΌΠΎ ΡΠ½ΡΡ‚ΡŒ Π±Π°ΠΌΠΏΠ΅Ρ€. НС Π±ΠΎΠΉΡ‚Π΅ΡΡŒ, Ρ‚Π°ΠΌ Π½ΠΈΡ‡Π΅Π³ΠΎ слоТного. Π―ΠΌΠ° ΠΈΠ»ΠΈ эстакада Ρ‚ΠΎΠΆΠ΅ Π½Π΅ Π½ΡƒΠΆΠ½Ρ‹, я всё Π΄Π΅Π»Π°Π» Π²ΠΎ Π΄Π²ΠΎΡ€Π΅ Ρƒ сСбя. Π’ ΠΏΠ΅Ρ€Π²ΡƒΡŽ ΠΎΡ‡Π΅Ρ€Π΅Π΄ΡŒ ΠΎΡ‚ΠΊΡ€ΡƒΡ‡ΠΈΠ²Π°Π΅ΠΌ Π³Π°ΠΉΠΊΠΈ Π·Π°Ρ‰ΠΈΡ‚Ρ‹ спСрСди(ΠΈΡ… 5 ΡˆΡ‚ΡƒΠΊ). Π—Π°Ρ‚Π΅ΠΌ Ρ‚Π°ΠΌ ΠΆΠ΅, снизу ΠΎΡ‚ΠΊΡ€ΡƒΡ‡ΠΈΠ²Π°Π΅ΠΌ ΠΏΠΎ 3 саморСза с ΠΊΠ°ΠΆΠ΄ΠΎΠΉ стороны, ΠΊΠΎΡ‚ΠΎΡ€Ρ‹Π΅ ΠΏΡ€ΠΈΠΊΡ€Π΅ΠΏΠ»ΡΡŽΡ‚ Π±Π°ΠΌΠΏΠ΅Ρ€ ΠΊ пластиковым Π±Ρ€Ρ‹Π·Π³ΠΎΠ²ΠΈΠΊΠ°ΠΌ. ΠŸΠΎΡ‚ΠΎΠΌ Π΅Ρ‰Ρ‘ ΠΏΠΎ 2 саморСза со стороны ΠΏΠ΅Ρ€Π΅Π΄Π½ΠΈΡ… колёс. Π”Π°Π»Π΅Π΅ ΠΏΠΎΠ΄Π½ΠΈΠΌΠ°Π΅ΠΌ ΠΊΠ°ΠΏΠΎΡ‚ ΠΈ ΠΎΡ‚ΠΊΡ€ΡƒΡ‡ΠΈΠ²Π°Π΅ΠΌ Π±ΠΎΠ»Ρ‚Ρ‹ ΠΏΠΎΠ΄ ΠΊΡ€Π΅ΡΡ‚ΠΎΠΎΠ±Ρ€Π°Π·Π½ΡƒΡŽ ΠΎΡ‚Π²Ρ‘Ρ€Ρ‚ΠΊΡƒ, ΠΊΠΎΡ‚ΠΎΡ€Ρ‹Π΅ Π΄Π΅Ρ€ΠΆΠ°Ρ‚ Π±Π°ΠΌΠΏΠ΅Ρ€ свСрху (Π²ΠΎΠ½ ΠΎΠ½ΠΈ Π½Π° Ρ„ΠΎΡ‚ΠΊΠ΅ ΠΏΠΎ Π±ΠΎΠΊΠ°ΠΌ ΠΎΡ‚ ΠΆΡ‘Π»Ρ‚ΠΎΠΉ этикСтки с надписью Β«Π’ΠžΠ ΠΠ˜ΠΠ“Β») Π”Π°Π»ΡŒΠ½Π΅ΠΉΡˆΠΈΠΌ Π΄Π²ΠΈΠΆΠ΅Π½ΠΈΠ΅ΠΌ стараСмся Π°ΠΊΠΊΡƒΡ€Π°Ρ‚Π½ΠΎ ΠΎΡ‚ΠΎΠ³Π½ΡƒΡ‚ΡŒ края Π±Π°ΠΌΠΏΠ΅Ρ€Π° ΠΎΡ‚ ΠΊΡ€Ρ‹Π»Π°, сначала с ΠΎΠ΄Π½ΠΎΠΉ стороны, ΠΏΠΎΡ‚ΠΎΠΌ с Π΄Ρ€ΡƒΠ³ΠΎΠΉ ΠΈ вуаля, Π±Π°ΠΌΠΏΠ΅Ρ€ Ρƒ нас Π² Ρ€ΡƒΠΊΠ°Ρ…).

Π’ΡƒΠΌΠ°Π½ΠΊΠΈ я ΠΊΡƒΠΏΠΈΠ» Π·Π°Ρ€Π°Π½Π΅Π΅ Π½Π° Ρ€Ρ‹Π½ΠΊΠ΅, Π²Ρ€ΠΎΠ΄Π΅ ΠΊΠ°ΠΊ ΠΎΡ‚ Ρ„ΠΈΡ€ΠΌΡ‹ дэпо Π½ΠΎ ΠΏΠΎ ΠΌΠΎΠ΅ΠΌΡƒ ΡΡƒΠ±ΡŠΠ΅ΠΊΡ‚ΠΈΠ²Π½ΠΎΠΌΡƒ мнСнию, ΠšΠΈΡ‚Π°ΠΉ, Ρ‚ΠΎΠ»ΡŒΠΊΠΎ Ρ…ΠΎΡ€ΠΎΡˆΠ΅Π³ΠΎ качСства. Π§Ρ‚ΠΎΠ±Ρ‹ ΠΈΡ… ΠΈΡΡ‚Π°Π»ΠΈΡ€ΠΎΠ²Π°Ρ‚ΡŒ, Π½ΡƒΠΆΠ½ΠΎ Π²Ρ‹Ρ€Π΅Π·Π°Ρ‚ΡŒ Π·Π°Π³Π»ΡƒΡˆΠΊΠΈ Π½Π° Π±Π°ΠΌΠΏΠ΅Ρ€Π΅ ΠΏΠΎΠ΄ Ρ‚ΡƒΠΌΠ°Π½ΠΊΠΈ, Π»ΡƒΡ‡ΡˆΠ΅ это Π΄Π΅Π»Π°Ρ‚ΡŒ большим канцСлярским Π½ΠΎΠΆΠΎΠΌ Π°ΠΊΠΊΡƒΡ€Π°Ρ‚Π½ΠΎ ΠΈ нСспСша. Π”Π°Π»Π΅Π΅ ΠΏΡ€ΠΈΠΊΡ€ΡƒΡ‡ΠΈΠ²Π°Π΅Ρ‚ Ρ‚ΡƒΠΌΠ°Π½ΠΊΠΈ ΠΊΠ°ΠΆΠ΄ΡƒΡŽ Π½Π° 3 саморСза, Π³Π»Π°Π²Π½ΠΎΠ΅ Π²Π½ΠΈΠΌΠ°Ρ‚Π΅Π»ΡŒΠ½ΠΎ, Ρ‡Ρ‚ΠΎΠ±Ρ‹ Π½ΠΈΠ· с Π²Π΅Ρ€Ρ…ΠΎΠΌ Π½Π΅ ΠΏΠ΅Ρ€Π΅ΠΏΡƒΡ‚Π°Ρ‚ΡŒ. ΠŸΡ€ΠΎΠ²ΠΎΠ΄ΠΈΠΌ, ΠΏΠΎΠ΄ΠΊΠ»ΡŽΡ‡Π°Π΅ΠΌ ΠΏΡ€ΠΎΠ²ΠΎΠ΄Π°, ΠΎΠΏΡΡ‚ΡŒ ΠΆΠ΅ ΠΏΠΎ схСмС, ΠΏΠΎΠ΄ΠΊΠ»ΡŽΡ‡Π°ΠΌ массу, Ρ‚ΡƒΡ‚, кстати, Π²ΠΊΡ€ΡƒΡ‡ΠΈΠ²Π°Ρ‚ΡŒ саморСзы Π² ΠΊΡƒΠ·ΠΎΠ² ΠΊΠ°ΠΊ написано Π² инструкции, Π½Π΅ ΡΠΎΠ²Π΅Ρ‚ΡƒΡŽ, ΠΏΠΎΡ‚ΠΎΠΌΡƒ Ρ‡Ρ‚ΠΎ Π½Π° ΠΊΡƒΠ·ΠΎΠ²Π΅ ΠΈ Ρ‚Π°ΠΊ Π΅ΡΡ‚ΡŒ мноТСство отвСрстий ΠΈΠ»ΠΈ мСталличСских ΠΏΡ€Π΅Π΄ΠΌΠ΅Ρ‚ΠΎΠ², ΠΊΡƒΠ΄Π° ΠΌΠΎΠΆΠ½ΠΎ ΠΏΡ€ΠΈΠΊΡ€ΡƒΡ‚ΠΈΡ‚ΡŒ ΠΏΡ€ΠΎΠ²ΠΎΠ΄ массы. ПослС этих манипуляций, Π΄Π΅Ρ€ΠΆΠΈΠΌ Π±Π°ΠΌΠΏΠ΅Ρ€ Π½Π° вСсу, ΠΏΠΎΠ΄ΠΊΠ»ΡŽΡ‡Π°Π΅ΠΌ Ρ‚ΡƒΠΌΠ°Π½ΠΊΠΈ ΠΈ ставим Π΅Π³ΠΎ Π½Π° мСсто Π² ΠΎΠ±Ρ€Π°Ρ‚Π½ΠΎΠΉ ΠΏΠΎΡΠ»Π΅Π΄ΠΎΠ²Π°Ρ‚Π΅Π»ΡŒΠ½ΠΎΡΡ‚ΠΈ.

Π”Π°Π»ΡŒΡˆΠ΅ всю ΠΏΡ€ΠΎΠ²ΠΎΠ΄ΠΊΡƒ(Ρ€Π΅Π»Π΅ ΠΈ ΠΏΡ€ΠΎΠ²ΠΎΠ΄, ΠΊΠΎΡ‚ΠΎΡ€Ρ‹ΠΉ протянули ΠΎΡ‚ ΠΊΠ½ΠΎΠΏΠΊΠΈ) ΠΏΠΎΠ΄ΠΊΠ»ΡŽΡ‡Π°Π΅ΠΌ ΠΏΠΎ схСмС, плюс ΠΈ минус я взял ΠΎΡ‚ аккумулятора ΠΈ наслаТдаСмся Ρ€Π΅Π·ΡƒΠ»ΡŒΡ‚Π°Ρ‚ΠΎΠΌ.

Π’Π°ΠΊ ΠΆΠ΅ Ρ…ΠΎΡ‡Ρƒ ΠΎΡ‚Π΄Π΅Π»ΡŒΠ½ΠΎ Π½Π°ΠΏΠΈΡΠ°Ρ‚ΡŒ ΠΏΡ€ΠΎ ΠΏΠΎΠ΄ΠΊΠ»ΡŽΡ‡Π΅Π½ΠΈΠ΅ ΠΊΠ½ΠΎΠΏΠΊΠΈ ΠΏΡ‚Ρ„. Если Π²Ρ‹ ΠΏΠΎΠΊΡƒΠΏΠ°Π΅Ρ‚Π΅ Π³ΠΎΡ‚ΠΎΠ²Ρ‹ΠΉ Π±Π»ΠΎΠΊ для ΠΏΠΎΠ΄ΠΊΠ»ΡŽΡ‡Π΅Π½ΠΈΡ Β«ΠΏΡ‚Ρ„ ΠΏΡ€ΠΈΠΎΡ€Π°Β», Ρ‚ΠΎΡ‚ Ρ‚Π°ΠΌ Π΅ΡΡ‚ΡŒ ΠΈ ΠΊΠ½ΠΎΠΏΠΊΠ° ΠΈ Ρ€Π΅Π»Π΅ ΠΈ нСобходимая инструкция, выглядит ΠΎΠ½ Ρ‚Π°ΠΊ:

ΠšΠΎΠΌΠΏΠ»Π΅ΠΊΡ‚ ПВЀ

ΠŸΠ΅Ρ€Π²Ρ‹ΠΌ, Ρ‡Ρ‚ΠΎ Π΄Π΅Π»Π°Π» я, это установка ΠΊΠ½ΠΎΠΏΠΊΠΈ Π² салонС, ΠΊΠ½ΠΎΠΏΠΊΠ° обычная, квадратная, ставится рядом с ΠΊΠ½ΠΎΠΏΠΊΠΎΠΉ ΠΎΠ±ΠΎΠ³Ρ€Π΅Π²Π° Π·Π°Π΄Π½Π΅Π³ΠΎ стСкла, дСлаСтся это Π΅Ρ‰Ρ‘ ΠΈ для Ρ‚ΠΎΠ³ΠΎ, Ρ‡Ρ‚ΠΎΠ±Ρ‹ ΠΌΠΎΠΆΠ½ΠΎ Π±Ρ‹Π»ΠΎ Π·Π°ΠΏΠΈΡ‚Π°Ρ‚ΡŒΡΡ ΠΎΡ‚ ΠΊΠΎΠ½Ρ‚Π°ΠΊΡ‚ΠΎΠ²(ΠΏΡ€ΠΎΠ²ΠΎΠ΄ΠΎΠ²) ΠΊΠ½ΠΎΠΏΠΊΠΈ ΠΎΠ±ΠΎΠ³Ρ€Π΅Π²Π°.

Π‘Ρ…Π΅ΠΌΠ° ΠΏΠΎΠ΄ΠΊΠ»ΡŽΡ‡Π΅Π½ΠΈΡ

Π‘Π½ΠΈΠΌΠ°Π΅ΠΌ ΠΊΠ½ΠΎΠΏΠΊΡƒ ΠΎΠ±ΠΎΠ³Ρ€Π΅Π²Π°, Π°ΠΊΠΊΡƒΡ€Π°Ρ‚Π½ΠΎ Π΅Ρ‘ ΠΏΠΎΠ΄Ρ†Π΅ΠΏΠΈΠ² Ρ‚ΠΎΠ½ΠΊΠΎΠΉ ΠΎΡ‚Π²Ρ‘Ρ€Ρ‚ΠΊΠΎΠΉ, вытягиваСм Π΅Ρ‘ вмСстС с Ρ„ΠΈΡˆΠΊΠΎΠΉ ΠΊΠΎΠ½Ρ‚Π°ΠΊΡ‚ΠΎΠ² ΠΈ Π²Ρ‹Π½ΠΈΠΌΠ°Π΅ΠΌ Π΅Ρ‘ ΠΈΠ· Ρ„ΠΈΡˆΠΊΠΈ.

ΠΊΠ½ΠΎΠΏΠΊΠ° с Ρ„ΠΈΡˆΠΊΠΎΠΉ

Π²Ρ‹Π½ΠΈΠΌΠ°Π΅ΠΌ ΠΊΠ½ΠΎΠΏΠΊΡƒ ΠΎΠ±ΠΎΠ³Ρ€Π΅Π²Π°

Π”Π°Π»Π΅Π΅ Π½Π°Ρ‡ΠΈΠ½Π°Π΅ΠΌ ΠΏΠΎΠ΄ΠΊΠ»ΡŽΡ‡Π°Ρ‚ΡŒ ΠΏΠΎ схСмС Π½Π°ΡˆΡƒ ΠΊΠ½ΠΎΠΏΠΊΡƒ ΠΏΡ‚Ρ„ ΠΎΡ‚ ΠΊΠ½ΠΎΠΏΠΊΠΈ ΠΎΠ±ΠΎΠ³Ρ€Π΅Π²Π°, для этого Π½ΡƒΠΆΠ΅Π½ ΠΌΡƒΠ»ΡŒΡ‚ΠΈΠΌΠ΅Ρ‚Ρ€ ΠΈΠ»ΠΈ хотя Π±Ρ‹ Π»Π°ΠΌΠΏΠΎΡ‡ΠΊΠ° 12Π² с 2-мя ΠΏΡ€ΠΎΠ²ΠΎΠ΄Π°ΠΌΠΈ.

1. Находим ΠΏΡ€ΠΎΠ²ΠΎΠ΄, ΠΊΠΎΡ‚ΠΎΡ€Ρ‹ΠΉ ΠΏΠΎΠ΄Ρ…ΠΎΠ΄ΠΈΡ‚ для подсвСтки ΠΊΠ½ΠΎΠΏΠΊΠΈ ΠΎΠ±ΠΎΠ³Ρ€Π΅Π²Π° стСкла, Ρ‚.Π΅. Π½Π° Π½Π΅Π³ΠΎ подаётся напряТСниС Π² 12Π² ΠΏΡ€ΠΈ Π²ΠΊΠ»ΡŽΡ‡Π΅Π½ΠΈΠΈ Π³Π°Π±Π°Ρ€ΠΈΡ‚ΠΎΠ², минусовой ΠΏΡ€ΠΎΠ²ΠΎΠ΄ ΠΌΡƒΠ»ΡŒΡ‚ΠΈΠΌΠ΅Ρ‚Ρ€Π° прислоняСм ΠΊ массС Π°Π²Ρ‚ΠΎ(ΠΊ Π³ΠΎΠ»ΠΎΠΌΡƒ ΠΌΠ΅Ρ‚Π°Π»Π»Ρƒ ΠΈΠ»ΠΈ ΠΊ Ρ‡Ρ‘Ρ€Π½ΠΎΠΌΡƒ ΠΏΡ€ΠΎΠ²ΠΎΠ΄Ρƒ Ρ„ΠΈΡˆΠΊΠΈ), Π²ΠΊΠ»ΡŽΡ‡Π°Π΅ΠΌ Π³Π°Π±Π°Ρ€ΠΈΡ‚Ρ‹(Π’ΠΠ–ΠΠž! Π—Π°ΠΆΠΈΠ³Π°Π½ΠΈΠ΅ Π΄ΠΎΠ»ΠΆΠ½ΠΎ Π±Ρ‹Ρ‚ΡŒ Π²Π«ΠΊΠ»ΡŽΡ‡Π΅Π½ΠΎ!) И Π½Π°Ρ‡ΠΈΠ½Π°Π΅ΠΌ ΠΈΡΠΊΠ°Ρ‚ΡŒ Π½Π° Ρ„ΠΈΡˆΠΊΠ΅ ΠΊΠ»Π΅ΠΌΠΌΡƒ(ΠΏΡ€ΠΎΠ²ΠΎΠ΄) Π³Π΄Π΅ появилось Π½Π°ΠΏΡ€ΡƒΠ³Π° 12Π², послС Ρ‚ΠΎΠ³ΠΎ, ΠΊΠ°ΠΊ нашли, Π²Ρ‹ΠΊΠ»ΡŽΡ‡ΠΈΡ‚Π΅ Π³Π°Π±Π°Ρ€ΠΈΡ‚Ρ‹, Ссли Π½Π°ΠΏΡ€ΡƒΠ³Π° ΠΏΡ€ΠΎΠΏΠ°Π»Π°, это Π½ΡƒΠΆΠ½Ρ‹ΠΉ Π½Π°ΠΌ ΠΏΡ€ΠΎΠ²ΠΎΠ΄, снимаСм с Π½Π΅Π³ΠΎ Π°ΠΊΠΊΡƒΡ€Π°Ρ‚Π½ΠΎ ΠΈΠ·ΠΎΠ»ΡΡ†ΠΈΡŽ, Ρ‡Ρ‚ΠΎΠ±Ρ‹ Π½Π΅ ΠΏΠΎΠ²Ρ€Π΅Π΄ΠΈΡ‚ΡŒ Π΅Π³ΠΎ ΠΈ ΠΏΠΎΠ΄ΠΊΡ€ΡƒΡ‡ΠΈΠ²Π°Π΅ΠΌ Ρ‚ΡƒΠ΄Π° ΠΏΡ€ΠΎΠ²ΠΎΠ΄ нашСй ΠΊΠ½ΠΎΠΏΠΊΠΈ ΠΏΡ‚Ρ„, ΠΊΠΎΡ‚ΠΎΡ€Ρ‹ΠΉ ΠΎΡ‚Π²Π΅Ρ‡Π°Π΅Ρ‚ Π·Π° подсвСтку(ΠΏΠΎ схСмС, ΠΏΡ€ΠΈΠ²Π΅Π΄Ρ‘Π½Π½ΠΎΠΉ Π²Ρ‹ΡˆΠ΅) ΠΎΠ½ числится, ΠΊΠ°ΠΊ Β«Π±Π΅Π»Ρ‹ΠΉ(ΠΆΡ‘Π»Ρ‚Ρ‹ΠΉ)Β».
2. По Ρ‚Π°ΠΊΠΎΠΉ ΠΆΠ΅ систСмС Π½Π°Ρ…ΠΎΠ΄ΠΈΠΌ ΠΏΡ€ΠΎΠ²ΠΎΠ΄, Π½Π° ΠΊΠΎΡ‚ΠΎΡ€ΠΎΠΌ появляСтся 12Π² ΠΏΡ€ΠΈ Π²ΠΊΠ»ΡŽΡ‡Π΅Π½ΠΈΠΈ заТигания, Ρ‚.Π΅. Π²Ρ‹ΠΊΠ»ΡŽΡ‡Π°Π΅ΠΌ Π³Π°Π±Π°Ρ€ΠΈΡ‚Ρ‹, Π²ΠΊΠ»ΡŽΡ‡Π°Π΅ΠΌ заТигания, Π½Π°Ρ…ΠΎΠ΄ΠΈΠΌ Π½ΡƒΠΆΠ½Ρ‹ΠΉ ΠΏΡ€ΠΎΠ²ΠΎΠ΄, провСряСм, ΠΎΠ½ ΠΈΠ»ΠΈ Π½Π΅Ρ‚ ΠΈ ΠΏΠΎΠ΄ΠΊΡ€ΡƒΡ‡ΠΈΠ²Π°Π΅ΠΌ ΠΊ Π½Π΅ΠΌΡƒ «красный(Ρ€ΠΎΠ·ΠΎΠ²Ρ‹ΠΉ)Β» ΠΏΡ€ΠΎΠ²ΠΎΠ΄ ΠΎΡ‚ ΠΊΠ½ΠΎΠΏΠΊΠΈ ΠΏΡ‚Ρ„ ΠΏΠΎ нашСй схСмС.
3. Π”Π°Π»ΡŒΡˆΠ΅ «масса», Ρ‚ΡƒΡ‚ всё Π»Π΅Π³ΠΊΠΎ, ΠΎΠ½ ΠΈ Ρ‚Π°ΠΌ ΠΈ Ρ‚Π°ΠΌ Β«Ρ‡Ρ‘Ρ€Π½Ρ‹ΠΉΒ».
4. ΠžΡΡ‚Π°Ρ‘Ρ‚ΡΡ ΠΎΠ΄ΠΈΠ½ ΠΏΡ€ΠΎΠ²ΠΎΠ΄ «синий», ΠΎΠ½ самый Π΄Π»ΠΈΠ½Π½Ρ‹ΠΉ, Π΅Π³ΠΎ Π²Π΅Π΄Ρ‘ΠΌ ΠΏΠΎΠ΄ ΠΊΠ°ΠΏΠΎΡ‚. Π›ΠΈΡ‡Π½ΠΎ я ΠΏΡ€ΠΎΠ²ΠΎΠ΄ΠΈΠ» Π΅Π³ΠΎ Ρ‡Π΅Ρ€Π΅Π· отвСрстия ΠΏΠΎΠ΄ ΠΏΠ΅Π΄Π°Π»ΡŒΠ½Ρ‹ΠΌ ΡƒΠ·Π»ΠΎΠΌ, Ρ‚Π°ΠΌ ΠΈΡ… ΠΌΠ½ΠΎΠ³ΠΎ. Π§Ρ‚ΠΎΠ±Ρ‹ ΠΈΡ… ΡƒΠ²ΠΈΠ΄Π΅Ρ‚ΡŒ ΠΈΠ· салона, Π½ΡƒΠΆΠ½ΠΎ Π½Π΅ΠΌΠ½ΠΎΠ³ΠΎ ΠΎΡ‚ΠΎΠ³Π½ΡƒΡ‚ΡŒ ΠΊΠΎΠ²Ρ€ΠΎΠ»ΠΈΠ½ ΠΏΠΎΠ΄ пСдалями.
5. УстанавливаСм Ρ€Π΅Π»Π΅ ΠΈΠ· ΠΊΠΎΠΌΠΏΠ»Π΅ΠΊΡ‚Π° ΠΏΠΎΠ΄ ΠΊΠ°ΠΏΠΎΡ‚ΠΎΠΌ.

Одно ΠΈΠ· мСст установки Ρ€Π΅Π»Π΅ ПВЀ

Π Π΅Π»Π΅ ΠΈΠΌΠ΅Π΅Ρ‚ своё ΡƒΡˆΠΊΠΎ, поэтому Π΅Π³ΠΎ ΠΊΡ€Π΅ΠΏΡ‘ΠΆ Π½Π΅ прСдставляСт ΠΈΠ· сСбя Ρ‡Ρ‚ΠΎ Ρ‚ΠΎ Ρ‚Ρ€ΡƒΠ΄Π½ΠΎΠ΅, просто ΠΎΡ‚Π²Π΅Ρ€Π½ΠΈΡ‚Π΅ ΡƒΠ΄ΠΎΠ±Π½ΡƒΡŽ Π²Π°ΠΌ Π³Π°ΠΉΠΊΡƒ крСплСния(Π½Π°ΠΏΡ€ΠΈΠΌΠ΅Ρ€, Π±Π»ΠΎΠΊΠ° абс, ΠΏΡ€ΠΎΠ΄Π΅Π½ΡŒΡ‚Π΅ это ΡƒΡˆΠΊΠΎ ΠΈ ΠΎΠΏΡΡ‚ΡŒ Π·Π°ΠΊΡ€ΡƒΡ‚ΠΈΡ‚Π΅).

Π”Π°Π»ΡŒΡˆΠ΅, ΠΏΠΎ ΠΏΠΎΠ΄ΠΊΠ»ΡŽΡ‡Π΅Π½ΠΈΡŽ, Ρ€Π΅Π»Π΅ ΠΈΠΌΠ΅Π΅Ρ‚ 4 ΠΊΠΎΠ½Ρ‚Π°ΠΊΡ‚Π°, ΠΎΠ½ΠΈ ΠΎΠ±ΠΎΠ·Π½Π°Ρ‡Π°ΡŽΡ‚ΡΡ Β«30, 87, 85, 86Β», Ссли Ρ€Π΅Π»Π΅ ΠΏΠ΅Ρ€Π΅Π²Π΅Ρ€Π½ΡƒΡ‚ΡŒ ΠΊΠΎΠ½Ρ‚Π°ΠΊΡ‚Π°ΠΌΠΈ Π²Π²Π΅Ρ€Ρ…, Ρ‚ΠΎ всС эти ΠΊΠΎΠ½Ρ‚Π°ΠΊΡ‚Ρ‹ Π±ΡƒΠ΄ΡƒΡ‚ подписаны ΡΠΎΠΎΡ‚Π²Π΅Ρ‚ΡΡ‚Π²ΡƒΡŽΡ‰ΠΈΠΌΠΈ Ρ†ΠΈΡ„Ρ€Π°ΠΌΠΈ.

Ρ€Π΅Π»Π΅

Π”Π°Π»Π΅Π΅, ΠΏΠΎΠ΄ΠΊΠ»ΡŽΡ‡Π°Π΅Ρ‚ ΠΊ Π½Π΅ΠΌΡƒ ΠΏΡ€ΠΎΠ²ΠΎΠ΄Π°, наш синий ΠΏΡ€ΠΎΠ²ΠΎΠ΄ ΠΎΡ‚ ΠΊΠ½ΠΎΠΏΠΊΠΈ ΠΏΠΎΠ΄ΠΊΠ»ΡŽΡ‡Π°Π΅ΠΌ ΠΊ 85-ΠΌΡƒ ΠΊΠΎΠ½Ρ‚Π°ΠΊΡ‚Ρƒ.
86-ΠΉ ΠΊΠΎΠ½Ρ‚Π°ΠΊΡ‚ это масса(я ΠΏΠΎΠ΄ΠΊΠ»ΡŽΡ‡Π°Π» ΠΎΡ‚ минусовой схСмы аккумулятора)
30-ΠΉ ΠΊΠΎΠ½Ρ‚Π°ΠΊΡ‚ ΠΊ плюсовой ΠΊΠ»Π΅ΠΌΠΌΠ΅ аккумулятора
87-ΠΉ, соотвСтствСнно ΠΊ Β«ΠΏΠ»ΡŽΡΡƒΒ» ΠΏΡ€ΠΎΡ‚ΠΈΠ²ΠΎΡ‚ΡƒΠΌΠ°Π½ΠΎΠΊ.

Relays — Как Ρ€Π°Π±ΠΎΡ‚Π°ΡŽΡ‚ Ρ€Π΅Π»Π΅

Как Ρ€Π°Π±ΠΎΡ‚Π°ΡŽΡ‚ Ρ€Π΅Π»Π΅

Магазин Ρ€Π΅Π»Π΅

Π Π΅Π»Π΅ β€” это ΠΏΠ΅Ρ€Π΅ΠΊΠ»ΡŽΡ‡Π°Ρ‚Π΅Π»ΠΈ, ΠΊΠΎΡ‚ΠΎΡ€Ρ‹Π΅ Ρ€Π°Π·ΠΌΡ‹ΠΊΠ°ΡŽΡ‚ ΠΈ Π·Π°ΠΌΡ‹ΠΊΠ°ΡŽΡ‚ Ρ†Π΅ΠΏΠΈ элСктромСханичСским ΠΈΠ»ΠΈ элСктронным способом. Π Π΅Π»Π΅ ΡƒΠΏΡ€Π°Π²Π»ΡΡŽΡ‚ ΠΎΠ΄Π½ΠΎΠΉ элСктричСской Ρ†Π΅ΠΏΡŒΡŽ, размыкая ΠΈ замыкая ΠΊΠΎΠ½Ρ‚Π°ΠΊΡ‚Ρ‹ Π² Π΄Ρ€ΡƒΠ³ΠΎΠΉ Ρ†Π΅ΠΏΠΈ. Как ΠΏΠΎΠΊΠ°Π·Ρ‹Π²Π°ΡŽΡ‚ схСмы Ρ€Π΅Π»Π΅, ΠΊΠΎΠ³Π΄Π° ΠΊΠΎΠ½Ρ‚Π°ΠΊΡ‚ Ρ€Π΅Π»Π΅ Π½ΠΎΡ€ΠΌΠ°Π»ΡŒΠ½ΠΎ Ρ€Π°Π·ΠΎΠΌΠΊΠ½ΡƒΡ‚ (НО), ΠΊΠΎΠ½Ρ‚Π°ΠΊΡ‚ остаСтся Ρ€Π°Π·ΠΎΠΌΠΊΠ½ΡƒΡ‚Ρ‹ΠΌ, ΠΊΠΎΠ³Π΄Π° Ρ€Π΅Π»Π΅ Π½Π΅ находится ΠΏΠΎΠ΄ напряТСниСм. Когда ΠΊΠΎΠ½Ρ‚Π°ΠΊΡ‚ Ρ€Π΅Π»Π΅ являСтся Π½ΠΎΡ€ΠΌΠ°Π»ΡŒΠ½ΠΎ Π·Π°ΠΌΠΊΠ½ΡƒΡ‚Ρ‹ΠΌ (НЗ), это ΠΎΠ·Π½Π°Ρ‡Π°Π΅Ρ‚, Ρ‡Ρ‚ΠΎ ΠΊΠΎΠ½Ρ‚Π°ΠΊΡ‚ Π·Π°ΠΌΠΊΠ½ΡƒΡ‚, ΠΊΠΎΠ³Π΄Π° Ρ€Π΅Π»Π΅ Π½Π΅ находится ΠΏΠΎΠ΄ напряТСниСм. Π’ любом случаС ΠΏΠΎΠ΄Π°Ρ‡Π° элСктричСского Ρ‚ΠΎΠΊΠ° Π½Π° ΠΊΠΎΠ½Ρ‚Π°ΠΊΡ‚Ρ‹ ΠΈΠ·ΠΌΠ΅Π½ΠΈΡ‚ ΠΈΡ… состояниС.

Π Π΅Π»Π΅ ΠΎΠ±Ρ‹Ρ‡Π½ΠΎ ΠΈΡΠΏΠΎΠ»ΡŒΠ·ΡƒΡŽΡ‚ΡΡ для ΠΏΠ΅Ρ€Π΅ΠΊΠ»ΡŽΡ‡Π΅Π½ΠΈΡ ΠΌΠ΅Π½ΡŒΡˆΠΈΡ… Ρ‚ΠΎΠΊΠΎΠ² Π² Ρ†Π΅ΠΏΠΈ управлСния ΠΈ ΠΎΠ±Ρ‹Ρ‡Π½ΠΎ Π½Π΅ ΡƒΠΏΡ€Π°Π²Π»ΡΡŽΡ‚ устройствами, ΠΏΠΎΡ‚Ρ€Π΅Π±Π»ΡΡŽΡ‰ΠΈΠΌΠΈ ΡΠ½Π΅Ρ€Π³ΠΈΡŽ, Π·Π° ΠΈΡΠΊΠ»ΡŽΡ‡Π΅Π½ΠΈΠ΅ΠΌ Π½Π΅Π±ΠΎΠ»ΡŒΡˆΠΈΡ… Π΄Π²ΠΈΠ³Π°Ρ‚Π΅Π»Π΅ΠΉ ΠΈ солСноидов, ΠΏΠΎΡ‚Ρ€Π΅Π±Π»ΡΡŽΡ‰ΠΈΡ… ΠΌΠ°Π»Ρ‹Π΅ Ρ‚ΠΎΠΊΠΈ. Π’Π΅ΠΌ Π½Π΅ ΠΌΠ΅Π½Π΅Π΅, Ρ€Π΅Π»Π΅ ΠΌΠΎΠ³ΡƒΡ‚ Β«ΡƒΠΏΡ€Π°Π²Π»ΡΡ‚ΡŒΒ» большими напряТСниями ΠΈ Ρ‚ΠΎΠΊΠ°ΠΌΠΈ, оказывая ΡƒΡΠΈΠ»ΠΈΠ²Π°ΡŽΡ‰ΠΈΠΉ эффСкт, ΠΏΠΎΡ‚ΠΎΠΌΡƒ Ρ‡Ρ‚ΠΎ нСбольшоС напряТСниС, ΠΏΡ€ΠΈΠ»ΠΎΠΆΠ΅Π½Π½ΠΎΠ΅ ΠΊ ΠΊΠ°Ρ‚ΡƒΡˆΠΊΠ΅ Ρ€Π΅Π»Π΅, ΠΌΠΎΠΆΠ΅Ρ‚ привСсти ΠΊ ΠΏΠ΅Ρ€Π΅ΠΊΠ»ΡŽΡ‡Π΅Π½ΠΈΡŽ ΠΊΠΎΠ½Ρ‚Π°ΠΊΡ‚ΠΎΠ² большим напряТСниСм.

Π—Π°Ρ‰ΠΈΡ‚Π½Ρ‹Π΅ Ρ€Π΅Π»Π΅ ΠΌΠΎΠ³ΡƒΡ‚ ΠΏΡ€Π΅Π΄ΠΎΡ‚Π²Ρ€Π°Ρ‚ΠΈΡ‚ΡŒ ΠΏΠΎΠ²Ρ€Π΅ΠΆΠ΄Π΅Π½ΠΈΠ΅ оборудования ΠΏΡƒΡ‚Π΅ΠΌ обнаруТСния элСктричСских ΠΎΡ‚ΠΊΠ»ΠΎΠ½Π΅Π½ΠΈΠΉ, Π² Ρ‚ΠΎΠΌ числС ΠΏΠ΅Ρ€Π΅Π³Ρ€ΡƒΠ·ΠΊΠΈ ΠΏΠΎ Ρ‚ΠΎΠΊΡƒ, минимального Ρ‚ΠΎΠΊΠ°, ΠΏΠ΅Ρ€Π΅Π³Ρ€ΡƒΠ·ΠΎΠΊ ΠΈ ΠΎΠ±Ρ€Π°Ρ‚Π½Ρ‹Ρ… Ρ‚ΠΎΠΊΠΎΠ². ΠšΡ€ΠΎΠΌΠ΅ Ρ‚ΠΎΠ³ΠΎ, Ρ€Π΅Π»Π΅ Ρ‚Π°ΠΊΠΆΠ΅ ΡˆΠΈΡ€ΠΎΠΊΠΎ ΠΈΡΠΏΠΎΠ»ΡŒΠ·ΡƒΡŽΡ‚ΡΡ для ΠΏΠ΅Ρ€Π΅ΠΊΠ»ΡŽΡ‡Π΅Π½ΠΈΡ пусковых ΠΊΠ°Ρ‚ΡƒΡˆΠ΅ΠΊ, Π½Π°Π³Ρ€Π΅Π²Π°Ρ‚Π΅Π»ΡŒΠ½Ρ‹Ρ… элСмСнтов, ΠΊΠΎΠ½Ρ‚Ρ€ΠΎΠ»ΡŒΠ½Ρ‹Ρ… Π»Π°ΠΌΠΏ ΠΈ Π·Π²ΡƒΠΊΠΎΠ²ΠΎΠΉ сигнализации.

ЭлСктромСханичСскиС Ρ€Π΅Π»Π΅ ΠΏΡ€ΠΎΡ‚ΠΈΠ² Ρ‚Π²Π΅Ρ€Π΄ΠΎΡ‚Π΅Π»ΡŒΠ½Ρ‹Ρ… Ρ€Π΅Π»Π΅

Π Π΅Π»Π΅ Π±Ρ‹Π²Π°ΡŽΡ‚ Π»ΠΈΠ±ΠΎ элСктромСханичСскими, Π»ΠΈΠ±ΠΎ ΠΏΠΎΠ»ΡƒΠΏΡ€ΠΎΠ²ΠΎΠ΄Π½ΠΈΠΊΠΎΠ²Ρ‹ΠΌΠΈ. Π’ элСктромСханичСских Ρ€Π΅Π»Π΅ (ЭМР) ΠΊΠΎΠ½Ρ‚Π°ΠΊΡ‚Ρ‹ Ρ€Π°Π·ΠΌΡ‹ΠΊΠ°ΡŽΡ‚ΡΡ ΠΈΠ»ΠΈ Π·Π°ΠΌΡ‹ΠΊΠ°ΡŽΡ‚ΡΡ ΠΏΠΎΠ΄ дСйствиСм ΠΌΠ°Π³Π½ΠΈΡ‚Π½ΠΎΠΉ силы. Π’ Ρ‚Π²Π΅Ρ€Π΄ΠΎΡ‚Π΅Π»ΡŒΠ½Ρ‹Ρ… Ρ€Π΅Π»Π΅ (SSR) ΠΊΠΎΠ½Ρ‚Π°ΠΊΡ‚Ρ‹ ΠΎΡ‚ΡΡƒΡ‚ΡΡ‚Π²ΡƒΡŽΡ‚, Π° ΠΏΠ΅Ρ€Π΅ΠΊΠ»ΡŽΡ‡Π΅Π½ΠΈΠ΅ ΠΏΠΎΠ»Π½ΠΎΡΡ‚ΡŒΡŽ элСктронноС. РСшСниС ΠΎΠ± использовании элСктромСханичСских ΠΈΠ»ΠΈ Ρ‚Π²Π΅Ρ€Π΄ΠΎΡ‚Π΅Π»ΡŒΠ½Ρ‹Ρ… Ρ€Π΅Π»Π΅ зависит ΠΎΡ‚ элСктричСских Ρ‚Ρ€Π΅Π±ΠΎΠ²Π°Π½ΠΈΠΉ прилоТСния, ΠΎΠ³Ρ€Π°Π½ΠΈΡ‡Π΅Π½ΠΈΠΉ ΠΏΠΎ стоимости ΠΈ ΠΎΠΆΠΈΠ΄Π°Π΅ΠΌΠΎΠ³ΠΎ срока слуТбы. Π₯отя Ρ‚Π²Π΅Ρ€Π΄ΠΎΡ‚Π΅Π»ΡŒΠ½Ρ‹Π΅ Ρ€Π΅Π»Π΅ стали ΠΎΡ‡Π΅Π½ΡŒ популярными, элСктромСханичСскиС Ρ€Π΅Π»Π΅ ΠΎΡΡ‚Π°ΡŽΡ‚ΡΡ распространСнными. МногиС Ρ„ΡƒΠ½ΠΊΡ†ΠΈΠΈ, выполняСмыС тяТСлым ΠΎΠ±ΠΎΡ€ΡƒΠ΄ΠΎΠ²Π°Π½ΠΈΠ΅ΠΌ, Ρ‚Ρ€Π΅Π±ΡƒΡŽΡ‚ ΠΊΠΎΠΌΠΌΡƒΡ‚Π°Ρ†ΠΈΠΈ. возмоТности элСктромСханичСских Ρ€Π΅Π»Π΅. Π’Π²Π΅Ρ€Π΄ΠΎΡ‚Π΅Π»ΡŒΠ½Ρ‹Π΅ Ρ€Π΅Π»Π΅ ΠΏΠ΅Ρ€Π΅ΠΊΠ»ΡŽΡ‡Π°ΡŽΡ‚ Ρ‚ΠΎΠΊ с ΠΏΠΎΠΌΠΎΡ‰ΡŒΡŽ Π½Π΅ΠΏΠΎΠ΄Π²ΠΈΠΆΠ½Ρ‹Ρ… элСктронных устройств, Ρ‚Π°ΠΊΠΈΡ… ΠΊΠ°ΠΊ выпрямитСли с ΠΊΡ€Π΅ΠΌΠ½ΠΈΠ΅Π²Ρ‹ΠΌ ΡƒΠΏΡ€Π°Π²Π»Π΅Π½ΠΈΠ΅ΠΌ.

Π­Ρ‚ΠΈ различия Π² Π΄Π²ΡƒΡ… Ρ‚ΠΈΠΏΠ°Ρ… Ρ€Π΅Π»Π΅ приводят ΠΊ прСимущСствам ΠΈ нСдостаткам ΠΊΠ°ΠΆΠ΄ΠΎΠΉ систСмы. ΠŸΠΎΡΠΊΠΎΠ»ΡŒΠΊΡƒ Ρ‚Π²Π΅Ρ€Π΄ΠΎΡ‚Π΅Π»ΡŒΠ½Ρ‹Π΅ Ρ€Π΅Π»Π΅ Π½Π΅ Π΄ΠΎΠ»ΠΆΠ½Ρ‹ ΠΏΠΎΠ΄Π°Π²Π°Ρ‚ΡŒ ΠΏΠΈΡ‚Π°Π½ΠΈΠ΅ Π½Π° ΠΊΠ°Ρ‚ΡƒΡˆΠΊΡƒ ΠΈΠ»ΠΈ Ρ€Π°Π·ΠΌΡ‹ΠΊΠ°Ρ‚ΡŒ ΠΊΠΎΠ½Ρ‚Π°ΠΊΡ‚Ρ‹, для Β«Π²ΠΊΠ»ΡŽΡ‡Π΅Π½ΠΈΡΒ» Ρ‚Π²Π΅Ρ€Π΄ΠΎΡ‚Π΅Π»ΡŒΠ½Ρ‹Ρ… Ρ€Π΅Π»Π΅ трСбуСтся мСньшСС напряТСниС. Π’ΠΎΡ‡Π½ΠΎ Ρ‚Π°ΠΊ ΠΆΠ΅ Ρ‚Π²Π΅Ρ€Π΄ΠΎΡ‚Π΅Π»ΡŒΠ½Ρ‹Π΅ Ρ€Π΅Π»Π΅ Π²ΠΊΠ»ΡŽΡ‡Π°ΡŽΡ‚ΡΡ ΠΈ Π²Ρ‹ΠΊΠ»ΡŽΡ‡Π°ΡŽΡ‚ΡΡ быстрСС, ΠΏΠΎΡ‚ΠΎΠΌΡƒ Ρ‡Ρ‚ΠΎ Π½Π΅Ρ‚ физичСских частСй, ΠΊΠΎΡ‚ΠΎΡ€Ρ‹Π΅ Π½ΡƒΠΆΠ½ΠΎ ΠΏΠ΅Ρ€Π΅ΠΌΠ΅Ρ‰Π°Ρ‚ΡŒ. Π₯отя отсутствиС ΠΊΠΎΠ½Ρ‚Π°ΠΊΡ‚ΠΎΠ² ΠΈ двиТущихся частСй ΠΎΠ·Π½Π°Ρ‡Π°Π΅Ρ‚, Ρ‡Ρ‚ΠΎ Ρ‚Π²Π΅Ρ€Π΄ΠΎΡ‚Π΅Π»ΡŒΠ½Ρ‹Π΅ Ρ€Π΅Π»Π΅ Π½Π΅ ΠΏΠΎΠ΄Π²Π΅Ρ€ΠΆΠ΅Π½Ρ‹ ΠΈΡΠΊΡ€Π΅Π½ΠΈΡŽ ΠΈ Π½Π΅ ΠΈΠ·Π½Π°ΡˆΠΈΠ²Π°ΡŽΡ‚ΡΡ, ΠΊΠΎΠ½Ρ‚Π°ΠΊΡ‚Ρ‹ Π² элСктромСханичСских Ρ€Π΅Π»Π΅ ΠΌΠΎΠΆΠ½ΠΎ Π·Π°ΠΌΠ΅Π½ΠΈΡ‚ΡŒ, Ρ‚ΠΎΠ³Π΄Π° ΠΊΠ°ΠΊ всС Ρ‚Π²Π΅Ρ€Π΄ΠΎΡ‚Π΅Π»ΡŒΠ½Ρ‹Π΅ Ρ€Π΅Π»Π΅ Π΄ΠΎΠ»ΠΆΠ½Ρ‹ Π±Ρ‹Ρ‚ΡŒ Π·Π°ΠΌΠ΅Π½Π΅Π½Ρ‹, ΠΊΠΎΠ³Π΄Π° какая-Π»ΠΈΠ±ΠΎ Ρ‡Π°ΡΡ‚ΡŒ Π²Ρ‹Ρ…ΠΎΠ΄ΠΈΡ‚ ΠΈΠ· строя. Из-Π·Π° конструкции Ρ‚Π²Π΅Ρ€Π΄ΠΎΡ‚Π΅Π»ΡŒΠ½Ρ‹Ρ… Ρ€Π΅Π»Π΅ сущСствуСт остаточноС элСктричСскоС сопротивлСниС ΠΈ/ΠΈΠ»ΠΈ ΡƒΡ‚Π΅Ρ‡ΠΊΠ° Ρ‚ΠΎΠΊΠ° нСзависимо ΠΎΡ‚ Ρ‚ΠΎΠ³ΠΎ, Ρ€Π°Π·ΠΎΠΌΠΊΠ½ΡƒΡ‚Ρ‹ ΠΈΠ»ΠΈ Π·Π°ΠΌΠΊΠ½ΡƒΡ‚Ρ‹ ΠΏΠ΅Ρ€Π΅ΠΊΠ»ΡŽΡ‡Π°Ρ‚Π΅Π»ΠΈ. Π’ΠΎΠ·Π½ΠΈΠΊΠ°ΡŽΡ‰ΠΈΠ΅ нСбольшиС ΠΏΠ΅Ρ€Π΅ΠΏΠ°Π΄Ρ‹ напряТСния ΠΎΠ±Ρ‹Ρ‡Π½ΠΎ Π½Π΅ ΠΏΡ€Π΅Π΄ΡΡ‚Π°Π²Π»ΡΡŽΡ‚ ΠΏΡ€ΠΎΠ±Π»Π΅ΠΌΡ‹; ΠΎΠ΄Π½Π°ΠΊΠΎ элСктромСханичСскиС Ρ€Π΅Π»Π΅ ΠΎΠ±Π΅ΡΠΏΠ΅Ρ‡ΠΈΠ²Π°ΡŽΡ‚ Π±ΠΎΠ»Π΅Π΅ чистоС состояниС Π’ΠšΠ› ΠΈΠ»ΠΈ Π’Π«ΠšΠ› ΠΈΠ·-Π·Π° ΠΎΡ‚Π½ΠΎΡΠΈΡ‚Π΅Π»ΡŒΠ½ΠΎ большого расстояния ΠΌΠ΅ΠΆΠ΄Ρƒ ΠΊΠΎΠ½Ρ‚Π°ΠΊΡ‚Π°ΠΌΠΈ, ΠΊΠΎΡ‚ΠΎΡ€ΠΎΠ΅ дСйствуСт ΠΊΠ°ΠΊ Ρ„ΠΎΡ€ΠΌΠ° изоляции.

Π₯отя Ρ‚Π²Π΅Ρ€Π΄ΠΎΡ‚Π΅Π»ΡŒΠ½Ρ‹Π΅ Ρ€Π΅Π»Π΅ ΠΎΠ±Π΅ΡΠΏΠ΅Ρ‡ΠΈΠ²Π°ΡŽΡ‚ Ρ‚Π΅ ΠΆΠ΅ Ρ€Π΅Π·ΡƒΠ»ΡŒΡ‚Π°Ρ‚Ρ‹, Ρ‡Ρ‚ΠΎ ΠΈ элСктромСханичСскиС Ρ€Π΅Π»Π΅, физичСская структура ΠΈ Ρ„ΡƒΠ½ΠΊΡ†ΠΈΠΎΠ½Π°Π»ΡŒΠ½Ρ‹Π΅ возмоТности Ρ‚Π²Π΅Ρ€Π΄ΠΎΡ‚Π΅Π»ΡŒΠ½Ρ‹Ρ… Ρ€Π΅Π»Π΅ ΠΎΡ‚Π»ΠΈΡ‡Π°ΡŽΡ‚ΡΡ ΠΎΡ‚ элСктромСханичСских Ρ€Π΅Π»Π΅.

ЭлСктромСханичСскиС Ρ€Π΅Π»Π΅

ΠžΡΠ½ΠΎΠ²Π½Ρ‹Π΅ части ΠΈ Ρ„ΡƒΠ½ΠΊΡ†ΠΈΠΈ элСктромСханичСских Ρ€Π΅Π»Π΅ Π²ΠΊΠ»ΡŽΡ‡Π°ΡŽΡ‚ Π² сСбя:

  1. Π Π°ΠΌΠ°: ΠŸΡ€ΠΎΡ‡Π½Π°Ρ Ρ€Π°ΠΌΠ°, содСрТащая ΠΈ ΠΏΠΎΠ΄Π΄Π΅Ρ€ΠΆΠΈΠ²Π°ΡŽΡ‰Π°Ρ части Ρ€Π΅Π»Π΅.
  2. ΠšΠ°Ρ‚ΡƒΡˆΠΊΠ°: ΠŸΡ€ΠΎΠ²ΠΎΠ»ΠΎΠΊΠ° Π½Π°ΠΌΠΎΡ‚Π°Π½Π° Π½Π° мСталличСский сСрдСчник. ΠšΠ°Ρ‚ΡƒΡˆΠΊΠ° ΠΏΡ€ΠΎΠ²ΠΎΠ΄Π° создаСт элСктромагнитноС ΠΏΠΎΠ»Π΅.
  3. Π―ΠΊΠΎΡ€ΡŒ: ПодвиТная Ρ‡Π°ΡΡ‚ΡŒ Ρ€Π΅Π»Π΅ А. Π―ΠΊΠΎΡ€ΡŒ Ρ€Π°Π·ΠΌΡ‹ΠΊΠ°Π΅Ρ‚ ΠΈ Π·Π°ΠΌΡ‹ΠΊΠ°Π΅Ρ‚ ΠΊΠΎΠ½Ρ‚Π°ΠΊΡ‚Ρ‹. ΠŸΡ€ΠΈΠΊΡ€Π΅ΠΏΠ»Π΅Π½Π½Π°Ρ ΠΏΡ€ΡƒΠΆΠΈΠ½Π° Π²ΠΎΠ·Π²Ρ€Π°Ρ‰Π°Π΅Ρ‚ ΡΠΊΠΎΡ€ΡŒ Π² исходноС ΠΏΠΎΠ»ΠΎΠΆΠ΅Π½ΠΈΠ΅.
  4. ΠšΠΎΠ½Ρ‚Π°ΠΊΡ‚Ρ‹: ΠŸΡ€ΠΎΠ²ΠΎΠ΄ΡΡ‰Π°Ρ Ρ‡Π°ΡΡ‚ΡŒ ΠΏΠ΅Ρ€Π΅ΠΊΠ»ΡŽΡ‡Π°Ρ‚Π΅Π»Ρ, которая Π·Π°ΠΌΡ‹ΠΊΠ°Π΅Ρ‚ (Π·Π°ΠΌΡ‹ΠΊΠ°Π΅Ρ‚) ΠΈΠ»ΠΈ Ρ€Π°Π·Ρ€Ρ‹Π²Π°Π΅Ρ‚ (Ρ€Π°Π·ΠΌΡ‹ΠΊΠ°Π΅Ρ‚) Ρ†Π΅ΠΏΡŒ.

Π Π΅Π»Π΅ Π²ΠΊΠ»ΡŽΡ‡Π°ΡŽΡ‚ Π΄Π²Π΅ Ρ†Π΅ΠΏΠΈ: Ρ†Π΅ΠΏΡŒ питания ΠΈ Ρ†Π΅ΠΏΡŒ ΠΊΠΎΠ½Ρ‚Π°ΠΊΡ‚Π°. ΠšΠ°Ρ‚ΡƒΡˆΠΊΠ° находится Π½Π° ΠΏΠΈΡ‚Π°ΡŽΡ‰Π΅ΠΉ сторонС; Π° ΠΊΠΎΠ½Ρ‚Π°ΠΊΡ‚Ρ‹ Ρ€Π΅Π»Π΅ находятся Π½Π° ΠΊΠΎΠ½Ρ‚Π°ΠΊΡ‚Π½ΠΎΠΉ сторонС. Когда ΠΊΠ°Ρ‚ΡƒΡˆΠΊΠ° Ρ€Π΅Π»Π΅ находится ΠΏΠΎΠ΄ напряТСниСм, Ρ‚ΠΎΠΊ, ΠΏΡ€ΠΎΡ‚Π΅ΠΊΠ°ΡŽΡ‰ΠΈΠΉ Ρ‡Π΅Ρ€Π΅Π· ΠΊΠ°Ρ‚ΡƒΡˆΠΊΡƒ, создаСт ΠΌΠ°Π³Π½ΠΈΡ‚Π½ΠΎΠ΅ ΠΏΠΎΠ»Π΅. Π‘ΡƒΠ΄ΡŒ Ρ‚ΠΎ Π² Π±Π»ΠΎΠΊΠ΅ постоянного Ρ‚ΠΎΠΊΠ°, Π³Π΄Π΅ ΠΏΠΎΠ»ΡΡ€Π½ΠΎΡΡ‚ΡŒ фиксирована, ΠΈΠ»ΠΈ Π² Π±Π»ΠΎΠΊΠ΅ ΠΏΠ΅Ρ€Π΅ΠΌΠ΅Π½Π½ΠΎΠ³ΠΎ Ρ‚ΠΎΠΊΠ°, Π³Π΄Π΅ ΠΏΠΎΠ»ΡΡ€Π½ΠΎΡΡ‚ΡŒ мСняСтся 120 Ρ€Π°Π· Π² сСкунду, основная функция остаСтся Π½Π΅ΠΈΠ·ΠΌΠ΅Π½Π½ΠΎΠΉ: магнитная ΠΊΠ°Ρ‚ΡƒΡˆΠΊΠ° притягиваСт пластину ΠΈΠ· ΠΆΠ΅Π»Π΅Π·Π°, которая являСтся Ρ‡Π°ΡΡ‚ΡŒΡŽ якоря. Один ΠΊΠΎΠ½Π΅Ρ† якоря ΠΏΡ€ΠΈΠΊΡ€Π΅ΠΏΠ»Π΅Π½ ΠΊ мСталличСской Ρ€Π°ΠΌΠ΅, Π²Ρ‹ΠΏΠΎΠ»Π½Π΅Π½Π½ΠΎΠΉ Ρ‚Π°ΠΊΠΈΠΌ ΠΎΠ±Ρ€Π°Π·ΠΎΠΌ, Ρ‡Ρ‚ΠΎ ΡΠΊΠΎΡ€ΡŒ ΠΌΠΎΠΆΠ΅Ρ‚ ΠΏΠΎΠ²ΠΎΡ€Π°Ρ‡ΠΈΠ²Π°Ρ‚ΡŒΡΡ, Π° Π΄Ρ€ΡƒΠ³ΠΎΠΉ ΠΊΠΎΠ½Π΅Ρ† Ρ€Π°Π·ΠΌΡ‹ΠΊΠ°Π΅Ρ‚ ΠΈ Π·Π°ΠΌΡ‹ΠΊΠ°Π΅Ρ‚ ΠΊΠΎΠ½Ρ‚Π°ΠΊΡ‚Ρ‹. ΠšΠΎΠ½Ρ‚Π°ΠΊΡ‚Ρ‹ Π±Ρ‹Π²Π°ΡŽΡ‚ Ρ€Π°Π·Π½Ρ‹Ρ… ΠΊΠΎΠ½Ρ„ΠΈΠ³ΡƒΡ€Π°Ρ†ΠΈΠΉ, Π² зависимости ΠΎΡ‚ количСства Ρ€Π°Π·ΠΌΡ‹ΠΊΠ°Ρ‚Π΅Π»Π΅ΠΉ, полюсов ΠΈ бросков, ΡΠΎΡΡ‚Π°Π²Π»ΡΡŽΡ‰ΠΈΡ… Ρ€Π΅Π»Π΅. НапримСр, Ρ€Π΅Π»Π΅ ΠΌΠΎΠΆΠ½ΠΎ ΠΎΠΏΠΈΡΠ°Ρ‚ΡŒ ΠΊΠ°ΠΊ однополюсноС, ΠΎΠ΄Π½ΠΎΠ½Π°ΠΏΡ€Π°Π²Π»Π΅Π½Π½ΠΎΠ΅ (SPST) ΠΈΠ»ΠΈ Π΄Π²ΡƒΡ…ΠΏΠΎΠ»ΡŽΡΠ½ΠΎΠ΅, ΠΎΠ΄Π½ΠΎΠ½Π°ΠΏΡ€Π°Π²Π»Π΅Π½Π½ΠΎΠ΅ (DPST).

Π­Ρ‚ΠΈ Ρ‚Π΅Ρ€ΠΌΠΈΠ½Ρ‹ Π΄Π°Π΄ΡƒΡ‚ ΠΌΠ³Π½ΠΎΠ²Π΅Π½Π½ΠΎΠ΅ прСдставлСниС ΠΎ конструкции ΠΈ функциях Ρ€Π°Π·Π»ΠΈΡ‡Π½Ρ‹Ρ… Ρ‚ΠΈΠΏΠΎΠ² Ρ€Π΅Π»Π΅.

  • Π Π°Π·Ρ€Ρ‹Π² — Π­Ρ‚ΠΎ количСство ΠΎΡ‚Π΄Π΅Π»ΡŒΠ½Ρ‹Ρ… мСст ΠΈΠ»ΠΈ ΠΊΠΎΠ½Ρ‚Π°ΠΊΡ‚ΠΎΠ², ΠΊΠΎΡ‚ΠΎΡ€Ρ‹Π΅ ΠΏΠ΅Ρ€Π΅ΠΊΠ»ΡŽΡ‡Π°Ρ‚Π΅Π»ΡŒ ΠΈΡΠΏΠΎΠ»ΡŒΠ·ΡƒΠ΅Ρ‚ для размыкания ΠΈΠ»ΠΈ замыкания ΠΎΠ΄Π½ΠΎΠΉ элСктричСской Ρ†Π΅ΠΏΠΈ. ВсС ΠΊΠΎΠ½Ρ‚Π°ΠΊΡ‚Ρ‹ Π»ΠΈΠ±ΠΎ ΠΎΠ΄ΠΈΠ½Π°Ρ€Π½Ρ‹Π΅, Π»ΠΈΠ±ΠΎ Π΄Π²ΠΎΠΉΠ½Ρ‹Π΅. ΠžΠ΄Π½ΠΎΡ€Π°Π·ΠΌΡ‹ΠΊΠ°ΡŽΡ‰ΠΈΠΉ ΠΊΠΎΠ½Ρ‚Π°ΠΊΡ‚ (SB) Ρ€Π°Π·Ρ€Ρ‹Π²Π°Π΅Ρ‚ ΡΠ»Π΅ΠΊΡ‚Ρ€ΠΈΡ‡Π΅ΡΠΊΡƒΡŽ Ρ†Π΅ΠΏΡŒ Π² ΠΎΠ΄Π½ΠΎΠΌ мСстС, Π° Π΄Π²ΠΎΠΉΠ½ΠΎΠΉ Ρ€Π°Π·ΠΌΡ‹ΠΊΠ°ΡŽΡ‰ΠΈΠΉ ΠΊΠΎΠ½Ρ‚Π°ΠΊΡ‚ (DB) Ρ€Π°Π·Ρ€Ρ‹Π²Π°Π΅Ρ‚ Π΅Π΅ Π² Π΄Π²ΡƒΡ… мСстах. ΠžΠ΄ΠΈΠ½Π°Ρ€Π½Ρ‹Π΅ Ρ€Π°Π·ΠΌΡ‹ΠΊΠ°ΡŽΡ‰ΠΈΠ΅ ΠΊΠΎΠ½Ρ‚Π°ΠΊΡ‚Ρ‹ ΠΎΠ±Ρ‹Ρ‡Π½ΠΎ ΠΈΡΠΏΠΎΠ»ΡŒΠ·ΡƒΡŽΡ‚ΡΡ ΠΏΡ€ΠΈ ΠΏΠ΅Ρ€Π΅ΠΊΠ»ΡŽΡ‡Π΅Π½ΠΈΠΈ ΠΌΠ°Π»ΠΎΠΌΠΎΡ‰Π½Ρ‹Ρ… устройств, Ρ‚Π°ΠΊΠΈΡ… ΠΊΠ°ΠΊ свСтовыС ΠΈΠ½Π΄ΠΈΠΊΠ°Ρ‚ΠΎΡ€Ρ‹. ΠšΠΎΠ½Ρ‚Π°ΠΊΡ‚Ρ‹ с Π΄Π²ΠΎΠΉΠ½Ρ‹ΠΌ Ρ€Π°Π·Ρ€Ρ‹Π²ΠΎΠΌ ΠΈΡΠΏΠΎΠ»ΡŒΠ·ΡƒΡŽΡ‚ΡΡ ΠΏΡ€ΠΈ ΠΏΠ΅Ρ€Π΅ΠΊΠ»ΡŽΡ‡Π΅Π½ΠΈΠΈ ΠΌΠΎΡ‰Π½Ρ‹Ρ… устройств, Ρ‚Π°ΠΊΠΈΡ… ΠΊΠ°ΠΊ солСноиды.
  • Полюс β€” это количСство ΠΏΠΎΠ»Π½ΠΎΡΡ‚ΡŒΡŽ ΠΈΠ·ΠΎΠ»ΠΈΡ€ΠΎΠ²Π°Π½Π½Ρ‹Ρ… Ρ†Π΅ΠΏΠ΅ΠΉ, ΠΊΠΎΡ‚ΠΎΡ€Ρ‹Π΅ Ρ€Π΅Π»Π΅ ΠΌΠΎΠ³ΡƒΡ‚ ΠΏΡ€ΠΎΡ…ΠΎΠ΄ΠΈΡ‚ΡŒ Ρ‡Π΅Ρ€Π΅Π· ΠΏΠ΅Ρ€Π΅ΠΊΠ»ΡŽΡ‡Π°Ρ‚Π΅Π»ΡŒ. ΠžΠ΄Π½ΠΎΠΏΠΎΠ»ΡŽΡΠ½Ρ‹ΠΉ ΠΊΠΎΠ½Ρ‚Π°ΠΊΡ‚ (SP) ΠΌΠΎΠΆΠ΅Ρ‚ ΠΎΠ΄Π½ΠΎΠ²Ρ€Π΅ΠΌΠ΅Π½Π½ΠΎ ΠΏΡ€ΠΎΠ²ΠΎΠ΄ΠΈΡ‚ΡŒ Ρ‚ΠΎΠΊ Ρ‚ΠΎΠ»ΡŒΠΊΠΎ ΠΏΠΎ ΠΎΠ΄Π½ΠΎΠΉ Ρ†Π΅ΠΏΠΈ. Π”Π²ΡƒΡ…ΠΏΠΎΠ»ΡŽΡΠ½Ρ‹ΠΉ ΠΊΠΎΠ½Ρ‚Π°ΠΊΡ‚ (Π”ΠŸ) ΠΌΠΎΠΆΠ΅Ρ‚ ΠΏΡ€ΠΎΠ²ΠΎΠ΄ΠΈΡ‚ΡŒ Ρ‚ΠΎΠΊ ΠΏΠΎ Π΄Π²ΡƒΠΌ ΠΈΠ·ΠΎΠ»ΠΈΡ€ΠΎΠ²Π°Π½Π½Ρ‹ΠΌ цСпям ΠΎΠ΄Π½ΠΎΠ²Ρ€Π΅ΠΌΠ΅Π½Π½ΠΎ. МаксимальноС количСство полюсов 12, Π² зависимости ΠΎΡ‚ конструкции Ρ€Π΅Π»Π΅.
  • Throw — Π­Ρ‚ΠΎ количСство Π·Π°ΠΊΡ€Ρ‹Ρ‚Ρ‹Ρ… ΠΊΠΎΠ½Ρ‚Π°ΠΊΡ‚ΠΎΠ² Π½Π° полюс, доступных Π½Π° ΠΏΠ΅Ρ€Π΅ΠΊΠ»ΡŽΡ‡Π°Ρ‚Π΅Π»Π΅. Π’Ρ‹ΠΊΠ»ΡŽΡ‡Π°Ρ‚Π΅Π»ΡŒ с ΠΎΠ΄Π½ΠΎΠΏΠΎΠ·ΠΈΡ†ΠΈΠΎΠ½Π½Ρ‹ΠΌ ΠΊΠΎΠ½Ρ‚Π°ΠΊΡ‚ΠΎΠΌ ΠΌΠΎΠΆΠ΅Ρ‚ ΡƒΠΏΡ€Π°Π²Π»ΡΡ‚ΡŒ Ρ‚ΠΎΠ»ΡŒΠΊΠΎ ΠΎΠ΄Π½ΠΎΠΉ Ρ†Π΅ΠΏΡŒΡŽ, Π° Π΄Π²ΡƒΡ…ΠΏΠΎΠ·ΠΈΡ†ΠΈΠΎΠ½Π½Ρ‹ΠΉ ΠΊΠΎΠ½Ρ‚Π°ΠΊΡ‚ ΠΌΠΎΠΆΠ΅Ρ‚ ΡƒΠΏΡ€Π°Π²Π»ΡΡ‚ΡŒ двумя.

Π’ΠΈΠΏΡ‹ Ρ€Π΅Π»Π΅: ЭлСктромСханичСскиС

  1. Π Π΅Π»Π΅ ΠΎΠ±Ρ‰Π΅Π³ΠΎ назначСния ΠΏΡ€Π΅Π΄ΡΡ‚Π°Π²Π»ΡΡŽΡ‚ собой элСктромСханичСскиС ΠΏΠ΅Ρ€Π΅ΠΊΠ»ΡŽΡ‡Π°Ρ‚Π΅Π»ΠΈ, ΠΎΠ±Ρ‹Ρ‡Π½ΠΎ управляСмыС ΠΌΠ°Π³Π½ΠΈΡ‚Π½ΠΎΠΉ ΠΊΠ°Ρ‚ΡƒΡˆΠΊΠΎΠΉ. Π Π΅Π»Π΅ ΠΎΠ±Ρ‰Π΅Π³ΠΎ назначСния Ρ€Π°Π±ΠΎΡ‚Π°ΡŽΡ‚ с ΠΏΠ΅Ρ€Π΅ΠΌΠ΅Π½Π½Ρ‹ΠΌ ΠΈΠ»ΠΈ постоянным Ρ‚ΠΎΠΊΠΎΠΌ ΠΏΡ€ΠΈ ΠΎΠ±Ρ‹Ρ‡Π½Ρ‹Ρ… напряТСниях, Ρ‚Π°ΠΊΠΈΡ… ΠΊΠ°ΠΊ 12 Π’, 24 Π’, 48 Π’, 120 Π’ ΠΈ 230 Π’, ΠΈ ΠΌΠΎΠ³ΡƒΡ‚ ΡƒΠΏΡ€Π°Π²Π»ΡΡ‚ΡŒ Ρ‚ΠΎΠΊΠ°ΠΌΠΈ Π² Π΄ΠΈΠ°ΠΏΠ°Π·ΠΎΠ½Π΅ ΠΎΡ‚ 2 Π΄ΠΎ 30 А. Π­Ρ‚ΠΈ Ρ€Π΅Π»Π΅ экономичны, Π»Π΅Π³ΠΊΠΎ замСняСмы ΠΈ Π΄ΠΎΠΏΡƒΡΠΊΠ°ΡŽΡ‚ ΡˆΠΈΡ€ΠΎΠΊΠΈΠΉ Π΄ΠΈΠ°ΠΏΠ°Π·ΠΎΠ½ ΠΊΠΎΠ½Ρ„ΠΈΠ³ΡƒΡ€Π°Ρ†ΠΈΠΉ ΠΏΠ΅Ρ€Π΅ΠΊΠ»ΡŽΡ‡Π°Ρ‚Π΅Π»Π΅ΠΉ.
  2. Π Π΅Π»Π΅ управлСния машиной Ρ‚Π°ΠΊΠΆΠ΅ ΡƒΠΏΡ€Π°Π²Π»ΡΡŽΡ‚ΡΡ ΠΌΠ°Π³Π½ΠΈΡ‚Π½ΠΎΠΉ ΠΊΠ°Ρ‚ΡƒΡˆΠΊΠΎΠΉ. Π­Ρ‚ΠΎ свСрхмощныС Ρ€Π΅Π»Π΅, ΠΈΡΠΏΠΎΠ»ΡŒΠ·ΡƒΠ΅ΠΌΡ‹Π΅ для управлСния стартСром ΠΈ Π΄Ρ€ΡƒΠ³ΠΈΠΌΠΈ ΠΏΡ€ΠΎΠΌΡ‹ΡˆΠ»Π΅Π½Π½Ρ‹ΠΌΠΈ ΠΊΠΎΠΌΠΏΠΎΠ½Π΅Π½Ρ‚Π°ΠΌΠΈ. Π₯отя ΠΎΠ½ΠΈ Π΄ΠΎΡ€ΠΎΠΆΠ΅, Ρ‡Π΅ΠΌ Ρ€Π΅Π»Π΅ ΠΎΠ±Ρ‰Π΅Π³ΠΎ назначСния, ΠΎΠ½ΠΈ, ΠΊΠ°ΠΊ ΠΏΡ€Π°Π²ΠΈΠ»ΠΎ, Π±ΠΎΠ»Π΅Π΅ Π΄ΠΎΠ»Π³ΠΎΠ²Π΅Ρ‡Π½Ρ‹. Π‘Π°ΠΌΡ‹ΠΌ большим прСимущСством Ρ€Π΅Π»Π΅ управлСния машиной ΠΏΠΎ ΡΡ€Π°Π²Π½Π΅Π½ΠΈΡŽ с Ρ€Π΅Π»Π΅ ΠΎΠ±Ρ‰Π΅Π³ΠΎ назначСния являСтся Ρ€Π°ΡΡˆΠΈΡ€ΡΠ΅ΠΌΠ°Ρ Ρ„ΡƒΠ½ΠΊΡ†ΠΈΠΎΠ½Π°Π»ΡŒΠ½ΠΎΡΡ‚ΡŒ Ρ€Π΅Π»Π΅ управлСния машиной Π·Π° счСт добавлСния аксСссуаров. Для Ρ€Π΅Π»Π΅ управлСния машинами доступСн ΡˆΠΈΡ€ΠΎΠΊΠΈΠΉ Π²Ρ‹Π±ΠΎΡ€ принадлСТностСй, Π²ΠΊΠ»ΡŽΡ‡Π°Ρ Π΄ΠΎΠΏΠΎΠ»Π½ΠΈΡ‚Π΅Π»ΡŒΠ½Ρ‹Π΅ полюса, трансформируСмыС ΠΊΠΎΠ½Ρ‚Π°ΠΊΡ‚Ρ‹, устройства подавлСния ΠΏΠ΅Ρ€Π΅Ρ…ΠΎΠ΄Π½Ρ‹Ρ… элСктричСских ΠΏΠΎΠΌΠ΅Ρ…, Π±Π»ΠΎΠΊΠΈΡ€ΠΎΠ²ΠΊΡƒ управлСния ΠΈ приспособлСния для синхронизации.
  3. Π“Π΅Ρ€ΠΊΠΎΠ½ΠΎΠ²Ρ‹Π΅ Ρ€Π΅Π»Π΅ ΠΏΡ€Π΅Π΄ΡΡ‚Π°Π²Π»ΡΡŽΡ‚ собой нСбольшой, ΠΊΠΎΠΌΠΏΠ°ΠΊΡ‚Π½Ρ‹ΠΉ, Π±Ρ‹ΡΡ‚Ρ€ΠΎΠ΄Π΅ΠΉΡΡ‚Π²ΡƒΡŽΡ‰ΠΈΠΉ ΠΏΠ΅Ρ€Π΅ΠΊΠ»ΡŽΡ‡Π°Ρ‚Π΅Π»ΡŒ с ΠΎΠ΄Π½ΠΈΠΌ Π·Π°ΠΌΡ‹ΠΊΠ°ΡŽΡ‰ΠΈΠΌ ΠΊΠΎΠ½Ρ‚Π°ΠΊΡ‚ΠΎΠΌ. Π“Π΅Ρ€ΠΊΠΎΠ½ΠΎΠ²Ρ‹Π΅ Ρ€Π΅Π»Π΅ Π³Π΅Ρ€ΠΌΠ΅Ρ‚ΠΈΡ‡Π½ΠΎ Π·Π°ΠΊΠ»ΡŽΡ‡Π΅Π½Ρ‹ Π² ΡΡ‚Π΅ΠΊΠ»ΡΠ½Π½ΡƒΡŽ ΠΎΠ±ΠΎΠ»ΠΎΡ‡ΠΊΡƒ, Ρ‡Ρ‚ΠΎ Π΄Π΅Π»Π°Π΅Ρ‚ ΠΊΠΎΠ½Ρ‚Π°ΠΊΡ‚Ρ‹ нСвосприимчивыми ΠΊ загрязнСниям, Π΄Ρ‹ΠΌΡƒ ΠΈΠ»ΠΈ Π²Π»Π°Π³Π΅, обСспСчиваСт Π½Π°Π΄Π΅ΠΆΠ½ΠΎΠ΅ ΠΏΠ΅Ρ€Π΅ΠΊΠ»ΡŽΡ‡Π΅Π½ΠΈΠ΅ ΠΈ ΡƒΠ²Π΅Π»ΠΈΡ‡ΠΈΠ²Π°Π΅Ρ‚ ΠΎΠΆΠΈΠ΄Π°Π΅ΠΌΡ‹ΠΉ срок слуТбы ΠΊΠΎΠ½Ρ‚Π°ΠΊΡ‚ΠΎΠ². ΠšΠΎΠ½Ρ†Ρ‹ ΠΊΠΎΠ½Ρ‚Π°ΠΊΡ‚Π°, ΠΊΠΎΡ‚ΠΎΡ€Ρ‹Π΅ часто ΠΏΠΎΠΊΡ€Ρ‹Ρ‚Ρ‹ Π·ΠΎΠ»ΠΎΡ‚ΠΎΠΌ ΠΈΠ»ΠΈ Π΄Ρ€ΡƒΠ³ΠΈΠΌ ΠΌΠ°Ρ‚Π΅Ρ€ΠΈΠ°Π»ΠΎΠΌ с Π½ΠΈΠ·ΠΊΠΈΠΌ сопротивлСниСм для ΠΏΠΎΠ²Ρ‹ΡˆΠ΅Π½ΠΈΡ проводимости, ΡΠ±Π»ΠΈΠΆΠ°ΡŽΡ‚ΡΡ ΠΈ Π·Π°ΠΊΡ€Ρ‹Π²Π°ΡŽΡ‚ΡΡ ΠΌΠ°Π³Π½ΠΈΡ‚ΠΎΠΌ. Π“Π΅Ρ€ΠΊΠΎΠ½ΠΎΠ²Ρ‹Π΅ Ρ€Π΅Π»Π΅ способны ΠΏΠ΅Ρ€Π΅ΠΊΠ»ΡŽΡ‡Π°Ρ‚ΡŒ ΠΏΡ€ΠΎΠΌΡ‹ΡˆΠ»Π΅Π½Π½Ρ‹Π΅ ΠΊΠΎΠΌΠΏΠΎΠ½Π΅Π½Ρ‚Ρ‹, Ρ‚Π°ΠΊΠΈΠ΅ ΠΊΠ°ΠΊ солСноиды, ΠΊΠΎΠ½Ρ‚Π°ΠΊΡ‚ΠΎΡ€Ρ‹ ΠΈ стартСры. Π“Π΅Ρ€ΠΊΠΎΠ½ΠΎΠ²Ρ‹Π΅ Ρ€Π΅Π»Π΅ состоят ΠΈΠ· Π΄Π²ΡƒΡ… Π³Π΅Ρ€ΠΊΠΎΠ½ΠΎΠ². Когда примСняСтся магнитная сила, такая ΠΊΠ°ΠΊ элСктромагнит ΠΈΠ»ΠΈ ΠΊΠ°Ρ‚ΡƒΡˆΠΊΠ°, ΠΎΠ½Π° создаСт ΠΌΠ°Π³Π½ΠΈΡ‚Π½ΠΎΠ΅ ΠΏΠΎΠ»Π΅, Π² ΠΊΠΎΡ‚ΠΎΡ€ΠΎΠΌ ΠΊΠΎΠ½Ρ†Ρ‹ язычков ΠΏΡ€ΠΈΠ½ΠΈΠΌΠ°ΡŽΡ‚ ΠΏΡ€ΠΎΡ‚ΠΈΠ²ΠΎΠΏΠΎΠ»ΠΎΠΆΠ½ΡƒΡŽ ΠΏΠΎΠ»ΡΡ€Π½ΠΎΡΡ‚ΡŒ. Когда ΠΌΠ°Π³Π½ΠΈΡ‚Π½ΠΎΠ΅ ΠΏΠΎΠ»Π΅ достаточно сильноС, сила притяТСния ΠΏΡ€ΠΎΡ‚ΠΈΠ²ΠΎΠΏΠΎΠ»ΠΎΠΆΠ½Ρ‹Ρ… полюсов ΠΏΡ€Π΅ΠΎΠ΄ΠΎΠ»Π΅Π²Π°Π΅Ρ‚ ΠΆΠ΅ΡΡ‚ΠΊΠΎΡΡ‚ΡŒ язычков ΠΈ сблиТаСт ΠΈΡ…. Когда магнитная сила исчСзаСт, язычки Π²ΠΎΠ·Π²Ρ€Π°Ρ‰Π°ΡŽΡ‚ΡΡ Π² исходноС ΠΎΡ‚ΠΊΡ€Ρ‹Ρ‚ΠΎΠ΅ ΠΏΠΎΠ»ΠΎΠΆΠ΅Π½ΠΈΠ΅. Π­Ρ‚ΠΈ Ρ€Π΅Π»Π΅ Ρ€Π°Π±ΠΎΡ‚Π°ΡŽΡ‚ ΠΎΡ‡Π΅Π½ΡŒ быстро ΠΈΠ·-Π·Π° нСбольшого расстояния ΠΌΠ΅ΠΆΠ΄Ρƒ ΠΊΠ°ΠΌΡ‹ΡˆΠ°ΠΌΠΈ.

Π’Π²Π΅Ρ€Π΄ΠΎΡ‚Π΅Π»ΡŒΠ½Ρ‹Π΅ Ρ€Π΅Π»Π΅

Π’Π²Π΅Ρ€Π΄ΠΎΡ‚Π΅Π»ΡŒΠ½Ρ‹Π΅ Ρ€Π΅Π»Π΅ состоят ΠΈΠ· Π²Ρ…ΠΎΠ΄Π½ΠΎΠΉ Ρ†Π΅ΠΏΠΈ , Ρ†Π΅ΠΏΠΈ управлСния ΠΈ Π²Ρ‹Ρ…ΠΎΠ΄Π½ΠΎΠΉ Ρ†Π΅ΠΏΠΈ . Входная Ρ†Π΅ΠΏΡŒ β€” это Ρ‡Π°ΡΡ‚ΡŒ корпуса Ρ€Π΅Π»Π΅, ΠΊ ΠΊΠΎΡ‚ΠΎΡ€ΠΎΠΉ ΠΏΠΎΠ΄ΠΊΠ»ΡŽΡ‡Π΅Π½ ΠΊΠΎΠΌΠΏΠΎΠ½Π΅Π½Ρ‚ управлСния. Π’Ρ…ΠΎΠ΄Π½ΠΎΠΉ ΠΊΠΎΠ½Ρ‚ΡƒΡ€ выполняСт Ρ‚Ρƒ ΠΆΠ΅ Ρ„ΡƒΠ½ΠΊΡ†ΠΈΡŽ, Ρ‡Ρ‚ΠΎ ΠΈ ΠΎΠ±ΠΌΠΎΡ‚ΠΊΠ° элСктромСханичСского Ρ€Π΅Π»Π΅. ЦСпь активируСтся, ΠΊΠΎΠ³Π΄Π° Π½Π° Π²Ρ…ΠΎΠ΄ Ρ€Π΅Π»Π΅ подаСтся напряТСниС, ΠΏΡ€Π΅Π²Ρ‹ΡˆΠ°ΡŽΡ‰Π΅Π΅ Π·Π°Π΄Π°Π½Π½ΠΎΠ΅ напряТСниС срабатывания Ρ€Π΅Π»Π΅. Входная Ρ†Π΅ΠΏΡŒ дСактивируСтся, ΠΊΠΎΠ³Π΄Π° ΠΏΡ€ΠΈΠ»ΠΎΠΆΠ΅Π½Π½ΠΎΠ΅ напряТСниС мСньшС ΡƒΠΊΠ°Π·Π°Π½Π½ΠΎΠ³ΠΎ минимального напряТСния ΠΎΡ‚ΠΊΠ»ΡŽΡ‡Π΅Π½ΠΈΡ Ρ€Π΅Π»Π΅. Π”ΠΈΠ°ΠΏΠ°Π·ΠΎΠ½ напряТСния ΠΎΡ‚ 3 Π’ Π΄ΠΎ 32 Π’ постоянного Ρ‚ΠΎΠΊΠ°, ΠΎΠ±Ρ‹Ρ‡Π½ΠΎ ΠΈΡΠΏΠΎΠ»ΡŒΠ·ΡƒΠ΅ΠΌΡ‹ΠΉ с Π±ΠΎΠ»ΡŒΡˆΠΈΠ½ΡΡ‚Π²ΠΎΠΌ Ρ‚Π²Π΅Ρ€Π΄ΠΎΡ‚Π΅Π»ΡŒΠ½Ρ‹Ρ… Ρ€Π΅Π»Π΅, Π΄Π΅Π»Π°Π΅Ρ‚ Π΅Π³ΠΎ ΠΏΡ€ΠΈΠ³ΠΎΠ΄Π½Ρ‹ΠΌ для Π±ΠΎΠ»ΡŒΡˆΠΈΠ½ΡΡ‚Π²Π° элСктронных схСм. ЦСпь управлСния β€” это Ρ‡Π°ΡΡ‚ΡŒ Ρ€Π΅Π»Π΅, которая опрСдСляСт, ΠΊΠΎΠ³Π΄Π° Π²Ρ‹Ρ…ΠΎΠ΄Π½ΠΎΠΉ ΠΊΠΎΠΌΠΏΠΎΠ½Π΅Π½Ρ‚ находится ΠΏΠΎΠ΄ напряТСниСм ΠΈΠ»ΠΈ обСсточиваСтся. ЦСпь управлСния Ρ„ΡƒΠ½ΠΊΡ†ΠΈΠΎΠ½ΠΈΡ€ΡƒΠ΅Ρ‚ ΠΊΠ°ΠΊ связь ΠΌΠ΅ΠΆΠ΄Ρƒ Π²Ρ…ΠΎΠ΄Π½ΠΎΠΉ ΠΈ Π²Ρ‹Ρ…ΠΎΠ΄Π½ΠΎΠΉ цСпями. Π’ элСктромСханичСских Ρ€Π΅Π»Π΅ эту Ρ„ΡƒΠ½ΠΊΡ†ΠΈΡŽ выполняСт ΠΊΠ°Ρ‚ΡƒΡˆΠΊΠ°. Выходная Ρ†Π΅ΠΏΡŒ Ρ€Π΅Π»Π΅ β€” это Ρ‡Π°ΡΡ‚ΡŒ Ρ€Π΅Π»Π΅, которая Π²ΠΊΠ»ΡŽΡ‡Π°Π΅Ρ‚ Π½Π°Π³Ρ€ΡƒΠ·ΠΊΡƒ ΠΈ выполняСт Ρ‚Ρƒ ΠΆΠ΅ Ρ„ΡƒΠ½ΠΊΡ†ΠΈΡŽ, Ρ‡Ρ‚ΠΎ ΠΈ мСханичСскиС ΠΊΠΎΠ½Ρ‚Π°ΠΊΡ‚Ρ‹ элСктромСханичСских Ρ€Π΅Π»Π΅. Однако Ρ‚Π²Π΅Ρ€Π΄ΠΎΡ‚Π΅Π»ΡŒΠ½Ρ‹Π΅ Ρ€Π΅Π»Π΅ ΠΎΠ±Ρ‹Ρ‡Π½ΠΎ ΠΈΠΌΠ΅ΡŽΡ‚ Ρ‚ΠΎΠ»ΡŒΠΊΠΎ ΠΎΠ΄ΠΈΠ½ Π²Ρ‹Ρ…ΠΎΠ΄Π½ΠΎΠΉ ΠΊΠΎΠ½Ρ‚Π°ΠΊΡ‚.

Π’Π²Π΅Ρ€Π΄ΠΎΡ‚Π΅Π»ΡŒΠ½Ρ‹Π΅ Ρ€Π΅Π»Π΅, ΠΏΠΎΠ΄ΠΎΠ±Π½Ρ‹Π΅ ΠΈΠ·ΠΎΠ±Ρ€Π°ΠΆΠ΅Π½Π½ΠΎΠΌΡƒ Π²Ρ‹ΡˆΠ΅, способны ΠΊΠΎΠΌΠΌΡƒΡ‚ΠΈΡ€ΠΎΠ²Π°Ρ‚ΡŒ высокиС напряТСния Π΄ΠΎ 600 Π’ ΠΏΠ΅Ρ€Π΅ΠΌΠ΅Π½Π½ΠΎΠ³ΠΎ Ρ‚ΠΎΠΊΠ° (срСднСквадратичноС Π·Π½Π°Ρ‡Π΅Π½ΠΈΠ΅). Π­Ρ‚ΠΈ Ρ€Π΅Π»Π΅ ΠΏΡ€Π΅Π΄Π½Π°Π·Π½Π°Ρ‡Π΅Π½Ρ‹ для ΠΊΠΎΠΌΠΌΡƒΡ‚Π°Ρ†ΠΈΠΈ Ρ€Π°Π·Π»ΠΈΡ‡Π½Ρ‹Ρ… Π½Π°Π³Ρ€ΡƒΠ·ΠΎΠΊ, Ρ‚Π°ΠΊΠΈΡ… ΠΊΠ°ΠΊ Π½Π°Π³Ρ€Π΅Π²Π°Ρ‚Π΅Π»ΡŒΠ½Ρ‹Π΅ элСмСнты, Π΄Π²ΠΈΠ³Π°Ρ‚Π΅Π»ΠΈ ΠΈ трансформаторы.

Π’ΠΈΠΏΡ‹ Ρ€Π΅Π»Π΅: Ρ‚Π²Π΅Ρ€Π΄ΠΎΡ‚Π΅Π»ΡŒΠ½Ρ‹Π΅

  1. Π Π΅Π»Π΅ Π½ΡƒΠ»Π΅Π²ΠΎΠΉ ΠΊΠΎΠΌΠΌΡƒΡ‚Π°Ρ†ΠΈΠΈ — Ρ€Π΅Π»Π΅ Π²ΠΊΠ»ΡŽΡ‡Π°Π΅Ρ‚ Π½Π°Π³Ρ€ΡƒΠ·ΠΊΡƒ ΠΏΡ€ΠΈ ΠΏΠΎΠ΄Π°Ρ‡Π΅ ΡƒΠΏΡ€Π°Π²Π»ΡΡŽΡ‰Π΅Π³ΠΎ (минимального Ρ€Π°Π±ΠΎΡ‡Π΅Π³ΠΎ) напряТСния ΠΈ напряТСнии Π½Π°Π³Ρ€ΡƒΠ·ΠΊΠΈ, Π±Π»ΠΈΠ·ΠΊΠΎΠΌ ΠΊ Π½ΡƒΠ»ΡŽ. Π Π΅Π»Π΅ с Π½ΡƒΠ»Π΅Π²Ρ‹ΠΌ ΠΏΠ΅Ρ€Π΅ΠΊΠ»ΡŽΡ‡Π΅Π½ΠΈΠ΅ΠΌ ΠΎΡ‚ΠΊΠ»ΡŽΡ‡Π°ΡŽΡ‚ Π½Π°Π³Ρ€ΡƒΠ·ΠΊΡƒ, ΠΊΠΎΠ³Π΄Π° напряТСниС управлСния снимаСтся ΠΈ Ρ‚ΠΎΠΊ Π² Π½Π°Π³Ρ€ΡƒΠ·ΠΊΠ΅ Π±Π»ΠΈΠ·ΠΎΠΊ ΠΊ Π½ΡƒΠ»ΡŽ. НаиболСС распространСны Ρ€Π΅Π»Π΅ с Π½ΡƒΠ»Π΅Π²Ρ‹ΠΌ ΠΏΠ΅Ρ€Π΅ΠΊΠ»ΡŽΡ‡Π΅Π½ΠΈΠ΅ΠΌ.
  2. Π Π΅Π»Π΅ ΠΌΠ³Π½ΠΎΠ²Π΅Π½Π½ΠΎΠ³ΠΎ Π²ΠΊΠ»ΡŽΡ‡Π΅Π½ΠΈΡ — Π½Π΅ΠΌΠ΅Π΄Π»Π΅Π½Π½ΠΎ Π²ΠΊΠ»ΡŽΡ‡Π°Π΅Ρ‚ Π½Π°Π³Ρ€ΡƒΠ·ΠΊΡƒ ΠΏΡ€ΠΈ Π½Π°Π»ΠΈΡ‡ΠΈΠΈ напряТСния срабатывания. Π Π΅Π»Π΅ ΠΌΠ³Π½ΠΎΠ²Π΅Π½Π½ΠΎΠ³ΠΎ Π²ΠΊΠ»ΡŽΡ‡Π΅Π½ΠΈΡ ΠΏΠΎΠ·Π²ΠΎΠ»ΡΡŽΡ‚ Π²ΠΊΠ»ΡŽΡ‡Π°Ρ‚ΡŒ Π½Π°Π³Ρ€ΡƒΠ·ΠΊΡƒ Π² любой ΠΌΠΎΠΌΠ΅Π½Ρ‚ Π΅Π΅ ΠΏΠΎΠ²Ρ‹ΡˆΠ΅Π½ΠΈΡ ΠΈ пониТСния.
  3. Π Π΅Π»Π΅ ΠΏΠΈΠΊΠΎΠ²ΠΎΠ³ΠΎ ΠΏΠ΅Ρ€Π΅ΠΊΠ»ΡŽΡ‡Π΅Π½ΠΈΡ — Π²ΠΊΠ»ΡŽΡ‡Π°Π΅Ρ‚ Π½Π°Π³Ρ€ΡƒΠ·ΠΊΡƒ, ΠΊΠΎΠ³Π΄Π° присутствуСт ΡƒΠΏΡ€Π°Π²Π»ΡΡŽΡ‰Π΅Π΅ напряТСниС, ΠΈ напряТСниС Π½Π°Π³Ρ€ΡƒΠ·ΠΊΠΈ находится Π½Π° ΠΏΠΈΠΊΠ΅. Π Π΅Π»Π΅ ΠΏΠΈΠΊΠΎΠ²ΠΎΠ³ΠΎ ΠΏΠ΅Ρ€Π΅ΠΊΠ»ΡŽΡ‡Π΅Π½ΠΈΡ Π²Ρ‹ΠΊΠ»ΡŽΡ‡Π°ΡŽΡ‚ΡΡ, ΠΊΠΎΠ³Π΄Π° исчСзаСт ΡƒΠΏΡ€Π°Π²Π»ΡΡŽΡ‰Π΅Π΅ напряТСниС ΠΈ Ρ‚ΠΎΠΊ Π² Π½Π°Π³Ρ€ΡƒΠ·ΠΊΠ΅ Π±Π»ΠΈΠ·ΠΎΠΊ ΠΊ Π½ΡƒΠ»ΡŽ.
  4. АналоговыС ΠΏΠ΅Ρ€Π΅ΠΊΠ»ΡŽΡ‡Π°ΡŽΡ‰ΠΈΠ΅ Ρ€Π΅Π»Π΅ — ΠΈΠΌΠ΅Π΅Ρ‚ бСсконСчноС количСство Π²ΠΎΠ·ΠΌΠΎΠΆΠ½Ρ‹Ρ… Π²Ρ‹Ρ…ΠΎΠ΄Π½Ρ‹Ρ… напряТСний Π² ΠΏΡ€Π΅Π΄Π΅Π»Π°Ρ… номинального Π΄ΠΈΠ°ΠΏΠ°Π·ΠΎΠ½Π° Ρ€Π΅Π»Π΅. АналоговыС ΠΏΠ΅Ρ€Π΅ΠΊΠ»ΡŽΡ‡Π°ΡŽΡ‰ΠΈΠ΅ Ρ€Π΅Π»Π΅ ΠΈΠΌΠ΅ΡŽΡ‚ Π²ΡΡ‚Ρ€ΠΎΠ΅Π½Π½ΡƒΡŽ схСму синхронизации, которая управляСт Π²Π΅Π»ΠΈΡ‡ΠΈΠ½ΠΎΠΉ Π²Ρ‹Ρ…ΠΎΠ΄Π½ΠΎΠ³ΠΎ напряТСния Π² зависимости ΠΎΡ‚ Π²Ρ…ΠΎΠ΄Π½ΠΎΠ³ΠΎ напряТСния. Π­Ρ‚ΠΎ позволяСт ΠΈΡΠΏΠΎΠ»ΡŒΠ·ΠΎΠ²Π°Ρ‚ΡŒ Ρ„ΡƒΠ½ΠΊΡ†ΠΈΡŽ Π²Ρ€Π΅ΠΌΠ΅Π½ΠΈ нарастания Π½Π°Π³Ρ€ΡƒΠ·ΠΊΠΈ. АналоговыС ΠΏΠ΅Ρ€Π΅ΠΊΠ»ΡŽΡ‡Π°ΡŽΡ‰ΠΈΠ΅ Ρ€Π΅Π»Π΅ Π²Ρ‹ΠΊΠ»ΡŽΡ‡Π°ΡŽΡ‚ΡΡ, ΠΊΠΎΠ³Π΄Π° исчСзаСт ΡƒΠΏΡ€Π°Π²Π»ΡΡŽΡ‰Π΅Π΅ напряТСниС ΠΈ Ρ‚ΠΎΠΊ Π² Π½Π°Π³Ρ€ΡƒΠ·ΠΊΠ΅ Π±Π»ΠΈΠ·ΠΎΠΊ ΠΊ Π½ΡƒΠ»ΡŽ.

A Π‘Ρ€ΠΎΠΊ слуТбы ΠΊΠΎΠ½Ρ‚Π°ΠΊΡ‚ΠΎΠ² Ρ€Π΅Π»Π΅

Π‘Ρ€ΠΎΠΊ слуТбы Ρ€Π΅Π»Π΅ зависит ΠΎΡ‚ Π΅Π³ΠΎ ΠΊΠΎΠ½Ρ‚Π°ΠΊΡ‚ΠΎΠ². ΠŸΡ€ΠΈ ΠΏΠ΅Ρ€Π΅Π³ΠΎΡ€Π°Π½ΠΈΠΈ ΠΊΠΎΠ½Ρ‚Π°ΠΊΡ‚ΠΎΠ² Π½Π΅ΠΎΠ±Ρ…ΠΎΠ΄ΠΈΠΌΠΎ Π·Π°ΠΌΠ΅Π½ΠΈΡ‚ΡŒ ΠΊΠΎΠ½Ρ‚Π°ΠΊΡ‚Ρ‹ Ρ€Π΅Π»Π΅ ΠΈΠ»ΠΈ всС Ρ€Π΅Π»Π΅. ΠœΠ΅Ρ…Π°Π½ΠΈΡ‡Π΅ΡΠΊΠ°Ρ Π΄ΠΎΠ»Π³ΠΎΠ²Π΅Ρ‡Π½ΠΎΡΡ‚ΡŒ β€” это количСство ΠΎΠΏΠ΅Ρ€Π°Ρ†ΠΈΠΉ (Ρ€Π°Π·ΠΌΡ‹ΠΊΠ°Π½ΠΈΠΉ ΠΈ Π·Π°ΠΌΡ‹ΠΊΠ°Π½ΠΈΠΉ), ΠΊΠΎΡ‚ΠΎΡ€Ρ‹Π΅ ΠΊΠΎΠ½Ρ‚Π°ΠΊΡ‚ ΠΌΠΎΠΆΠ΅Ρ‚ Π²Ρ‹ΠΏΠΎΠ»Π½ΠΈΡ‚ΡŒ Π±Π΅Π· элСктричСского Ρ‚ΠΎΠΊΠ°. ΠœΠ΅Ρ…Π°Π½ΠΈΡ‡Π΅ΡΠΊΠΈΠΉ срок слуТбы Ρ€Π΅Π»Π΅ ΠΎΡ‚Π½ΠΎΡΠΈΡ‚Π΅Π»ΡŒΠ½ΠΎ Π²Π΅Π»ΠΈΠΊ, прСдлагая Π΄ΠΎ 1 000 000 срабатываний. ЭлСктричСская Π΄ΠΎΠ»Π³ΠΎΠ²Π΅Ρ‡Π½ΠΎΡΡ‚ΡŒ Ρ€Π΅Π»Π΅ β€” это количСство ΠΎΠΏΠ΅Ρ€Π°Ρ†ΠΈΠΉ (Ρ€Π°Π·ΠΌΡ‹ΠΊΠ°Π½ΠΈΠΉ ΠΈ Π·Π°ΠΌΡ‹ΠΊΠ°Π½ΠΈΠΉ), ΠΊΠΎΡ‚ΠΎΡ€Ρ‹Π΅ ΠΊΠΎΠ½Ρ‚Π°ΠΊΡ‚Ρ‹ ΠΌΠΎΠ³ΡƒΡ‚ Π²Ρ‹ΠΏΠΎΠ»Π½ΡΡ‚ΡŒ с элСктричСским Ρ‚ΠΎΠΊΠΎΠΌ ΠΏΡ€ΠΈ Π΄Π°Π½Π½ΠΎΠΌ номинальном Ρ‚ΠΎΠΊΠ΅. Π‘Ρ€ΠΎΠΊ слуТбы ΠΊΠΎΠ½Ρ‚Π°ΠΊΡ‚ΠΎΠ² Ρ€Π΅Π»Π΅ составляСт ΠΎΡ‚ 100 000 Π΄ΠΎ 500 000 Ρ†ΠΈΠΊΠ»ΠΎΠ².

Π§Ρ‚ΠΎ Ρ‚Π°ΠΊΠΎΠ΅ Ρ€Π΅Π»Π΅?

(Π’Π΅Ρ€Π½ΡƒΡ‚ΡŒΡΡ ΠΊ Ρ€Π΅Π»Π΅)

ПониманиС Ρ€Π΅Π»Π΅ ΠΈ схСм ΠΏΠΎΠ΄ΠΊΠ»ΡŽΡ‡Π΅Π½ΠΈΡ


БпСциалисты ΠΏΠΎ Π·Π°Ρ‰ΠΈΡ‚Π΅ Ρ†Π΅ΠΏΠ΅ΠΉ.


Π’Π΅Π»Π΅Ρ„ΠΎΠ½: +61 3 9521 6133 Β Β Β  Ѐакс: +61 3 9521 6177 Β Β Β  Π’Π΅Π±-сайт: www.swe-check.com.au Как это Ρ€Π°Π±ΠΎΡ‚Π°Π΅Ρ‚ ΠΈ Ρ‡Ρ‚ΠΎ Ρ‚Π°ΠΊΠΎΠ΅ Ρ€Π΅Π»Π΅?

  • Π—Π°Ρ‡Π΅ΠΌ ΠΈΡΠΏΠΎΠ»ΡŒΠ·ΠΎΠ²Π°Ρ‚ΡŒ Ρ€Π΅Π»Π΅?
  • Π—Π°Ρ‡Π΅ΠΌ ΠΈΡΠΏΠΎΠ»ΡŒΠ·ΠΎΠ²Π°Ρ‚ΡŒ Ρ€Π΅Π»Π΅ Π² Π°Π²Ρ‚ΠΎΠΌΠΎΠ±ΠΈΠ»Π΅?
  • Как ΠΏΠΎΠ½ΡΡ‚ΡŒ элСктричСскиС характСристики Ρ€Π΅Π»Π΅
  • ΠŸΠΎΡ‡Π΅ΠΌΡƒ Π·Π°Ρ‰ΠΈΡ‚Π½Ρ‹Π΅ устройства ΠΈΡΠΏΠΎΠ»ΡŒΠ·ΡƒΡŽΡ‚ΡΡ Π² Ρ€Π΅Π»Π΅?
  • Π’ Ρ‡Π΅ΠΌ Ρ€Π°Π·Π½ΠΈΡ†Π° ΠΌΠ΅ΠΆΠ΄Ρƒ 4- ΠΈ 5-ΠΊΠΎΠ½Ρ‚Π°ΠΊΡ‚Π½Ρ‹ΠΌΠΈ Ρ€Π΅Π»Π΅?
  • Π‘Ρ…Π΅ΠΌΡ‹ 4- ΠΈ 5-ΠΊΠΎΠ½Ρ‚Π°ΠΊΡ‚Π½ΠΎΠ³ΠΎ Ρ€Π΅Π»Π΅
  • Для Ρ‡Π΅Π³ΠΎ ΠΈΡΠΏΠΎΠ»ΡŒΠ·ΡƒΡŽΡ‚ΡΡ Ρ€Π΅Π»Π΅ ISO?
  • Π’ΠΈΠΏΡ‹ Ρ€Π΅Π»Π΅
  • Π‘Ρ…Π΅ΠΌΡ‹ ΠΈ Π΄ΠΈΠ°Π³Ρ€Π°ΠΌΠΌΡ‹ Ρ€Π΅Π»Π΅
  • Π§Ρ‚ΠΎ Ρ‚Π°ΠΊΠΎΠ΅ Ρ€Π΅Π»Π΅ ΠΈ ΠΊΠ°ΠΊ ΠΎΠ½ΠΎ Ρ€Π°Π±ΠΎΡ‚Π°Π΅Ρ‚?

    Π Π΅Π»Π΅ прСдставляСт собой ΠΏΠ΅Ρ€Π΅ΠΊΠ»ΡŽΡ‡Π°Ρ‚Π΅Π»ΡŒ с элСктричСским ΠΏΡ€ΠΈΠ²ΠΎΠ΄ΠΎΠΌ. ΠžΠ±Ρ‹Ρ‡Π½ΠΎ ΠΎΠ½ΠΈ ΠΈΡΠΏΠΎΠ»ΡŒΠ·ΡƒΡŽΡ‚ элСктромагнит (ΠΊΠ°Ρ‚ΡƒΡˆΠΊΡƒ) для управлСния своим Π²Π½ΡƒΡ‚Ρ€Π΅Π½Π½ΠΈΠΌ мСханичСским ΠΌΠ΅Ρ…Π°Π½ΠΈΠ·ΠΌΠΎΠΌ ΠΏΠ΅Ρ€Π΅ΠΊΠ»ΡŽΡ‡Π΅Π½ΠΈΡ (ΠΊΠΎΠ½Ρ‚Π°ΠΊΡ‚Π°ΠΌΠΈ). Когда ΠΊΠΎΠ½Ρ‚Π°ΠΊΡ‚ Ρ€Π΅Π»Π΅ Ρ€Π°Π·ΠΎΠΌΠΊΠ½ΡƒΡ‚, это Π²ΠΊΠ»ΡŽΡ‡ΠΈΡ‚ ΠΏΠΈΡ‚Π°Π½ΠΈΠ΅ Ρ†Π΅ΠΏΠΈ, ΠΊΠΎΠ³Π΄Π° активируСтся ΠΊΠ°Ρ‚ΡƒΡˆΠΊΠ°.

    ΠŸΡ€ΠΈΠ²Π΅Π΄Π΅Π½Π½Ρ‹Π΅ Π½ΠΈΠΆΠ΅ ΠΏΡ€ΠΈΠΌΠ΅Ρ€Ρ‹ схСм Ρ€Π΅Π»Π΅ ΠΏΠΎΠΊΠ°Π·Ρ‹Π²Π°ΡŽΡ‚, ΠΊΠ°ΠΊ Ρ€Π°Π±ΠΎΡ‚Π°Π΅Ρ‚ Ρ€Π΅Π»Π΅.

    Π£ нас Π΅ΡΡ‚ΡŒ большой Π²Ρ‹Π±ΠΎΡ€ Ρ€Π΅Π»Π΅ ΠΈ Π΄Π΅Ρ€ΠΆΠ°Ρ‚Π΅Π»Π΅ΠΉ Ρ€Π΅Π»Π΅, доступных для ΠΏΠΎΠΊΡƒΠΏΠΊΠΈ. прямо с нашСго сайта.

    Π—Π°Ρ‡Π΅ΠΌ ΠΈΡΠΏΠΎΠ»ΡŒΠ·ΠΎΠ²Π°Ρ‚ΡŒ Ρ€Π΅Π»Π΅?

    Π Π΅Π»Π΅ ΠΏΠΎΠ·Π²ΠΎΠ»ΡΡŽΡ‚ слаботочной Ρ†Π΅ΠΏΠΈ ΡƒΠΏΡ€Π°Π²Π»ΡΡ‚ΡŒ ΠΎΠ΄Π½ΠΎΠΉ ΠΈΠ»ΠΈ нСсколькими ΡΠΈΠ»ΡŒΠ½ΠΎΡ‚ΠΎΡ‡Π½Ρ‹ΠΌΠΈ цСпями. Π Π΅Π»Π΅ ΠΎΠ±Π΅ΡΠΏΠ΅Ρ‡ΠΈΠ²Π°ΡŽΡ‚ эти прСимущСства;
    1. Для ΠΏΠΎΠ΄ΠΊΠ»ΡŽΡ‡Π΅Π½ΠΈΡ ΠΏΠ΅Ρ€Π΅ΠΊΠ»ΡŽΡ‡Π°Ρ‚Π΅Π»Ρ управлСния ΠΊ Ρ€Π΅Π»Π΅ ΠΌΠΎΠΆΠ½ΠΎ ΠΈΡΠΏΠΎΠ»ΡŒΠ·ΠΎΠ²Π°Ρ‚ΡŒ Π±ΠΎΠ»Π΅Π΅ Ρ‚ΠΎΠ½ΠΊΠΈΠ΅ ΠΊΠ°Π±Π΅Π»ΠΈ, Ρ‡Ρ‚ΠΎ позволяСт ΡΡΠΊΠΎΠ½ΠΎΠΌΠΈΡ‚ΡŒ вСс, мСсто ΠΈ ΡΡ‚ΠΎΠΈΠΌΠΎΡΡ‚ΡŒ.
    2. Π Π΅Π»Π΅ ΠΏΠΎΠ·Π²ΠΎΠ»ΡΡŽΡ‚ Π½Π°ΠΏΡ€Π°Π²Π»ΡΡ‚ΡŒ ΠΏΠΈΡ‚Π°Π½ΠΈΠ΅ ΠΊ устройству ΠΏΠΎ ΠΊΡ€Π°Ρ‚Ρ‡Π°ΠΉΡˆΠ΅ΠΌΡƒ Ρ€Π°ΡΡΡ‚ΠΎΡΠ½ΠΈΡŽ, Ρ‚Π΅ΠΌ самым сниТая ΠΏΠΎΡ‚Π΅Ρ€ΠΈ напряТСния.
    3. КабСль большого сСчСния Π½Π΅ΠΎΠ±Ρ…ΠΎΠ΄ΠΈΠΌΠΎ ΠΈΡΠΏΠΎΠ»ΡŒΠ·ΠΎΠ²Π°Ρ‚ΡŒ Ρ‚ΠΎΠ»ΡŒΠΊΠΎ для ΠΏΠΎΠ΄ΠΊΠ»ΡŽΡ‡Π΅Π½ΠΈΡ источника питания (Ρ‡Π΅Ρ€Π΅Π· Ρ€Π΅Π»Π΅) ΠΊ устройству.

    Π—Π°Ρ‡Π΅ΠΌ ΠΈΡΠΏΠΎΠ»ΡŒΠ·ΠΎΠ²Π°Ρ‚ΡŒ Ρ€Π΅Π»Π΅ Π² Π°Π²Ρ‚ΠΎΠΌΠΎΠ±ΠΈΠ»Π΅?

    ИспользованиС Ρ€Π΅Π»Π΅ Π² Π°Π²Ρ‚ΠΎΠΌΠΎΠ±ΠΈΠ»Π΅ Π΄Π°Π΅Ρ‚ нСсколько прСимущСств. Π’ΠΎ-ΠΏΠ΅Ρ€Π²Ρ‹Ρ…, использованиС Ρ€Π΅Π»Π΅ ΠΎΠ·Π½Π°Ρ‡Π°Π΅Ρ‚, Ρ‡Ρ‚ΠΎ схСма ΠΏΠ΅Ρ€Π΅ΠΊΠ»ΡŽΡ‡Π΅Π½ΠΈΡ Π½Π΅ Ρ‚Ρ€Π΅Π±ΡƒΠ΅Ρ‚ ΠΏΠ΅Ρ€Π΅ΠΊΠ»ΡŽΡ‡Π°Ρ‚Π΅Π»Ρ ΠΈΠ»ΠΈ кабСля с высоким Π½ΠΎΠΌΠΈΠ½Π°Π»ΡŒΠ½Ρ‹ΠΌ Ρ‚ΠΎΠΊΠΎΠΌ, Ρ‡Ρ‚ΠΎ сниТаСт ΡΡ‚ΠΎΠΈΠΌΠΎΡΡ‚ΡŒ ΠΈ вСс. Π’ΠΎ-Π²Ρ‚ΠΎΡ€Ρ‹Ρ…, Ρ€Π΅Π»Π΅ ΠΌΠΎΠΆΠ½ΠΎ Ρ€Π°Π·ΠΌΠ΅ΡΡ‚ΠΈΡ‚ΡŒ Π² любом мСстС автомобиля, Ρ‡Ρ‚ΠΎΠ±Ρ‹ ΠΎΠ±Π΅ΡΠΏΠ΅Ρ‡ΠΈΡ‚ΡŒ ΡΡ„Ρ„Π΅ΠΊΡ‚ΠΈΠ²Π½ΡƒΡŽ ΠΏΠ΅Ρ€Π΅Π΄Π°Ρ‡Ρƒ энСргии Π½Π° элСктричСскоС ΠΎΠ±ΠΎΡ€ΡƒΠ΄ΠΎΠ²Π°Π½ΠΈΠ΅, ΠΊΠΎΡ‚ΠΎΡ€Ρ‹ΠΌ Π²Ρ‹ управляСтС. Π Π΅Π»Π΅ идСально подходят для управлСния ΠΌΠ½ΠΎΠ³ΠΈΠΌΠΈ цСпями Π² Π°Π²Ρ‚ΠΎΠΌΠΎΠ±ΠΈΠ»Π΅, Ρ‚Π°ΠΊΠΈΠΌΠΈ ΠΊΠ°ΠΊ Ρ„Π°Ρ€Ρ‹, элСктродвигатСли, ΠΎΠ±ΠΎΠ³Ρ€Π΅Π²Π°Ρ‚Π΅Π»ΡŒ ΠΈ Ρ‚. Π΄.

    Как ΠΏΠΎΠ½ΡΡ‚ΡŒ элСктричСскиС характСристики Ρ€Π΅Π»Π΅

    Π‘Π°Π·ΠΎΠ²Ρ‹Π΅ Ρ€Π΅Π»Π΅ ΠΈΠΌΠ΅ΡŽΡ‚ элСктричСскиС характСристики ΠΊΠ°ΠΊ для ΠΊΠ°Ρ‚ΡƒΡˆΠΊΠΈ, Ρ‚Π°ΠΊ ΠΈ для Π²Π½ΡƒΡ‚Ρ€Π΅Π½Π½ΠΈΡ… ΠΏΠ΅Ρ€Π΅ΠΊΠ»ΡŽΡ‡Π°ΡŽΡ‰ΠΈΡ… ΠΊΠΎΠ½Ρ‚Π°ΠΊΡ‚ΠΎΠ². НоминальноС напряТСниС ΠΊΠ°Ρ‚ΡƒΡˆΠΊΠΈ напряТСниС, Π½Π΅ΠΎΠ±Ρ…ΠΎΠ΄ΠΈΠΌΠΎΠ΅ для ΠΏΡ€Π°Π²ΠΈΠ»ΡŒΠ½ΠΎΠΉ Ρ€Π°Π±ΠΎΡ‚Ρ‹ ΠΊΠ°Ρ‚ΡƒΡˆΠΊΠΈ. Π‘Ρ…Π΅ΠΌΠ° Π²ΠΊΠ»ΡŽΡ‡Π΅Π½ΠΈΡ Ρ€Π΅Π»Π΅ Ρ‚Π°ΠΊΠΆΠ΅ ΠΈΠΌΠ΅Π΅Ρ‚ напряТСниС ΠΈ Π°ΠΌΠΏΠ΅Ρ€ Ρ€Π΅ΠΉΡ‚ΠΈΠ½Π³. Π­Ρ‚ΠΎ максимальноС Π·Π½Π°Ρ‡Π΅Π½ΠΈΠ΅ ΠΊΠΎΠ½Ρ‚Π°ΠΊΡ‚ΠΎΠ² ΠΏΠ΅Ρ€Π΅ΠΊΠ»ΡŽΡ‡Π°Ρ‚Π΅Π»Ρ, ΠΊΠΎΡ‚ΠΎΡ€ΠΎΠ΅ НЕЛЬЗЯ ΠΏΡ€Π΅Π²Ρ‹ΡˆΠ°Ρ‚ΡŒ. Π”Π²ΠΎΠΉΠ½ΠΎΠΉ бросок Ρ€Π΅Π»Π΅ часто ΠΈΠΌΠ΅ΡŽΡ‚ элСктричСскиС характСристики с двумя ΠΏΠ΅Ρ€Π΅ΠΊΠ»ΡŽΡ‡Π°Ρ‚Π΅Π»ΡΠΌΠΈ. Один для Π½ΠΎΡ€ΠΌΠ°Π»ΡŒΠ½ΠΎ Ρ€Π°Π·ΠΎΠΌΠΊΠ½ΡƒΡ‚ΠΎΠΉ ΠΊΠ»Π΅ΠΌΠΌΡ‹, Π΄Ρ€ΡƒΠ³ΠΎΠΉ для Π½ΠΎΡ€ΠΌΠ°Π»ΡŒΠ½ΠΎ Π·Π°ΠΊΡ€Ρ‹Ρ‚Ρ‹ΠΉ Ρ‚Π΅Ρ€ΠΌΠΈΠ½Π°Π». Ρ‚.Π΅. Н/О: 35 А ΠΏΡ€ΠΈ 14 Π’ постоянного Ρ‚ΠΎΠΊΠ°, Н/Π—: 20 А ΠΏΡ€ΠΈ 14 Π’ постоянного Ρ‚ΠΎΠΊΠ°.

    ΠŸΠΎΡ‡Π΅ΠΌΡƒ Π² Ρ€Π΅Π»Π΅ ΠΈΡΠΏΠΎΠ»ΡŒΠ·ΡƒΡŽΡ‚ΡΡ устройства Π·Π°Ρ‰ΠΈΡ‚Ρ‹?

    Π Π΅Π»Π΅ ΠΌΠΎΠ³ΡƒΡ‚ ΡΠΎΠ·Π΄Π°Π²Π°Ρ‚ΡŒ ΡΠΈΠ»ΡŒΠ½Ρ‹Π΅ скачки напряТСния, ΠΊΠΎΠ³Π΄Π° ΠΎΠ½ΠΈ Π²Ρ‹ΠΊΠ»ΡŽΡ‡Π΅Π½Ρ‹ ΠΈΠ·-Π·Π° обСсточивания ΠΊΠ°Ρ‚ΡƒΡˆΠΊΠΈ. РСзисторы ΠΈΠ»ΠΈ Π΄ΠΈΠΎΠ΄Ρ‹ ΠΈΠ½ΠΎΠ³Π΄Π° ΡƒΡΡ‚Π°Π½Π°Π²Π»ΠΈΠ²Π°ΡŽΡ‚ΡΡ Π½Π° ΠΊΠ°Ρ‚ΡƒΡˆΠΊΡƒ Ρ€Π΅Π»Π΅, Ρ‡Ρ‚ΠΎΠ±Ρ‹ ΠΎΡΡ‚Π°Π½ΠΎΠ²ΠΈΡ‚ΡŒ / ΡƒΠΌΠ΅Π½ΡŒΡˆΠΈΡ‚ΡŒ эти всплСски, ΠΈΠ΄ΡƒΡ‰ΠΈΠ΅ ΠΎΠ±Ρ€Π°Ρ‚Π½ΠΎ Π² ΡƒΠΏΡ€Π°Π²Π»Π΅Π½ΠΈΠ΅. Ρ†Π΅ΠΏΠΈ ΠΈ ΠΏΠΎΠ²Ρ€Π΅Π΄ΠΈΡ‚ΡŒ Ρ‡ΡƒΠ²ΡΡ‚Π²ΠΈΡ‚Π΅Π»ΡŒΠ½Ρ‹Π΅ ΠΊΠΎΠΌΠΏΠΎΠ½Π΅Π½Ρ‚Ρ‹. РСзисторы Π±ΠΎΠ»Π΅Π΅ Π΄ΠΎΠ»Π³ΠΎΠ²Π΅Ρ‡Π½Ρ‹, Ρ‡Π΅ΠΌ Π΄ΠΈΠΎΠ΄Ρ‹, Π½ΠΎ Π½Π΅ Ρ‚Π°ΠΊ эффСктивны ΠΏΡ€ΠΈ устранСниС скачков напряТСния. Π’Π°ΠΌ Π½Π΅ΠΎΠ±Ρ…ΠΎΠ΄ΠΈΠΌΠΎ ΠΎΡ†Π΅Π½ΠΈΡ‚ΡŒ Ρ‡ΡƒΠ²ΡΡ‚Π²ΠΈΡ‚Π΅Π»ΡŒΠ½ΠΎΡΡ‚ΡŒ ΠΊΠΎΠΌΠΏΠΎΠ½Π΅Π½Ρ‚ΠΎΠ² Π² Ρ†Π΅ΠΏΠΈ управлСния ΠΏΡ€ΠΈ принятиС Ρ€Π΅ΡˆΠ΅Π½ΠΈΡ ΠΎ Ρ‚ΠΎΠΌ, трСбуСтся Π»ΠΈ / ΠΊΠ°ΠΊΠΎΠΉ Ρ‚ΠΈΠΏ Π·Π°Ρ‰ΠΈΡ‚Ρ‹.

    ΠŸΡ€ΠΈΠΌΠ΅Ρ€Ρ‹ схСм ΠΏΠΎΠ΄ΠΊΠ»ΡŽΡ‡Π΅Π½ΠΈΡ 4-ΠΊΠΎΠ½Ρ‚Π°ΠΊΡ‚Π½ΠΎΠ³ΠΎ Π½ΠΎΡ€ΠΌΠ°Π»ΡŒΠ½ΠΎ Ρ€Π°Π·ΠΎΠΌΠΊΠ½ΡƒΡ‚ΠΎΠ³ΠΎ Ρ€Π΅Π»Π΅

    Π’ Ρ‡Π΅ΠΌ Ρ€Π°Π·Π½ΠΈΡ†Π° ΠΌΠ΅ΠΆΠ΄Ρƒ 4-ΠΊΠΎΠ½Ρ‚Π°ΠΊΡ‚Π½Ρ‹ΠΌ ΠΈ 5-ΠΊΠΎΠ½Ρ‚Π°ΠΊΡ‚Π½Ρ‹ΠΌ Ρ€Π΅Π»Π΅?

    Π Π°Π·Π½ΠΈΡ†Π° ΠΌΠ΅ΠΆΠ΄Ρƒ 4-ΠΊΠΎΠ½Ρ‚Π°ΠΊΡ‚Π½Ρ‹ΠΌ ΠΈ 5-ΠΊΠΎΠ½Ρ‚Π°ΠΊΡ‚Π½Ρ‹ΠΌ Ρ€Π΅Π»Π΅ Π·Π°ΠΊΠ»ΡŽΡ‡Π°Π΅Ρ‚ΡΡ Π² Ρ‚ΠΎΠΌ, Ρ‡Ρ‚ΠΎ 4-ΠΊΠΎΠ½Ρ‚Π°ΠΊΡ‚Π½ΠΎΠ΅ Ρ€Π΅Π»Π΅ ΠΈΡΠΏΠΎΠ»ΡŒΠ·ΡƒΠ΅Ρ‚ΡΡ для управлСния ΠΎΠ΄Π½ΠΈΠΌ Ρ†Π΅ΠΏΠΈ, Ρ‚ΠΎΠ³Π΄Π° ΠΊΠ°ΠΊ 5-ΠΊΠΎΠ½Ρ‚Π°ΠΊΡ‚Π½ΠΎΠ΅ Ρ€Π΅Π»Π΅ ΠΏΠ΅Ρ€Π΅ΠΊΠ»ΡŽΡ‡Π°Π΅Ρ‚ ΠΏΠΈΡ‚Π°Π½ΠΈΠ΅ ΠΌΠ΅ΠΆΠ΄Ρƒ двумя цСпями.

    4-ΠΊΠΎΠ½Ρ‚Π°ΠΊΡ‚Π½ΠΎΠ΅ Ρ€Π΅Π»Π΅
    4-ΠΊΠΎΠ½Ρ‚Π°ΠΊΡ‚Π½ΠΎΠ΅ Ρ€Π΅Π»Π΅ ΠΈΡΠΏΠΎΠ»ΡŒΠ·ΡƒΠ΅Ρ‚ 2 ΠΊΠΎΠ½Ρ‚Π°ΠΊΡ‚Π° (85 ΠΈ 86) для управлСния ΠΊΠ°Ρ‚ΡƒΡˆΠΊΠΎΠΉ ΠΈ 2 ΠΊΠΎΠ½Ρ‚Π°ΠΊΡ‚Π° (30 ΠΈ 87), ΠΊΠΎΡ‚ΠΎΡ€Ρ‹Π΅ ΠΏΠ΅Ρ€Π΅ΠΊΠ»ΡŽΡ‡Π°ΡŽΡ‚ ΠΏΠΈΡ‚Π°Π½ΠΈΠ΅ Π² ΠΎΠ΄Π½ΠΎΠΉ Ρ†Π΅ΠΏΠΈ. Доступны 2 Ρ‚ΠΈΠΏΠ° 4-ΠΊΠΎΠ½Ρ‚Π°ΠΊΡ‚Π½Ρ‹Ρ… Ρ€Π΅Π»Π΅; Π½ΠΎΡ€ΠΌΠ°Π»ΡŒΠ½ΠΎ ΠΎΡ‚ΠΊΡ€Ρ‹Ρ‚Ρ‹ΠΉ ΠΈΠ»ΠΈ Π½ΠΎΡ€ΠΌΠ°Π»ΡŒΠ½ΠΎ Π·Π°ΠΊΡ€Ρ‹Ρ‚Ρ‹ΠΉ. ΠΠΎΡ€ΠΌΠ°Π»ΡŒΠ½ΠΎ Ρ€Π°Π·ΠΎΠΌΠΊΠ½ΡƒΡ‚ΠΎΠ΅ Ρ€Π΅Π»Π΅ Π²ΠΊΠ»ΡŽΡ‡Π°Π΅Ρ‚ ΠΏΠΈΡ‚Π°Π½ΠΈΠ΅ Ρ†Π΅ΠΏΠΈ, ΠΊΠΎΠ³Π΄Π° активируСтся ΠΊΠ°Ρ‚ΡƒΡˆΠΊΠ°. ΠΠΎΡ€ΠΌΠ°Π»ΡŒΠ½ΠΎ Π·Π°ΠΌΠΊΠ½ΡƒΡ‚ΠΎΠ΅ Ρ€Π΅Π»Π΅ ΠΎΡ‚ΠΊΠ»ΡŽΡ‡Π°Π΅Ρ‚ ΠΏΠΈΡ‚Π°Π½ΠΈΠ΅ Ρ†Π΅ΠΏΠΈ, ΠΊΠΎΠ³Π΄Π° активируСтся ΠΊΠ°Ρ‚ΡƒΡˆΠΊΠ°.

    5-ΠΊΠΎΠ½Ρ‚Π°ΠΊΡ‚Π½ΠΎΠ΅ Ρ€Π΅Π»Π΅
    5-ΠΊΠΎΠ½Ρ‚Π°ΠΊΡ‚Π½ΠΎΠ΅ Ρ€Π΅Π»Π΅ ΠΈΠΌΠ΅Π΅Ρ‚ 2 ΠΊΠΎΠ½Ρ‚Π°ΠΊΡ‚Π° (85 ΠΈ 86) для управлСния ΠΊΠ°Ρ‚ΡƒΡˆΠΊΠΎΠΉ ΠΈ 3 ΠΊΠΎΠ½Ρ‚Π°ΠΊΡ‚Π° (30, 87 ΠΈ 87A), ΠΊΠΎΡ‚ΠΎΡ€Ρ‹Π΅ ΠΏΠ΅Ρ€Π΅ΠΊΠ»ΡŽΡ‡Π°ΡŽΡ‚ ΠΏΠΈΡ‚Π°Π½ΠΈΠ΅ ΠΌΠ΅ΠΆΠ΄Ρƒ двумя схСмы. Они ΠΈΠΌΠ΅ΡŽΡ‚ ΠΊΠ°ΠΊ Π½ΠΎΡ€ΠΌΠ°Π»ΡŒΠ½ΠΎ ΠΎΡ‚ΠΊΡ€Ρ‹Ρ‚Ρ‹Π΅, Ρ‚Π°ΠΊ ΠΈ Π½ΠΎΡ€ΠΌΠ°Π»ΡŒΠ½ΠΎ Π·Π°ΠΊΡ€Ρ‹Ρ‚Ρ‹Π΅ ΡΠΎΠ΅Π΄ΠΈΠ½ΠΈΡ‚Π΅Π»ΡŒΠ½Ρ‹Π΅ ΠΊΠΎΠ½Ρ‚Π°ΠΊΡ‚Ρ‹. Когда ΠΊΠ°Ρ‚ΡƒΡˆΠΊΠ° Π°ΠΊΡ‚ΠΈΠ²ΠΈΡ€ΠΎΠ²Π°Π½Π°, ΠΌΠΎΡ‰Π½ΠΎΡΡ‚ΡŒ Π±ΡƒΠ΄Π΅Ρ‚ ΠΏΠ΅Ρ€Π΅ΠΊΠ»ΡŽΡ‡Π°Ρ‚ΡŒΡΡ с Π½ΠΎΡ€ΠΌΠ°Π»ΡŒΠ½ΠΎ Π·Π°ΠΊΡ€Ρ‹Ρ‚ΠΎΠ³ΠΎ ΡˆΡ‚ΠΈΡ„Ρ‚Π° Π½Π° Π½ΠΎΡ€ΠΌΠ°Π»ΡŒΠ½ΠΎ ΠΎΡ‚ΠΊΡ€Ρ‹Ρ‚Ρ‹ΠΉ ΡˆΡ‚ΠΈΡ„Ρ‚.

    Для Ρ‡Π΅Π³ΠΎ ΠΈΡΠΏΠΎΠ»ΡŒΠ·ΡƒΡŽΡ‚ΡΡ Ρ€Π΅Π»Π΅ ISO? Π Π΅Π»Π΅

    ISO ΠΏΡ€Π΅Π΄Π½Π°Π·Π½Π°Ρ‡Π΅Π½Ρ‹ для использования Π² Π°Π²Ρ‚ΠΎΠΌΠΎΠ±ΠΈΠ»ΡŒΠ½ΠΎΠΉ ΠΏΡ€ΠΎΠΌΡ‹ΡˆΠ»Π΅Π½Π½ΠΎΡΡ‚ΠΈ ΠΈ ΠΈΠΌΠ΅ΡŽΡ‚ ΡΡ‚Π°Π½Π΄Π°Ρ€Ρ‚Π½ΡƒΡŽ схСму элСктричСских соСдинСний. Ρ‚Π΅Ρ€ΠΌΠΈΠ½Π°Π»Ρ‹. Π’ Π±ΠΎΠ»Π΅Π΅ Π½ΠΎΠ²Ρ‹Ρ… Ρ€Π΅Π»Π΅ ISO 280 ΠΈΡΠΏΠΎΠ»ΡŒΠ·ΡƒΠ΅Ρ‚ΡΡ ΡˆΡ‚Ρ‹Ρ€Π΅Π²Π°Ρ ΠΊΠ»Π΅ΠΌΠΌΠ° мСньшСго Ρ€Π°Π·ΠΌΠ΅Ρ€Π° ΡˆΠΈΡ€ΠΈΠ½ΠΎΠΉ 2,8 ΠΌΠΌ, ΠΊΠΎΡ‚ΠΎΡ€ΡƒΡŽ ΠΌΠΎΠΆΠ½ΠΎ ΠΈΡΠΏΠΎΠ»ΡŒΠ·ΠΎΠ²Π°Ρ‚ΡŒ Π² ΠΊΠΎΠΌΠΏΠ°ΠΊΡ‚Π½ΠΎΠΌ корпусС. Π±Π»ΠΎΠΊΠΈ распрСдСлСния питания ΠΈ Π΄Π΅Ρ€ΠΆΠ°Ρ‚Π΅Π»ΠΈ.

    Π‘Ρ‚Π°Π½Π΄Π°Ρ€Ρ‚Π½Ρ‹Π΅ Ρ€Π΅Π»Π΅ ISO

    ΠœΡ‹ ΠΏΡ€ΠΎΠ΄Π°Π΅ΠΌ 4-ΠΊΠΎΠ½Ρ‚Π°ΠΊΡ‚Π½Ρ‹Π΅ Π½ΠΎΡ€ΠΌΠ°Π»ΡŒΠ½ΠΎ Ρ€Π°Π·ΠΎΠΌΠΊΠ½ΡƒΡ‚Ρ‹Π΅ (SPST) ΠΈ 5-ΠΊΠΎΠ½Ρ‚Π°ΠΊΡ‚Π½Ρ‹Π΅ ΠΏΠ΅Ρ€Π΅ΠΊΠ»ΡŽΡ‡Π°ΡŽΡ‰ΠΈΠ΅ (SPDT) Ρ€Π΅Π»Π΅ с рСзистором. Π·Π°Ρ‰ΠΈΡ‚Π° Π½Π° 12Π’ ΠΈΠ»ΠΈ 24Π’. Если Π²Π°ΠΌ трСбуСтся диодная Π·Π°Ρ‰ΠΈΡ‚Π°, ΠΎΠ±Ρ€Π°Ρ‚ΠΈΡ‚Π΅ΡΡŒ Π² ΠΎΡ‚Π΄Π΅Π» ΠΏΡ€ΠΎΠ΄Π°ΠΆ.

    ΠœΠΈΠΊΡ€ΠΎΡ€Π΅Π»Π΅ ISO
    (4 ΠΊΠΎΠ½Ρ‚Π°ΠΊΡ‚Π°)

    ΠœΠΈΠΊΡ€ΠΎΡ€Π΅Π»Π΅ ISO
    (5 ΠΊΠΎΠ½Ρ‚Π°ΠΊΡ‚ΠΎΠ²)

    Мини-Ρ€Π΅Π»Π΅ ISO
    (4 ΠΊΠΎΠ½Ρ‚Π°ΠΊΡ‚Π°)

    Мини-Ρ€Π΅Π»Π΅ ISO
    (5 ΠΊΠΎΠ½Ρ‚Π°ΠΊΡ‚ΠΎΠ²)

    Макси-Ρ€Π΅Π»Π΅ ISO
    (4 ΠΊΠΎΠ½Ρ‚Π°ΠΊΡ‚Π°)

    НовыС Ρ€Π΅Π»Π΅ ISO 280

    ΠœΡ‹ ΠΏΡ€ΠΎΠ΄Π°Π΅ΠΌ ΠΊΠ°ΠΊ 4-ΠΊΠΎΠ½Ρ‚Π°ΠΊΡ‚Π½ΠΎΠ΅ Π½ΠΎΡ€ΠΌΠ°Π»ΡŒΠ½ΠΎ Ρ€Π°Π·ΠΎΠΌΠΊΠ½ΡƒΡ‚ΠΎΠ΅ (SPST), Ρ‚Π°ΠΊ ΠΈ 5-ΠΊΠΎΠ½Ρ‚Π°ΠΊΡ‚Π½ΠΎΠ΅ ΠΏΠ΅Ρ€Π΅ΠΊΠ»ΡŽΡ‡Π°ΡŽΡ‰Π΅Π΅ (SPDT) Ρ€Π΅Π»Π΅ с рСзистором. Π·Π°Ρ‰ΠΈΡ‚Π° Π½Π° 12Π’ ΠΈΠ»ΠΈ 24Π’. Если Π²Π°ΠΌ трСбуСтся диодная Π·Π°Ρ‰ΠΈΡ‚Π°, ΠΎΠ±Ρ€Π°Ρ‚ΠΈΡ‚Π΅ΡΡŒ Π² ΠΎΡ‚Π΄Π΅Π» ΠΏΡ€ΠΎΠ΄Π°ΠΆ.

    Π£Π»ΡŒΡ‚Ρ€Π°ΠΌΠΈΠΊΡ€ΠΎΡ€Π΅Π»Π΅ ISO 280
    (4 ΠΊΠΎΠ½Ρ‚Π°ΠΊΡ‚Π°)

    ΠœΠΈΠΊΡ€ΠΎΡ€Π΅Π»Π΅ ISO 280
    (4 ΠΊΠΎΠ½Ρ‚Π°ΠΊΡ‚Π°)

    ΠœΠΈΠΊΡ€ΠΎΡ€Π΅Π»Π΅ ISO 280
    (5 ΠΊΠΎΠ½Ρ‚Π°ΠΊΡ‚ΠΎΠ²)

    Мини-Ρ€Π΅Π»Π΅ ISO 280
    (4 ΠΊΠΎΠ½Ρ‚Π°ΠΊΡ‚Π°)

    Мини-Ρ€Π΅Π»Π΅ ISO
    (5 ΠΊΠΎΠ½Ρ‚Π°ΠΊΡ‚ΠΎΠ²)

    ΠŸΡ€ΠΈΠΌΠ΅Ρ€Ρ‹ схСм ΠΏΠΎΠ΄ΠΊΠ»ΡŽΡ‡Π΅Π½ΠΈΡ Π½ΠΎΡ€ΠΌΠ°Π»ΡŒΠ½ΠΎ Ρ€Π°Π·ΠΎΠΌΠΊΠ½ΡƒΡ‚ΠΎΠ³ΠΎ Ρ€Π΅Π»Π΅

    ΠŸΡ€ΠΈΠΌΠ΅Ρ€ 1. 4-ΠΊΠΎΠ½Ρ‚Π°ΠΊΡ‚Π½ΠΎΠ΅ (Π½ΠΎΡ€ΠΌΠ°Π»ΡŒΠ½ΠΎ Ρ€Π°Π·ΠΎΠΌΠΊΠ½ΡƒΡ‚ΠΎΠ΅) Ρ€Π΅Π»Π΅ с ΠΏΠ΅Ρ€Π΅ΠΊΠ»ΡŽΡ‡Π°Ρ‚Π΅Π»Π΅ΠΌ Π½Π° плюсовой сторонС Ρ†Π΅ΠΏΠΈ управлСния.

    ΠŸΡ€ΠΈΠΌΠ΅Ρ€ 2. 4-ΠΊΠΎΠ½Ρ‚Π°ΠΊΡ‚Π½ΠΎΠ΅ (Π½ΠΎΡ€ΠΌΠ°Π»ΡŒΠ½ΠΎ Ρ€Π°Π·ΠΎΠΌΠΊΠ½ΡƒΡ‚ΠΎΠ΅) Ρ€Π΅Π»Π΅ с ΠΏΠ΅Ρ€Π΅ΠΊΠ»ΡŽΡ‡Π°Ρ‚Π΅Π»Π΅ΠΌ Π½Π° ΠΎΡ‚Ρ€ΠΈΡ†Π°Ρ‚Π΅Π»ΡŒΠ½ΠΎΠΉ сторонС Ρ†Π΅ΠΏΠΈ управлСния.

    ΠŸΡ€ΠΈΠΌΠ΅Ρ‡Π°Π½ΠΈΠ΅. Π­Ρ‚ΠΈ схСмы Π±Ρ‹Π»ΠΈ ΡƒΠΏΡ€ΠΎΡ‰Π΅Π½Ρ‹, Ρ‡Ρ‚ΠΎΠ±Ρ‹ ΠΏΡ€ΠΎΠΈΠ»Π»ΡŽΡΡ‚Ρ€ΠΈΡ€ΠΎΠ²Π°Ρ‚ΡŒ Ρ„ΡƒΠ½ΠΊΡ†ΠΈΡŽ Ρ€Π΅Π»Π΅ ΠΈ, ΡΠ»Π΅Π΄ΠΎΠ²Π°Ρ‚Π΅Π»ΡŒΠ½ΠΎ, Π½Π΅ Π²ΠΊΠ»ΡŽΡ‡Π°ΡŽΡ‚ ΠΏΡ€Π΅Π΄ΠΎΡ…Ρ€Π°Π½ΠΈΡ‚Π΅Π»ΠΈ. Π·Π°Ρ‰ΠΈΡ‚Ρ‹, которая потрСбуСтся. ΠšΠ»Π΅ΠΌΠΌΡ‹ ΠΊΠ°Ρ‚ΡƒΡˆΠΊΠΈ Ρ€Π΅Π»Π΅ Π½Π΅ ΠΈΠΌΠ΅ΡŽΡ‚ полярности, Ссли Ρ‚ΠΎΠ»ΡŒΠΊΠΎ ΠΊΠ°Ρ‚ΡƒΡˆΠΊΠ° Ρ€Π΅Π»Π΅ Π½Π΅ Π·Π°Ρ‰ΠΈΡ‰Π΅Π½Π° Π΄ΠΈΠΎΠ΄Π° (Π²Π½ΡƒΡ‚Ρ€ΠΈ Ρ€Π΅Π»Π΅), ΠΈ Π² этом случаС ΠΊΠ»Π΅ΠΌΠΌΠ° ΠΊΠ°Ρ‚ΡƒΡˆΠΊΠΈ, ΠΏΠΎΠ΄ΠΊΠ»ΡŽΡ‡Π΅Π½Π½Π°Ρ ΠΊ Π°Π½ΠΎΠ΄Ρƒ Π΄ΠΈΠΎΠ΄Π°, Π΄ΠΎΠ»ΠΆΠ½Π° Π±Ρ‹Ρ‚ΡŒ ΠΏΠΎΠ΄ΠΊΠ»ΡŽΡ‡Π΅Π½Π° ΠΊ минусу.

    Π”Π΅Ρ€ΠΆΠ°Ρ‚Π΅Π»ΠΈ Ρ€Π΅Π»Π΅

    ΠœΡ‹ ΠΏΡ€ΠΎΠ΄Π°Π΅ΠΌ ΡˆΠΈΡ€ΠΎΠΊΠΈΠΉ ассортимСнт Π΄Π΅Ρ€ΠΆΠ°Ρ‚Π΅Π»Π΅ΠΉ для Ρ€Π΅Π»Π΅ ISO ΠΈ ISO 280, ΠΊΠΎΡ‚ΠΎΡ€Ρ‹Π΅ подходят для ΠΎΠ΄Π½ΠΎΠ³ΠΎ ΠΈΠ»ΠΈ Π½Π΅ΡΠΊΠΎΠ»ΡŒΠΊΠΈΡ… Ρ€Π΅Π»Π΅. Ρ€Π΅Π»Π΅. Π£ нас Ρ‚Π°ΠΊΠΆΠ΅ Π΅ΡΡ‚ΡŒ большой Π²Ρ‹Π±ΠΎΡ€ Π±Π»ΠΎΠΊΠΎΠ² распрСдСлСния питания, ΠΊΠΎΡ‚ΠΎΡ€Ρ‹Π΅ ΠΌΠΎΠ³ΡƒΡ‚ Π±Ρ‹Ρ‚ΡŒ оснащСны смСсь Ρ€Π΅Π»Π΅, ΠΏΡ€Π΅Π΄ΠΎΡ…Ρ€Π°Π½ΠΈΡ‚Π΅Π»Π΅ΠΉ ΠΈ автоматичСских Π²Ρ‹ΠΊΠ»ΡŽΡ‡Π°Ρ‚Π΅Π»Π΅ΠΉ.

    ΠŸΠΎΡΠΌΠΎΡ‚Ρ€Π΅Ρ‚ΡŒ наши Π΄Π΅Ρ€ΠΆΠ°Ρ‚Π΅Π»ΠΈ Ρ€Π΅Π»Π΅ ISO

    ΠŸΠΎΡΠΌΠΎΡ‚Ρ€ΠΈΡ‚Π΅ наши Π΄Π΅Ρ€ΠΆΠ°Ρ‚Π΅Π»ΠΈ Ρ€Π΅Π»Π΅ ISO 280

    Π‘Π°ΠΌΡ‹ΠΉ большой Π΄ΠΈΠ°ΠΏΠ°Π·ΠΎΠ½ Β Β β€’ Π’Ρ‹ΡΡˆΠ΅Π΅ качСство β€’ ΠšΠΎΠ½ΠΊΡƒΡ€Π΅Π½Ρ‚Π½ΠΎΠ΅ Ρ†Π΅Π½ΠΎΠΎΠ±Ρ€Π°Π·ΠΎΠ²Π°Π½ΠΈΠ΅ β€’ БыстроС обслуТиваниС Β Β β€’ ΠœΠΈΡ€ΠΎΠ²Ρ‹Π΅ Π±Ρ€Π΅Π½Π΄Ρ‹

    Π‘Ρ…Π΅ΠΌΠ° Π°Π²Ρ‚ΠΎΠΌΠΎΠ±ΠΈΠ»ΡŒΠ½ΠΎΠ³ΠΎ Ρ€Π΅Π»Π΅! Π‘Ρ…Π΅ΠΌΠ° ΠΏΠΎΠ΄ΠΊΠ»ΡŽΡ‡Π΅Π½ΠΈΡ 4- ΠΈ 5-ΠΊΠΎΠ½Ρ‚Π°ΠΊΡ‚Π½ΠΎΠ³ΠΎ Ρ€Π΅Π»Π΅

    ΠΠ²Ρ‚ΠΎΠΌΠΎΠ±ΠΈΠ»ΡŒΠ½ΠΎΠ΅ Ρ€Π΅Π»Π΅ прСдставляСт собой элСктричСский ΠΏΠ΅Ρ€Π΅ΠΊΠ»ΡŽΡ‡Π°Ρ‚Π΅Π»ΡŒ, ΠΊΠΎΡ‚ΠΎΡ€Ρ‹ΠΉ Π²ΠΊΠ»ΡŽΡ‡Π°Π΅Ρ‚ΡΡ ΠΈ Π²Ρ‹ΠΊΠ»ΡŽΡ‡Π°Π΅Ρ‚ΡΡ с ΠΏΠΎΠΌΠΎΡ‰ΡŒΡŽ элСктричСского Ρ‚Ρ€ΠΈΠ³Π³Π΅Ρ€Π°. Он состоит ΠΈΠ· Π΄Π²ΡƒΡ… Ρ†Π΅ΠΏΠ΅ΠΉ, ΠΊΠ°Ρ‚ΡƒΡˆΠΊΠΈ ΠΈ высокоампСрной Ρ†Π΅ΠΏΠΈ. КаТдая Ρ†Π΅ΠΏΡŒ ΠΈΠΌΠ΅Π΅Ρ‚ свои Π½ΠΎΠΌΠ΅Ρ€Π° ΠΊΠΎΠ½Ρ‚Π°ΠΊΡ‚ΠΎΠ² ΠΈ схСму ΠΏΠΎΠ΄ΠΊΠ»ΡŽΡ‡Π΅Π½ΠΈΡ.

    НиТС ΠΏΡ€ΠΈΠ²Π΅Π΄Π΅Π½ΠΎ ΠΏΠΎΠ»Π½ΠΎΠ΅ руководство ΠΏΠΎ Π°Π²Ρ‚ΠΎΠΌΠΎΠ±ΠΈΠ»ΡŒΠ½Ρ‹ΠΌ схСмам Ρ€Π΅Π»Π΅, Ρ‚Π°ΠΊΠΈΠΌ ΠΊΠ°ΠΊ Ρ‡Π΅Ρ‚Ρ‹Ρ€Π΅Ρ…- ΠΈ пятиконтактная схСма ΠΏΠΎΠ΄ΠΊΠ»ΡŽΡ‡Π΅Π½ΠΈΡ Ρ€Π΅Π»Π΅, Π½Π° понятном языкС.

    Π‘ΠΎΠ΄Π΅Ρ€ΠΆΠ°Π½ΠΈΠ΅

    Π‘Ρ…Π΅ΠΌΠ° Π°Π²Ρ‚ΠΎΠΌΠΎΠ±ΠΈΠ»ΡŒΠ½ΠΎΠ³ΠΎ Ρ€Π΅Π»Π΅

    DIN-Standard-Code-For-Relay-Terminals

    Π‘Ρ…Π΅ΠΌΠ° ΠΏΠΎΠ΄ΠΊΠ»ΡŽΡ‡Π΅Π½ΠΈΡ Π°Π²Ρ‚ΠΎΠΌΠΎΠ±ΠΈΠ»ΡŒΠ½ΠΎΠ³ΠΎ Ρ€Π΅Π»Π΅ Π½Π΅ слоТна, это ΠΎΡ‡Π΅Π½ΡŒ просто. ВсС, Ρ‡Ρ‚ΠΎ Π²Π°ΠΌ Π½ΡƒΠΆΠ½ΠΎ Π·Π½Π°Ρ‚ΡŒ, это основы схСмы ΠΏΠΎΠ΄ΠΊΠ»ΡŽΡ‡Π΅Π½ΠΈΡ Π°Π²Ρ‚ΠΎΠΌΠΎΠ±ΠΈΠ»ΡŒΠ½ΠΎΠ³ΠΎ Ρ€Π΅Π»Π΅.

    Π—Π΄Π΅ΡΡŒ, Π² этом Ρ€Π°Π·Π΄Π΅Π»Π΅, я объясняю схСму ΠΏΠΎΠ΄ΠΊΠ»ΡŽΡ‡Π΅Π½ΠΈΡ Ρ‡Π΅Ρ‚Ρ‹Ρ€Π΅Ρ…- ΠΈ пятиконтактного Π°Π²Ρ‚ΠΎΠΌΠΎΠ±ΠΈΠ»ΡŒΠ½ΠΎΠ³ΠΎ Ρ€Π΅Π»Π΅. ΠžΡΡ‚Π°Π²Π°ΠΉΡ‚Π΅ΡΡŒ со ΠΌΠ½ΠΎΠΉ, здСсь я расскаТу Π²Π°ΠΌ Π½Π΅ΠΌΠ½ΠΎΠ³ΠΎ ΠΎΠ± Π°Π²Ρ‚ΠΎΠΌΠΎΠ±ΠΈΠ»ΡŒΠ½Ρ‹Ρ… Ρ€Π΅Π»Π΅ΠΉΠ½Ρ‹Ρ… ΠΊΠ»Π΅ΠΌΠΌΠ°Ρ….

    Π’ ΠΏΡ€ΠΎΡˆΠ»ΠΎΠΌ Π²Π΅ΠΊΠ΅ Π² Π“Π΅Ρ€ΠΌΠ°Π½ΠΈΠΈ Π±Ρ‹Π»Π° создана организация DIN Standard. Π­Ρ‚ΠΎ Ρ€Π°ΡΡˆΠΈΡ„Ρ€ΠΎΠ²Ρ‹Π²Π°Π΅Ρ‚ΡΡ ΠΊΠ°ΠΊ Deutsches Institut fΓΌr Normung, Ρ‡Ρ‚ΠΎ ΠΎΠ·Π½Π°Ρ‡Π°Π΅Ρ‚ НСмСцкий институт стандартизации. Π­Ρ‚Π° Π½Π°Ρ†ΠΈΠΎΠ½Π°Π»ΡŒΠ½Π°Ρ организация присваиваСт стандартныС Π½ΠΎΠΌΠ΅Ρ€Π° (извСстный стандарт DIN) практичСски всСм областям Ρ‚Π΅Ρ…Π½ΠΈΠΊΠΈ, охватывая ΠΏΠΎΡ‡Ρ‚ΠΈ Ρ‚Ρ€ΠΈΠ΄Ρ†Π°Ρ‚ΡŒ тысяч Π½ΠΎΠΌΠ΅Ρ€ΠΎΠ² DIN. ΠžΡ€Π³Π°Π½ΠΈΠ·Π°Ρ†ΠΈΡ DIN Ρ‚Π°ΠΊΠΆΠ΅ Π΄Π°Π΅Ρ‚ стандартный ΠΊΠΎΠ΄ DIN для Π°Π²Ρ‚ΠΎΠΌΠΎΠ±ΠΈΠ»Π΅ΠΉ.

    Код стандарта для элСктричСских ΠΊΠ»Π΅ΠΌΠΌ извСстСн ΠΊΠ°ΠΊ стандарт DIN 72552 ΠΈ ΠΏΠΎΡ‡Ρ‚ΠΈ принят Π²ΠΎ всСм ΠΌΠΈΡ€Π΅. DIN стандартизируСт ΠΏΠΎΡ‡Ρ‚ΠΈ ΠΊΠ°ΠΆΠ΄Ρ‹ΠΉ Ρ‚Π΅Ρ€ΠΌΠΈΠ½Π°Π» Π² Π°Π²Ρ‚ΠΎΠΌΠΎΠ±ΠΈΠ»Π΅ с ΠΊΠΎΠ΄ΠΎΠΌ. Π—Π΄Π΅ΡΡŒ ΠΌΡ‹ обсудим Ρ‚ΠΎΠ»ΡŒΠΊΠΎ ΠΊΠ»Π΅ΠΌΠΌΡ‹ Ρ€Π΅Π»Π΅. НиТС прСдставлСна ​​схСма ΠΏΠΎΠ΄ΠΊΠ»ΡŽΡ‡Π΅Π½ΠΈΡ Ρ‡Π΅Ρ‚Ρ‹Ρ€Π΅Ρ…- ΠΈ пятиконтактного Π°Π²Ρ‚ΠΎΠΌΠΎΠ±ΠΈΠ»ΡŒΠ½ΠΎΠ³ΠΎ Ρ€Π΅Π»Π΅.

    5-контактная схСма ΠΏΠΎΠ΄ΠΊΠ»ΡŽΡ‡Π΅Π½ΠΈΡ Ρ€Π΅Π»Π΅ Π½Π° корпусС Ρ€Π΅Π»Π΅

    Π’ соотвСтствии со стандартом DIN 72552 ΠΊΠ°ΠΆΠ΄Ρ‹ΠΉ ΠΊΠΎΠ½Ρ‚Π°ΠΊΡ‚ Ρ€Π΅Π»Π΅ ΠΈΠΌΠ΅Π΅Ρ‚ Π½ΠΎΠΌΠ΅Ρ€Π° 85, 86, 30, 87 ΠΈ 87a.

    Π’Π°ΠΌ Π½Π΅ΠΎΠ±Ρ…ΠΎΠ΄ΠΈΠΌΠΎ Π·Π½Π°Ρ‚ΡŒ, Ρ‡Ρ‚ΠΎ Ρ€Π΅Π»Π΅ ΠΈΠΌΠ΅Π΅Ρ‚ Π΄Π²Π΅ Ρ†Π΅ΠΏΠΈ, Ρ†Π΅ΠΏΡŒ ΠΊΠ°Ρ‚ΡƒΡˆΠΊΠΈ ΠΈ Ρ†Π΅ΠΏΡŒ высокого Ρ‚ΠΎΠΊΠ°. Π’ Ρ€Π΅Π»Π΅ ΠΊΠΎΠ½Ρ‚Π°ΠΊΡ‚Ρ‹ 85 ΠΈ 86 ΡΡ‡ΠΈΡ‚Π°ΡŽΡ‚ΡΡ ΠΊΠΎΠ½Ρ‚Π°ΠΊΡ‚Π°ΠΌΠΈ Ρ†Π΅ΠΏΠΈ ΠΊΠ°Ρ‚ΡƒΡˆΠΊΠΈ, Π° ΠΊΠΎΠ½Ρ‚Π°ΠΊΡ‚Ρ‹ 30, 87 ΠΈ 87a ΡΡ‡ΠΈΡ‚Π°ΡŽΡ‚ΡΡ цСпями с высокой силой Ρ‚ΠΎΠΊΠ°.

    НомСра Π²Ρ‹Π²ΠΎΠ΄ΠΎΠ² Ρ†Π΅ΠΏΠΈ ΠΊΠ°Ρ‚ΡƒΡˆΠΊΠΈ

    ΠžΠ±Ρ‹Ρ‡Π½ΠΎ ΠΊΠ»Π΅ΠΌΠΌΠ° -85 ΠΏΠΎΠ΄ΠΊΠ»ΡŽΡ‡Π°Π΅Ρ‚ΡΡ ΠΊ Π·Π°Π·Π΅ΠΌΠ»Π΅Π½Π½ΠΎΠΌΡƒ источнику питания, Π° ΠΊΠ»Π΅ΠΌΠΌΠ° +86 — ΠΊ источнику горячСго питания для создания ΠΏΠΎΠ»Π½ΠΎΠΉ Ρ†Π΅ΠΏΠΈ.

    НомСра ΠΊΠΎΠ½Ρ‚Π°ΠΊΡ‚ΠΎΠ² Ρ†Π΅ΠΏΠΈ высокого Ρ‚ΠΎΠΊΠ°

    КлСмма 30 считаСтся ΠΎΠ±Ρ‰Π΅ΠΉ ΠΊΠ»Π΅ΠΌΠΌΠΎΠΉ. Π§Π΅Ρ€Π΅Π· эту ΠΊΠ»Π΅ΠΌΠΌΡƒ Π½Π° Ρ€Π΅Π»Π΅ поступаСт 12-Π²ΠΎΠ»ΡŒΡ‚ΠΎΠ²ΠΎΠ΅ ΠΏΠΈΡ‚Π°Π½ΠΈΠ΅ (ΠΌΠΎΠΆΠ½ΠΎ ΡΠΊΠ°Π·Π°Ρ‚ΡŒ Power-In Wire).

    И Π²Ρ‚ΠΎΡ€ΠΎΠΉ ΠΏΡ€ΠΎΠ²ΠΎΠ΄ ΠΊΠ»Π΅ΠΌΠΌΡ‹ 87 Ρ†Π΅ΠΏΠΈ высокой силы Ρ‚ΠΎΠΊΠ° ΠΏΠΎΠ»ΡƒΡ‡Π°Π΅Ρ‚ ΠΏΠΈΡ‚Π°Π½ΠΈΠ΅ ΠΎΡ‚ Ρ€Π΅Π»Π΅ ΠΈ посылаСт Π΅Π³ΠΎ Π½Π° ΠΊΠΎΠΌΠΏΠΎΠ½Π΅Π½Ρ‚ (ΠΌΠΎΠΆΠ½ΠΎ ΡΠΊΠ°Π·Π°Ρ‚ΡŒ, Power out Wire).

    БущСствуСт Ρ‚Π°ΠΊΠΆΠ΅ Ρ‚Ρ€Π΅Ρ‚ΠΈΠΉ ΠΏΡ€ΠΎΠ²ΠΎΠ΄, ΠΊΠΎΡ‚ΠΎΡ€Ρ‹ΠΉ Π²Ρ…ΠΎΠ΄ΠΈΡ‚ Π² ΡΠΈΠ»ΠΎΠ²ΡƒΡŽ Ρ†Π΅ΠΏΡŒ 5-ΠΊΠΎΠ½Ρ‚Π°ΠΊΡ‚Π½ΠΎΠ³ΠΎ Ρ€Π΅Π»Π΅, ΠΊΠΎΡ‚ΠΎΡ€Ρ‹ΠΉ называСтся ΠΊΠ»Π΅ΠΌΠΌΠΎΠΉ 87a. Π­Ρ‚ΠΎ Π΄ΠΎΠΏΠΎΠ»Π½ΠΈΡ‚Π΅Π»ΡŒΠ½Π°Ρ пятая ΠΊΠ»Π΅ΠΌΠΌΠ°, которая ΠΎΠ±Ρ‹Ρ‡Π½ΠΎ Π½Π΅ ΠΈΡΠΏΠΎΠ»ΡŒΠ·ΡƒΠ΅Ρ‚ΡΡ Π² Π°Π²Ρ‚ΠΎΠΌΠΎΠ±ΠΈΠ»ΡŒΠ½Ρ‹Ρ… Ρ€Π΅Π»Π΅. Он ΠΈΡΠΏΠΎΠ»ΡŒΠ·ΡƒΠ΅Ρ‚ΡΡ для Ρ†Π΅ΠΏΠ΅ΠΉ, ΠΊΠΎΡ‚ΠΎΡ€Ρ‹ΠΌ трСбуСтся ΠΏΠΈΡ‚Π°Π½ΠΈΠ΅, ΠΊΠΎΠ³Π΄Π° Ρ€Π΅Π»Π΅ Π½Π΅ Π°ΠΊΡ‚ΠΈΠ²ΠΈΡ€ΠΎΠ²Π°Π½ΠΎ.

    Π‘Ρ…Π΅ΠΌΠ° ΠΏΠΎΠ΄ΠΊΠ»ΡŽΡ‡Π΅Π½ΠΈΡ 4-ΠΊΠΎΠ½Ρ‚Π°ΠΊΡ‚Π½ΠΎΠ³ΠΎ Ρ€Π΅Π»Π΅

    Π‘Ρ…Π΅ΠΌΠ° ΠΏΠΎΠ΄ΠΊΠ»ΡŽΡ‡Π΅Π½ΠΈΡ Π°Π²Ρ‚ΠΎΠΌΠΎΠ±ΠΈΠ»ΡŒΠ½ΠΎΠ³ΠΎ Ρ€Π΅Π»Π΅

    Π‘Ρ…Π΅ΠΌΠ° ΠΏΠΎΠ΄ΠΊΠ»ΡŽΡ‡Π΅Π½ΠΈΡ 4-ΠΊΠΎΠ½Ρ‚Π°ΠΊΡ‚Π½ΠΎΠ³ΠΎ Ρ€Π΅Π»Π΅

    Π§Π΅Ρ‚Ρ‹Ρ€Π΅Ρ…ΠΊΠΎΠ½Ρ‚Π°ΠΊΡ‚Π½ΠΎΠ΅ Ρ€Π΅Π»Π΅ ΠΈΠΌΠ΅Π΅Ρ‚ Ρ‡Π΅Ρ‚Ρ‹Ρ€Π΅ ΠΊΠΎΠ½Ρ‚Π°ΠΊΡ‚Π°, Π΄Π²Π° ΠΊΠΎΠ½Ρ‚Π°ΠΊΡ‚Π° для Ρ†Π΅ΠΏΠ΅ΠΉ ΠΊΠ°Ρ‚ΡƒΡˆΠΊΠΈ ΠΈ Π΄Π²Π° ΠΊΠΎΠ½Ρ‚Π°ΠΊΡ‚Π° для Ρ†Π΅ΠΏΠ΅ΠΉ с высоким Ρ‚ΠΎΠΊΠΎΠΌ. Π”Π²Π΅ ΠΊΠ»Π΅ΠΌΠΌΡ‹ Ρ†Π΅ΠΏΠΈ ΠΊΠ°Ρ‚ΡƒΡˆΠΊΠΈ β€” 85 ΠΈ 86, Π³Π΄Π΅ ΠΊΠ»Π΅ΠΌΠΌΠ° 85 β€” ΠΎΡ‚Ρ€ΠΈΡ†Π°Ρ‚Π΅Π»ΡŒΠ½Ρ‹ΠΉ ΠΊΠΎΠ½Ρ‚Π°ΠΊΡ‚ ΠΊΠ°Ρ‚ΡƒΡˆΠΊΠΈ Ρ€Π΅Π»Π΅, Π° ΠΊΠ»Π΅ΠΌΠΌΠ° 86 β€” ΠΏΠΎΠ»ΠΎΠΆΠΈΡ‚Π΅Π»ΡŒΠ½Ρ‹ΠΉ ΠΊΠΎΠ½Ρ‚Π°ΠΊΡ‚ ΠΊΠ°Ρ‚ΡƒΡˆΠΊΠΈ Ρ€Π΅Π»Π΅.

    Аналогично, Π΄Π²Π΅ ΠΊΠ»Π΅ΠΌΠΌΡ‹ высокоампСрной Ρ†Π΅ΠΏΠΈ Ρ€Π΅Π»Π΅ — 30 ΠΈ 87, Π² ΠΊΠΎΡ‚ΠΎΡ€ΠΎΠΉ ΠΊΠ»Π΅ΠΌΠΌΠ° 30 являСтся ΠΎΠ±Ρ‰ΠΈΠΌ ΠΊΠΎΠ½Ρ‚Π°ΠΊΡ‚ΠΎΠΌ, Ρ‡Ρ‚ΠΎ ΠΎΠ·Π½Π°Ρ‡Π°Π΅Ρ‚, Ρ‡Ρ‚ΠΎ это Π²Ρ…ΠΎΠ΄Π½ΠΎΠΉ ΠΊΠΎΠ½Ρ‚Π°ΠΊΡ‚, Ρ‡Π΅Ρ€Π΅Π· эту ΠΊΠ»Π΅ΠΌΠΌΡƒ Ρ‚ΠΎΠΊ поступаСт Π² Ρ€Π΅Π»Π΅, нСзависимо ΠΎΡ‚ Ρ‚ΠΎΠ³ΠΎ, ΠΏΠΎΠ»ΠΎΠΆΠΈΡ‚Π΅Π»ΡŒΠ½Ρ‹ΠΉ ΠΎΠ½ ΠΈΠ»ΠΈ ΠΎΡ‚Ρ€ΠΈΡ†Π°Ρ‚Π΅Π»ΡŒΠ½Ρ‹ΠΉ, зависит ΠΎΡ‚ спрос Π½Π° источник питания.

    КлСмма А 87 являСтся Π½ΠΎΡ€ΠΌΠ°Π»ΡŒΠ½ΠΎ Ρ€Π°Π·ΠΎΠΌΠΊΠ½ΡƒΡ‚Ρ‹ΠΌ ΠΊΠΎΠ½Ρ‚Π°ΠΊΡ‚ΠΎΠΌ ΠΈ замыкаСтся ΠΏΡ€ΠΈ ΠΏΠΎΠ΄Π°Ρ‡Π΅ питания Π½Π° Ρ€Π΅Π»Π΅. Π•Π³ΠΎ Ρ‚Π°ΠΊΠΆΠ΅ Π½Π°Π·Ρ‹Π²Π°ΡŽΡ‚ Π²Ρ‹Ρ…ΠΎΠ΄Π½ΠΎΠΉ ΠΊΠ»Π΅ΠΌΠΌΠΎΠΉ, Ρ‡Ρ‚ΠΎ ΠΎΠ·Π½Π°Ρ‡Π°Π΅Ρ‚, Ρ‡Ρ‚ΠΎ Ρ‚ΠΎΠΊΠΈ выходят ΠΈΠ· Ρ€Π΅Π»Π΅ Ρ‡Π΅Ρ€Π΅Π· эту ΠΊΠ»Π΅ΠΌΠΌΡƒ Π½Π° ΠΊΠΎΠΌΠΏΠΎΠ½Π΅Π½Ρ‚.

    Π‘Ρ…Π΅ΠΌΠ° ΠΏΠΎΠ΄ΠΊΠ»ΡŽΡ‡Π΅Π½ΠΈΡ 5-ΠΊΠΎΠ½Ρ‚Π°ΠΊΡ‚Π½ΠΎΠ³ΠΎ Ρ€Π΅Π»Π΅

    Π‘Ρ…Π΅ΠΌΠ° ΠΏΠΎΠ΄ΠΊΠ»ΡŽΡ‡Π΅Π½ΠΈΡ 5-ΠΊΠΎΠ½Ρ‚Π°ΠΊΡ‚Π½ΠΎΠ³ΠΎ Ρ€Π΅Π»Π΅

    Π’ пятиконтактном Ρ€Π΅Π»Π΅ имССтся ΠΏΡΡ‚ΡŒ Π²Ρ‹Π²ΠΎΠ΄ΠΎΠ², Π΄Π²Π° ΠΈΠ· ΠΊΠΎΡ‚ΠΎΡ€Ρ‹Ρ… ΠΏΡ€Π΅Π΄Π½Π°Π·Π½Π°Ρ‡Π΅Π½Ρ‹ для Ρ†Π΅ΠΏΠΈ ΠΊΠ°Ρ‚ΡƒΡˆΠΊΠΈ, Π° Ρ‚Ρ€ΠΈ β€” для Ρ†Π΅ΠΏΠΈ высокого Ρ‚ΠΎΠΊΠ°. Π”Π²Π° ΠΏΡ€ΠΎΠ²ΠΎΠ΄Π° Ρ†Π΅ΠΏΠΈ ΠΊΠ°Ρ‚ΡƒΡˆΠΊΠΈ — это ΠΊΠ»Π΅ΠΌΠΌΡ‹ 85 ΠΈ 86, Π² ΠΊΠΎΡ‚ΠΎΡ€Ρ‹Ρ… ΠΊΠ»Π΅ΠΌΠΌΠ° 85 считаСтся ΠΎΡ‚Ρ€ΠΈΡ†Π°Ρ‚Π΅Π»ΡŒΠ½Ρ‹ΠΌ ΠΊΠΎΠ½Ρ‚Π°ΠΊΡ‚ΠΎΠΌ, Π° ΠΊΠ»Π΅ΠΌΠΌΠ° 86 считаСтся ΠΏΠΎΠ»ΠΎΠΆΠΈΡ‚Π΅Π»ΡŒΠ½Ρ‹ΠΌ ΠΊΠΎΠ½Ρ‚Π°ΠΊΡ‚ΠΎΠΌ.

    ЦСпь большой силы Ρ‚ΠΎΠΊΠ° пятиконтактного Ρ€Π΅Π»Π΅ состоит ΠΈΠ· Ρ‚Ρ€Π΅Ρ… ΠΊΠ»Π΅ΠΌΠΌ 30, 87 ΠΈ 87Π°. КлСмма 30 являСтся ΠΎΠ±Ρ‰ΠΈΠΌ ΠΊΠΎΠ½Ρ‚Π°ΠΊΡ‚ΠΎΠΌ, ΠΏΠΈΡ‚Π°Π½ΠΈΠ΅ ΠΎΡ‚ аккумулятора поступаСт Ρ‡Π΅Ρ€Π΅Π· эту ΠΊΠ»Π΅ΠΌΠΌΡƒ Π½Π° аккумулятор. КлСмма 87 являСтся Ρ€Π°Π·ΠΎΠΌΠΊΠ½ΡƒΡ‚Ρ‹ΠΌ ΠΊΠΎΠ½Ρ‚Π°ΠΊΡ‚ΠΎΠΌ ΠΈ всСгда остаСтся Ρ€Π°Π·ΠΎΠΌΠΊΠ½ΡƒΡ‚ΠΎΠΉ, замыкаСтся ΠΏΡ€ΠΈ ΠΏΠΎΠ΄Π°Ρ‡Π΅ питания Π½Π° Ρ€Π΅Π»Π΅. Π§Π΅Ρ€Π΅Π· эту ΠΊΠ»Π΅ΠΌΠΌΡƒ ΠΏΠΈΡ‚Π°Π½ΠΈΠ΅ поступаСт Π½Π° ΠΊΠΎΠΌΠΏΠΎΠ½Π΅Π½Ρ‚, ΠΊΠΎΠ³Π΄Π° Ρ€Π΅Π»Π΅ находится ΠΏΠΎΠ΄ напряТСниСм.

    Аналогично, ΠΊΠ»Π΅ΠΌΠΌΠ° 87a являСтся Π½ΠΎΡ€ΠΌΠ°Π»ΡŒΠ½ΠΎ Π·Π°ΠΌΠΊΠ½ΡƒΡ‚Ρ‹ΠΌ ΠΊΠΎΠ½Ρ‚Π°ΠΊΡ‚ΠΎΠΌ, ΠΈ Π΅Π΅ ΠΊΠΎΠ½Ρ‚Π°ΠΊΡ‚Ρ‹ Ρ€Π°Π·ΠΌΡ‹ΠΊΠ°ΡŽΡ‚ΡΡ, ΠΊΠΎΠ³Π΄Π° Π½Π° Ρ€Π΅Π»Π΅ подаСтся ΠΏΠΈΡ‚Π°Π½ΠΈΠ΅. Π­Ρ‚ΠΎ ΠΎΠ·Π½Π°Ρ‡Π°Π΅Ρ‚, Ρ‡Ρ‚ΠΎ ΠΏΠΎ ΡƒΠΌΠΎΠ»Ρ‡Π°Π½ΠΈΡŽ ΠΎΠ½ пропускаСт Ρ‚ΠΎΠΊ Π±Π΅Π· Π²ΠΊΠ»ΡŽΡ‡Π΅Π½ΠΈΡ Ρ€Π΅Π»Π΅ ΠΈ останавливаСт Ρ‚ΠΎΠΊ, ΠΊΠΎΠ³Π΄Π° Ρ€Π΅Π»Π΅ находится ΠΏΠΎΠ΄ напряТСниСм. Он ΠΈΡΠΏΠΎΠ»ΡŒΠ·ΡƒΠ΅Ρ‚ΡΡ Π² прилоТСниях, Π³Π΄Π΅ трСбуСтся Ρ‚ΠΎΠΊ, ΠΊΠΎΠ³Π΄Π° Ρ€Π΅Π»Π΅ Π½Π΅ находится ΠΏΠΎΠ΄ напряТСниСм.

    Π‘Ρ…Π΅ΠΌΠ° ΠΏΠΎΠ΄ΠΊΠ»ΡŽΡ‡Π΅Π½ΠΈΡ 4-ΠΊΠΎΠ½Ρ‚Π°ΠΊΡ‚Π½ΠΎΠ³ΠΎ Ρ€Π΅Π»Π΅ для освСщСния

    Π‘Ρ…Π΅ΠΌΠ° ΠΏΠΎΠ΄ΠΊΠ»ΡŽΡ‡Π΅Π½ΠΈΡ 4-ΠΊΠΎΠ½Ρ‚Π°ΠΊΡ‚Π½ΠΎΠ³ΠΎ Ρ€Π΅Π»Π΅ для освСщСния

    КлСмма 86 ΠΏΡ€ΠΎΠ²ΠΎΠ΄Π° Ρ†Π΅ΠΏΠΈ ΠΊΠ°Ρ‚ΡƒΡˆΠΊΠΈ Ρ€Π΅Π»Π΅ ΠΏΠΎΠ΄ΠΊΠ»ΡŽΡ‡Π΅Π½Π° ΠΊ источнику горячСго питания, Π° ΠΊΠ»Π΅ΠΌΠΌΠ° 85 ΠΏΠΎΠ΄ΠΊΠ»ΡŽΡ‡Π΅Π½Π° ΠΊ Π²Ρ‹ΠΊΠ»ΡŽΡ‡Π°Ρ‚Π΅Π»ΡŽ освСщСния, ΠΊΠΎΡ‚ΠΎΡ€Ρ‹ΠΉ часто являСтся Π½Π°Π·Π΅ΠΌΠ½Ρ‹ΠΌ источником питания.

    ΠžΡ‚ Π±Π»ΠΎΠΊΠ° ΠΏΡ€Π΅Π΄ΠΎΡ…Ρ€Π°Π½ΠΈΡ‚Π΅Π»Π΅ΠΉ Π½Π° ΠΊΠ»Π΅ΠΌΠΌΡƒ 30 Ρ†Π΅ΠΏΠΈ большой силы Ρ‚ΠΎΠΊΠ° ΠΈΠ΄Π΅Ρ‚ ΠΎΠΏΠ»Π°Π²Π»Π΅Π½Π½Ρ‹ΠΉ ΠΏΡ€ΠΎΠ²ΠΎΠ΄, ΠΏΡ€ΠΈ этом вторая ΠΊΠ»Π΅ΠΌΠΌΠ° 87 ΠΏΠΎΠ΄ΠΊΠ»ΡŽΡ‡Π°Π΅Ρ‚ΡΡ ΠΊ Ρ„Π°Ρ€Π΅. ЦСпь ΠΊΠ°Ρ‚ΡƒΡˆΠΊΠΈ потрСбляСт Ρ‚ΠΎΠΊ ΠΏΠΎΡ‡Ρ‚ΠΈ 0,5 Π°ΠΌΠΏΠ΅Ρ€Π°, Ρ‡Ρ‚ΠΎ Π½Π΅ являСтся Π½Π°Π³Ρ€ΡƒΠ·ΠΊΠΎΠΉ Π½Π° Π²Ρ‹ΠΊΠ»ΡŽΡ‡Π°Ρ‚Π΅Π»ΡŒ свСта, Π² Ρ€Π΅Π·ΡƒΠ»ΡŒΡ‚Π°Ρ‚Π΅ Π²Ρ‹ΠΊΠ»ΡŽΡ‡Π°Ρ‚Π΅Π»ΡŒ Π½Π΅ сгорит.

    Π’Π°ΠΊΠΈΠΌ ΠΎΠ±Ρ€Π°Π·ΠΎΠΌ, ΠΊΠΎΠ³Π΄Π° Ρ†Π΅ΠΏΡŒ ΠΊΠ°Ρ‚ΡƒΡˆΠΊΠΈ Π°ΠΊΡ‚ΠΈΠ²ΠΈΡ€ΠΎΠ²Π°Π½Π°, Ρ†Π΅ΠΏΡŒ высокой силы Ρ‚ΠΎΠΊΠ° сомкнСтся, ΠΈ Ρ‚ΠΎΠΊ Π±ΡƒΠ΄Π΅Ρ‚ Ρ‚Π΅Ρ‡ΡŒ нСпосрСдствСнно ΠΎΡ‚ соСдинСния Π±Π°Ρ‚Π°Ρ€Π΅ΠΈ Π±Π»ΠΎΠΊΠ° ΠΏΡ€Π΅Π΄ΠΎΡ…Ρ€Π°Π½ΠΈΡ‚Π΅Π»Π΅ΠΉ ΠΊ Ρ„Π°Ρ€Π°ΠΌ, ΠΈ Π»Π°ΠΌΠΏΠΎΡ‡ΠΊΠ° загорится.

    Π‘Ρ…Π΅ΠΌΠ° ΠΏΠΎΠ΄ΠΊΠ»ΡŽΡ‡Π΅Π½ΠΈΡ 4-ΠΊΠΎΠ½Ρ‚Π°ΠΊΡ‚Π½ΠΎΠ³ΠΎ Ρ€Π΅Π»Π΅ для Π·Π²ΡƒΠΊΠΎΠ²ΠΎΠ³ΠΎ сигнала

    Π‘Ρ…Π΅ΠΌΠ° ΠΏΠΎΠ΄ΠΊΠ»ΡŽΡ‡Π΅Π½ΠΈΡ 4-ΠΊΠΎΠ½Ρ‚Π°ΠΊΡ‚Π½ΠΎΠ³ΠΎ Ρ€Π΅Π»Π΅ для Π·Π²ΡƒΠΊΠΎΠ²ΠΎΠ³ΠΎ сигнала

    КлСмма 86 Ρ†Π΅ΠΏΠΈ ΠΊΠ°Ρ‚ΡƒΡˆΠΊΠΈ Ρ€Π΅Π»Π΅ ΠΏΠΎΠ΄ΠΊΠ»ΡŽΡ‡Π΅Π½Π° ΠΊ ΠΏΠΎΠ»ΠΎΠΆΠΈΡ‚Π΅Π»ΡŒΠ½ΠΎΠΌΡƒ источнику питания аккумуляторной Π±Π°Ρ‚Π°Ρ€Π΅ΠΈ, Π° ΠΊΠ»Π΅ΠΌΠΌΠ° 85 ΠΏΠΎΠ΄ΠΊΠ»ΡŽΡ‡Π΅Π½Π° ΠΊ Π½Π°ΠΊΠ»Π°Π΄ΠΊΠ΅ Π·Π²ΡƒΠΊΠΎΠ²ΠΎΠ³ΠΎ сигнала Π½Π° Ρ€ΡƒΠ»Π΅Π²ΠΎΠΌ колСсС. , ΠΊΠΎΡ‚ΠΎΡ€Ρ‹ΠΉ являСтся Π½Π°Π·Π΅ΠΌΠ½Ρ‹ΠΌ источником питания. КлСмма 30 силовой Ρ†Π΅ΠΏΠΈ Ρ€Π΅Π»Π΅
    соСдиняСтся с ΠΏΠ»Π°Π²ΠΊΠΈΠΌ ΠΏΡ€ΠΎΠ²ΠΎΠ΄ΠΎΠΌ ΠΎΡ‚ Π±Π»ΠΎΠΊΠ° ΠΏΡ€Π΅Π΄ΠΎΡ…Ρ€Π°Π½ΠΈΡ‚Π΅Π»Π΅ΠΉ, Π° ΠΊΠ»Π΅ΠΌΠΌΠ° 87 силовой Ρ†Π΅ΠΏΠΈ ΠΏΠΎΠ΄ΠΊΠ»ΡŽΡ‡Π°Π΅Ρ‚ΡΡ ΠΊ Π·Π²ΡƒΠΊΠΎΠ²ΠΎΠΌΡƒ сигналу.

    ΠŸΡ€ΠΈΠΌΠ΅Ρ‡Π°Π½ΠΈΠ΅:

    Какой Π±Ρ‹ источник питания (ΠΏΠΎΠ»ΠΎΠΆΠΈΡ‚Π΅Π»ΡŒΠ½ΠΎΠ΅ ΠΈΠ»ΠΈ ΠΎΡ‚Ρ€ΠΈΡ†Π°Ρ‚Π΅Π»ΡŒΠ½ΠΎΠ΅ напряТСниС) Π²Ρ‹ Π½Π΅ ΠΏΠΎΠ΄ΠΊΠ»ΡŽΡ‡ΠΈΠ»ΠΈ ΠΊ ΠΎΠ±Ρ‰Π΅ΠΉ ΠΊΠ»Π΅ΠΌΠΌΠ΅ Ρ†Π΅ΠΏΠΈ высокого Ρ‚ΠΎΠΊΠ° ΠΊΠ»Π΅ΠΌΠΌΡ‹ 30. Он Π±ΡƒΠ΄Π΅Ρ‚ Π²Ρ‹Π²ΠΎΠ΄ΠΈΡ‚ΡŒ эту ΠΌΠΎΡ‰Π½ΠΎΡΡ‚ΡŒ Π½Π° ΠΊΠ»Π΅ΠΌΠΌΡƒ 87 ΠΈΠ»ΠΈ 87a ΠΏΡ€ΠΎΠ²ΠΎΠ΄Π° Π½Π°Π³Ρ€ΡƒΠ·ΠΊΠΈ.

    НапримСр, Ссли Π²Ρ‹ Π΄Π°Π΄ΠΈΡ‚Π΅ Ρ‚Π΅Ρ€ΠΌΠΈΠ½Π°Π»Ρƒ 30 источников горячСго питания. Π’ обСсточСнном состоянии Ρ€Π΅Π»Π΅ ΠΏΠΎΠ΄Π°Π΅Ρ‚ горячий источник питания Π½Π° ΠΊΠ»Π΅ΠΌΠΌΡƒ 87a, Π° Π² Π°ΠΊΡ‚ΠΈΠ²Π½ΠΎΠΌ состоянии ΠΎΠ½ΠΎ ΠΏΠΎΠ΄Π°Π΅Ρ‚ горячий источник питания Π½Π° ΠΊΠ»Π΅ΠΌΠΌΡƒ 87.

    Π’ΠΎ ΠΆΠ΅ самоС относится ΠΈ ΠΊ Π·Π°Π·Π΅ΠΌΠ»Π΅Π½Π½Ρ‹ΠΌ источникам питания.

    Π₯ΠΎΡ€ΠΎΡˆΠΈΠΉ способ Π·Π°ΠΏΠΎΠΌΠ½ΠΈΡ‚ΡŒ схСму ΠΏΠΎΠ΄ΠΊΠ»ΡŽΡ‡Π΅Π½ΠΈΡ Ρ€Π΅Π»Π΅

    Π£ Π½Π΅ΠΊΠΎΡ‚ΠΎΡ€Ρ‹Ρ… людСй Π²ΠΎΠ·Π½ΠΈΠΊΠ°ΡŽΡ‚ трудности с ΠΎΠΏΡ€Π΅Π΄Π΅Π»Π΅Π½ΠΈΠ΅ΠΌ Π½ΠΎΠΌΠ΅Ρ€ΠΎΠ² ΠΊΠΎΠ½Ρ‚Π°ΠΊΡ‚ΠΎΠ² ΠΈ ΠΊΠ»Π΅ΠΌΠΌ. Π― ΠΈΡΠΏΠΎΠ»ΡŒΠ·ΡƒΡŽ ΠΌΠ΅Ρ‚ΠΎΠ΄, ΠΊΠΎΡ‚ΠΎΡ€Ρ‹ΠΉ ΠΏΠΎΠΌΠΎΠ³Π°Π΅Ρ‚ ΠΌΠ½Π΅ Π·Π°ΠΏΠΎΠΌΠ½ΠΈΡ‚ΡŒ ΠΊΠΎΠ½Ρ‚Π°ΠΊΡ‚Π½Ρ‹Π΅ ΠΊΠ»Π΅ΠΌΠΌΡ‹.

    ΠžΠ±Ρ‹Ρ‡Π½ΠΎ ΠΊΠΎΠ½Ρ‚Π°ΠΊΡ‚Ρ‹ Ρ€Π΅Π»Π΅ ΠΈΠΌΠ΅ΡŽΡ‚ Π½ΠΎΠΌΠ΅Ρ€Π° 30, 85, 86 ΠΈ 87 (Π² 4-ΠΊΠΎΠ½Ρ‚Π°ΠΊΡ‚Π½ΠΎΠΌ Ρ€Π΅Π»Π΅).